Download Actual problems of infectious diseases and HIV infection Tests Drug

Survey
yes no Was this document useful for you?
   Thank you for your participation!

* Your assessment is very important for improving the workof artificial intelligence, which forms the content of this project

Document related concepts

Prenatal testing wikipedia , lookup

Compartmental models in epidemiology wikipedia , lookup

Syndemic wikipedia , lookup

Dysprosody wikipedia , lookup

Dental emergency wikipedia , lookup

Infection wikipedia , lookup

Marburg virus disease wikipedia , lookup

Pandemic wikipedia , lookup

Infection control wikipedia , lookup

List of medical mnemonics wikipedia , lookup

Transcript
Actual problems of infectious diseases and HIV infection
Tests
1.
Drug of choice for cholera prophylaxis is:
A.*oxytetracycline
B. chloramphenicol
C. erythromycin
D.penicillin
E.none of these
2.
The function of glucose in ORS (oral rehydration solution):
A.increase Na+ absorption by Co-transport
B. gives sweet taste to ORS
C. increase osmalality of ORS
D.*increase Na+ K- pump activity
E.increase Ca+ absorption
3.
El-Tor vibrio may be differentiated from classical vibrio by the fact that El-Tor
vibrio:
A.agglutinate chicken and sheep RBC
B. resistant to classical phage IV
C. resistant to polymixin B-5 unit disc
D.*all of the above
E.none of these
4.
Chemo-prophylaxis for cholera is administrating:
A.*doxycycline 300 mg once
B. metrogyl 400 mg 3 tablets
C. vancomycin 1 mg stat
D.kanamycin 500 mg stat
E.lincomycin 1 g
5.
The average incubation period of cholera is:
A.24 hours
B. *48 hours
C. 72 hours
D.96 hours
E.12 hours
6.
Which is not essential in cholera epidemic:
A.notification
B. oral rehydration therapy and tetracycline
C. chlorination of well every week
D.*isolation
E.chemo-prophylaxis
7.
Oral rehydration therapy does not contain:
A.sodium chloride
B. *calcium lactate
C. bicarbonate
D.glucose
E.none of these
8.
Best method to treat diarrhoea in child is:
9.
10.
11.
12.
13.
14.
15.
A.intra venous fluide
B. *ORS
C. antibiotics
D.bowel binders
E.lavage of stomach
ORS contains how much potassium:
A.*20
B. 30
C. 40
D.10
E.50
Certificate to cholera vaccination is valid after:
A.5 days
B. *10 days
C. 15 days
D.20 days
E.25 days
Drug of choice for treating cholera in a pregnant women is:
A.tetracycline
B. doxycycline
C. *furazolidone
D.cotrimoxozole
E.none of these
Best emergency sanitary measure to control cholera is:
A.disinfection of stool
B. mass vaccination
C. *provision of chlorinated water
D.chemoprophylaxis
E.none of these
Drug of choice in cholera treatment is:
A.*tetracycline
B. sulphadiazine
C. erythromycin
D.ampicillin
E.none of these
A contact carrier in cholera has following characteristic:
A.gall bladder is infected
B. stool is not positive for vibrio cholera
C. does not play any role in spread of infection
D.*duration of carrier state is less than 10 days
E.none of these
Quantity of NaCl in an ORS packet for making 1 litre of oral rehydration fluid is:
A.*3,5 gram
B. 2,5 gram
C. 1,5 gram
D.2 gram
16.
17.
18.
19.
20.
21.
22.
23.
E.3 gram
A freshly prepared oral rehydration solution should not be used after:
A.4 hours
B. 6 hours
C. 12 hours
D.*24 hours
E.48 hours
Regarding cholera vaccine which one of following is true:
A.it is given at interval of 6 months
B. long lasting immunity
C. not useful in epidemics
D.*not given orally
E.is high effective
ORS rehydration fluid does not contain:
A.NaCl
B. *calcium lactate
C. bicarbonate
D.glucose
E.none of these
What is the transport medium for cholera:
A.tellurinate medium
B. chacko-nair medium
C. *venkatraman-ramakrishna medium
D.Mc-Leods medium
E.none of these
Which of the following about cholera is true:
A.inavasive
B. endotoxin is released
C. *vibriocidal antibody titre measure prevalence
D.all of these
E.none of these
Vibrio cholera was discovered by:
A.*Koch
B. Mechnicov
C. Johnsnow
D.Virchow
E.Jenner
The characteristic feature of El-Tor cholera are all except:
A.more of subclinical cases
B. mortality is less
C. *secondary attack rate is high in family
D.El-Tor vibrio is harder and able to survive longer
E.severity is less
The growth factor required for growth of vibrio paraheamolyticus is:
A.*saline
B. tryptophan
24.
25.
26.
27.
28.
29.
30.
31.
C. bile
D.citrate
E.sugar
True about vibrio cholera is:
A.*very resistant to alkaline PH
B. nutritionally fastidious
C. best growth at 24 oC
D.rod shaped bacilli
E.all of these
The following are true about vibrio cholera except:
A.*produces indole and reduces nitrares
B. dies rapidly at low temperature
C. synthesises neuraminidases
D.vaccine confirms long immunity
E.none of these
True about epidemiology of cholera is:
A.*chemoprophylaxis is not effective
B. boiling of water can’t destroy organism
C. food can transport disease
D.vaccination give 90 % protection
E.rehydration is not effective
What percentage of fluid loss will be in IV degree of dehydration?
A.4-8 % of body weight
B. 6-9 % of body weight
C. 3-6 % of body weight
D.*Over 10 % of body weight
E.Over 15 % of body weight
At what percent of fluid loss will be I degree of dehydration?
A.*3-6 % of body weight
B. 6-9 % of body weight
C. 1-3 % of body weight
D.0,5-2 % of body weight
E.2-7 % of body weight
At I degree of dehydration the loss of liquid is:
A.0,5-1,5 % of body weight
B. 6-9 % of body weight
C. 3-6 % of body weight
D.5-8 % of body weight
E.*1-3 % of body weight
At what percent of fluid loss will be II degree of dehydration?
A.3-6 % of body weight
B. Over 10 % of body weight
C. *6-9 % of body weight
D.4-8 % of body weight
E.10-15 % of body weight
At what degree of dehydration, there will be “metabolic violation”:
A.Subcompensated
B. *Negative
C. Irreversible
D.Moderate metabolic acidosis
E.Insignificant metabolic alkalosis
32.
What time is it necessary to complete primary rehydration at dehydration shock?
A.3-5 hrs
B. 0.5 hrs
C. 2-3 hrs
D.*1-1.5 hrs
E.4-6 hrs
33.
What from the below mentioned preparations, can be used for the treatment of
primary rehydration?
A.*Rehydron
B. Acesalt
C. Khlosalt
D.Kvartasalt
E.Lactosalt
34.
What from the below mentioned preparations, can be used for the treatment of
primary rehydration?
A.Acesalt
B. Trisalt
C. *Oralit
D.Cryoplasma
E.Lactosalt
35.
What from the below mentioned preparations can be used for the treatment of
primary rehydration?
A.Lactosalt
B. *Disalt
C. Acesalt
D.Trisalt
E.Khlosalt
36.
What clinically atypical forms of cholera do you know?
A.*Very rapid of the children and elderly persons
B. “Choleric typhoid”, acute subclinical, for the children and elderly persons
C. Dry, very rapid, “choleric typhoid”, subclinical for the children and elderly persons
D.Very rapid “choleric typhoid”, acute, subclinical, for the children and elderly
persons
37.
In a settlement was found out a few cases of cholerA. Who must be insulated?
A.with dysfunction of intestine
B. Patients with cholera
C. Carriers
D.*Persons contact with the sick patient
E.Persons with hyperthermia
38.
Who must be admitted in the hospital from the focus of cholera?
A.Carriers
B. Patients with cholera
C. *Persons with dysfunction of intestine
D.Contact persons
E.Persons with high temperature
39.
In the break out of cholera it is necessary to carry out such measures, except:
A.Hyper chlorination of drinking water
B. An active discovery of patients by rounds
C. Obligatory hospitalization, inspection and treatment of patients and vibrio tests
D.Revealing and isolation of contact persons
E.*Vaccine prophylaxis
40.
With the purpose of specific prophylaxis of cholera is used:
A.*Cholerogen-toxoid
B. Vaccine
C. Nitrofuranes
D.Immunoprotein
E.Antibiotics
41.
In the different places of settlement found out a few cases of disease of cholerA.
Who from the contacts of cholera patient is sent in an insulator?
A.Vibrio positive
B. Patients with cholera
C. *Contact with the patient persons
D.Persons with dysfunction of intestine
E.Persons with high temperature
42.
Which from the below is a complication of cholera?
A.Collapse
B. Infectious-toxic shock
C. Acute renal insufficiency
D.*Dehydration shock
E.Status typhosis
43.
What salt solutions do not contain potassium?
A.Trisalt
B. Lactosalt
C. *Disalt
D.Qudrosalt
E.Khlosalt
44.
For the rehydration in dehydration shock it is necessary to conduct the permanent
careful account of all losses of liquid in each:
A.4 hrs
B. 30 hrs
C. 3 hrs
D.*2 hrs
E.5 hrs
45.
In 1 liter of Trisalt solution, the concentration of potassium chloride is:
A.3 g/l
B. 1.5 g/l
C. *1.0 g/l
46.
47.
48.
49.
50.
51.
52.
53.
D.2 g/l
E.2.5 g/l
The essential therapy for cholera is.
A.Diet
B. Antibacterial preparations
C. Correction of dysbacteriosis
D.Desintoxication
E.*Primary rehydration
The main principle of therapy for re-hydration in cholera is.
A.Determining the definitive degree of dehydration from clinical data
B. Amount of lost liquid which was preceded at time of hospitalization
C. Application of isotonic polyglucal solution
D.Simultaneous introduction of liquid in more than one vessel
E.*All are correct
Duration of therapy of primary rehydration in cholera is.
A.30 minutes
B. *2 hours
C. 6 hours
D.12 hours
E.1 days
Amount of solutions necessary for the primary rehydration in cholera is.
A.*Accordingly to the degree of dehydration at time of hospitalization
B. In accordance with the loss of liquid
C. 2 l
D.5 l
E.10 l
Amount of solutions necessary for the secondary rehydration in cholera is.
A.*Accordingly to the degree of dehydration at the time of hospitalization
B. In accordance with the loss of liquid
C. 2 l
D.5 l
E.10 l
What from is the given measures during the secondary rehydration?
A.Determining degree of dehydration from clinical data
B. Amount of lost liquid, which was preceded at the time of hospitalization
C. Application of isotonic crystalloid solutions
D.Simultaneous introduction of liquid in a few vessels
E.*Amount of liquid loss
What solutions must be applied for compensatory rehydration in cholera?
A.Colloid
B. Hypertensive epitonic polyionic crystalloid
C. *Isotonic polyionic crystalloid
D.Reosorbilact
E.Isotonic solution of glucose
Method of etiotropic therapy of cholera is.
A.Glucocorticoids
B. Antiviral
C. *Antibiotics
D.Rehydration
E.Vaccine
54.
In the different places of settlement it is found out a few cases of cholerA. Who
from such place is directed to an insulator?
A.Patients with a cholera
B. Transmitters
C. *Persons who had contact with the patient
D.Persons with dysfunction of gastro-intestinal tract
E.Persons who left the place on infection
55.
For cholera prophylaxis drug is:
A.erythromycin
B. ampicillin
C. *tetracycline
D.biseptol
E.penicillin
56.
What mechanism is typical for salmonellosis.
A.*Fecal-oral
B. Contact
C. Transmissive
D.Air-drop
E.All possible
57.
In order to prevent salmonellosis should be.
A.Disinfection
B. Vaccination
C. Chemoprophylaxis
D.*Sanitary and epidemiological control over food
E.All these measures are not undertaken
58.
What group of infectious diseases salmonellosis belong to?
A.Sapronoz
B. *Zoonosis
C. Antroponoz
D.Zooantroponoz
E.The group is not defined
59.
Salmonella is classified by.
A.*O-antigen and H-antigen
B. O-antigen and Vi-antigen
C. H-antigen and Vi-antigen
D.O-antigen, H-antigen and Vi-antigen
E.O-antigen, H-antigen, Vi-antigen and HBsAg
60.
What salmonella is adapted to humans.
A.*S. typhi
B. S. newport
C. S. cholerae-suis
D.S. abortus-ovis
61.
62.
63.
64.
65.
66.
67.
E.S. gallinarum-pullorum
What is the level of morbidity of salmonellosis nowadays in Ukraine.
A.Not registered
B. Epidemic
C. *Sporadic
D.Annual outbreaks
E.In endemic focus only
How often chronic carriering formed after salmonellosis.
A.Not formed
B. *in 0,1-1 %
C. in 8-10 %
D.in 20-30 %
E.in 50-80 %
The source of agent in salmonellosis is.
A.Cats
B. *Farm animals
C. Rodents
D.Soil
E.Feces of patients
Greatest epidemiological role in spreding of salmonella belong to.
A.*Cattle
B. Gray rats
C. Mice
D.Fish
E.Man
What is mechanism of transmission of salmonellosis.
A.Vertical
B. Parenteral
C. Air-drop
D.Contact
E.*Fecal-oral
What is most important factor in salmonellosis transmission.
A.Boiled meat
B. Fish
C. Water
D.Sex
E.*Eggs
What route of transmission is not inherent to Salmonella typhimurium.
A.Milk
B. Contact home
C. Water
D.*Sex
E.Food
68.
What typical dietary factor in spreading of salmonellosis.
A.Juices
B. Alcohol
C. *Meat products
D.Salad
E.Water
69.
What season is typical for salmonellosis.
A.Spring
B. Winter and spring
C. Autumn
D.Winter
E.*Summer-autumn
70.
What is the kind of immunity after salmonellosis.
A.Inheredited
B. *Type specific
C. Short term
D.Not formed
E.Passive
71.
What type of outbreaks appear in salmonellosis.
A.Water
B. Home
C. Farm
D.*Food
E.Milk
72.
What preventive and antepidemic activities in salmonella focused on the first
link of epidemic process.
A.*Veterinarian measures
B. Revealing, hospitalization and treatment of sick people
C. Systematic sanitary-hygienic control
D.Disinfection
E.Vaccination
73.
The rules of discharging of salmonellosis patients from a hospital .
A.One-time negative bacteriological investigation of stool
B. *Three negative bacteriological investigation of stool
C. 14 days normal body temperature and the double negative bacteriological study
stool and urine
D.Clinical recovery and normalization rectomanoscopy picture
E.Normalization rectomanoscopy picture and in the absence of antiserum to RNGA
74.
Demands according more than 3 months salmonella carrier who are working in
food production.
A.Dyspanserization
B. Recently released from work
C. Rehospitalization
D.*Do not allow to work
E.Do nothing
75.
All laboratory and instrumental tests are needed to confirming the diagnosis of
food poisoning, except:
A.*General blood analysis
B. Coprogram
C. Occupied emptying
D.Occupied sources
E.Serum researches with the autoculture of substance
76.
The etiologic diagnosis of acute intestinal infections can be confirmed thus,
except for:
A.Separation of pathogen from patients and from remainder of suspicious product
B. To obtain identical cultures of bacteria from a few patients from those which
consumed that meal
C. *Separation of identical cultures from different materials (washings, vomiting
mass, excrement) at one patient at the bacterial semination them no less than 105/g
and diminishing of this index in the process of convalescence
D.Presence at the selected culture of Escherichia’s and staphylococcus enterotoxin
E.Positive agglutination reaction or other immunological reactions with autoshtames
of possible pathogen, which testify to growth of title of antibodies on the blood
serum of patient in the dynamics of disease
77.
What is necessary for bacteriological confirmation of clostridial
gastroenterocolitis diagnosis?
A.Endo‘s medium, thermostat
B. Ploskirev‘s medium and blood agar
C. Blaurock‘s medium, thermostat
D.Endo‘s medium, anaerostat
E.*Blaurock‘s media, anaerostat
78.
Which from the listed products can become the causal factor of toxic food-borne
infection?
A.Decorative cakes
B. *Galantine
C. Cheese
D.Fresh bread
E.Tea
79.
What inoculums material should be taken to discharge the toxins?
A.*Suspected food
B. Urine
C. Stool
D.Vomiting mass
E.Medullar
80.
What is the duration of incubation period in food poisoning?
A.*2 hours – 24 hours
B. 3 days
C. 1 week
D.1 month
E.1 years
81.
What methods can confirm the diagnosis of food poisoning?
A.Diagnostic confirmation requires isolating staphyloccocci from the urine
B. Diagnostic confirmation requires isolating staphyloccocci from the stool
C. Diagnostic confirmation requires isolating staphyloccocci from the liquor
D.*Diagnostic confirmation requires isolating staphyloccocci from the suspected
food
E.All above it
82.
When the specific complication of typhoid fever like intestinal bleeding may
appier?
A.On the 1st week of illness
B. On the 2nd week of illness
C. *On the 3rd week of illness
D.On the 4th week of illness
E.On the any week of illness
83.
When the specific complication of typhoid fever like perforation of a bowel may
appier?
A.On the 1st week of illness
B. On the 2nd week of illness
C. *On the 3rd week of illness
D.On the 4th week of illness
E.On the any week of illness
84.
Who is the source of typhoid fever?
A.*Sick people
B. Sick agricultural animals
C. Sick rodents
D.Soil
E.Defecating of patients
85.
Salmonella typhi contains:
A.Only O-antigen and Н-antigen
B. Only O-antigen and Vi-antigen
C. Only H-antigen and Vi-antigen
D.*O-antigen, H-antigen and Vi-antigen
E.O-antigen, H-antigen, Vi- antigen and HBsAg
86.
When is it possible to stop etiotropic treatment in typhoid fever?
A.At once after normalization of temperature of body
B. After normalization of sizes of liver and spleen
C. After disappearance of roseollas
D.In 10 days after disappearance of roseollas
E.*After the 10th day of normalisation of body temperature
87.
By what method is it possible to find out bacterial carriering in typhoid fever?
A.Coproculture
B. Reaction of agglutination of Vidall
C. Indirect hemaglutination test with О-аntigen
D.Indirect hemaglutination test with a Н-antigen
E.*Indirect hemaglutination test with a Vi-antigen
88.
Typhoid bacilli are usually cultured from:
A.*Blood, stool, urine
B. Blood, urine, sputum
C. Stool, liquor, urine
D.Blood, stool, sputum
E.Stool, liquor, sputum
89.
A suspected case of typhoid fever of 1st week is admitted in the hospital. What
examination (laboratory diagnosis) do you suggest for this patient?
A.Coproculture
B. Reaction of agglutination of Vidall
C. Indirect hemaglutination test with О-, Н-, Vi-аntigens
D.Urinoculture
E.*Hemoculture
90.
A suspected case of typhoid fever of 3nd week is admitted in the hospital. What
examination (laboratory diagnosis) do you suggest for this patient?
A.Coproculture
B. Reaction of agglutination of Vidall
C. Urinoculture
D.Hemoculture
E.*All about it
91.
What from the transferred signs is not characteristic for a typhoid rash?
A.*Papular, disappears together with normalization of temperature of body
B. Appears on a 7-10th day, roseolla-type
C. Located mainly on a abdomen and lateral surfaces of trunk, observed at the half of
patients
D.The amount of elements is limited, pours in addition
E.Roseolla-type, sometimes saved longer than fever
92.
What symptom is not typical for typhoid on the second week of illness?
A.Constipation
B. Headache
C. Fever
D.Relative bradycardia
E.*Cramps
93.
What changes in general blood analysis are typical for typhoid fever?
A.*Leykopenia, aneosiniphilia, lymph-, monocytosis, enhanceable RSE
B. Leykocytosis, hypereosinophilia, thrombocytosis, enhanceable RSE
C. Hypochromatic anaemia, leycocytosis, appearance of young forms, RSE is not
changed
D.Hyperchromatic anaemia, leycocytosis, appearance of young forms, enhanceable
RSE
E.Leykopenia, lymphopenia, thrombocytosis, enhanceable RSE
94.
What does the diagnostic titre of reaction of Vі-haemaglutination testify to?
A.*Typhoid bacterial-carrier
B. Epidemic typhus
C. Meningococcaemia
D.Malaria
E.Brucellosis
95.
What is the duration of observation in typhoid fever?
96.
97.
98.
99.
100.
101.
102.
A.14 days
B. *21 days
C. 7 days
D.30 days
E.Does not look after
Reconvalescents of typhoid fever may discharged from a clinic after:
A.Non-permanent negative bacteriologic examination of defecating
B. *21th day of normal temperature of body and 3-multiple negative bacteriologic
examination of excrement and urine
C. of 14th day of normal temperature of body and 2-multiple negative bacteriologic
examination of excrement and urine
D.Clinical convalescence and normalization of rectal manoscopic picture
E.Normalisations of rectal manoscopic picture and in default of title of antibodies in
RNGA
In focus of typhoid fever doing, except for:
A.Thermometry
B. Coproculture
C. Vidal reaction
D.*Haemoculture
E.Urine culture
What from the adopted ways of transmission is characteristic for typhoid fever?
A.*Alimentary
B. Contact
C. Transmission
D.Air-drop
E.Vertical
What environments do typhoid sticks grow on well?
A.Chicken embryos
B. water-whey nourishing environment
C. *Bilious clear soup
D.Meat-peptone gelose + cistin
E.To the Bismute-sulfate gelose
Name of the basic factors of pathogen of typhoid stick?
A.*Vi-antigen and endotoxin
B. Exotoxin
C. Vi-antigen
D.Enzymes of pathogenicity
E.Endotoxin
Duration of latent period of typhoid fever?
A.3-7 days
B. *9-21 day
C. From a few hovers to 2-3 days
D.From 12 to 100 days
E.From a few hovers to 17 days
The place of reproduction of typhoid bacterium in the organism of human is:
A.Stomach
B. *Lymphatic formations of colon
C. Blood
D.Bilious ways
E.Mucous membrane of colon
103.
What from the indicated pathology anatomic phases is not characteristic for
typhoid?
A.*Catarrhal inflammation
B. Cerebral-type of swelling
C. Necrosis
D.Ulcers
E.Clean ulcers
104.
Food poisoning due to Staphylococcus aureus has an incubation period of:
A.*1 - 6 hours
B. 6 – 12 hours
C. 12 – 18 hour
D.more then 18 hours
E.less then 1 hour
105.
Material which should be taken for bacteriological examination in case of food
poisoning include:
A.Suspicion food products
B. Vomiting mass
C. Stool of patient
D.Washing mass
E.*All of above
106.
The immediate treatment for toxic food borne infection is:
A.Gastric lavage
B. Sorbents
C. Antibiotics
D.*Both A & B
E.All of above
107.
Which of the following is not a causative agent of food poisoning:
A.Staphylococcus aureus
B. Bacillus cereus
C. Streptococcus haemoliticus
D.Clostridium perfringens
E.*Yersinia pestis
108.
What is the main clinical symptom of food poisoning:
A.Headache
B. High fever
C. Constipation
D.Diarrhea
E.*Vomiting
109.
Pathogenesis of food borne infections involves the production of:
A.*Enterotoxins
B. Endotoxins
C. Necrotoxins
D.All of above
E.None of above
110.
The following are characteristic features of Staphylococcus aureus food
poisoning except:
A.*Optimum temperature for toxin formation is 37
B. Intra dietetic toxinns are responsible for intestinal symptoms
C. Toxins can be destroyed by boiling for 30 min
D.Incubation period is 1-6 hours
E.All of above
111.
Differential diagnosis of food poisoning is done with:
A.Cholera
B. Shigellosis
C. Salmonellosis
D.Rota viral infection
E.*All of above
112.
Which of the following is frequent complication of food poisoning:
A.*Acute heard insufficiency
B. Acute renal insufficiency
C. Acute lung insufficiency
D.Acute brain insufficiency
E.All of above
113.
Etiology agent of botulism is:
A.Campylobacter
B. Balantidium coli
C. *Cl. botulinum
D.Cl. perfrigens
E.St. Aureus
114.
What groups of symptoms are occurs in the clinic of botulism?
A.Vomiting, high temperature
B. *Dysfunction of speech and vision, breathing, dysphagia
C. Sickness, general weakness
D.High temperature, diarrhea, speech dysfunction
E.Diarrhea, vomiting, dysfunction of vision
115.
What special treatment used in beginning of the botulism?
A.*Antibiotic therapy
B. Disintoxication therapy
C. Hormonal therapy
D.Sulfonamides therapy
E.Vaccine therapy
116.
What material should be taken to find botulotoxin?
A.Stool
B. Urine
C. *Blood
D.Vomiting masses
E.All above it
117.
What specific treatment used in botulism?
118.
119.
120.
121.
A.Antibiotic therapy
B. Disintoxication therapy
C. Hormonal therapy
D.*Serum therapy
E.Vaccine therapy
What is transmissive factor of botulism?
A.Water
B. Milk
C. Air
D.*Soil
E.Animals
Drug of choice for sanation of the carriers of amoeba cysts can be.
A.Monomicyn
B. Delagil
C. Tetracycline
D.*Yatren
E.Ursosan
What is the duration of clinical reconvalescense in acute amoebiasis.
A.3-6 months
B. 1-3 months
C. *6-12 months
D.12-18 months
E.18-24 months
What agent causes balantidiasis.
A.*B. Coli
B. B. Enterocolitica
C. S. Derby
D.S. Boydi
E.L. Canicola
122.
What group of pathogens balantidiasis belong to.
A.Hlamidia
B. Mycoplasma
C. *Simplest
D.Worming
E.Rickettsiae
123.
Who is the reservour of the causative agent in balantidiasis.
A.*Pig
B. Cow
C. Sheep
D.Goat
E.Human
124.
In what disease mucous, erythrocytes, eosinophils, plasma cells and crystals
Charcot-Leiden were in stool analysis reveals.
A.*Intestinal amoebiasis
B. Intestinal yersiniosis
C. Shigellosis
D.Balantidiasis
E.Enterohaemorrhagic esherichiosis
125.
What is the main method of taking of material for parasitological examination in
case of intestinal amoebiasis.
A.In the next day of defecation
B. After processing of desinfectants
C. *Immediately after the defecation
D.1-2 hours after processing with Lyugol solution
E.2-3 hours after processing with iron hematoxylin
126.
What is the pathological changes in intestine in balantidiasis.
A.No changes
B. Ulcer
C. *Hyperemia, edema
D.Edema
E.Hyperemia without edema
127.
What is the incubation period in balantidiasis.
A.7-14 days
B. 5-10 days
C. *1-3 months
D.1-3 weeks
E.3-6 weeks
128.
What complication is typical for balantidiasis.
A.Intestinal bleeding
B. Cachexia
C. Perforated ulcer
D.Abscess of bowel
E.*All the above mentioned
129.
What are the stages of life-cycle of balantidium.
A.Cyst
B. Vegetative and spore
C. *Vegetative and cyst
D.Spore
E.Vegetative
130.
What is the mechanism of transmision in balantidiasis.
A.Contact
B. *Fecal-oral
C. Air-drop
D.Transmissive
E.Vertical
131.
What group of infectious diseases balantidiasis belongs to.
A.*Intestinal
B. Blood infection
C. Sapronosis
D.External covers
E.Antroponosis
132.
What clinical forms of balantidiasis most often may appear.
A.*Mild
B. Acute
C. Subclinical
D.Chronic
E.All of the above
133.
The complications of balantidiasis can be all except:
A.Hypochromic anemia
B. Intestinal bleeding
C. Perforation ulcers
D.*Liver abscess
E.Cachexia
134.
When a patient can be discharged from a hospital after the treatment of
balantidiasis?
A.*After clinical recovering and two negative results of microscopic study of stool
B. After clinical recovering and two negative results of bacteriological research of
stool
C. After clinical recovering and a negative result microscopic study of stool
D.After clinical recovering and normalization in the total blood
E.After clinical recovery, indicators of the overall normalization of blood and two
negative results of bacteriological research stool
135.
How long a recoveres after balantidiasis should be observed and followed-up?
A.3 months
B. 6 months
C. *1 year
D.2 years
E.5 years
136.
Name the cause of giardiasis:
A.L. Canicola
B. B. Coli
C. *L. Іntestinalis
D.S. Boidii
E.L. pomona
137.
For verification of diagnosis balantidiasis use:
A.Bacterioscopy
B. Bacteriology
C. Fluoroexam
D.*Protozooscopy
E.USD
138.
Name the cause of amoebiasis.
A.*E. Histolytica
B. L. Canicola
C. S. Derby
D.S. Boydii
E.B. Enterocolitica
139.
What is the group of pathogens of amoebiasis?
A.Mycoplasma
B. Chlamidai
C. *The simplest
D.Rickettsiae
E.Worms
140.
Who is a source of the causative agents of amoebiasis?
A.*People
B. Cows
C. Sheep
D.Pigs
E.Camels
141.
What part of lower GI tract is affected with amoebiasis most often?
A.*Descending and ascending colon
B. Sigmoid and rectum
C. Duodenun and jejunum
D.Transversal colon
E.Small intestine
142.
The causative agent of amoebiasis during life cycle can be in any form except:
A.Mature cyst
B. Immature cyst
C. Trophozoit
D.Quadrinucleate cyst
E.*Spore
143.
What group pathogenic agents of giardiasis belongs to:
A.*Simplest
B. Worming
C. Rickettsiae
D.Mycoplasma
E.Hlamidia
144.
Most of vegitative forms of amoebiasis can be found in stool of:
A.*Patients with acute intestinal amoebiasis
B. Reconvalences after acute intestinal amoebiasis
C. Patients with amebic liver abscesses
D.Patients with chronic recurrent intestinal amoebiasis in remission stage
E.All the above
145.
In what form the causative agent of acute intestinal amoebiasis could be found in
stool:
A.Minor vegetative form
B. Loomenal form
C. Cyst
D.*Tissue form
E.All the above listed forms
146.
What ulcers are specific for amoebiasis:
A.*Edematose ulcers with undermining, surrounded by aflushing zone located on the
intact mucosa
B. Smooth ulcers with undermining, located on the hyperemic mucosa
C. Necrotic ulcers, located on the hyperemic mucosa
D.Edematose ulcers with undermining located on the intact mucosa
E.Small lesions on the basis of infiltration covered with white coat
147.
What kind of mucous membrane can be found between amoebiatic ulcers of
colon:
A.Hyperemia without edema
B. Lividity, without edema
C. Hyperemia, edema
D.Regular colored edema
E.*Do not change
148.
What is the duration of incubation period for intestinal amoebiasis:
A.3-5 days
B. 4-6 days
C. *From 1 week to several months
D.1-2 years
E.From 3 months to 1 year
149.
What are the known clinical forms of amoebiasis, except:
A.Enteric
B. Skin
C. Liver abscess
D.*Myocarditis
E.Lung abscess
150.
Intestinal amoebiasis can be characterized by such complications, ecxept:
A.Amoeboma
B. Intestinal bleeding
C. Perforation of ulcers
D.*Meningitis
E.Stricture of colon
151.
What is the mechanism of amoebiasis transmission:
A.Vector borne
B. *Fecal-oral
C. Air-dropping
D.Wound
E.Vertical
152.
Which group does delagil belong to:
A.Indirect ant amoeboid
B. Direct anti amoeboid
C. *Tissue ant amoeboid
D.Product with universal effect
E.Do not belong to any of these groups
153.
How long clinical recovering lasts in acute amoebiasis.
A.3-6 months
B. 1-3 months
C. *6-12 months
D.12-18 months
E.18-24 months
154.
Drug of choice for sanation of the carriers of amoeba cysts can be.
A.Monomicyn
B. Delagil
C. Tetracycline
D.*Yatren
E.Ursosan
155.
In what disease mucous, erythrocytes, eosinophils, plasma cells and crystals
Charcot-Leiden in stool analysis were reveals.
A.*Intestinal amoebiasis
B. Intestinal yersiniosis
C. Shigellosis
D.Balantidiasis
E.Enterohaemorrhagic esherichiosis
156.
What is the main method of material taking for parasitological examination in
intestinal amoebiasis.
A.In the next day of defecation
B. After processing of desinfectants
C. *1-2 hours after processing with Lyugol solution
D.Immediately after the defecation
E.2-3 hours after processing with iron hematoxylin
157.
Name the cause of amoebiasis.
A.L. Canicola
B. *E. Histolytica
C. S. Derby
D.S. Boydii
E.B. enterocolitica
158.
What group of pathogens the agent of amoebiasis belongs to?
A.Mycoplasma
B. Chlamidia
C. *Simplest
D.Rickettsiae
E.Worms
159.
What is a source of the causal agent of amoebiasis?
A.*People
B. Cows
C. Sheep
D.Pigs
E.Camels
160.
What part of GI tract is affected in amoebiasis most often?
A.*Descending and ascending colon
B. Sigmoid and rectum
C. Doudenum and jejunum
D.Transversal colon
E.Small intestine
161.
The causal agent of amoebiasis during life cycle can be in any form except:
A.Mature cyst
B. Immature cyst
C. Trophosoit
D.Quadrinucleate cyst
E.*Spore
162.
Most of vegitative forms of amoebiasis can be found in stool of:
A.Reconvalence acute intestinal amoebiasis
B. Cystonic after use laxative
C. *The patient acute intestinal amoebiasis
D.Patients with chronic recurrent intestinal amoebiasis in remission stage
E.Patients with amebic liver abscesses
163.
In what form the causative agent is found in stool in acute intestinal amoebiasis:
A.*Tissue forms
B. Minor vegetative form
C. Loomenal form
D.Cysts
E.All the above listed forms
164.
What ulcers are specific for amebiasis:
A.Smooth ulcers with undermining, located on the hyperemic mucosa
B. Necrotic ulcers, located on the hyperemic mucosa
C. *Edematose ulcers with undermining, surrounded by aflushing zone located on the
intact mucosa
D.Edematose ulcers with undermining located on the intact mucosa
E.Small lesions on the basis of infiltration covered with white coat
165.
What kind of ulcers are present at аmebiasis?
A.Smooth sharp edges, placed on a hyperemic mucus membrane
B. *Fillings out sharp edges, surrounded by the area of hyperemia, are placed on the
unchanged mucus membrane
C. Even edges, placed on a hyperemic mucus membrane
D.Fillings out sharp edges, placed on the unchanged mucus membrane
E.Fillings out sharp edges, surrounded by the area of hyperemia, are placed on the
unchanged mucus membrane
166.
What are the known clinical forms of amoebiasis, except:
A.Enteric
B. Skin
C. Liver abscess
D.*Myocarditis
E.Lung abscess
167.
When it is possible to discharge the patients after amebiasis from infectious
hospital?
A.*After clinical convalescence, in default of in incandescence of mucous,
еosinophils, crystals of Charkot-Leiden and two negative results of parasitological
research of excrements
B. After clinical convalescence, in default of in incandescence of blood and three
negative results of parasitological research of excrements
C. After clinical convalescence, in default of leukocytosis and two negative results of
parasitological research of excrements
D.After clinical convalescence and normalization of indexes of global analysis of
blood
E.After clinical convalescence, normalization of indexes of global analysis of blood
and two negative results of bacteriological examination of excrement
168.
How often treatment of cyst amebae carrier should be done?
A.Three times a year
B. Once a year
C. *Twice a year
D.Do not done
E.Four times a year
169.
How to increase posibility of lamblia cysts in fresh feces and vegetative forms in
duodenal content revealing?
A.Cultivation in thermostat
B. By the method of floatation in bilious clear soup
C. Cultivation in anaerobic chamber
D.*By the applications of phase-contrast and lumencense microscopy with the help of
methylen-orange
E.To sow on a nourishing environment
170.
What drug is more effective in treatment of giardiasis?
A.Ursohol
B. Delagil
C. *Ornidazol
D.Tetracyclin
E.Enteroseptol
171.
What agent causes balantidiasis.
A.*B. Coli
B. B. Enterocolitica
C. S. Derby
D.S. boydi
E.L. canicola
172.
What group of pathogens balantidiasis belong to.
A.Hlamidium
B. Mycoplasmas
C. *Simplest
D.Worming
E.Rickettsiae
173.
Who is the source in balantidiasis.
A.*Pig
B. Cow
C. Sheep
D.Goat
E.Bear
174.
175.
176.
177.
178.
179.
180.
181.
What is the pathological changes in intestine in balantidiasis.
A.No changes
B. Ulcer
C. *Hyperemia, edema
D.Edema
E.Hyperemia without edema
What is the incubation period in balantidiasis.
A.7-14 days
B. 5-10 days
C. 1-3 months
D.*1-3 weeks
E.3-6 weeks
What complication is typical for balantidiasis.
A.Intestinal bleeding
B. Cachexia
C. Perforated ulcer
D.Abscess of bowel
E.*All the above mentioned
What are the forms of balantidium.
A.Cyst
B. Vegetative and spore
C. *Vegetative and cyst
D.Spore
E.Vegetative
What is the transmissive mechanism in balantidiasis.
A.Contact
B. *Fecal-oral
C. Air-drop
D.Transmissive
E.Vertical
What group of infectious diseases balantidiasis belongs to.
A.*Intestinal
B. Blood infection
C. Sapronosis
D.External covers
E.Antroponosis
What clinical forms of balantidiasis are seen most often.
A.Acute
B. Subclinical
C. *Mild
D.Chronic
E.All of the above
What is the possible complications of balantidiasis, except:
A.Hypochromic anaemia
B. Enterorrhagia
C. Perforation of ulcer
D.*Abscess of liver
E.Cachexia
182.
When would you discharge a patient with balandiasis from a hospital?
A.*After clinical recovery and two negative results of parasitological research of
excrement
B. After clinical recovery and two negative results of bacteriological examination of
excrement
C. After clinical recovery and one negative result of parasitological research of
excrement
D.After clinical recovery and normalization of indexes of global analysis of blood
E.After clinical recovery, normalization of indexes of global analysis of blood and
two negative results of bacteriological examination of excrement
183.
How long does lasts recovering after a balantidiasis?
A.6 months
B. 3 months
C. *1 year
D.2 years
E.5 years
184.
With what serum reactions it is possible to confirm the diagnosis of
balantidiasis?
A.*Complement link reaction, reaction in gel precipitation, reaction of
immobilization
B. Reaction of indirect gemagglutination, immune fluorescent reaction
C. Complement link reaction, reaction of indirect hemagglutination
D.Complement link reaction, immune fluorescent reaction, reaction of indirect
gemagglutination
E.Complement link reaction, reaction of indirect gemagglutination
185.
What test is more frequently used for verification of balantidias:
A.Virusological
B. Bacteriological
C. X-rays
D.*Research on protozoa
E.Ultrasound
186.
The clinical forms of balantidiasis are all, except?
A.*Mild rapid
B. Acute
C. Subclinical
D.Chronic continues
E.Chronic recurrent
187.
What group of pathogenic agents giardiasis belongs to?
A.Worming
B. *The simplest
C. Rickettsiae
D.Mycoplasmas
E.Hlamidii
188.
What from the given preparations can applied for etiotropic therapy of
amebiosis?
A.Osarsol
B. Metronidazol
C. Tetracycline
D.Delagil
E.*All are correct
189.
Choose the universal drug for amebiasis treatment.
A.Osarsol
B. *Metronidazole
C. Tetracycline
D.Delagil
E.All are correct
190.
What is the mechanism of ascariasis transmission?
A.Percutaneous
B. Transmissive
C. *Fecal-oral
D.Parenteral
E.Air-drop
191.
What is the phase of ascariasis pathogenesis?
A.Bacteremia
B. *Early (migratory)
C. Extraintestinal
D.Toxinemia
E.Parenchymatous diffusion
192.
What is the phase of ascariasis pathogenesis?
A.Bacteremia
B. Toxinemia
C. Extraintestinal
D.*Late (intestinal)
E.Parenchymatous diffusion
193.
What is the epidemiology of enterobiosis?
A.Zoonosis
B. Wound helminthiasis
C. Percutaneous helminthiasis
D.*Contagious helminthiasis
E.Transmissive helminthiasis
194.
What is the place of the parazitising of the agent in strongyloidosis?
A.*Upper sections of a small intestine
B. Large intestine (sigmoid)
C. Large intestine (caecum)
D.Bile ducts
E.Liver
195.
What is the source on the invasion in enterobiasis?
A.Wild animals
B. Dogs, cats
196.
C.
D.
E.
197.
198.
199.
200.
201.
202.
C. *Humans
D.Mollusks
E.Fish
What are the ways of the infection in ancylostomosis?
A.*Peroral
B.
Parentral
Air-drop
Transmissive
All are corect
What are the main clinical sighs of the early stage of ancylostostomiasis?
A.Dermatitis (polymorphic rash, itch)
B. Damage of the respiratory tract (bronchitis, laryngotracheitis, eosinophilic
infiltrates)
C. Fever
D.Eosinophilia (30-60 %)
E.*All are corect
What are the principal clinical sighs of the chronic stage of ancylostostomiasis?
A.Hepasplenomegaly
B. B 12 deficiency anemia
C. *Nausea, vomiting, diarrhea, abdominal pain (gastroduodenitis)
D.Fever with attacks in 48 hours
E.Pericarditis
What are the principal clinical sighs of the chronic stage of ancylostostomiasis?
A.Constipation, abdominal pain
B. Hepasplenomegaly
C. *Iron deficiency anemia
D.Fever with attacks in 48 hours
E.Pericarditis
What is epidemiology of strongyloidiasis?
A.Anthroponosis
B. Geogelminthiasis
C. The way of the infection is peroral
D.The way of the infection is percutaneous
E.*All right
What are the clinical forms of the late stage of strongyloidiasis?
A.Gastrointestinal form, allergic-toxic form
B. Allergic-toxic form, meningoencephalitic form
C. *Gastrointestinal form, allergic-toxic form, mixed form
D.Meningeal, meningoencephalitic form
E.Acute hepatic insufficiency, gastrointestinal form,
What is the agent of pork worm disease?
A.*Taenia soleum
B. Hymenolepis nana
C. Taeniarhynchus saginatus
D.Trichinella spiralis
E.Dyphylobotrium latum
203.
204.
205.
206.
207.
208.
209.
210.
What are the principal clinical syndromes in pork worm disease?
A.Cholestatic syndrome
B. Renal syndrome
C. *Dyspeptic syndrome
D.Meningeal syndrome
E.Respiratory syndrome
What are the principal clinical syndromes in pork worm disease?
A.Renal syndrome
B. Cholestatic syndrome
C. *General toxic syndrome
D.Meningeal syndrome
E.Respiratory syndrome
What is the epidemiology of cystecercosis?
A.The infection is an endogenic (autoinvasion)
B. The human is the definitive host
C. The pigs are an intermediate host
D.The human is an intermediate host
E.*All are correct
What methods are used for diagnostics of cystecercosis?
A.*Ophthalmoscope
B. Biochemistry methods
C. Cardiogram
D.Biological test
E.Intracutaneous test
What methods are used for diagnostics of cystecercosis?
A.Cardiogram
B. Biochemistry methods
C. *Computer tomography
D.Biological test
E.Intracutaneous test
What methods are used for diagnostics of cystecercosis?
A.Biological test
B. Biochemistry methods
C. Cardiogram
D.*Roentgenograms
E.Intracutaneous test
What is epidemiology of fascioliasis?
A.It is anthroponosis
B. *It is zoonosis
C. The definitive host is camel and cattle
D.The intermediate hosts are the pigs
E.The intermediate hosts are mice
What is epidemiology of fascioliasis?
A.It is anthroponosis
B. It is sapronosis
C. *The definitive host is human and cattle
211.
212.
food:
D.The intermediate hosts are the pigs
E.The intermediate hosts are mice
What is epidemiology of fascioliasis?
A.It is anthroponosis
B. It is sapronosis
C. The definitive host is mice and cattle
D.The intermediate hosts are the pigs
E.*The intermediate hosts are mollusks
The invasion of the human with Dyphillobotirum latum is possible in use into the
A.*Fresh water fish
B. Sea fish
C. Pork
D.Beef
E.Milk
213.
What is the duration of the life of the adult forms in echinoccosis?
A.*It is continued from 6 months till 1 year
B. It is continued till 1 month
C. It is continued from 1 year till 3 years
D.It is continued more than 3 years
E.It is continued 7 days
214.
What is epidemiology of hymenolepidosis?
A.It is percutaneous helminthiasis
B. It is not contagious helminthiasis
C. *It is peroral helminthiasis
D.It is transmissive helminthiasis
E.It is especially dangerous disease
215.
What is epidemiology of hymenolepidosis?
A.It is not peroral helminthiasis
B. It is percutaneous helminthiasis
C. *It is contagious helminthiasis
D.It is transmissive helminthiasis
E.It is especially dangerous disease
216.
What is the possible localization of hydatid cyst?
A.Liver
B. Lungs
C. Brain
D.Ovaries
E.*All are correct
217.
How to increase frequency of findings of cyst of lamblias in fresh incandescence
and vegetative forms in duodenal content?
A.Cultivation in thermostat
B. By the method of floatation in bilious clear soup
C. Cultivation in anaerobic chamber
D.*By the applications of phase-contrast and lumencense microscopy with the help of
methylen-orange
218.
219.
220.
221.
222.
223.
224.
225.
E.To sow on a nourishing environment
What preparation is it more expedient to apply for giardiasis treatment?
A.Ursohol
B. Delagil
C. *Ornidazol
D.Tetracyclin
E.Enteroseptol
Name the cause of giardiasis:
A.L. Canicola
B. B. Coli
C. *L. Іntestinalis
D.S. Boidii
E.L. pomona
The most effective means of filariasis control will be:
A.Yatren therapy
B. Insecticidal measures against culex mosquitoes
C. Provision of underground drainage
D.*Personal prophylaxix
E.All mentioned above
Causative agent of Giardiasis is:
A.G. lamblia
B. G. intestinalis
C. *G. lamblia and G. intestinalis
D.B. coli
E.None of the above
Cryptosporidiosis is:
A.Blood borne infection
B. Respiratory infection
C. *Intestinal infection
D.Infection of external covers
E.Helminthiasis
Balantidiasis is caused by:
A.*B. coli
B. B. hominis
C. E. intestinalis
D.Cyclospora cayetanensis
E.B. melitensis
Trichinellosis develops after:
A.Bite of a tick
B. Drinking of contaminated water
C. *Ingestion of the infected meat
D.Bite of a dog
E.All of the above
Which of the following species of Trichinella are distributed world wide:
A.T. nelsoni
B. T. spiralis
226.
227.
228.
229.
230.
231.
232.
233.
C. T. nativa
D.*All mentioned above
E.None
What is the main symptom of the Trichinellosis:
A.Rash
B. Muscle pain
C. Edema of eyelids
D.Nodules in muscles
E.*All mentioned above
Which of the following is the largest intestinal helminthes in human:
A.*D. latum
B. S. stercoralis
C. Anisakis simplex
D.E. vermicularis
E.T. saginatus
Loffler’s syndrome in ascariasis is due to:
A.Inflitration of payer’s patches by eosinophills
B. Invasion of gallblader by A.lumbricoides pathogen
C. *inflitration of lung tissue by eosinophills
D.Inflitration of liver by eosinophills
E.None of the above
Which drug can be used in pregnancy in case of ascariasis?
A.Albendazole
B. Mebendazole
C. Pyrantel pamoate
D.Ivermectin
E.*Piperasin adipinat
Which of the following is known as pinworm?
A.*E. vermicularis
B. E. duodenale
C. N. americanus
D.T. solium
E.All of the above
What is larva currents?
A.Dead larva
B. Floating larva
C. *Running larva
D.Slipping larva
E.None of the above
All of the following are the blood flukes except:
A.Schistosoma japonicum
B. Fasciola gigantica
C. Clonorchis sinensis
D.Fasciola hepatica
E.*Echinococcus granulosis
Chyluria is the complication of:
A.*Lymphatic filariasis
B. Abdominal angiostrongyliasis
C. Enterobiasis
D.Trichuriasis
E.Amebiasis
234.
Drug of choice for the treatment of the lymphatic filariasis is:
A.Albendazole
B. Glucorticoids
C. *Diethylcarbamazine
D.Metronidazole
E.Chloramphenicol
235.
The distinctive pattern of movement of filarial worms in lymphatic vessels is
known as:
A.Filaria jumping sign
B. *Filaria dance sign
C. Filaria swim sign
D.Filaria escape sign
E.Filaria flying sign
236.
Onchoceriasis is also known as:
A.Tropical pulmonary eosinophillia
B. *River blindness
C. Guinea worm infection
D.African eye worm disease
E.Ricketsiosis
237.
B-12 deficiency is cause by which of the following:
A.Echinococus granulosis
B. T. saginata
C. E. multilocularis
D.*Diphyllobothrium latum
E.Ascaris lumbricoideus
238.
Ascaridiosis is:
A.Bacterial infection
B. Viral infection
C. Protozoosis
D.Fungal infection
E.*Helminthosis
239.
Ascaridiosis belongs to:
A.*Nematodosis
B. Trematodosis
C. Cestodosis
D.Ricketsiosis
E.Mycosis
240.
Toxocarosis is:
A.Bacterial infection
B. Viral infection
C. Protozoosis
241.
242.
243.
244.
245.
246.
247.
248.
D.Fungal infection
E.*Helminthosis
Toxocarosis belongs to:
A.Trematodosis
B. Cestodosis
C. Nematodosis
D.Ricketsiosis
E.Mycosis
Enterobiosis is:
A.Bacterial infection
B. Viral infection
C. Protozoosis
D.Fungal infection
E.*Helminthosis
Enterobiosis belongs to:
A.Trematodosis
B. *Nematodosis
C. Cestodosis
D.Ricketsiosis
E.Mycosis
Trichinosis is:
A.Bacterial infection
B. Viral infection
C. Protozoosis
D.Fungal infection
E.*Helminthosis
Trichinosis belongs to:
A.*Nematodosis
B. Trematodosis
C. Cestodosis
D.Ricketsiosis
E.Mycosis
Strongyloidosis is:
A.Bacterial infection
B. Viral infection
C. Protozoosis
D.Fungal infection
E.*Helminthosis
Strongyloidosis belongs to:
A.Trematodosis
B. *Nematodosis
C. Cestodosis
D.Ricketsiosis
E.Mycosis
Schistosomosis is:
A.Bacterial infection
249.
250.
251.
252.
253.
254.
255.
B. Viral infection
C. Protozoosis
D.Fungal infection
E.*Helminthosis
Schistosomosis belongs to:
A.Nematodosis
B. *Trematodosis
C. Cestodosis
D.Ricketsiosis
E.Mycosis
Fasciolosis is:
A.Bacterial infection
B. Viral infection
C. Protozoosis
D.Fungal infection
E.*Helminthosis
Fasciolosis belongs to:
A.Nematodosis
B. *Trematodosis
C. Cestodosis
D.Ricketsiosis
E.Mycosis
Opisthorchosis is:
A.Bacterial infection
B. Viral infection
C. *Helminthosis
D.Protozoosis
E.Fungal infection
Opisthorchosis belongs to:
A.Nematodosis
B. *Trematodosis
C. Cestodosis
D.Ricketsiosis
E.Mycosis
Echinococcosis is:
A.Bacterial infection
B. Viral infection
C. Protozoosis
D.Fungal infection
E.*Helminthosis
Echinococcosis belongs to:
A.Nematodosis
B. Trematodosis
C. *Cestodosis
D.Ricketsiosis
E.Mycosis
256.
257.
258.
259.
260.
261.
262.
263.
Alveococcosis is:
A.Bacterial infection
B. *Helminthosis
C. Viral infection
D.Protozoosis
E.Fungal infection
Alveococcosis belongs to:
A.Nematodosis
B. Trematodosis
C. *Cestodosis
D.Ricketsiosis
E.Mycosis
Diphyllobothriosis is:
A.Bacterial infection
B. Viral infection
C. *Helminthosis
D.Protozoosis
E.Fungal infection
Diphyllobothriosis belongs to:
A.Nematodosis
B. Trematodosis
C. *Cestodosis
D.Ricketsiosis
E.Mycosis
Teniarinchosis is:
A.Bacterial infection
B. Viral infection
C. Protozoosis
D.Fungal infection
E.*Helminthosis
Teniarinchosis belongs to:
A.Nematodosis
B. Trematodosis
C. *Cestodosis
D.Ricketsiosis
E.Mycosis
Teniosis is:
A.Bacterial infection
B. Viral infection
C. *Helminthosis
D.Protozoosis
E.Fungal infection
Teniosis belongs to:
A.Nematodosis
B. Trematodosis
C. *Cestodosis
D.Ricketsiosis
E.Mycosis
264.
Cycticercosis is:
A.Bacterial infection
B. *Helminthosis
C. Viral infection
D.Protozoosis
E.Fungal infection
265.
Cycticercosis belongs to:
A.Nematodosis
B. Trematodosis
C. *Cestodosis
D.Ricketsiosis
E.Mycosis
266.
What clinical forms of balantidiasis are seen most often.
A.*Mild
B. Acute
C. Subclinical
D.Chronic
E.All of the above
267.
Give recommendations for a patient in reconvalensent period of viral hepatitis
during a clinical supervision.
A.A medical supervision during 6 month
B. Biochemical inspection
C. Abstain from hard physical load
D.Temporal contra-indications for prophylactic inoculations
E.*All the above
268.
What is etiotropic therapy of viral hepatitis.
A.Ribavirin
B. Interferon
C. Inductors of interferon
D.Zefix
E.*All the above
269.
Choose the remedies for etiotropic therapy of viral hepatitis.
A.*Interferons
B. Vaccine
C. Normal human immune globulin
D.Hepatoprotectors
E.Glucocorticoids
270.
Choose the remedies for etiotropic therapy for viral hepatitis.
A.Antibiotics
B. *Interferons
C. Probiotics
D.Vaccine
E.Normal human immunoprotein
271.
The criteria for application of etiotropic therapy in viral hepatitis is.
A.Protracted motion of HBV, HVD
B. Any form of HV
C. Biochemical activity
D.Presence of virus replication
E.*All the above
272.
The criteria for application of etiotropic therapy for the patient with HCV.
A.Clinical displays are insignificant
B. Icterus is absent
C. Moderate biochemical activity
D.There is anti-HCV in blood
E.*RNA of HCV +
273.
Factors which are indications of successful interferon therapy in HV infections
are all, except.
A.Level of ALaT not more than 2-3 norm
B. Low titre of HCV after the treatment
C. Absence of cholestasis
D.2th and 4th genotypes of HCV
E.*Expressed fibrosis
274.
Indirect action of interferon therapy.
A.Influenza-like syndrome
B. Nausea
C. Itching
D.Para-hypnosis
E.*All the above||
275.
.Give recommendation for a patient in reconvalensent period of viral hepatitis
during a clinical supervision after isolation.
A.*Medical supervision during 6 months, periodic biochemical inspections.
B. Control bacteriological examinations
C. Full labor investigation
D.To continue prophylactic inoculations
E.Supervision is not needed
276.
Indirect action of interferons.
A.Flatulence
B. Diarrhea
C. Nausea
D.Depression
E.*All the above
277.
Indirect action of interferon therapy are all except.
A.Influenza-like syndrome
B. Nausea
C. Depression
D.Intensification of autoimmune diseases
E.*Progress of fibrosis
278.
. Basic principles of antiviral therapy for viral hepatitis.
A.Individual selection of dose and rhythm of application of preparations
B. Duration of introduction of preparations
279.
280.
281.
282.
283.
284.
285.
C. Control of amount of erytrocytes, leucocytes and thrombocytes, in blood
D.Control of iron level in blood
E.*All the above
. Contra-indications for antiviral therapy of viral hepatitis.
A.Decompensatory cirrhosis of liver
B. Thrombocytopenia <50000 in 1 мм3
C. Psychic disorders
D.Leucocytopenia <1500 in 1 мм3
E.*All the above
Contra-indications for antiviral therapy of viral hepatitis.
A.A.Decompensatory cirrhosis of liver
B. Autoimmune disease
C. Alcoholism and other drug addictions
D.D.Coinfection by HIV
E.*All the above
Choose the indexes of efficiency of interferon therapy.
A.*Disappearance| of markers of viral replication
B. Improvement of the general state
C. Normalization of the liver size
D.Disappearance of icterus
E.All the above
Choose the indexes of efficiency of interferon therapy.
A.Improvement of the general state
B. *Normalization of activity of ALaT
C. Normalization of the liver size
D.Disappearance of icterus
E.All the above
Types of answer for interferon therapy are.
A.Stable remission
B. Unsteady
C. Partial answer
D.Absence of answer
E.*All the above
The characteristic of an unsteady answer of interferon therapy are.
A.Disappearance of markers of viral replication upon completion of course of therapy
B. Normalization of activity of ALaT during the course of therapy
C. An origin of relapse in next 6 months
D.Disappearance of icterus
E.*All the above
That characteristic of a partial answer of interferon therapy are all, except.
A.*Disappearance of markers of viral replication
B. Normalization of activity of ALaT is upon completion of course of therapy
C. Disappearance of icterus
D.Normalization the state of patient
E.Normalization of the size of liver
286.
When is interferon therapy effective in the the patient.
A.Normalization of the state of patient
B. Normalization of activity of ALaT upon completion of course of therapy
C. Disappearance of icterus
D.Normalization of the size of liver
E.*The markers of viral replication, are determined upon completion of course of
therapy
287.
What laboratory work-up is needed for confirming the diagnosis of viral
hepatitis.
A.Total analysis of blood
B. Determination of level of bilirubin
C. Determination of activity of aminotransferase
D.*Determination of markers of HV in IFA
E.All the above
288.
What laboratory and instrumental examinations are needed for confirming the
diagnosis of viral hepatitis.
A.Complete analysis of blood
B. Ultrasound of abdominal region
C. Determination of activity of aminotransferase
D.*Determination of antigen of viruses
E.Duodenal probing
289.
What is incubation period for hepatitis B:
A.45 days
B. *180 days
C. 360 days
D.90 days
E.25 days
290.
All the hepatitis have parenteral route of transmission except:
A.*A
B. B
C. C
D.D
E.TTV
291.
Chronic course is common for viral hepatitis except:
A.*A
B. B
C. C
D.D
E.B+C
292.
All the following medicines are interferons except:
A.Intron
B. Roferon
C. Reaferon
D.Leukinferon
E.*Cycloferon
293.
All the following medicines are hepatoprotective agents except:
A.Carsil
B. Silibor
C. Legalon
D.*Lomusol
E.Arginine
294.
What is mechanism of transmission in viral hepatitis A:
A.Contact
B. Transmissive
C. Vertical
D.*Fecal-oral
E.Air-drop
295.
What is mechanism of transmission in viral hepatitis B:
A.*Contact
B. Transmissive
C. Alimentary
D.Fecal-oral
E.Air-drop
296.
What is mechanism of transmission in viral hepatitis C:
A.*Contact
B. Transmissive
C. Alimentary
D.Fecal-oral
E.Air-drop
297.
What is mechanism of transmission in viral hepatitis D:
A.*Contact
B. Transmissive
C. Alimentary
D.Fecal-oral
E.Air-drop
298.
What is mechanism of transmission in viral hepatitis E:
A.Contact
B. Sexual
C. Alimentary
D.*Fecal-oral
E.Air-drop
299.
At inspection of a 8 weeks term pregnant woman HBsAg was found. Level of
bilirubin of blood and activity of ALAT were normal. What is necessary to do?
A.*To save pregnancy and conduct the inoculation to newborn against hepatitis B
B. Termination of pregnancy
C. Termination of pregnancy and conduct treatment by interferon
D.To save pregnancy and conduct treatment by lamivudin
E.To save pregnancy and ultrasonic inspection of the fetus
300.
Diagnosed a patient: chronic hepatitis in the stage of integration. What markers
will be in
A.patient in this stage disease?
B. HBeAg
C. Antibodies to HBeAg
D.DNA OF HBV
E.Viral DNA-polimerase
F. *HBsAg, anti-НBе
301.
As etiotropic therapy of sharp and chronic viral hepatitis B utillize:
A.Corticosteroid
B. Immunomodulate preparations
C. Cytostatics
D.Antibiotics
E.*Antiviral preparations
302.
When did order a patient with virus hepatitis?
A.Non-permanent negative virologic research of defecating
B. 21 days of normal temperature of body
C. Of normalization of level of bilirubinum
D.Clinical convalescence and normalization of level of transaminasis
E.*Clinical convalescence and no more than triple increase of level of transaminasis
303.
Give recommendation for a patient in reconvalensent period of viral hepatitis
during a clinical supervision after isolation.
A.A medical supervision during 6 month
B. Biochemical inspection
C. Abstain from hard physical load
D.Temporal contra-indications for prophylactic inoculations
E.*All the above
304.
Give recommendation for a patient in reconvalensent period of viral hepatitis
during a clinical supervision after isolation.
A.*Medical supervision during 6 months, periodic biochemical inspections.
B. Control bacteriological examinations
C. Full labor investigation
D.To continue prophylactic inoculations
E.Supervision is not needed
305.
Etiotropic therapy of viral hepatitis is.
A.Ribavirin
B. Interferon
C. Inductors of interferon
D.Zefix
E.*All the above.
306.
Choose the remedies for etiotropic therapy of viral hepatitis.
A.*Ribavirin
B. Vaccine
C. Normal human immunoprotein
D.Hepatoprotector
E.Glucocorticoid
307.
Choose the remedies for etiotropic therapy for viral hepatitis.
A.Antibiotics
B. *Interferon
C. Probiotics
D.Vaccine
E.Normal human immunoprotein
308.
The criteria for application of etiotropic therapy in viral hepatitis is.
A.Protracted motion of HBV, HVD
B. Any form of HV
C. Biochemical activity
D.Presence of virus replication
E.*All the above.
309.
The criteria for application of etiotropic therapy for the patient with HCV.
A.Clinical displays are insignificant
B. Icterus is absent
C. Moderate biochemical activity
D.There is anti-HCV in blood
E.*RNA of HCV +
310.
Factors which are indications of successful interferon therapy in HV infections
are all, except.
A.Level of ALaT not more than 2-3 norm
B. Low titre of HCV after the treatment
C. Absence of cholestasis
D.2th and 4th genotypes of HCV
E.*Expressed fibrosis
311.
Indirect action of interferon therapy.
A.Influenza-like syndrome
B. Nausea
C. Itching
D.Para-hypnosis
E.*All the above
312.
Indirect action of interferons.
A.Flatulence
B. Diarrhea
C. Nausea
D.Depression
E.*All the above
313.
Indirect action of interferon therapy are all except.
A.Influenza-like syndrome
B. Nausea
C. Depression
D.Intensification of autoimmune diseases
E.*Progress of fibrosis
314.
Basic principles of antiviral therapy for viral hepatitis.
A.Individual selection of dose and rhythm of application of preparations
B. Duration of introduction of preparations
C. Control of amount of erytrocytes, leucocytes and thrombocytes, in blood
D.Control of iron level in blood
315.
316.
317.
318.
319.
320.
321.
322.
E.*All the above
Contra-indications for antiviral therapy of viral hepatitis.
A.Decompensatory cirrhosis of liver
B. Thrombocytopenia <50000 in 1 мм3
C. Psychic disorders
D.Leucocytopenia <1500 in 1 мм3
E.*All the above
Contra-indications for antiviral therapy of viral hepatitis.
A.Decompensatory cirrhosis of liver
B. Autoimmune disease
C. Alcoholism and other drug addictions
D.Coinfection by HIV
E.*All the above
Choose the indexes of efficiency of interferon therapy.
A.*Disappearance| of markers of viral replication
B. Improvement of the general state
C. Normalization of the liver size
D.Disappearance of icterus
E.All the above
Choose the indexes of efficiency of interferon therapy.
A.Improvement of the general state
B. *Normalization of activity of ALaT
C. Normalization of the liver size
D.Disappearance of icterus
E.All the above
Types of answer for interferon therapy are.
A.Stable remission
B. Unsteady
C. Partial answer
D.Absence of answer
E.*All the above
The characteristic of an unsteady answer of interferon therapy are.
A.Disappearance of markers of viral replication upon completion of course of therapy
B. Normalization of activity of ALaT during the course of therapy
C. An origin of relapse in next 6 months
D.Disappearance of icterus
E.*All the above
That characteristic of a partial answer of interferon therapy are all, except.
A.*Disappearance of markers of viral replication
B. Normalization of activity of ALaT is upon completion of course of therapy
C. Disappearance of icterus
D.Normalization the state of patient
E.Normalization of the size of liver
When is interferon therapy effective in the the patient.
A.Normalization of the state of patient
B. Normalization of activity of ALaT upon completion of course of therapy
C. Disappearance of icterus
D.Normalization of the size of liver
E.*The markers of viral replication, are determined upon completion of course of
therapy
323.
What laboratory work-up is needed for confirming the diagnosis of viral
hepatitis.
A.General blood analysis
B. Determination of level of bilirubin
C. Determination of activity of aminotransferase
D.*Determination of markers of HV in IFA
E.All the above
324.
What laboratory and instrumental examinations are needed for confirming the
diagnosis of viral hepatitis.
A.General blood analysis
B. Ultrasound of abdominal region
C. Determination of activity of aminotransferase
D.*Determination of antigen of viruses
E.Duodenal probing
325.
What is maximal incubation period for hepatitis B:
A.45 days
B. *180 days
C. 360 days
D.90 days
E.25 days
326.
All the hepatitis have parenteral route of transmission except:
A.*A
B. B
C. C
D.D
E.TTV
327.
Chronic course is common for viral hepatitis except:
A.*A
B. B
C. C
D.D
E.B+C
328.
All the following medicines are interferons except:
A.Intron
B. Roferon
C. Reaferon
D.Leukinferon
E.*Cycloferon
329.
All the following medicines are hepatoprotective agents except:
A.Carsil
B. Silibor
C. Legalon
330.
331.
332.
333.
334.
335.
336.
337.
D.*Lomusol
E.Arginine
What is incubation period for hepatitis A:
A.*45 days
B. 180 days
C. 360 days
D.90 days
E.25 days
What group of infectious diseases hepatitis A belong to:
A.External covers
B. *Intestinal
C. Blood
D.Wound
E.Transmissive
What group of infectious diseases hepatitis B belong to:
A.*External covers
B. Intestinal
C. Blood
D.Wound
E.Transmissive
What group of infectious diseases hepatitis C belong to:
A.*External covers
B. Intestinal
C. Blood
D.Wound
E.Transmissive
What group of infectious diseases hepatitis D belong to:
A.*External covers
B. Intestinal
C. Blood
D.Wound
E.Transmissive
What group of infectious diseases hepatitis E belong to:
A.External covers
B. *Intestinal
C. Blood
D.Wound
E.Transmissive
What is transmissive factor for hepatitis A:
A.Blood
B. Semen
C. *Water
D.Air
E.Milk of mother
What is transmissive factor for hepatitis B:
A.Food
338.
339.
340.
341.
342.
343.
344.
B. Milk
C. Water
D.Air
E.*Milk of mother
What recommendations for a patient in reconvalensent period of viral hepatitis.
A.A medical supervision during 6 month
B. Biochemical inspection
C. Abstain from hard physical load
D.Temporal contra-indications for prophylactic inoculations
E.*All the above
Medicines for etiotropic therapy of viral hepatitis.
A.Ribavirin
B. Interferon
C. Inductors of interferon
D.Zefix
E.*All the above
What drugs is etiotropic therapy of viral hepatitis.
A.*Interferons
B. Vaccine
C. Normal human immune globulin
D.Hepatoprotectors
E.Glucocorticoids
What criteria for application of etiotropic therapy in viral hepatitis is.
A.Protracted motion of HBV, HVD
B. Any form of HV
C. Biochemical activity
D.Presence of virus replication
E.*All the above
What criteria for application of etiotropic therapy for the patient with HCV.
A.Clinical displays are insignificant
B. Icterus is absent
C. Moderate biochemical activity
D.There is anti-HCV in blood
E.*RNA of HCV +
Signs of efficiency of interferon therapy in HV infections are all, except.
A.Level of ALaT not more than 2-3 norm
B. Low titre of HCV after the treatment
C. olestasis
D.2th and 4th genotypes of HCV
E.*Expressed fibrosis
What is indirect action of interferon therapy.
A.Influenza-like syndrome
B. Nausea
C. Itching
D.Para-hypnosis
E.*All the above||
345.
346.
347.
348.
349.
350.
351.
352.
353.
What recommendation for viral hepatitis patient in recovering period.
A.*Medical supervision during 6 months, periodic biochemical inspections.
B. Control bacteriological examinations
C. Full labor investigation
D.To continue prophylactic inoculations
E.Supervision is not needed
What is indirect action of interferons.
A.Flatulence
B. Diarrhea
C. Nausea
D.Depression
E.*All the above
Indirect action of interferon therapy are all except.
A.Influenza-like syndrome
B. Nausea
C. Depression
D.Intensification of autoimmune diseases
E.*Progress of fibrosis
What is basic principles of antiviral therapy for viral hepatitis.
A.Individual selection of dose and rhythm of application of preparations
B. Duration of introduction of preparations
C. Control of amount of erytrocytes, leucocytes and thrombocytes, in blood
D.Control of iron level in blood
E.*All the above
What is contra-indications for antiviral therapy of viral hepatitis.
A.Decompensatory cirrhosis of liver
B. Thrombocytopenia <50000 in 1 мм3
C. Psychic disorders
D.Leucocytopenia <1500 in 1 мм3
E.*All the above
What is contra-indications for antiviral therapy of viral hepatitis.
A.A.Decompensatory cirrhosis of liver
B. B. Autoimmune disease
Alcoholism and other drug addictions
A.D.Coinfection by HIV
B. *E. All the above
Choose the indexes of efficiency of interferon therapy.
A.*Disappearance of markers of viral replication
B. Improvement of the general state
C. Normalization of the liver size
D.Disappearance of icterus
E.All the above
Choose the indexes of efficiency of interferon therapy.
A.Improvement of the general state
B. *Normalization of activity of ALaT
C. Normalization of the liver size
354.
355.
356.
357.
358.
359.
360.
361.
D.Disappearance of icterus
E.All the above
What is types of answer for interferon therapy.
A.Stable remission
B. Unsteady
C. Partial answer
D.Absence of answer
E.*All the above
What is organism answer of interferon therapy.
A.Disappearance of markers of viral replication upon completion of course of therapy
B. Normalization of activity of ALaT during the course of therapy
C. Relapse in next 6 months
D.Disappearance of icterus
E.*All the above
Answer of interferon therapy are all, except.
A.*Disappearance of markers of viral replication
B. Normalization of ALaT activity
C. Disappearance of icterus
D.Normalization of patient general condition
E.Normalization of a liver size
What investigation for confirming of viral hepatitis diagnosis.
A.Total analysis of blood
B. Determination of bilirubin level
C. Determination of aminotransferases activity
D.*Determination of HV markers by IFA
E.All the above
What examination is helpful in confirming of viral hepatitis diagnosis.
A.Complete analysis of blood
B. Ultrasound of abdominal region
C. Determination of activity of aminotransferase
D.*Determination of antigen of viruses
E.Duodenal probing
What is duration of incubation period for B hepatitis :
A.45 days
B. *180 days
C. 360 days
D.90 days
E.25 days
Parenteral way of transmission is present in spreding of hepatitis except:
A.*A
B. B
C. C
D.D
E.TTV
Chronic course is possible for viral hepatitis except:
A.*A
362.
363.
364.
365.
366.
367.
368.
B. B
C. C
D.D
E.B+C
All the following medicines are interferons except:
A.Intron
B. Roferon
C. Reaferon
D.Leukinferon
E.*Cycloferon
All the following medicines are hepatoprotective agents except:
A.Carsil
B. Silibor
C. Legalon
D.*Lomusol
E.Arginine
What is mechanism of viral hepatitis A transmission:
A.Contact
B. Transmissive
C. Vertical
D.*Fecal-oral
E.Air-drop
What is mechanism of hepatitis B transmission:
A.*Contact
B. Transmissive
C. Alimentary
D.Fecal-oral
E.Air-drop
What is mechanism hepatitis C transmission:
A.*Contact
B. Transmissive
C. Alimentary
D.Fecal-oral
E.Air-drop
What rash is present in case of haemorrhagic fevers with kidneys syndrome?
A.Roseolar
B. Maculo-papular
C. Punctuate
D.*Petechial
E.Rash is not present
What rash is present in case of Congo hemorrhagic fever?
A.Roseola
B. Maculo-papular
C. Punctulate
D.*Petechial
E.Rashes not is characteristic
369.
What rashes is present in case of Crimea hemorrhagic fever?
A.Roseolar
B. Maculo-papular
C. Punctulate
D.*Petechial
E.Rashes not is characteristic
370.
How long the rash is present in case of haemorrhagic fever with kidneys
syndrome?
A.*During all feverish period
B. Before the reconvalescense
C. Before development of clinical features of kidneys insufficiency
D.During whole disease
E.Appears yet in a latent period and disappears in the period of early reconvalescense
371.
A kidney syndrome at haemorrhagic fever with kidneys syndrome shows up
usually:
A.Only laboratory changes
B. Only on BRIDLES
C. *By pain in lumbar area, positive Pasternatsky symptom, development of oliguria
D.By fever, polyuria, dyspepsia
E.By paradoxical ischuria
372.
What changes in biochemical blood test inherent for haemorrhagic fever with
kidneys syndrome?
A.Increase level of urea and bilirubin
B. The level of urea and kreatinine falls
C. The level of kreatinine grows and urea falls
D.The level of urea grows and kreatinine falls
E.*The level of urea and kreatinine grows
373.
What changes in haemogram is typical for haemorrhagic fever with kidneys
syndrome?
A.Normochromic anaemia, leucocytosis with atypical mononucleosis,
thrombocytopenia enhanceable ESR
B. Erythrocytosis, lymphocytosis,ESR is enhanceable
C. Normochromic anaemia, leucopenia with neutrophylosis, thrombocytopenia
enhanceable ESR
D.*Hypochromic anaemia, leucocytosis with neutrophylosis, thrombocytopenia
enhanceable ESR
E.Hyperchromic anaemia, leucocytosis with neutrophylosis, thrombocytopenia
mionectic ESR
374.
Polyuria in haemorrhagic fever with kidneys syndrome is a sign of:
A.*Recovering
B. . Chronic process
C. Unfavorable flow of illness
D.Development of complications
E.Complete convalescence
375.
In most patients with Congo hemorrhagic what type of fever is:
A.Wunderlich’s type
376.
377.
378.
379.
380.
381.
382.
B. Botkin’s type
C. Undulating
D.Intermittent
E.*Two-humped
What changes in haemogram inherent Congo hemorrhagic fever?
A.Normochomic anaemia, leucocytosis mononuclear
B. Erythrocytosis, lymphocytosis
C. *Hypochromic anemia, erythrofilosis
D.Hypochromic anemia, neutrofilosis
E.Hyperchromic anemia, neutrofilosis
What is typical for the Lassa hemorrhagic fever:
A.Effect of cardiovascular system
B. Development of acute hepatic insufficiency
C. Hundred-per-cent lethality
D.*Defeat of breathing organs
E.Development of paresis and paralysis
Confirm diagnosis of haemorrhagic fever with kidneys syndrome by a way of:
A.Only virological methods
B. Only bacteriological methods
C. Bacteriological and serum methods
D.Proper epidemiological information
E.*Virologic and serum methods
Confirm the diagnosis of Lassa hemorrhagic fever by a way of:
A.Only virological methods
B. Only bacteriological methods
C. Bacteriological and serum methods
D.Proper epidemiological information
E.*Virologic and serum methods
Confirm the diagnosis of Congo hemorrhagic fever by a way of:
A.Only virological methods
B. Only bacteriological methods
C. Bacteriological and serum methods
D.Proper epidemiological information
E.*Virologic and serum methods
Confirm the diagnosis of Ebola fever by a way of:
A.Growth of viruses on chicken embryons
B. Only bacteriological methods
C. Bacteriological and serum methods
D.Proper epidemiological information
E.*Selection of virus on the Vero culture
Confirm the diagnosis of Omsk fever by a way of:
A.Growth of virus on chicken embryons
B. Only bacteriological methods
C. Bacteriological and serum methods
D.Proper epidemiological information
E.*Selection of virus on the Vero culture
383.
Confirm the diagnosis of Marburg fever by a way of:
A.Growth on chicken embryos
B. Only bacteriological methods
C. Bacteriological and serum methods
D.Proper epidemiologys information
E.*Selection of virus on the Vero culture
384.
What etiothropic means use at treatment of haemorrhagic fever with kidneys
syndrome:
A.Benzylpenicillin
B. Dopamine
C. *Virolex
D.Dexamethazone
E.Etamsylatum
385.
What etiothropic means use at treatment of patients with Lassa fever:
A.Benzylpenicillin
B. Dopamine
C. *Ribavirin
D.Dexamethazole
E.Etamsylatum
386.
What etiothropic means use at treatment of patients with Marburg fever:
A.Benzypenicillin
B. Dopamine
C. *Ribavirin
D.Dexamethazone
E.Etamsylatum
387.
What etiothropic means use at treatment of patients with Congo fever:
A.Benzylpenicillin
B. Dopamine
C. *Ribavirin
D.Dexamethazone
E.Etamsylatum
388.
What etiothropic means use at treatment of patients with Ebola fever:
A.Benzylpenicillin
B. Dopamine
C. *Virolex
D.Dexamethazone
E.Etamsylatum
389.
What etiothropic means use at treatment of patients with Crimea fever:
A.Benzylpenicillin
B. Dopamine
C. *Ribavirin
D.Dexamethazone
E.Etamsylatum
390.
Specific prevention of hemorrhagic fevers may perform by:
A.Live vaccine
B. Killed vaccine
C. Specific immunoglobulin
D.*Do not develop
E.Polivalent vaccine
391.
Who is the source of the causal agent in the Crimean-Congo haemorrhagic fever?
A.Rodents, cattle, birds
B. Tikes
C. *Rodents, cattle, birds, sick people
D.Sick man, reconvalescent, bacteriocarrier
E.Rodents, cattle, birds, sick people, bacteriocarries
392.
The source of infection of Omsk‘s hemorrhagic fever are muskrat, water rats and
other rodents. Who are the carriers?
A.Bee and flea
B. *Pliers and flea
C. Mosquitoes
D.Fly
E.Pliers and mosquitoes
393.
Specific prevention of Crimean-Congo haemorrhagic fever are:
A.*Vaccine and human immunoglobulin
B. Serum
C. Serum and human immunoglobulin
D.Do not developed
E.Antibacterial drugs
394.
How to prevent malaria?
A.Follow the rules of personal hygiene
B. Do not drink raw water
C. Vaccination
D.*Chemoprophylaxis
E.Heating of food
395.
What group of infectious diseases malaria belong to?
A.Intestinal
B. Respiratory
C. External covers
D.Transmissive
E.*Blood
396.
What is the treatment of malaria attacks.
A.Antibiotics
B. Serum transfer
C. Delagil
D.Primaquine
E.*Delagil + Primaquine
397.
In a survey of donor blood microhametosis are found. Assign treatment.
A.Delagil
B. *Primaquine
C. Antibiotics
D.Fluoroquinolone
E.Sulfanilamide
398.
399.
400.
401.
402.
403.
404.
405.
Malaria must be differentiated primarily with such diseases:
A.Pyelonephritis
B. Sepsis
C. Viral hepatitis
D.Leptospirosis
E.*All of the above
The diagnosis of malaria can be confirmed by:
A.Microscopy of urine
B. Haemoculture
C. Bacteriology of stool
D.Serological reactions
E.*Parazitoscopy of blood
Rules of hospitalization of patients with malaria:
A.*In separate room
B. In a respiratory infections department
C. In the Meltser’s ward
D.Patients are not hospitalized
E.In a intestinal infections department
Why early relapse in malaria develops?
A.The immune deficiency
B. *Due to erythrocytic forms of shizonts
C. Releasing of tissue shizonts to the blood stream
D.Fresh contamination
E.Availability of a blood gamonts
Why late relapses in malaria develops?
A.Due to erythrocytic forms of shizonts
B. *Releasing of tissue shizonts to the blood stream
C. Fresh contamination
D.Availability of a blood gamonts
E.The immune deficiency
By what drug is possible to prevent early recurrence of malaria:
A.*Delagil
B. Immunoglobulin
C. Antibiotics
D.Primahin
E.Glucocorticoids
By what drug is possible to prevent relapse of malaria:
A.Delagil
B. Immunoglobulin
C. Antibiotics
D.*Primahin
E.Glucocorticoids
Radical treatment of malaria include:
A.Primaquine within 2 weeks
B. Glucocorticoids
C. *Delagil + primahin
D.Serum transfer
406.
Indications for the appointment of hematoshizotrop antimalarial drugs:
A.*Attack of malaria
B. Preventing late relapse
C. Prophylactic course after returning from areas difficult to malaria
D.Antyretsidiv course for the rekonvalescents
E.During a check-up
407.
Indications for the appointment of histoshizotrop antimalarial drugs:
A.Attack of malaria
B. *Prevention of late relapse
C. Complications of malaria
D.Malaria chemoprophylaxis for a period of stay in endemic areas
E.During a check-up
408.
What do you need to give to the patients with malaria‘s coma?
A.Glucocorticoids
B. Antishok treatment
C. *Intravenous delagil
D.Blood transfusion
E.Oxygen
409.
Specific complications of malaria, except:
A.Hemoglobinuria fever
B. Spleen destruction
C. Malaria‘s coma
D.*Intestinal perforation
E.Hemolytic anaemia
410.
Reconvalences after malaria may descharged from a clinic no earlier than:
A.After 2 weeks till complete clinical recovery
B. After 2 weeks of a radical course of therapy
C. *After a radical course of therapy with a negative results of parazitoscopy
D.After 3 weeks with a negative blood culture results
E.After 3 weeks, if the negative results of planting feces
411.
Activities on contact with import cases of malaria:
A.Parasitoscopy of blood
B. The direction of the contact in the detention facility for 5 days
C. Chemoprophylaxis
D.Vaccination
E.*Do not hold
412.
What is malaria prevention for those who have returned from disadvantaged
areas:
A.Introduction of human immunoglobulin
B. Interferon
C. 6-days-prevention by streptomycin or tetracycline
D.*Primaquine
E.All the above
413.
What measures must be taken in relation to people who had a contact with
malaria case:
414.
415.
416.
417.
418.
419.
420.
A.Microscopy of periferal blood
B. Isolation during 5 days
C. Chimioprophylaxis
D.Houses rounds
E.*Does not conduct
Who would be a subject for inspection on malaria?
A.Recovering of malaria persons
B. Persons, returning from endemic regions of malaria
C. Patients with fever more than 5 days
D.Patients with spleenomegaly
E.*All above enumerated
Methods for identification of sources of malaria:
A.Stool culture test
B. Haemoculture
C. *Microscopic assessment of blood
D.Byurne test
E.All above enumerated
When should begin treatment of malaria patients?
A.*Immediately after hospitalization
B. After taking of material for research
C. After of final diagnosis
D.After laboratory and instrumental research
E.All answers are correct
Delagil appoint in case of malaria in such doses:
A.0,5 g 3 times a day 3 days
B. 0,5 g during a week
C. *In the first day 1 g, then 0,5 g every 6 h
D.0,5 g a day during a month
E.0,5 g 2 times a day 3 days
There are etiotropic drugs of malaria, except:
A.Delagilum
B. Prymahin
C. *Ceresyn
D.Quinine
E.Fansydar
The radical course of treatment of malaria includes:
A.Five-day therapy of delagilum
B. Prymahin during 2 weeks
C. C. Delagilum + prymahin + fansydar
D.*Delagilum + prymahin
E.Delagilum + fansydar
What do you need for treatment of chlorochyn resistent forms of malaria?
A.Bactrimum
B. Dapson
C. Fansydar
D.Meflohin
421.
422.
423.
424.
425.
426.
427.
428.
E.*All enumerated
The most effective antibiotics at the treatment of patients with leptospirosis are:
A.*Penicillin
B. Macrolids
C. Cefalosporins
D.Ftorhinolons
E.Sulfanilamids
What daily doses of penicillin for leptospirosis treatment:
A.2-3 million units
B. *3-12 million units
C. 10-20 million units
D.20-40 million units
E.Over 40 million units
All of these is an epidemic dangerous to leptospirosis except:
A.Farm animals
B. Wide rodents
C. Domestic animals
D.Foxes
E.*Humans
How long lasts the incubation period of leptospirosis:
A.2 month
B. 1-7 days
C. *7-14 days
D.14-21 days
E.2-3 days
Which serotypes of leptospirosis caused the disease more frequent:
A.L. interogans
B. L. grippotyphosa
C. L. canicola
D.*L. icterohaemorrhagia
E.L. Pomona
Agglutinines in leptospirosis arrive at a maximal titre:
A.On the third day of illness
B. *On the third week of illness and later
C. On the fourth week of illness
D.On the second month of illness
E.On the second week of illness
Rules of hospitalization of patients with infectious mononucleosis:
A.Patients are not hospitalized
B. In a ward for the infections of respiratory tracts
C. *In a separate ward
D.In a ward for the infections of external covers
E.In a ward for intestinal infections
What symptoms are not characteristic of infectious mononucleosis?
A.Hepatomegaly
B. *Oliguria
C. Limfocytosis
D.Tonsillitis
E.Splenomegaly
429.
How much atypical mononucleares present in infection mononucleosis?
A.Less than 5 cells
B. *10 and more cells
C. 5 and more cells
D.Less than10 cells
E.50 % cells
430.
What is most possible complication occur in infectious mononucleosis?
A.Oliguria
B. Autoimmune process
C. Meningitis
D.*Spleen rupture
E.Pneumonia
431.
What additional inspections must be conducted to the patient with infectious
mononucleosis?
A.*IFA on HIV-infection, bacteriology inspection on diphtheria
B. IFA on HIV-infection, bacteriology inspection on a scarlet fever
C. Bacteriology inspection on diphtheria and typhoid fever
D.Burne and Rihth-Heddlson reactions
E.Paul-Bunnel reaction and punction of lymphatic node
432.
What from the following symptoms are not characteristic of infectious
mononucleosis?
A.Fever
B. *Defeat of kidneys
C. Lymphadenopathy
D.Tonsillitis
E.Increasing of liver and spleen
433.
For what disease characterize changes in a blood (presence of lymphomonocytes
and atypical mononuclears?
A.Flu
B. *Infectious mononucleosis
C. Measels
D.AIDS
E.Diphtheria
434.
In a survey of donor blood found microhametosis. Assign treatment.
A.Delagil
B. *Primaquine
C. Antibiotics
D.Fluoroquinolone
E.Sulfanilamide
435.
Diagnosed a patient: chronic hepatitis in the stage of integration. What markers
will be in patient in this stage disease?
A.HBeAg
B. Antibodies to HBeAg
C. DNA OF HBV
D.Viral DNA-polimerase
E.*HBsAg, anti-НBе
436.
As etiotropic therapy of sharp and chronic viral hepatitis B utillize:
A.Corticosteroid
B. Immunomodulate preparations
C. Cytostatics
D.Antibiotics
E.*Antiviral preparations
437.
Give recommendation for a patient in reconvalensent period of viral hepatitis
during a clinical supervision after isolation.
A.*Medical supervision during 6 months, periodic biochemical inspections.
B. Control bacteriological examinations
C. Full labor investigation
D.To continue prophylactic inoculations
E.Supervision is not needed
438.
Etiotropic therapy of viral hepatitis is.
A.Ribavirin
B. Interferon
C. Inductors of interferon
D.Zefix
E.*All the above
439.
Choose the remedies for etiotropic therapy of viral hepatitis.
A.*Ribavirin
B. Vaccine
C. Normal human immunoprotein
D.Hepatoprotector
E.Glucocorticoid
440.
Choose the remedies for etiotropic therapy for viral hepatitis.
A.Antibiotics
B. *Interferon
C. Probiotics
D.Vaccine
E.Normal human immunoprotein
441.
The criteria for application of etiotropic therapy in viral hepatitis is.
A.Protracted motion of HBV, HVD
B. Any form of HV
C. Biochemical activity
D.Presence of virus replication
E.*All the above
442.
The criteria for application of etiotropic therapy for the patient with HCV.
A.Clinical displays are insignificant
B. Icterus is absent
C. Moderate biochemical activity
D.There is anti-HCV in blood
E.*RNA of HCV +
443.
Factors which are indications of successful interferon therapy in HV infections
are all, except.
A.Level of ALaT not more than 2-3 norm
B. Low titre of HCV after the treatment
C. Absence of cholestasis
D.2th and 4th genotypes of HCV
E.*Expressed fibrosis
444.
Indirect action of interferon therapy.
A.Influenza-like syndrome
B. Nausea
C. Itching
D.Para-hypnosis
E.*All the above||
445.
Indirect action of interferons.
A.Flatulence
B. Diarrhea
C. Nausea
D.Depression
E.*All the above
446.
Indirect action of interferon therapy are all except.
A.Influenza-like syndrome
B. Nausea
C. Depression
D.Intensification of autoimmune diseases
E.*Progress of fibrosis
447.
Basic principles of antiviral therapy for viral hepatitis.
A.Individual selection of dose and rhythm of application of preparations
B. Duration of introduction of preparations
C. Control of amount of erytrocytes, leucocytes and thrombocytes, in blood
D.Control of iron level in blood
E.*All the above
448.
Contra-indications for antiviral therapy of viral hepatitis.
A.Decompensatory cirrhosis of liver
B. Thrombocytopenia <50000 in 1 мм3
C. Psychic disorders
D.Leucocytopenia <1500 in 1 мм3
E.*All the above
449.
Contra-indications for antiviral therapy of viral hepatitis.
A.Decompensatory cirrhosis of liver
B. Autoimmune disease
C. Alcoholism and other drug addictions
D.Coinfection by HIV
E.*All the above
450.
Choose the indexes of efficiency of interferon therapy.
A.*Disappearance| of markers of viral replication
B. Improvement of the general state
C. Normalization of the liver size
D.Disappearance of icterus
E.All the above
451.
Choose the indexes of efficiency of interferon therapy.
A.Improvement of the general state
B. *Normalization of activity of ALaT
C. Normalization of the liver size
D.Disappearance of icterus
E.All the above
452.
Types of answer for interferon therapy are.
A.Stable remission
B. Unsteady
C. Partial answer
D.Absence of answer
E.*All the above
453.
The characteristic of an unsteady answer of interferon therapy are.
A.Disappearance of markers of viral replication upon completion of course of therapy
B. Normalization of activity of ALaT during the course of therapy
C. An origin of relapse in next 6 months
D.Disappearance of icterus
E.*All the above
454.
That characteristic of a partial answer of interferon therapy are all, except.
A.*Disappearance of markers of viral replication
B. Normalization of activity of ALaT is upon completion of course of therapy
C. Disappearance of icterus
D.Normalization the state of patient
E.Normalization of the size of liver
455.
When is interferon therapy effective in the the patient.
A.Normalization of the state of patient
B. Normalization of activity of ALaT upon completion of course of therapy
C. Disappearance of icterus
D.Normalization of the size of liver
E.*The markers of viral replication, are determined upon completion of course of
therapy
456.
What laboratory work-up is needed for confirming the diagnosis of viral
hepatitis.
A.Total analysis of blood
B. Determination of level of bilirubin
C. Determination of activity of aminotransferase
D.*Determination of markers of HV in IFA
E.All the above
457.
What laboratory and instrumental examinations are needed for confirming the
diagnosis of viral hepatitis.
A.Complete analysis of blood
B. Ultrasound of abdominal region
C. Determination of activity of aminotransferase
458.
459.
460.
461.
462.
463.
464.
465.
D.*Determination of antigen of viruses
E.Duodenal probing
What is incubation period for hepatitis B:
A.45 days
B. *180 days
C. 360 days
D.90 days
E.25 days
All the hepatitis have parenteral route of transmission except:
A.*A
B. B
C. C
D.D
E.TTV
Chronic course is common for viral hepatitis except:
A.*A
B. B
C. C
D.D
E.B+C
All the following medicines are interferons except:
A.Intron
B. Roferon
C. Reaferon
D.Leukinferon
E.*Cycloferon
All the following medicines are hepatoprotective agents except:
A.Carsil
B. Silibor
C. Legalon
D.*Lomusol
E.Arginine
Phage symptom in case of yellow fever is:
A.Pain in right iliac area
B. Enanthema on a soft palate
C. *Replacement of tachicardia on expressed bradicardia
D.Hemorrhages in a conjunctiva
E.Yellow hands
Hemograme in the second period of yellow fever:
A.Leukocytosis
B. Normal global analysis of blood
C. *Leukopenia, neutropenia
D.Leukopenia, neutrophilosis
E.Leukocytosis, lymphomonocytosis
Whatever complication meets at the yellow fever:
A.*Liver insufficiency
B. Kidney insufficiency
C. Infectious-toxic shock
D.Myocarditis
E.Edema of lungs
466.
Unlike leptospirosis in case of yellow fever is absent:
A.Hemorrhagic syndrome
B. Kidney insufficiency
C. Іntoxication syndrome
D.Міalglic syndrome
E.*Hepatic insufficiency
467.
For confirmation of yellow fever diagnosis use:
A.Bacteriological analysis of blood
B. Bacteriological examination of urine
C. *Virological hemanalysis
D.Biochemical blood test
E.Global analysis of blood
468.
In the initial period of hemorrhagic fever with a kidney syndrome a characteristic
sign is:
A.High temperatures
B. Pains in gastrocnemius muscles and positive Pasternatsky symptom
C. *Pains in joints and positive Pasternatsky symptom
D.Hemorragic syndrome
E.Dyspepsia phenomena
469.
An initial period at the hemorrhagic fever with a kidneys syndrome lasts:
A.Few hours
B. One day
C. *Three days
D.one week
E.Two weeks
470.
When a violations of diuresis at patients with hemorrhagic fever with a kidneys
syndrome appear:
A.In initial period
B. Don’t appear
C. During all periods of disease
D.*In climax period
E.In recovering period
471.
General view of patient with the hemorrhagic fever with a kidneys syndrome:
A.Skinning covers
B. *Pallor of nasolabial triangle, hyperemia of neck and overhead half of trunk
C. Hyperemia of person, scleritis, conjunctivitis
D.Grayish color of person
E.Icteric color of skin
472.
In a biochemical blood test of patients with the hemorrhagic fever with a kidneys
syndrome not typically:
A.High level of urea
B. Decline of potassium level
C. *Bilirubinemia
D.Increasing of kreatinine
E.Increasing of nitrogen
473.
For confirmation of diagnosis of hemorragic fever with a kidney syndrome use:
A.Bacteriological method
B. Virological method
C. *Reaction of immunofluorescence
D.Reaction of braking of hemagglutination
E.Research of blood drop under a microscope
474.
For treatment of patients with the hemorrhagic fever with a kidney syndrome
does not use:
A.Glucocorticoids
B. Anabolic steroid
C. Disintoxication facilities
D.*Dihydration facilities
E.Antihistaminics
475.
For the initial period of the Congo hemorrhagic fever not characteristically:
A.Fever
B. Pains in joints and muscles
C. Severe pain of head
D.*Oliguria
E.Dizziness
476.
At an objective review for the Congo hemorrhagic fever characteristically:
A.*Mucosal hyperemia of person
B. Pallor of person
C. Puffiness of person
D.Ochrodermia of person
E.Exanthema on face
477.
The most characteristic symptom in the climax period of the Congo hemorrhagic
fever is:
A.*Hemorrhagic syndrome
B. Hepatic insufficiency
C. Dyspepsia phenomena
D.Sharp kidney insufficiency
E.Мeningeal syndrome
478.
In a case of Congo fever in a general blood analysis is not typical:
A.. Leukocytosis
B. *Leukopenia
C. Neutropenia
D.Thrombocytopenia
E.Increasing of ESR
479.
What laboratory and instrumental examinations should used for flu diagnosis?
A.Complete analysis of blood
B. X-ray of organs of thoraxic cavity
C. Analysis sputum
D.*Determination of viruses by the method of immunofluorescence
E.Biochemical blood test
480.
Virus causing hemorrhagic cystitis, diarrhea and conjunctivitis:
A.RSV
B. Rhinovirus
C. *Adenovirus
D.Rotavirus
E.Flu
481.
What is conduct specific passive immunnoprophylaxis of flu?
A.Live antenuated vaccine
B. Inactive parenteral vaccine
C. *Immune protein
D.Remantadin
E.Antibiotics of wide spectrum of action|
482.
Duration of isolation of patient with influenza complications?
A.4 days
B. 7 days
C. *10 days
D.17 days
E.20 days
483.
How is the urgent prophylaxis of scarlet fever conducted?
A.Vaccination
B. *Isolation of children, who had contact with a patient
C. Chemioprophylaxis
D.Disinfection
E.Non-admission of contact with carrier of B-streptococcus
484.
What level is necessary to reduce the temperature of patient’s body with
hyperthermia?
A.39 °C
B. *38 °C
C. 37,5 °C
D.37 °C
E.38,5 °C
485.
What made specific passive flu immunization?
A.Living intranasal vaccine
B. Parenteral inactivated vaccine
C. * Immune globulin
D.Remantadin
E.Antibiotic
486.
What pathogen causes severe acute respiratory syndrome?
A.Bocavirus
B. Rheovirus
C. Metapneumovirus
D.Adenovirus
E.*Coronavirus
487.
Which family owned influenza?
A.Pallidum
488.
489.
490.
491.
492.
493.
494.
B. Legionella
C. Tohovirus
D.* Ortomixovirus
E.Rickettsiae
The intensity of intoxication caused by flu depends on?
A.Hemagglutinin
B. Neuraminidase
C. * Rybonucleoproteid
D.Membrane proteins
E.S-protein
What determs the immunosuppressive action of influenza virus?
A.*Hemagglutinin
B. S-protein
C. Neuraminidase
D.RNA polymerase
E.The membrane protein
What is antigenic shift of influenza virus?
A.*Antigenic changes of rybonucleoproteid
B. Recombination hemagglutinin and neuraminidase
C. Antigenic changes of the virus within serovars
D.Genetic recombination between different strains of influenza virus
E.Variability neuraminidase
What medium used for culturing influenza virus?
A.*Chicken embryos and cell culture
B. Gall broth
C. Meat pepton agar
D.Medium containing blood
E.Water-serum culture medium
Which of these protein of influenza virus is capable to hemolise red blood cells?
A.RNA polymerase
B. S-protein
C. Neuraminidase
D.Membrane protein
E.* Hemagglutinin
Specify contagious period in flu?
A.The end of the incubation period + the entire period of the disease
B. End of the incubation period
C. Only during the height
D.* The period of convalescence
E.The crisis period + 7 days normal body temperature
What is a gateway for the flu virus?
A.*Cylindrical mucosal epithelium of respiratory tract
B. Solitary follicles
C. Mucous of tonsils
D.Epithelial cells of the skin
E.Mucous membranes of the digestive tract
495.
What is antigenic drift of influenza virus?
A.*Partial change in antigenic specificity of hemagglutinin
B. Recombination hemagglutinin and neuraminidase
C. Antigenic changes of the virus within serovars
D.Genetic recombination between different strains of influenza virus
E.Variability neuraminidase
496.
What is the main preventive measure in case of contact with sick with the flu?
A.Vaccination
B. * Chemoprophylaxis
C. Vitaminoprophylaxis
D.Admission antipyretics
E.Admission immunostimulators
497.
Which of these vaccines is the least reactivity?
A.Fullvirion
B. Split vaccines
C. * Subunit
D.Live
E.Inactivated
498.
What is the most common form of adenovirus infection?
A.Tracheobronchitis
B. Laryngitis
C. Pneumonia
D.* Pharyngoconjunctive fever
E.Rhinitis
499.
What is most often clinically manifested flu?
A.Nasopharyngitis
B. Bronchospasm
C. Rhinitis
D.Acute respiratory failure
E.* Laryngotracheitis
500.
What is most evident parainfluenza infection?
A.Rhinitis
B. * Laryngitis
C. Tracheobronchitis
D.Lymphadenopathy
E.Bronchospasm
501.
Which agents of acute respiratory infections contain DNA?
A.Respiratory virus syntytsialnyy
B. Influenza virus
C. Parainfluenza virus
D.* Adenovirus
E.Rhinovirus
502.
Which of acute respiratory diseases transmitted not only through airborne, but
the fecal-oral transmission mechanism?
A.Rhinoviral disease
B. Parainfluenza
503.
504.
505.
506.
507.
508.
509.
510.
C. * Adenoviral disease
D.Respiratory syntytsial disease
E.Flu
What is the duration of incubation period of adenoviral infection?
A.Several hours - 1 day
B. 1-2 days
C. * 1-14 days
D.2-3 days
E.14-21 day
What is characteristic of adenoviral conjunctivitis unlike diphtheria eyes?
A.Conjunctiva of eyelids bright red tape snug, hard shot
B. Complete loss of vision due panoftalmit
C. * Conjunctiva of eyelids bright red, covered with films that are easily removed
D.Photophobia, pain during palpation
E.Conjunctiva with haemorrhages
Which of the following is characteristic of adenovirus infection?
A.Bronchiolitis
B. Bronchiolitis and pneumonia
C. Real croup, pneumonia
D.* Membranous conjunctivitis and pharyngitis
E.Bronchiolitiasis and false croup
Which family pathogen of parainfluenza belong to?
A.Pneumoviridae
B. * Paramyxoviridae
C. Ortomyxoviridae
D.Adenoviridae
E.Reoviridae
What is theduration of incubation period in parainfluenza?
A.Several hours - 2 days
B. 7-9 days
C. 9-14 days
D.* 7-2 days
E.14-21 day
What is most often damaged in parainfluenza?
A.*The mucous membrane of the larynx and trachea
B. Lower respiratory tract
C. Maxillary sinus
D.Conjunctiva
E.Nasal mucosa
What complication is typical for parainfluenza?
A.Real croup
B. * False croup
C. Sinusitis
D.Pneumonia
E.Pharyngitis
Which family pathogen of respiratory syncytial infection belong to?
A.Picornaviridae
B. Orthomyxoviridae
C. Adenoviridae
D.Reoviridae
E.* Paramyxoviridae
511.
Which of acute respiratory disease is characterized by the development of
pulmonary edema?
A.Adenovirus infection
B. * Flu
C. Respiratory syncytial infection
D.Parainfluenza
E.Psittacosis
512.
What changes in general blood analisis is characteristic in rhinovirus infection?
A.*The number of leukocytes and ESR are not changed
B. Significant leukocytosis and increased ESR
C. Leukocytosis with neutrocytosis
D.Leukopenia with lymphocytosis
E.Leukocytosis, anemia, thrombocytopenia
513.
What is the possible complication of respiratory syncytial infection?
A.Etmoiditis
B. Otitis
C. Pneumonia
D.Sinusitis
E.* All of the above
514.
Which family agent of rhinovirus infection belong to?
A.Retroviridae
B. Paramyxoviridae
C. *Picornaviridae
D.Adenoviridae
E.Reoviridae
515.
What is the duration of incubation period of rhinovirus infection?
A.Several hours-1 day
B. 1-6 days
C. *7-10 days
D.10-14 days
E.14-20 days
516.
Which family pathogen of metapnevmovirus infection owned to?
A.Retroviridae
B. * Paramyxoviridae
C. Picornaviridae
D.Adenoviridae
E.Reoviridae
517.
What is the most effective drug in the treatment of metapneumovirus infection?
A.Ganacyclovir
B. Acyclovir
C. Tamiflu
D.Remantadin
E.* Ribavirin
518.
What transfer factor is not typical for coronavirus infection?
A.Air
B. Water
C. * Semen
D.Blood
E.Urine
519.
People of what ages is most sensitive for bocavirus infection?
A.*Children from neonatal period up to 1 year
B. Children from 1 to 5 years
C. Children from 6 to 10 years
D.Teens
E.Adults
520.
For what viral infection convulsions may occur?
A.Adenovirus infection
B. Respiratory syntytsialna
C. Rynovirusna infection
D.Parainfluenza
E.* Flu
521.
What are the symptoms of croup syndrome?
A.*Inspiratory dyspnea
B. Expiratory wheeze
C. Gruff "barking" cough
D.Spastic cough
E.Availability of films in the oropharynx
522.
The usage of what drug is not appropriate in mild and moderate forms of flu?
A.Antiviral
B. Expectorant
C. * Antibiotics
D.Immunostimulators
E.Desensitizing
523.
What is the most effective drug in the treatment of influenza?
A.Ganacyclovir
B. Acyclovir
C. * Ingavirin
D.Remantadin
E.Amizon
524.
For what infection meningeal syndrome is most typical?
A.Adenovirus infection
B. Parainfluenza
C. Rhinovirus infection
D.* Flu
E.Respiratory syncytial disease
525.
What infectious disease is characterized by the local reaction of the nasal mucosa
with hyperemia, edema and significant secretion?
526.
527.
528.
529.
530.
531.
532.
A.*Rhinovirus disease
B. Typhoid fever
C. Malaria
D.Varicella
E.Flu
What viruse more often cause nosocomial infection?
A.Adenoviruses
B. Parainfluenza virus
C. Rhinovirus
D.Influenza viruses
E.* Coronavirus
Methods of laboratory diagnosis of acute respiratory viral infections except:
A.Virological
B. Serum
C. * Blood culture
D.PCR
E.Flu
Specific methods of laboratory diagnosis of influenza:
A.Cultivation of the virus in chicken eggs or tissue cultures
B. Detection of virus antigens by immunofluorescence in nasal washings from
C. Detection of antibodies to the virus in paired sera of blood
D.Detection of virus antigens by fluorescent microscopy in smears from the nasal
mucosa
E.* All of the above
What laboratory tests of influenza?
A.Immunofluorescence method of nasopharyngeal swabs
B. General blood
C. Bacteriological investigation of sputum
D.Biological test on laboratory animals
E.*All of the above
What are the indications for antibiotics therapy in flu?
A.Very severe course
B. Presence of complications
C. Some age groups (children, elderly people age)
D.The presence of foci of chronic bacterial infections
E.* All of the above
What drug is used as etiotropic therapy in adenoviral infection?
A.Paracetamol
B. Aspirin
C. Ceftriaxone
D.* Dezoxirybonukleaz
E.All of the above
What indicates possible complications in flu?
A.Duration of hyperthermia over 5 days
B. Leukocytosis
C. Netrophilosis
D.Accelerated ESR
E.* All of the above
533.
What is color in fluorescent microscope preparation in case of influenza?
A.Red
B. * Green
C. Blue
D.Yellow
E.Violet
534.
What is the entrance gate at infectious mononucleosis?
A.Mucosa of colon
B. Mucosa of digestive tract
C. Epithelial cells of skin
D.Peyer‘s plate and follicles
E.*Mucosa of nazo-pharig
535.
What is preparation of specific therapy for viral neuro infection?
A.* Acyclovir
B. Cefataxime
C. Ceftriaxone
D.Gentamycin
E.Furazolidon
536.
What is preparation for specific therapy of viral neiroinfection?
A.Lazix
B. Cefotaksim
C. Ceftriakson
D.* Acyclovir
E.Prednisolon
537.
What is drug for specific therapy of widespread form of diphtheria of
nasopharynx.
A.Macrolids per os
B. Penicillin i/m
C. Cortycosteroid
D.* Antidiphterial serum i/v
E.Antitoxic therapy
538.
What is the first dose of antidiphterial serum for a 6 years child with a
diphtherial widespread croup:
A.* 40 AО
B. 15 AО
C. 20 AО
D.80 AО
E.60 AО
539.
What is the first dose of antitoxic antidiphtherial serum for a patient with
diphtheria of pharynx?
A.120 thousand of AО
B. 80 thousand of AО
C. * 30 thousand of AО
D.50 thousand of AО
540.
541.
542.
543.
544.
545.
E.150 thousand of AО
At the end of treatment of patients with tonsillitis it is recommended to enter:
A.500 000 of Bicyllin-3 intramuscular
B. * 1 500 000 of Bicyllin-5 intramuscular
C. 1 000 000 of Bicyllin-5 intramuscular
D.1 500 000 of Bicyllin-3 intramuscular
E.500 000 of Bicyllin-3 intramuscular
Before revaccination from diphtheria of adult persons, are recommended:
A.* To explore an immune type
B. To use antibiotics
C. To use antihistamines
D.5 years after last revaccination
E.10 years after last revaccination
Before revaccination from diphtheria of adult persons, are recommended:
A.* To explore an immune type
B. To use antibiotics
C. To use antihistamines
D.5 years after last revaccination
E.10 years after last revaccination
What is the exciter of tonsillitis (angina).
A.* Streptococcus of group A
B. Streptococcus of group B
C. Streptococcus of group C
D.Streptococcus of group D
E.Streptococcus of group E
Choose, what changes are characteristic for diphtheria of tonsils.
A.Tonsils enlarged, edematous, on-the-spot of tonsills are some heaved up
subephithelial abscesses yellow-white color
B. In lacunes of tonsils are a pus as yellow-white coat
C. Tonsils are hyperemic, hypertrophied, on both are necrotizing areas dark grey
color, after removing layer by layer of which the deep defect of mucus shell
appeared with an uneven bottom
D.One tonsils hyperemic and filling out, on its surface there is a grey-white coat,
under it coat – bleeding ulcer with a smooth bottom
E.* One tonsil is enlarged, on him dense grey-white color coat, which becomes
separated from hardness, mucus bleeds under him
Choose, what changes are characteristic for a follicle tonsillitis (angina).
A.* Tonsils enlarged, edematous, on-the-spot of tonsils are some heaved up
subephithelial abscesses yellow-white color
B. In lacunes of tonsils are a pus as yellow-white coat
C. Tonsils are hyperemic, hypertrophied, on both are necrotizing areas dark grey
color, after removing layer by layer of which the deep defect of mucus shell
appeared with an uneven bottom
D.One tonsils hyperemic and filling out, on its surface there is a grey-white coat,
under it coat – bleeding ulcer with a smooth bottom
546.
547.
548.
549.
550.
E.One tonsil is enlarged, on him dense grey-white color coat, which becomes
separated from hardness, mucus bleeds under him
Choose, what changes are characteristic for a lacunars tonsillitis (angina).
A.Tonsils enlarged, edematous, on-the-spot of tonsils are some heaved up
subephithelial abscesses yellow-white color
B. * In lacunes of tonsils are a pus as yellow-white coat
C. Tonsils are hyperemic, hypertrophied, on both are necrotizing areas dark grey
color, after removing layer by layer of which the deep defect of mucus shell
appeared with an uneven bottom
D.One tonsils hyperemic and filling out, on its surface there is a grey-white coat,
under it coat – bleeding ulcer with a smooth bottom
E.One tonsil is enlarged, on him dense grey-white color coat, which becomes
separated from hardness, mucus bleeds under him
Choose, what changes are characteristic for a ulcers-necrotic tonsillitis (angina).
A.Tonsils enlarged, edematous, on-the-spot of tonsils are some heaved up
subephithelial abscesses yellow-white color
B. In lacunes of tonsils are a pus as yellow-white coat
C. * Tonsils are hyperemic, hypertrophied, on both are necrotizing areas dark grey
color, after removing layer by layer of which the deep defect of mucus shell
appeared with an uneven bottom
D.One tonsils hyperemic and filling out, on its surface there is a grey-white coat,
under it coat – bleeding ulcer with a smooth bottom
E.One tonsil is enlarged, on him dense grey-white color coat, which becomes
separated from hardness, mucus bleeds under him
Choose, what changes are characteristic for a Vensan-Plaut‘s tonsillitis.
A.Tonsils enlarged, edematous, on-the-spot of tonsils are some heaved up
subephithelial abscesses yellow-white color
B. In lacunes of tonsils are a pus as yellow-white coat
C. Tonsils are hyperemic, hypertrophied, on both are necrotizing areas dark grey
color, after removing layer by layer of which the deep defect of mucus shell
appeared with an uneven bottom
D.* One tonsils hyperemic and filling out, on its surface there is a grey-white coat,
under it coat – bleeding ulcer with a smooth bottom
E.One tonsil is enlarged, on him dense grey-white color coat, which becomes
separated from hardness, mucus bleeds under him
Complication of diphtheria of larynx is:
A.Myocarditis
B. Paresis of auditory nerve
C. Nephrosonephritis
D.* Cereals
E.Poliomyelitis
Complications of 4-5th week of diphtheria are:
A.Encephalitis
B. Bulbar disorders, pancreatitis, hepatitis
C. * Poliomyelitis, myocarditis
D.Nephrosonephritis
551.
are:
552.
553.
554.
555.
556.
557.
558.
E.Stenotic laryngotracheitis
Complications which often develop on the first week of diphtheria of otopharynx
A.Poliomyelitis
B. Asphyxia
C. Insufficiency of glandulars
D.hepatospleenomegaly
E.* Paresis of soft palate
Diphtheria planned vaccination begin in:
A.In first days after birth of child
B. * In 3 month age
C. In 6-month age
D.In 1 year
E.In 6 years
Early complications of diphtheria of otopharynx is:
A.* Paresis of soft palate
B. Pneumonia
C. Asphyxia
D.Croup
E.Poliomyelitis
Especially high titre of ant diptheria antitoxic antibodies testifies in:
A.Recovering
B. Acute period of diphtheria
C. * Bacteriocarriering
D.Forming of immunity to diphtheria
E.About nothing does not testify
Etiology agent of meningitis is:
A.* Neisseria meningitides
B. Entamoeba histolytica
C. Vibro cholerae
D.Clostridium botulinum
E.Campylobacter pylori
For corynebacterium diphtheria is typical:
A.Contain endotoxin only
B. * Exotoxin production
C. Exotoxin does not product
D.Enterotoxin production
E.Myelotoxin production
For the treatment of acidosis in meningococcal meningitis is better to use.
A.10-20 % glucose solution
B. 10 % chloride solution
C. * 4 % sodium bicarbonate solution
D.Albumen
E.Concentrated dry plasma
What disease is typical changes in blood (presence of atypical mononucleares)?
A.Flu
559.
560.
561.
562.
563.
564.
565.
B. * Infectious mononucleosis
C. Measles
D.AIDS
E.Diphtheria
What etiotropic (antistreptococcal) facilities are the most effective :
A.Furazolidonum
B. Gentamicin
C. * Benzilpenicilin and Oxacillinum
D.Benzilpenicilin and Furazolidonum
E.Doksiciklin and Gentamicin
How is it possible to specify the diagnosis of meningococcal meningitis.
A.Meningitis is primary
B. Presence of a lot of cells in the CSF
C. Presence of gram-negative diplococcus in CSF
D.Meningococes from the throat
E.* All the above
How long is the incubation period of a tonsillitis (angina)?
A.From a few hours to 5 days
B. From a few hours to 4 days
C. From a few hours to 3 days
D.* From a few hours to 2 days
E.From a few hours to 1 days
How long is the period of fever in patients with a tonsillitis (angina)?
A.1-2 days
B. 2-3 days
C. * 3-5 days
D.5-7 days
E...More than week
Name the exciter of acute tonsillitis:
A.α -hemolytic streptococcus of group A
B. γ -hemolytic streptococcus of group A
C. α -hemolytic streptococcus of group C
D.*β -hemolytic streptococcus of group C
E.β -hemolytic streptococcus of group A
The source of exciter of tonsillitis is:
A.patient with angina
B. patient with erysipelas
C. patient with the scarlet fever
D.healthy transmitter of hemolytic streptococcus
E.*all listed above
What is the basic mechanism of transmission of tonsillitis?
A.*air-droplet
B. alimentary
C. contact
D.transmisive
E.vertical
566.
How long last the incubation period in tonsillitis?
A.from a few hours to 5 days
B. from a few hours to 4 days
C. from a few hours to 3 days
D.*from a few hours to 2 days
E.from a few hours to 1 days
567.
Angina begins sharply, from headache, increase of temperature of body, dull
ache in joints and chill. What other characteristic syndrome of appears simultaneously
(rarer in the end of 1st days)?
A.nausea
B. vomiting
C. *pain in the throat
D.pain in the stomach
E.tachycardia
568.
A frequent and early symptom of tonsillitis is an enlargement of lymph nodes,
their pain. Which group of lymph nodes enlarge first of all?
A.posterior-neck
B. occypital
C. supraclavicular
D.*submandibular
E.anterior-neck
569.
What is the average duration of fever period in tonsillitis?
A.days
B. 2-3 days
C. *3-5 days
D.5-7 days
E.More one week
570.
What kinds of angina can be distinguish according to the changes in a tonsils?
A.Catarrhal, follicle and lacunar
B. *Catarrhal, follicle, lacunar and necrotic-ulcerous
C. Catarrhal, follicle, lacunar, pellicle and necrotic-ulcerous
D.Follicle, lacunar and necrotic-ulcerous
E.Follicle, lacunar, pellicle and necrotic-ulcerous
571.
What is the medicine for specific therapy of widespread form of nasopharynx
diphtheria:
A.Macrolids per os
B. Penicillin i/m
C. Cortycosteroid
D.*Antidiphterial serum i/v
E.Antitoxic therapy
572.
Specify the correct method of serum introduction after the Bezredko method:
A.1,0 ml of divorced 1:100 hypodermic – through 30 min. 0,1 ml of undivorced
hypodermic – through 30 min. all dose of intramuscle
B. 0,1 ml of divorced 1:1 000 endermic – through 30 min. 0,1 ml of divorced 1:10
hypodermic – through 30 min. all dose of intramuscle
C. 0,1 ml of undivorced endermic – through 30 min. 0,1 ml hypodermic – through 30
min. all dose of intramuscle
D.*0,1 ml of divorced 1:100 endermic – through 30 min. 0,1 ml of undivorced
hypodermic – through 30 min. all dose of intramuscle
E.1,0 ml of divorced 1:10 hypodermic – through 30 min. 0,1 ml of undivorced
hypodermic – through 30 min. all dose of intramuscle
573.
What is characteristic signs of raid at diphtheria?
A.One-sided, grey-white, on-the-spot crateriform ulcers
B. *Grey-white, dense with clear edges and brilliant surface
C. Yellow-white, fragile, perilacunar is located
D.One-sided, yellow-white, in lacunas
E.White, fragile, is easily taken off by a spatula
574.
What is the exciter of diphtheria:
A.Virus of Epshtein-Barr
B. *Leffler Bacillus
C. Corynebacteria ulcerans
D.Fusiform stick
E.Corynebacteria xerosis
575.
Etiology agent of meningitis are accept:
A.Staphylococci
B. Neisseria meningitides
C. Mycobacterium tuberculosis
D.Viruses
E.*Entamoeba histolytica
576.
Wich of these symptoms are often present in patients with meningitis?
A.Profuse watery diarrhea, vomiting, dehydratation, muscular cramps
B. *Fever, headache, stiff neck, vomiting, confusion, irritability
C. Headache, dry cough, algor
D.Abdominal pain, diarrhea, constipation, flatulence
E.Algor, high temperature, headache
577.
What measures it’s necessary to perform in the focus of viral
meningoencephalitis?
A.Bacteriological inspection of contact
B. Phagoprophylaxis
C. Supervision during 2 weeks
D.Chemoprophylaxis
E.*Immunization
578.
What is used as specific prophylaxis of viral meningoencephalitis.
A.Immune globulin
B. *. Vaccine
C. Antibioticsnatoxin
D.Serum
E.Nothing
579.
What laboratory methods should be taken for diagnosis of meningoencephalitis?
A.Lumbar puncture
B. *Biopsy of tissues
C. Urine examination
D.Coprogram
E.Serologic detection
580.
What clinical forms of tick encephalitis present accept:
A.Meningeal
B. Meningoencephalitic
C. *General
D.Feverish
E.Meningoencephalopoliemielitic
581.
What drugs of choice at etiotropic therapy of viral meningoencephalitis.
A.Penicillins
B. Amynoglicosides
C. *Interferons
D.Phtorhinilons
E.Nitrofuranes
582.
Viral meningoencephalitis and DIC-syndrome require above all things.
A.Administration of diuretics
B. Administration of antihistaminic preparations
C. Administration of vitamins
D.*Administration of analgesic
E.Administration of heparin
583.
Source of meningoencephalitis are accept:
A.People
B. Ticks
C. *Fish
D.Fleas
E.Mosquitoes
584.
How is it possible to specify the diagnosis of viral meningoencephalitis after
cerebrospinal puncture.
A.Some increasing of chlorides in CSF
B. Presence of a lot of lymphocytes in CSF
C. *All the above
D.Some increasing of protein in CSF
E.Neurolymph under high pressure
585.
Etiology agent of meningitis is:
A.*Neisseria meningitides
B. Entamoeba histolytica
C. Vibro cholerae
D.Clostridium botulinum
E.Campylobacter pylori
586.
Witch of these symptoms are often present in patients with meningitis?
A.Algor, high temperature, headache
B. Profuse watery diarrhea, vomiting, dehydratation, muscular cramps
C. Abdominal pain, diarrhea, constipation, flatulence
D.Headache, dry cough, algor
E.*Prodromal respiratory illness or sore throat, fever, headache, stiff neck, vomiting,
confusion, irritability
587.
What laboratory methods should be taken to discharge meningitis?
A.*Lumbar puncture
B. Serologic detection
C. Urine examination
D.Coprograma
E.Biopsy of tissues
588.
Source of meningitis is:
A.Animals
B. Birds
C. Fish
D.Pediculus humanus
E.*People
589.
How is it possible to specify the diagnosis of meningococcal meningitis.
A.Meningitis is primary
B. Presence of a lot of cells in the CSF
C. Presence of gram-negative diplococcus in CSF
D.Meningococes from the throat
E.*All the above
590.
What are the rules at taking of smear material on the discovery of meningococal
infection?
A.The taken away material at drawing out must not touch only mucus shell of cheeks
and tongue
B. The taken away material at drawing out must not touch only teeth and tongue
C. The taken away material at drawing out must not touch only teeth, mucus shell of
cheeks
D.*The taken away material at drawing out must not touch teeth, mucus shell of
cheeks and tongue
E.The taken away material at drawing out can touch teeth, mucus shell of cheeks and
tongue
591.
What temperature terms is it needed for cultivation of meningococcal on
artificial mediums?
A.23-40 °C
B. 35-43 °C
C. *35-37 °C
D.23-35 °C
E.37-39 °C
592.
When does the laboratory give the results of bacteriological examination of
smear from throat?
A.On 2th days
B. On 3th days
C. *On 4th days
D.On 5th days
E.On 6th days
593.
What is taken for serum research for confirmation of meningococcal infection?
A.*Blood
B. Mucus
C. Urine
D.CSF
E.Saliva
594.
What antibiotics preparations of choice of etiotropic therapy at a meningococcal
infection.
A.* Benzylpenicillin and it derivatives
B. Gentamycin
C. Cefazolin
D.Sulfolamide
E.Ciprofloxacin
595.
In what dose should| benzyl penicillin be administered at meningococcal
meningitis?
A.From a calculation 100-300 thousands unit on 1 kg of mass of body on days
B. *From a calculation 200-500 thousands unit on 1 kg of mass of body on days
C. From a calculation 500-700 thousands unit on 1 kg of mass of body on days
D.From a calculation 700-900 thousands unit on 1 kg of mass of body on days
E.Regardless of mass of body
596.
In what daily interval should the dose of benzylpenicillin at meningococcal
meningitis administered.
A.2 hrs
B. *4 hrs
C. 6 hrs
D.5 hrs
E.8 hrs
597.
Which preparation has a bacteriostatic action, and is more expedient to begin
etiotropic therapy in the case of infectious toxic shock.
A.From benzylpenicillin and its derivatives
B. From ciprofloxacin
C. From gentamycin
D.From ciprofloxacin
E.*From levomycitin of succinate
598.
A patient is sick with meningococcal meningitis. He take a massive dose of
penicillin. 4 days temperature of body 36,6-36,8 °C. Meningeal signs are negative. When is
it possible to stop the antibiotic therapy.
A.*At a cytosis in a CSF 100 and less, lymphocytes prevail
B. After 10 days from the beginning antibiotic therapy
C. After 7 days from the beginning antibiotic therapy
D.At a cytosis 100 and less, neutrophil prevail
E.From 6 days from the beginning antibiotic
599.
A patient with meningococcal meningitis gets penicillin during 7 days. The
temperature of body is normal 4 days. Meningeal signs are negative. When is it possible to
stop the antibiotic therapy.
A.In default of leucocytosis displacement in blood
B. *. At a cytosis in a neurolymph 100 and less, lymphocyte prevail
C. At a cytosis in a neurolymph 100 and less, neutrophil prevail
D.At a cytosis in a neurolymph 150, lymphocyte prevail
E.At once immediately
600.
For the treatment of acidosis at meningococcal meningitis is better to use.
A.10-20 % glucose solution
B. 10 % chloride solution
C. *4 % sodium bicarbonate solution
D.Albumin
E.Concentrated dry plasma
601.
Meningococemia and ID-syndrome require above all things.
A.Administration of diuretics
B. Administration of analgesic
C. *Administration of heparin
D.Administration of vitamins
E.Administration of antihistaminic preparations
602.
What is used as specific prophylaxis in the period of epidemic spreading of
meningococcal infection.
A.Immun globulin
B. Serum
C. *Vaccine
D.Anatoxin
E.Nothing
603.
What measures are conducted in the place of meningococcal infection?
A.Supervision during 2 weeks
B. Phagoprophylaxis
C. Immunization
D.*Bacteriological inspection of contact
E.Chemoprophylaxis
604.
What complication has developed in patient with diphtheria of mouth pellicle
severe form was diagnosed. On the 6th day of disease when pain in the heart region,
palpitation were appeared. Pulse – 120 per 1 min, systolic noise on apex of heart. On ECG
is incomplete blockade of left leg of Giss bunch?
A.*Early infectious-toxic myocarditis
B. Myocardial dystrophy
C. Heart attack of myocardium
D.Acute cardio-vessel insufficiency
E.Stenosis of mitral valve
605.
What group of infectious diseases diphtheria belong to?
A.Sapronosis
B. Zoonosis
C. *Anthroponosis
D.Zooanthroponosis
E.A group is not certain
606.
What is the properties of сorynebacterium diphtheria:
A.Contain endotoxin only
B. *Exotoxin products
C. Exotoxin does not product
D.An enterotoxin products
E.Myelotoxin products
607.
The source of infection at diphtheria is:
A.*Sick people and carriers
B. Sick agricultural animals
C. Rodents
D.Mosquitoes
E.Aerosol of saliva and epipharyngeal mucous of patients
608.
What is mechanism of transmission of Corynebacterium diphtheria?
A.Vertical
B. Transmissive
C. *Air-drop
D.Contact
E.Parenteral
609.
Especially high titre of ant diptherial antitoxic antibodies testifies to:
A.Incubation
B. Acute period of diphtheria
C. *Bacteriocarriering
D.Forming of immunity to diphtheria
E.About nothing does not testify
610.
What group of infectious diseases by L. Gromashevsky classification diphtheria
belong to?
A.External covers
B. Blood
C. Intestinal
D.*Respiratory
E.Transmissive
611.
What is transmissive factors in diphtheria?
A.Blood
B. Water
C. *Saliva
D.Urine
E.Exrements
612.
What is seasonal character of diphtheria?
A.Spring-summer
B. Summer-autumn
C. *Autumn-winter
D.Winter-spring
E.Spring-autumn
613.
Before revaccination from diphtheria of adult persons, they are recommended:
A.*To explore an immune type
B. To use antibiotics
C. To use antihistamines
D.5 years after last revaccination
E.10 years after last revaccination
614.
Diphtheria planned vaccination begin in:
A.In first days after birth of child
B. *In 3 month age
C. In 6-month age
D.In 1 year
E.In 6 years
615.
In preschool is case of disease on diphtheriA. What prophylactic measures must
be conducted above all things?
A.Urgent hospitalization
B. Urgent vaccination
C. *Quarantines measures
D.Urgent by chemical prophylactic antibiotics
E.Introduction of antidiphterial whey
616.
At a child 6 years with a diphtherial widespread croup the first dose of
antidiphterial serum makes:
A.*40 AО
B. 15 AО
C. 20 AО
D.80 AО
E.60 AО
617.
What material it’s necessary to take for bacteriologic examination in suspicion
on diphtheria?
A.Excrement
B. Blood
C. Urine
D.*Mucous
E.Neurolymph
618.
What is main complication of diphtheria of larynx:
A.Myocarditis
B. Paresis of auditory nerve
C. Nephrosonephritis
D.*Croup
E.Poliomyelitis
619.
What complications more often develops during the first week of diphtheria of
otopharynx:
A.Poliomyelitis
B. Asphyxia
C. Paratonsillitis
D.Hepatospleenomegaly
E.*Paresis of soft palate
620.
What is early complications of diphtheria of otopharynx:
A.*Paresis of soft palate
B. Pneumonia
C. Asphyxia
D.Croup
E.Poliomyelitis
621.
What complications more often develops during 4-5th week of diphtheria:
A.Encephalitis
B. Bulbar disorders, pancreatitis, hepatitis
C. *Poliomyelitis, myocarditis
D.Nephrosonephritis
E.Stenotic laryngotracheitis
622.
What are the rules of hospitalization of patients with infectious mononucleosis?
A.Patients are not hospitalized
B. In a chamber for the infections of respiratory tracts
C. *In a separate chamber
D.In a chamber for the infections of external covers
E.In a chamber for intestinal infections
623.
What additional inspections must be conducted to the patient with infectious
mononucleosis?
A.*IFA on HIV-infection, bacteriology inspection on diphtheria
B. IFA on HIV-infection, bacteriology inspection on a rabbit-fever
C. Bacteriology inspection on diphtheria and typhoid
D.Reaction of Burne and Rihth-Heddlson
E.Reaction of Paul-Bunnel and punction of lymphatic knot
624.
What from the following symptoms are not characteristic of infectious
mononucleosis?
A.Fever
B. *Defeat of kidneys
C. Lymphadenopathy
D.Tonsillitis
E.Increasing of liver and spleen
625.
For what disease characterize changes in a blood (presence of lymphomonocytes
and atypical mononuclears)?
A.Flu
B. *Infectious mononucleosis
C. Measels
D.AIDS
E.Diphtheria
626.
What additional test should hold for the patient with infectious mononucleosis?
A.Burne and Wright-Hadlson‘s reactions
B. ELISA-test, bacteriological test for tularemia
C. Bacteriological test for diphtheria and typhoid fever
D.*ELISA-test, bacteriological test for diphtheria
E.Paul-Burne reaction and lymph node puncture
627.
What the most possible complication occurs during infectious mononucleosis?
A.Meningitis
B. Autoimmune alopecia
C. Encephalitis
D.*Splenic rupture
E.Obstruction of respiratory tract
628.
The source of infection at infectious mononucleosis is:
A.*Sick people and carriers
B. Sick agricultural animals
C. Rodents
D.Mosquitoes
E.Aerosol of saliva and epipharyngeal mucous of patients
629.
What is seasonal character of infectious mononucleosis?
A.Spring-summer
B. Summer-autumn
C. Autumn-winter
D.*Winter-spring
E.Spring-autumn
630.
After the disease which was accompanied by the fever and pharyngalgias, there
were an odynophagia, dysarthria, weakness and violation of motions in hands and feet,
hyporeflexia, violation of sensitiveness in extremities to the polyneurotic type. What
disease does it follow to think about above all things?
A.Neuropathy of hypoglossus
B. *Diphtherial polyneuropathy
C. Neuropathy of glossopharyngeus nerve
D.Trunk encephalitis
E.Pseudobulbar syndrome
631.
What is immediately investigation in suspicious of diphtheria:
A.Strokes with tonsills, nose or other areas for the exposure of diphtherial stick
B. IFA
C. *Microscopy (painting by Neiser)
D.Haemoculture
E.RDHA with a diphtherial diagnosticum
632.
Etiology agent of meningitis is:
A.*Neisseria meningitides
B. Entamoeba histolytica
C. Vibrio cholerae
D.Clostridium botulinum
E.Campylobacter pylori
633.
Wich of these symptoms are often present in patients with meningitis?
A.Algor, high temperature, headache
B. Profuse watery diarrhea, vomiting, dehydratation, muscular cramps
C. Abdominal pain, diarrhea, constipation, flatulence
D.Headache, dry cough, algor
E.*Prodromal respiratory illness or sore throat, fever, headache, stiff neck, vomiting,
confusion, irritability
634.
What group of infectious diseases meningococcal infection belong to:
A.Intestinal
B. Blood
C. *Respiratory
D.Transmissive
E.External covers
635.
What is the mechanism of transmission of meningococcal infection?
A.Fecal-oral
B. Contact
C. Transmissive
D.*Air-drop
E.Vertical
636.
What is seasonal character of meningococcal infection?
A.Summer-autumn
B. Autumn-winter
C. *Winter-spring
D.Winter
E.Summer
637.
What clinical form of meningococcal infection more often may happened?
A.Meningococcemia
B. Meningitis
C. Meningoencephalitis
D.*Nasopharengitis
E.Pneumonia
638.
What syndrome may appear in severe meningococcemia?
A.Paul-Bunnel
B. Plaut-Vincent
C. Jarish-Gersgeimer
D.Gien-Barre
E.*Waterhause-Friedrichsen
639.
What laboratory methods should be taken to discharge meningitis?
A.*Lumbar puncture
B. Serologic detection
C. Urine examination
D.Coprograma
E.Biopsy of tissues
640.
Source of meningitis is:
A.Animals
B. Birds
C. Fish
D.Pediculus humanus
E.*People
641.
How is it possible to specify the diagnosis of meningococcal meningitis.
A.Meningitis is primary
B. Presence of a lot of cells in the CSF
C. Presence of gram-negative diplococcus in CSF
D.Meningococes from the throat
E.*All the above
642.
What are the rules at taking of smear material on the discovery of meningococal
infection?
A.The taken away material at drawing out must not touch only mucus shell of cheeks
and tongue
B. The taken away material at drawing out must not touch only teeth and tongue
C. The taken away material at drawing out must not touch only teeth, mucus shell of
cheeks
D.*The taken away material at drawing out must not touch|| teeth, mucus shell of
cheeks and tongue
E.The taken away material|| at drawing out can touch|| teeth, mucus shell of cheeks
and tongue
643.
When does the laboratory give the results of bacteriological examination of
smear from throat?
A.On 2th days
B. On 3th days
C. *On 4th days
D.On 5th days
E.On 6th days
644.
What is taken for serum research for confirmation of meningococcal infection?
A.*Blood
B. Mucus
C. Urine
D.CSF
E.Saliva
645.
What antibiotics preparations of choice of etiotropic therapy at a meningococcal
infection.
A.*Benzylpenicillin and it derivatives
B. Gentamycin
C. Cefazolin
D.Sulfolamide
E.Ciprofloxacin
646.
In what dose should| benzyl penicillin be administered at meningococcal
meningitis?
A.From a calculation 100-300 thousands unit on 1 kg of mass of body on days
B. *From a calculation 200-500 thousands unit on 1 kg of mass of body on days
C. From a calculation 500-700 thousands unit on 1 kg of mass of body on days
D.From a calculation 700-900 thousands unit on 1 kg of mass of body on days
E.Regardless of mass of body
647.
In what daily interval should the dose of benzylpenicillin at meningococcal
meningitis administered.
A.2 hrs
B. *4 hrs
C. 6 hrs
D.5 hrs
E.8 hrs
648.
Which preparation has a bacteriostatic action, and is more expedient to begin
etiotropic therapy in the case of infectious toxic shock.
A.From benzylpenicillin and its derivatives
B. From ciprofloxacin
C. From gentamycin
D.From ciprofloxacin
E.*From laevomycitin succinate
649.
For the treatment of acidosis at meningococcal meningitis is better to use.
A.10-20 % glucose solution
B. 10 % chloride solution
C. *4 % sodium bicarbonate solution
D.Albumen
E.Concentrated dry plasma
650.
Meningococсemia and DIC-syndrome require above all things.
A.Administration of diuretics
B. Administration of analgesic
C. *Administration of heparin
D.Administration of vitamins
E.Administration of antihistaminic preparations
651.
What is used as specific prophylaxis in the period of epidemic spreading of
meningococcal infection.
A.Immune globulin
B. Serum
C. *Vaccine
D.Anatoxin
E.Nothing
652.
A patient with meningococcal meningitis gets penicillin during 7 days. Last 4
days temperature of body is normal. Meningeal signs are absent. When is it possible to
abolish an antibiotic?
A.*At cytosis in liquor 100 and less, lymphocytes prevails
B. At absence of leukocytosis and stab-nucleus shift in a blood
C. At cytosis in liquor 100 and more less, neutrophils prevails
D.At cytosis in liquor 150, lymphocytes prevails
E.At once
653.
What measures are conducted in the place of meningococcal infection?
A.Supervision during 2 weeks
B. Phagoprophylaxis
C. Immunization
D.*Bacteriological inspection of contact
E.Chemoprophylaxis
654.
Violations of electrolyte balance show up at leptospirosis:
A.*Metabolic acidosis
B. By a metabolic alkalosis
C. Respirator acidosis
D.By a respiratory alkalosis
E.All above enumerated
655.
The decline of arteriotony at a leptospirosis is not caused:
A.By expansion of vessels under the action of toxin
B. Hypovolemia
C. By myocarditis
D.*Hypercalcgesty
E.By adrenal insufficiency
656.
All are the clinical signs of measles except:
A.Acute beginning of high fever
B. *Icterus
C. Maculo-papular rash
D.Sequential appearance of rash
E.Scaling
657.
For how long a patient with complicated form of measles should be isolated:
A.For 4 days from the beginning of rash
B. For 7 days from the beginning of rash
C. *For 10 days from the beginning of rash
D.For 17 days from the beginning
E.For 20 days from the beginning of illness
658.
Term of contagious period of patient diagnosed with uncomplicated form of
measles
A.Until clinical recovery
B. After rash starts disappearing
C. Before appearance of rash
D.*4 days from the beginning of rash
E.10 days from the beginning of illness
659.
What is the duration of quarantine in child's establishment in case of rubella?
A.5 days after the isolation of the last patient
B. 11 days
C. 21 day
D.10 days
E.No need for quarantine
F. *5 days after isolation of the last child
660.
What is duration of contagious period for a patient with epidemic parotitis?
A.21 days
B. First week of illness
C. First 10 days from the beginning of disease
D.Whole period of clinical symptoms
E.*First 9 days of disease.
661.
What measures should be taken in regards to persons, who were in contact with a
patient diagnosed with epidemic parotitis?
A.Observation after contact people during a maximal length of incubation period
B. Quorantin in child's establishment
C. Isolation of people who were in contact with ill from 11th to the 21t day of illness
D.Isolation of children up to 10 years old, who were not ill with epidemic parotitis,
for 21 day from a moment of contact
E.*All above enumerated
662.
What is the duration of contagious period for a patient diagnosed with scarlet
fever?
A.10 days from the beginning of illness
B. Until patient is discharged from the hospital
C. Until rash is present
D.*Till the 22d day from the beginning of illness
E.Not contagious
663.
What is duration period of supervision after ill with scarlet fever?
A.*7 days from time of contact
B. 21 day
C. Till patient’s rash is present
D.Till patient is discharged from permanent establishment
E.Not conducted
664.
Methods of specific prophylaxis of scarlet fever:
A.Isolation of ill
B. Vaccination
C. Use of antibiotics
D.Disinfection
E.*Does not exist
665.
What are the anti epidemic measures in regards to people who were in contact
with chicken-pox patient:
A.*Separation and limit of contacts with others
B. Vaccination
C. Use of antibiotics
D.Disinfection
E.Does not exist
666.
Measures of urgent prophylaxis for unvaccinated children who have never been
ill with measles in case of exposure to an ill with measles
A.Separation from the source
B. *Vaccination
C. Administration of antibiotics
D.Disinfection
E.Does not exist
667.
Measures of urgent prophylaxis of measles for contacts which have never been
ill, but were vaccinated against measles
A.Separation from the source
B. Vaccination
C. Use of antibiotics
D.Use of immunoglobulin
E.*No need to conduct
668.
Measures of urgent prophylaxis of measles for people who had been ill with
measles, but never have been vaccinated
A.Separation from the ill
B. Vaccination
C. Use of immunoglobulin
D.*Use of antibiotics
E.No need to conduct
669.
Possible side effects at application of antibiotics are all, except.
A.Stomach-ache
B. Nausea, vomit
C. Diarrhea
D.*Fever
670.
671.
672.
673.
674.
675.
676.
677.
E.Skin rash
Contra-indication for the application of antibiotics are all, except.
A.An increased sensitiveness to preparation
B. Severy disorders of liver
C. Severy disorders of kidneys
D.Period of pregnancy and lactation
E.*Prolonged fever
Principles of etiotropic therapy of sepsis.
A.Administration of antibiotics quick as possible
B. Administration of antibiotics in maximal therapeutic doses
C. In accordance to credible microbiological diagnosis
D.An account of possible of therapeutic concentration of antibiotic in field of
infection
E.*All the above
Immune modulator therapy of sepsis are all, except.
A.Ronkoleykin
B. Interferon
C. Inductors of interferon
D.*Vaccine
E.Normal human immune globuline
Basic principles of antibiotics therapy.
A.A selection of antibiotic after the sensitiveness of the selected exciter
B. selection of antibiotic according to the diagnosis of certain infectious disease
clinically
C. Choose the most active preparation
D.Choose the least toxic preparation
E.*All the above
Basic principles of antibiotics therapy of sepsis.
A.A selection of preparation from data of bacteriostatic
B. Determination of dose, method and multiple of introduction the preparation
C. Timeliness and definite duration of introduction input of antibiotic
D.D Combining antibiotics between itself for enhancement of antibacterial effect
E.*E. All the above
Agglutinines at a leptospirosis arrive at a maximal titre:
A.On the third day of illness
B. *On the third week of illness and later
C. On the fourth week of illness
D.On the second month of illness
E.To the second week of illness
At a leptospirosis the exposure of antibodies is considered reliable in a titre:
A.1:70 and anymore
B. 1:80 and anymore
C. 1:60 and anymore
D.*1:100 and anymore
E.1:40 and anymore
At treatment of patients with leptospirosis antibiotics consider most effective:
A.*Penicillin
B. Macrolids
C. Cefalosporins
D.Ftorhinolons
E.Sul'fanilamids
678.
Etiotropic therapy of leptospirosis includes:
A.Oxyhinolons
B. Sulfanilamids
C. Nitrofurans
D.Antitocidns
E.*Antibiotics
679.
What from antibiotics more expedient to use for treatment of leptospirosis?
A.Macrolids
B. Tetracyclins
C. Aminoglicosids
D.*Penicillins
E.Metrogil
680.
A patient has a severy icteric form of leptospirosis. What from antibiotics is
better to appoint as etiotropic therapy?
A.Yunidoks
B. Tetracyclinum
C. Azitromicin
D.Rovamicin
E.*Penicillin
681.
Treatment of leptospirosis:
A.Desintoxication, dehydratation, antibiotics, glukokorticosteroids
B. *Antileptospirosis immunoprotein, antibiotics, nosotropic facilities
C. Antibiotics, rehadratation, sorbtion preparation, vitamins
D.Diet № 7, antibiotics, desintoxication facilities
E.Antibiotics, diuretic, interferons
682.
Daily allowance doses of penicillin at treatment of leptospirosis:
A.2-3 million units
B. *3-12 million units
C. 20 million units
D.40 million units
E.Over 40 million units
683.
With the purpose of immunotherapy it is better to apply at treatment of
leptospirosis:
A.Immunodepressants
B. Antihistaminics
C. Horse whey
D.*Antileptospirosis human immunoprotein
E.Antileptospirosis neat immunoprotein
684.
What from antibiotics are more effective in treatment of icteric form of
leptospirosis:
A.*Penicillins
B. Aminoglicozids
C. Tetracyclins
D.Macrolids
E.Metrogil
685.
For the prophylaxis of leptospirosis use:
A.*Active vaccine
B. Anavaccine
C. Toxoid
D.All
E.Antibiotics
686.
Vaccinations against leptosprosis perform for:
A.All
B. Only villagers
C. Only to the habitants of endemic districts
D.*Only to the persons busy in the stock-raising
E.It is not conducted
687.
Which serotypes of leptospirosis caused the disease more frequent:
A.L. interogans
B. L. grippotyphosa
C. L. canicola
D.*L. icterohaemorrhagia
E.L. Pomona
688.
All of these have an epidemic dangerous exept:
A.Farm animals
B. Wide rodents
C. Domestic animals
D.Foxes
E.*Human
689.
How long lasts the leptospirosis incubation period:
A.2 month
B. 1-7 days
C. *7-14 days
D.14-21 days
E.2-3 days
690.
The main of antibiotics which is used in treatment of leptospirosis:
A.Tetracyclin
B. Tetraolean
C. Erythromycin
D.Streptomycin
E.*Penicillin
691.
When there can be such specific complication of typhoid fever, like to intestinal
bleeding?
A.On the 1st week of illness
B. On the 2nd week of illness
C. *On the 3rd week of illness
D.On the 4th week of illness
E.On the any week of illness
692.
When there can be such specific complication of typhoid fever, like to
perforation of bowel?
A.On the 1st week of illness
B. On the 2nd week of illness
C. *On the 3rd week of illness
D.On the 4th week of illness
E.On the any week of illness
693.
Who is the source of typhoid fever?
A.*Sick people
B. Sick agricultural animals
C. Sick rodents
D.Soil
E.Defecating of patients
694.
Salmonella typhi contains:
A.Only O-antigen and Н-antigen
B. Only O-antigen and Vi-antigen
C. Only H-antigen and Vi-antigen
D.*O-antigen, H-antigen and Vi-antigen
E.O-antigen, H-antigen, Vi- antigen and HBsAg
695.
When is it possible to abolish etiotropic preparations in a patient with typhoid
fever?
A.At once after normalization of temperature of body
B. After normalization of sizes of liver and spleen
C. After disappearance of roseollas
D.In 10 days after disappearance of roseollas
E.*After the 10th day of normal temperature of body
696.
By which method is it possible to find out bacterial carrier in case of typhoid
fever?
A.Coproculture
B. Reaction of agglutination of Vidall
C. Indirect hemaglutination test with О-аntigen
D.Indirect hemaglutination test with a Н-antigen
E.*Indirect hemaglutination test with a Vi-antigen
697.
Typhoid bacilli are usually cultured from:
A.*Blood, stool, urine
B. Blood, urine, sputum
C. Stool, liquor, urine
D.Blood, stool, sputum
E.Stool, liquor, sputum
698.
A suspected case of typhoid fever of 1st week is admitted in the hospital. What
examination (laboratory diagnosis) do you suggest for this patient?
A.Coproculture
B. Reaction of agglutination of Vidall
C. Indirect hemaglutination test with О-, Н-, Vi-аntigens
D.Urinoculture
E.*Hemoculture
699.
A suspected case of typhoid fever of 3nd week is admitted in the hospital. What
examination (laboratory diagnosis) do you suggest for this patient?
A.Coproculture
B. Reaction of agglutination of Vidall
C. Urinoculture
D.Hemoculture
E.*All about it
700.
What from the transferred signs is not characteristic for a typhoid rash?
A.*Papular, disappears together with normalization of temperature of body
B. Appears on a 7-10th day, rosella-type
C. Located mainly on a abdomen and lateral surfaces of trunk, observed at the half of
patients
D.The amount of elements is limited, pours in addition
E.rosella-type, sometimes saved longer than fever
701.
What ever symptom is not characteristic for typhoid on the second week of
illness?
A.Constipation
B. Headache
C. Fever
D.Relative bradycardia
E.*Cramps
702.
What changes in general analysis of blood are characteristic for typhoid?
A.*Leykopenia, aneosiniphilia, lymph-, monocytosis, enhanceable RSE
B. Leykocytosis, hypereosinophilia, thrombocytosis, enhanceable RSE
C. Hypochromatic anaemia, leycocytosis, appearance of young forms, RSE is not
changed
D.Hyperchromatic anaemia, leycocytosis, appearance of young forms, enhanceable
RSE
E.Leykopenia, lymphopenia, thrombocytosis, enhanceable RSE
703.
What does the diagnostic titre of reaction of Vі-haemaglutination testify to?
A.*About typhoid bacterial-carrier
B. About the period of height of the epidemic typhus
C. About meningococcaemia
D.About a malaria
E.About the latent period of brucellosis
704.
What term of looking after the hearth of typhoid?
A.14 days
B. *21 days
C. 7 days
D.30 days
E.Does not look after
705.
Reconvalescente of typhoid fever may go out from clinic after:
A.Non-permanent negative bacteriologic examination of defecating
B. *21th day of normal temperature of body and 3-multiple negative bacteriologic
examination of excrement and urine
706.
707.
708.
709.
710.
711.
712.
C. of 14th day of normal temperature of body and 2-multiple negative bacteriologic
examination of excrement and urine
D.Clinical convalescence and normalization of rectal manoscopic picture
E.Normalisations of rectal manoscopic picture and in default of title of antibodies in
RNGA
In epydfocus of typhoid fever doing, except for:
A.Daily thermometery
B. Coprologic culture
C. Reaction of Vidal
D.*Haemoculture
E.Urine culture
For establishment of transmitter of Salmonella typhi utilize:
A.Test of Cuverkalov
B. RA with O- and Н-antigen
C. PCR
D.Bacteriologic examination and reaction of Vidal
E.*Bacteriologic examination and RNGA with a Vi-antigen
What from the adopted ways of transmission is characteristic for typhoid?
A.*Alimentary
B. Contact
C. Transmission
D.Air-drop
E.Vertical
What environments do typhoid sticks grow on well?
A.Chicken embryos
B. water-whey nourishing environment
C. *Bilious clear soup
D.Meat-peptone gelose + cistin
E.To the Bismute-sulfate gelose
Name of the basic factors of pathogen of typhoid stick?
A.*Vi-antigen and endotoxin
B. Exotoxin
C. Vi-antigen
D.Enzymes of pathogenicity
E.Endotoxin
Duration of latent period at typhoid?
A.3-7 days
B. *9-21 day
C. From a few hovers to 2-3 days
D.From 12 to 100 days
E.From a few hovers to 17 days
The part of reproduction of typhoid bacterium in the organism of man is:
A.Stomach
B. *Lymphatic formations of colon
C. Blood
D.Bilious ways
E.Mucous membrane of colon
713.
What from the adopted phases of pathogenesis is not characteristic for typhoid?
A.*Swelling, edema of mucous membrane of overhead respiratory tracts
B. Stage of penetration
C. Stage of lymphodefence reactions
D.Stage of bacteriaemia
E.Stage of intoxication
714.
What from the indicated pathology anatomic phases is not characteristic for
typhoid?
A.*Catarrhal inflammation of amygdales
B. Cerebral-type of swelling
C. Necrosis
D.Ulcers
E.Clean ulcers
715.
What is entrance gates of typhoid fever agent?
A.Mucous membrane of amygdales
B. Mucous membrane of nasopharynx
C. Epithelial cells of skin
D.Mucous membrane of colon
E.*Mucous membrane of digestive tract
716.
Who is the source of epidemic typhus?
A.Patients with epidemic typhus
B. Patients with disease Brill-Zinsser
C. *Patients with epidemic typhus and disease Brill-Zinsser
D.Patients with Brill-Zinsser disease and Sachs disease
E.Patients with epidemic typhus and abdominal typhoid
717.
During what time the lice can transfer the epidemic typhus?
A.Up to 10 days
B. Up to 15 days
C. Up to 20 days
D.*Up to 30 days
E.Up to 40 days
718.
What is Brill's disease?
A.Vertiacal borne
B. *Remote relapse typhus
C. Early relapse typhus
D.Re-infection rickettsia
E.Self nozological unit
719.
Often, in patient with epidemic typhus is tongue‘s tremor when protrusion that
sticked on the lower teeth. What term did it call?
A.Symptom of Heller
B. Conjunctivitis
C. *Govorov-Godele symptom
D.Zorohovich-Chiari symptom
E.Enantema Rosenberg‘s
720.
What is not typical for epidemic typhus exanthema?
A.*Arise on 7-10-day of illness
B. Has rosy-petehia nature
C. Localized mainly on the lateral surface of the torso and limbs flexion surfaces
D.It can grab his hands and feet, but never on the face
E.Abundant
721.
What is not typical for epidemic typhus exanthema?
A.Disappear with the drop in temperature
B. Never appeared another rash
C. Roseola saved up to 6 days, petehii – 12
D.Leave a little pigmentation and poor peeling
E.*Single elements
722.
What is not typical of blood in the severe epidemic typhus?
A.Neutrofil leukocytosis
B. Academy of Sciences or hypleozinofil
C. Limphopeniya
D.*Anemia
E.The increase in ESR
723.
What is not typical for the analysis of urine in the severe epidemic typhus?
A.Proteinuria
B. Single-cylinder hyaline
C. *Multiple granular cylinders
D.A small number of erythrocytes
E.A small number of leukocytes
724.
When is serological diagnosis possible in patients with epidemic typhus?
A.From the 1st day of illness
B. On the 2nd day illness
C. From the 3-4th day illness
D.From 4-5th day of illness
E.*From the 5 to 7th day of illness
725.
What diagnostic titer response agglutination test with typhus rickettsia?
A.1:40 and above
B. 1:80 and above
C. *1:160 or higher
D.1:320 or higher
E.1:640 or higher
726.
In with titre will be positive reaction of agglutination in patient with epidemic
typhus in droplets test when Mosing?
A.*1:40 and above
B. 1:80 and above
C. 1:160 or higher
D.1:320 or higher
E.1:640 or higher
727.
For contact persons in the centre of the epidemic typhus establish surveillance
over:
A.21 days
B. 25 days
C. *51 days
D.72 days
E.3 months
728.
During the contact person with the Brill‘s disease establish surveillance over:
A.21 days
B. *25 days
C. 51 days
D.72 days
E.3 months
729.
In the case of head lice carry out sanitation: hair cut, followed by incineration,
processing hair. What kind of insecticide products for this use?
A.0,5 % solution of water emulsions karbofosa
B. 0,5 % metilatsetofos
C. 10 % solution of liquid neutral metilatsetofosa soap
D.0,5 % water emulsion dikrezilu
E.*3 % soap RHTSG
730.
When patients after epidemic typhus may go out from clinic?
A.*After clinical recovery, but not before the 12-day normal temperature
B. After a full clinical recovery
C. After clinical recovery, but not before the 12-day period following the lifting of
antibiotics
D.At the 12-day normal body temperature
E.After clinical recovery, but not earlier than the 9-day normal body temperature
731.
Whit antibiotics are less effective from the transferred at the epidemic fever?
A.Tetracyclin
B. Metacyclin
C. *Levomicetyn
D.Vibramycinum
E.Doxyciclin
732.
Who is the source of epidemic typhus?
A.Patients with epidemic typhus
B. Patients with disease Brill-Zinsser
C. *Patients with epidemic typhus and disease Brill-Zinsser
D.Patients with Brill-Zinsser disease and Sachs disease
E.Patients with epidemic typhus and abdominal typhoid
733.
When sick people gets epidemic typhus infection, which period affects more?
A.Over the past 2 days, the incubation period and 2-3 days after lowering temperature
B. All hectic period and 2-3 days after lowering temperature
C. 2-3 days after lowering temperature
D.*Over the past 2 days, the incubation period, all febrile period and 2-3 days after
lowering temperature
E.Over the past 2 days, the incubation period and the hectic period
734.
On which period the maximal growth of infection occurs during epidemic typhus
disease?
A.At the incubation period
B. *At the 1th week of illness
C. At the 2nd week of illness
D.At the 3rd week of illness
E.At the time of recovery
735.
During what time the lice can transfer the epidemic typhus?
A.Up to 10 days
B. Up to 15 days
C. Up to 20 days
D.*Up to 30 days
E.Up to 40 days
736.
What is Brill's disease?
A.Vertiacal borne
B. *Remote relapse typhus
C. Early relapse typhus
D.Re-infection rickettsia
E.Self nozological unit
737.
Often, in patient with epidemic typhus arise transition petehies in the
conjunctivA. What term did it call?
A.Symptom of Heller
B. Conjunctivitis
C. Symptom of Govorova-Godele
D.*Symptom of Zorohovich-Chiari-Avtsyna
E.Enantema Rosenberg‘s
738.
Often, in patient with epidemic typhus arises petehies on mucosal soft palate.
What term did it call?
A.Symptom of Heller
B. Conjunctivitis
C. Symptom of Govorova-Godele
D.Symptom of Zorohovich-Kiari
E.*Enantema Rosenberg‘s
739.
Often, in patient with epidemic typhus is tongue‘s tremor when protrusion that
sticked on the lower teeth. What term did it call?
A.Symptom of Heller
B. Conjunctivitis
C. *Symptom of Govorova-Godele
D.Symptom of Zorohovich-Chiari
E.Enantema Rosenberg‘s
740.
What is not typical for epidemic typhus exanthema?
A.*Arise on 7-10-day of illness
B. Has rosy-petehia nature
C. Localized mainly on the lateral surface of the torso and limbs flexion surfaces
D.It can grab his hands and feet, but never on the face
E.Abundant
741.
What is not typical for epidemic typhus exanthema?
A.Disappear with the drop in temperature
B. Never appeared another rash
C. Roseola saved up to 6 days, petehii – 12
D.Leave a little pigmentation and poor peeling
E.* Arise on 7-10-day of illness
742.
What is not typical of blood in the severe epidemic typhus?
A.Neutrofil leukocytosis
B. Academy of Sciences or hypleozinofil
C. Limphopeniya
D.*Anemia
E.The increase in ESR
743.
What is not typical for the analysis of urine in the severe epidemic typhus?
A.Proteinuria
B. Single-cylinder hyaline
C. *Multiple granular cylinders
D.A small number of erythrocytes
E.A small number of leukocytes
744.
When is serological diagnosis possible in patients with epidemic typhus?
A.From the 1st day of illness
B. On the 2nd day illness
C. From the 3-4th day illness
D.From 4-5th day of illness
E.*From the 5 to 7th day of illness
745.
What diagnostic titer response agglutination test with typhus rickettsia?
A.1:40 and above
B. 1:80 and above
C. *1:160 or higher
D.1:320 or higher
E.1:640 or higher
746.
In with titre will be positive reaction of agglutination in patient with epidemic
typhus in droplets test when Mosing?
A.*1:40 and above
B. 1:80 and above
C. 1:160 or higher
D.1:320 or higher
E.1:640 or higher
747.
In the family of the patient with epidemic typhus, were lice in the children. With
the help of any of these events could prevent the subsequent spread of the disease?
A.*Monitoring and complete sanitation of contact in the centre
B. The use of chemoprophylaxis
C. The use of antibiotics
D.Isolation contact
E.Check-up
748.
When you can stopped etiotropic medications treatment of the patient with
epidemic typhus?
A.Immediately after the normalization of body temperature
B. After the normalization of the liver and spleen
C. *After a 2-day normal body temperature
D.After the disappearance of roseola
749.
over:
E.Within 10 days after the disappearance of roseola
For contact persons in the centre of the epidemic typhus establish surveillance
A.21 days
B. 25 days
C. *51 days
D.72 days
E.3 months
750.
During the contact person with the Brill‘s disease establish surveillance over:
A.21 days
B. *25 days
C. 51 days
D.72 days
E.3 months
751.
In the case of head lice carry out sanitation: hair cut, followed by incineration,
processing hair. What kind of insecticide products for this use?
A.0,5 % solution of water emulsions karbofosa
B. 0,5 % metilatsetofos
C. 10 % solution of liquid neutral metilatsetofosa soap
D.0,5 % water emulsion dikrezilu
E.*3 % soap RHTSG
752.
When patients after epidemic typhus may go out from clinic?
A.*After clinical recovery, but not before the 12-day normal temperature
B. After a full clinical recovery
C. After clinical recovery, but not before the 12-day period following the lifting of
antibiotics
D.At the 12-day normal body temperature
E.After clinical recovery, but not earlier than the 9-day normal body temperature
753.
Phage symptom in case of yellow fever is:
A.Pain in right iliac area
B. Enanthema on a soft palate
C. *Replacement of tachicardia on expressed bradicardia
D.Hemorrhages in a conjunctiva
E.Yellow hands
754.
Hemograme in the second period of yellow fever:
A.Leukocytosis
B. Normal global analysis of blood
C. *Leukopenia, neutropenia
D.Leukopenia, neutrophilosis
E.Leukocytosis, lymphomonocytosis
755.
Whatever complication meets at the yellow fever:
A.*Liver insufficiency
B. Kidney insufficiency
C. Infectious-toxic shock
D.Myocarditis
E.Edema of lungs
756.
Unlike leptospirosis in case of yellow fever is absent:
A.Hemorrhagic syndrome
B. Kidney insufficiency
C. Іntoxication syndrome
D.Міalglic syndrome
E.*Hepatic insufficiency
757.
For confirmation of yellow fever diagnosis use:
A.Bacteriological analysis of blood
B. Bacteriological examination of urine
C. *Virological hemanalysis
D.Biochemical blood test
E.Global analysis of blood
758.
In the initial period of hemorrhagic fever with a kidney syndrome a characteristic
sign is:
A.High temperatures
B. Pains in gastrocnemius muscles and positive Pasternatsky symptom
C. *Pains in joints and positive Pasternatsky symptom
D.Hemorragic syndrome
E.Dyspepsia phenomena
759.
An initial period at the hemorrhagic fever with a kidneys syndrome lasts:
A.Few hours
B. Day
C. *To three days
D.Week
E.Two weeks
760.
Whether there is violation of diuresis at patients with hemorrhagic fever with a
kidneys syndrome:
A.In an initial period
B. It is not
C. It is in all periods of disease
D.*It is in climax period
E.It is in the period of recovering
761.
General view of patient with the hemorrhagic fever with a kidneys syndrome:
A.Skinning covers
B. *Pallor of nasolabial triangle, hyperemia of neck and overhead half of trunk
C. Hyperemia of person, scleritis, conjunctivitis
D.Grayish color of person
E.Icteric color of skin
762.
In the biochemical blood test at patients with the hemorrhagic fever with a
kidneys syndrome not characteristically:
A.High level of urea
B. Decline of potassium level
C. *Bilirubinemia
D.Increasing of kreatinine
E.Increasing of nitrogen
763.
For confirmation of diagnosis of hemorragic fever with a kidney syndrome use:
A.Bacteriological method
B. Virological method
C. *Reaction of immunofluorescence
D.Reaction of braking of hemagglutination
E.Research of blood drop under a microscope
764.
For treatment of patients with the hemorrhagic fever with a kidney syndrome
does not use:
A.Glucocorticoids
B. Anabolic steroid
C. Disintoxication facilities
D.*Dihydration facilities
E.Antihistaminics
765.
For the initial period of the Congo hemorrhagic fever not characteristically:
A.Fever
B. Pains in joints and muscles
C. Severe pain of head
D.*Oliguria
E.Dizziness
766.
At an objective review for the Congo hemorrhagic fever characteristically:
A.*Mucosal hyperemia of person
B. Pallor of person
C. Puffiness of person
D.Ochrodermia of person
E.Exanthema on face
767.
The most characteristic symptom in the climax period of the Congo hemorrhagic
fever is:
A.*Hemorrhagic syndrome
B. Hepatic insufficiency
C. Dyspepsia phenomena
D.Sharp kidney insufficiency
E.Мeningeal syndrome
768.
In the general blood analysis of in case of Congo hemorrhagic fever is not
typically:
A.Leukocytosis
B. *Leukopenia
C. Neutropenia
D.Thrombocytopenia
E.Increasing of ESR
769.
What rashes in case of haemorrhagic fevers with kidneys syndrome?
A.Roseola
B. Maculo-papular
C. Punctuate
D.*Petechial
E.Rashes is not characteristic
770.
What rashes present in case of Congo hemorrhagic fever?
A.Roseola
B. Maculo-papular
C. Punctulate
D.*Petechial
E.Rashes not is characteristic
771.
What rashes present in case of Crimea hemorrhagic fever?
A.Roseola
B. Maculo-papular
C. Punctulate
D.*Petechial
E.Rashes not is characteristic
772.
How long the rash is present in case of haemorrhagic fever with kidneys
syndrome?
A.*During all feverish period
B. Before the reconvalescense
C. Before development of clinical features of kidneys insufficiency
D.During whole disease
E.Appears yet in a latent period and disappears in the period of early reconvalescense
773.
A kidney syndrome at haemorrhagic fever with kidneys syndrome shows up
usually:
A.Only laboratory changes
B. Only on BRIDLES
C. *By pain in lumbar area, positive Pasternatsky symptom, development of oliguria
D.By fever, polyuria, dyspepsia
E.By paradoxical ischuria
774.
What changes in biochemical blood test inherent for haemorrhagic fever with
kidneys syndrome?
A.Increase level of urea and bilirubin
B. The level of urea and kreatinine falls
C. The level of kreatinine grows and urea falls
D.The level of urea grows and kreatinine falls
E.*The level of urea and kreatinine grows
775.
What changes in haemogram inherent for haemorrhagic fever with kidneys
syndrome?
A.Normochromic anaemia, leucocytosis with atypical mononucleosis,
thrombocytopenia enhanceable ESR
B. erythrocytosis, lymphocytosis,ESR is enhanceable
C. Normochromic anaemia, leucopenia with neutrophylosis, thrombocytopenia
enhanceable ESR
D.*Hypochromic anaemia, leucocytosis with neutrophylosis, thrombocytopenia
enhanceable ESR
E.Hyperchromic anaemia, leucocytosis with neutrophylosis, thrombocytopenia
mionectic ESR
776.
The period of polyuria at haemorrhagic fever with kidneys syndrome is a sign of:
A.*Recovering
B. Chronic process
C. Unfavorable flow of illness
D.Development of complications
E.Complete convalescence
777.
In most patients with Congo hemorrhagic fever temperature curve is:
A.Wunderlich type
B. Botkin type
C. Undulating
D.Intermittent
E.*Two-humped
778.
With appearance of hemorrhagic syndrome at Congo fever temperature of body
always:
A.Normalize
B. Grows critically
C. *Goes down
D.Does not change
E.Grows gradually
779.
What changes in haemogram inherent Congo hemorrhagic fever?
A.Normochomic anaemia, leucocytosis mononuclear
B. Erythrocytosis, lymphocytosis
C. *Hypochromic anemia, erythrofilosis
D.Hypochromic anemia, neutrofilosis
E.Hyperchromic anemia, neutrofilosis
780.
What is typical for the Lassa hemorrhagic fever:
A.Effect of cardiovascular system
B. Development of acute hepatic insufficiency
C. Hundred-per-cent lethality
D.*Defeat of breathing organs
E.Development of paresis and paralysis
781.
Confirm diagnosis of haemorrhagic fever with kidneys syndrome by a way of:
A.Only virological methods
B. Only bacteriological methods
C. Bacteriological and serum methods
D.Proper epidemiological information
E.*Virologic and serum methods
782.
Confirm the diagnosis of Lassa hemorrhagic fever by a way of:
A.Only virological methods
B. Only bacteriological methods
C. Bacteriological and serum methods
D.Proper epidemiological information
E.*Virologic and serum methods
783.
Confirm the diagnosis of Congo hemorrhagic fever by a way of:
A.Only virological methods
B. Only bacteriological methods
C. Bacteriological and serum methods
D.Proper epidemiological information
E.*Virologic and serum methods
784.
Confirm the diagnosis of Ebola fever by a way of:
A.Growth of viruses on chicken embryons
B. Only bacteriological methods
C. Bacteriological and serum methods
D.Proper epidemiological information
E.*Selection of virus on the Vero culture
785.
Confirm the diagnosis of Omsk fever by a way of:
A.Growth of virus on chicken embryons
B. Only bacteriological methods
C. Bacteriological and serum methods
D.Proper epidemiological information
E.*Selection of virus on the Vero culture
786.
Confirm the diagnosis of Marburg fever by a way of:
A.Growth on chicken embryos
B. Only bacteriological methods
C. Bacteriological and serum methods
D.Proper epidemiologys information
E.*Selection of virus on the Vero culture
787.
What etiothropic means use at treatment of haemorrhagic fever with kidneys
syndrome:
A.Benzylpenicillin
B. Dopamine
C. *Virolex
D.Dexamethazone
E.Etamsylatum
788.
What etiothropic means use at treatment of patients with Lassa fever:
A.Benzylpenicillin
B. Dopamine
C. *Ribavirin
D.Dexamethazole
E.Etamsylatum
789.
What etiothropic means use at treatment of patients with Omsk fever:
A.Benzylpenicillin
B. Dopamine
C. *Ribavirin
D.Dexamethazone
E.Etamsylatum
790.
What etiothropic means use at treatment of patients with Marburg fever:
A.Benzypenicillin
B. Dopamine
C. *Ribavirin
D.Dexamethazone
E.Etamsylatum
791.
What etiothropic means use at treatment of patients with Congo fever:
A.Benzylpenicillin
B. Dopamine
C. *Ribavirin
D.Dexamethazone
E.Etamsylatum
792.
What etiothropic means use at treatment of patients with Ebola fever:
A.Benzylpenicillin
B. Dopamine
C. *Virolex
D.Dexamethazone
E.Etamsylatum
793.
What etiothropic means use at treatment of patients with Crimea fever:
A.Benzylpenicillin
B. Dopamine
C. *Ribavirin
D.Dexamethazone
E.Etamsylatum
794.
Specific prevention of hemorrhagic fevers:
A.The live vaccine
B. Killed vaccine
C. The specific immunoglobulin
D.*Do not developed
E.Polivalent vaccine
795.
Who is the source of the causal agent in the Crimean-Congo haemorrhagic fever?
A.Rodents, cattle, birds
B. Iksod and gamazov mites
C. *Rodents, cattle, birds, sick people
D.The sick man, reconvalenc, bacteriocarries
E.Rodents, cattle, birds, sick people, bacteriocarries
796.
The source of infection of Omsk‘s hemorrhagic fever are muskrat, water rats and
other rodents. Who are the carriers?
A.Bee and flea
B. *Pliers and flea
C. Mosquitoes
D.Fly
E.Pliers and mosquitoes
797.
Specific prevention of Crimean-Congo haemorrhagic fever are:
A.*Vaccine and human immunoglobulin
B. Serum
C. Serum and human immunoglobulin
D.Do not developed
E.Antibacterial drugs
798.
Those who have been in contact with sick haemorrhagic fevers, as well as those
who had bite by the ticks in endemic areas are introducing:
A.Specific vaccine
B. The specific immunoglobulin in doses of 10-15 ml vaccine
C. The specific immunoglobulin in doses of 10-15 ml
D.*The specific immunoglobulin in doses 5-7,5 ml
E.Nothing
799.
Hemograme in the second period of yellow fever:
A.Leukocytosis
B. Normal global analysis of blood
C. *Leukopenia, neutropenia
D.Leukopenia, neutrophilosis
E.Leukocytosis, lymphomonocytosis
800.
What ever complication meets at the yellow fever:
A.*Liver insufficiency
B. Kidney insufficiency
C. Infectious-toxic shock
D.Myocarditis
E.Edema of lungs
801.
In the initial period of hemorrhagic fever with a kidney syndrome a characteristic
sign is:
A.High temperatures
B. Pains in gastrocnemius muscles and positive Pasternatsky symptom
C. *Pains in the joints and positive Pasternatsky symptom
D.Hemorragic syndrome
E.Dyspepsia phenomena
802.
For treatment of patients with the hemorrhagic fever with a kidney syndrome do
not use:
A.Glucocorticoids
B. Anabolic steroid
C. Disintoxication facilities
D.*Dihydration facilities
E.Antihistamins
803.
For the initial period of the Congo hemorrhagic fever not characteristically:
A.Fever
B. Pains in joints and muscles
C. Severe pain of head
D.*Oliguria
E.Dizziness
804.
The most characteristic symptom in the climax period of the Congo hemorrhagic
fever is:
A.*Hemorrhagic syndrome
B. Hepatic insufficiency
C. Dyspepsia phenomena
D.Sharp kidney insufficiency
E.Мeningeal syndrome
805.
In the global analysis of blood in case of Congo hemorrhagic fever not
characteristically:
A.Leukocytosis
B. *Leukopenia
C. Neutropenia
D.Thrombocytopenia
E.Increasing of ESR
806.
What rashes in case of haemorrhagic fevers with kidneys syndrome?
A.Roseola
B. Maculo-papular
C. Punctuate
D.*Petechial
E.Rashes is not characteristic
807.
What does change in biochemical blood test in the patient with haemorrhagic
fever with kidneys syndrome?
A.Increase level of urea and bilirubin
B. The level of urea and kreatinine falls
C. The level of kreatinine grows and urea falls
D.The level of urea grows and kreatinine falls
E.*The level of urea and kreatinine grows
808.
What does change in haemogram in the patient with haemorrhagic fever with
kidneys syndrome?
A.Normochromic anaemia, leucocytosis with atypical mononucleosis,
thrombocytopenia enhanceable ESR
B. erythrocytosis, lymphocytosis,ESR is enhanceable
C. Normochromic anaemia, leucopenia with neutrophylosis, thrombocytopenia
enhanceable ESR
D.*Hypochromic anaemia, leucocytosis with neutrophylosis, thrombocytopenia
enhanceable ESR
E.Hyperchromic anaemia, leucocytosis with neutrophylosis, thrombocytopenia
mionectic ESR
809.
The temperature curve in most patients with Congo hemorrhagic fever is:
A.Wunderlich type
B. Botkin type
C. Undulating
D.Intermittent
E.*Two-humped
810.
What is typical for the Lassa hemorrhagic fever:
A.Effect of cardiovascular system
B. Development of acute hepatic insufficiency
C. Hundred-per-cent lethality
D.*Defeat of breathing organs
E.Development of paresis and paralysis
811.
What etiothropic means use at treatment of patients with Lassa fever:
A.Benzylpenicillin
B. Dopamine
C. *Ribavirin
D.Dexamethazole
E.Etamsylatum
812.
What etiothropic means use at treatment of haemorrhagic fever with kidneys
syndrome:
A.Benzylpenicillin
B. Dopamine
C. *Virolex
D.Dexamethazone
E.Etamsylatum
813.
Specific prevention of hemorrhagic fevers:
A.The live vaccine
B. Killed vaccine
C. The specific immunoglobulin
D.*Do not developed
E.Polivalent vaccine
814.
Who is the source of the causal agent in the Crimean-Congo haemorrhagic fever?
A.Rodents, cattle, birds
B. Iksod and gamazov mites
C. *Rodents, cattle, birds, sick people
D.The sick man, reconvalenc, bacteriocarries
E.Rodents, cattle, birds, sick people, bacteriocarries
815.
Phage symptom in case of yellow fever is:
A.Pain in right iliac area
B. Enanthema on a soft palate
C. *Replacement of tachicardia on expressed bradicardia
D.Hemorrhages in a conjunctiva
E.Yellow hands
816.
General view of patient with the hemorrhagic fever with a kidneys syndrome:
A.Skinning covers
B. *Pallor of nasolabial triangle, hyperemia of neck and overhead half of trunk
C. Hyperemia of person, scleritis, conjunctivitis
D.Grayish color of person
E.Icteric color of skin
817.
For confirmation of diagnosis of hemorrhagic fever with a kidney syndrome use:
A.Bacteriological method
B. Virological method
C. *Reaction of immunofluorescence
D.Reaction of braking of hemagglutination
E.Research of blood drop under a microscope
818.
For treatment of patients with the hemorrhagic fever with a kidney syndrome
does not use:
A.Corticosteroids
B. Anabolic steroids
C. Disintoxication facilities
D.*Dehydration facilities
E.Antihistaminics
819.
For the initial period of the Congo hemorrhagic fever not characteristic:
A.Fever
B. Pains in joints and muscles
C. Severe pain of head
D.*Oliguria
E.Dizziness
820.
At an objective examination for the Congo hemorrhagic fever character:
A.*Mucosal hyperemia of person
B. Pallor of person
C. Puffiness of person
D.Ochrodermia of person
E.Exanthema on face
821.
The most characteristic symptom in the climax period of the Congo hemorrhagic
fever is:
A.*Hemorrhagic syndrome
B. Hepatic insufficiency
C. Dyspepsia phenomena
D.Sharp kidney insufficiency
E.Мeningeal syndrome
822.
In the global analysis of blood in case of Congo hemorrhagic fever not
characteristic:
A.Leukocytosis
B. *Leukopenia
C. Neutropenia
D.Thrombocytopenia
E.Increasing of ESR
823.
What rashes in case of haemorrhagic fevers with kidneys syndrome?
A.Roseola
B. Maculo-papular
C. Punctuate
D.*Petechial
E.Rashes is not characteristic
824.
What rashes present in case of Congo hemorrhagic fever?
A.Roseola
B. Maculo-papular
C. Punctulate
D.*Petechial
E.Rashes not is characteristic
825.
What rashes present in case of Crimea hemorrhagic fever?
A.Roseola
B. Maculo-papular
C. Punctulate
D.*Petechial
E.Rashes not is characteristic
826.
How long the rash is present in case of hemorrhagic fever with kidneys
syndrome?
A.*During all feverish period
B. Before the convalescence
C. Before development of clinical features of kidneys insufficiency
D.During whole disease
E.Appears yet in a latent period and disappears in the period of early reconvalescense
827.
What changes in biochemical blood test inherent for hemorrhagic fever with
kidneys syndrome?
A.Increase level of urea and bilirubin
B. The level of urea and kreatinine falls
C. The level of kreatinine grows and urea falls
D.The level of urea grows and kreatinine falls
E.*The level of urea and kreatinine increase
828.
What changes in blood analysis inherent for hemorrhagic fever with kidneys
syndrome?
A.Normochromic anaemia, leucocytosis with atypical mononucleosis,
thrombocytopenia increased ESR
B. erythrocytosis, lymphocytosis,ESR is increased
C. Normochromic anaemia, leucopenia with neutrophylosis, thrombocytopenia
increased ESR
D.*Hypochromic anaemia, leucocytosis with neutrophylosis, thrombocytopenia
increased ESR
E.Hyperchromic anaemia, leucocytosis with neutrophylosis, thrombocytopenia
mionectic ESR
829.
The period of polyuria at haemorrhagic fever with kidneys syndrome is a sign of:
A.*Recovering
B. Chronic process
C. Unfavorable flow of illness
D.Development of complications
E.Complete convalescence
830.
With appearance of hemorrhagic syndrome at Congo fever temperature of body
always:
A.Normal
B. Grows critically
C. *Goes down
D.Does not change
E.Grows gradually
831.
What changes in blood analysis inherent at Congo hemorrhagic fever?
A.Normochromic anaemia, leucocytosis mononuclear
B. Erythrocytosis, lymphocytosis
C. *Hypochromic anemia, erythrophilosis
D.Hypochromic anemia, neutrophilosis
E.Hyperchromic anemia, neutrophilosis
832.
What is typical for the Lassa hemorrhagic fever:
A.Effect of cardiovascular system
B. Development of acute hepatic insufficiency
C. Hundred-per cent lethality
D.*Defeat of breathing organs
E.Development of paresis and paralysis
833.
Confirm the diagnosis of hemorrhagic fever with kidneys syndrome by a way of:
A.Only virological methods
B. Only bacteriological methods
C. Bacteriological and serum methods
D.Proper epidemiological information
E.*Virologic and serum methods
834.
Confirm the diagnosis of Lassa hemorrhagic fever by a way of:
A.Only virological methods
B. Only bacteriological methods
C. Bacteriological and serum methods
D.Proper epidemiological information
E.*Virologic and serum methods
835.
Confirm the diagnosis of Congo hemorrhagic fever by a way of:
A.Only virological methods
B. Only bacteriological methods
C. Bacteriological and serum methods
D.Proper epidemiological information
E.*Virologic and serum methods
836.
Confirm the diagnosis of Ebola fever by a way of:
A.Growth of viruses on chicken embryos
B. Only bacteriological methods
C. Bacteriological and serum methods
D.Proper epidemiological information
E.*Selection of virus on the Vero culture
837.
Confirm the diagnosis of Omsk fever by a way of:
A.Growth of virus on chicken embryons
B. Only bacteriological methods
C. Bacteriological and serum methods
D.Proper epidemiological information
E.*Selection of virus on the Vero culture
838.
The measures of urgent prophylaxis of plague.
A.Administration of human immunoglobulin
B. Chlorochin (delagil) 0,25 g 2 times in week
C. *6-day’s prophylaxis with streptomycin or tetracycline
D.In first 5 days intake antibiotics of penicillin or tetracycline origin
E.Іnterferon
839.
The rules of hospitalization of patients with plague:
A.To separate ward
B. To ward for respiratory infections
C. *To ward cubicle
D.Patient’s are not hospitalized
E.To ward for intestinal infections
840.
Patient T., drives in a country unhappy on a plague. Conduct measures on a
specific prophylaxis.
A.Human immunoglobulin
B. Interferon
C. Bacteriophage
D.*Dry living vaccine
E.Live measles vaccine
841.
842.
843.
844.
845.
846.
847.
848.
Preparations for urgent prophylaxis of plague:
A.Injection of human immunoglobulin
B. *Streptomycin or tetracycline
C. Human immunoglobulin
D.Dry living vaccine or tetracycline generations.
E.Interferon
Y. pestis is transmitted more frequently by:
A.*Flea
B. Water
C. Air
D.Food storage
E.Tick
The duration of incubation period of plague is:
A.3 to 8 days;
B. 2 to 12 days;
C. 2 to 10 days;
D.1 to 8 days.
E.*2 to 6 days;
What is the main feature of septicemic plague?
A.*Massive bacteriemia
B. Headache
C. Pain in the abdominal
D.Throat ache
E.Bleeding
What drug is first step of choice for the treatment of plague?
A.Amoxicillin
B. *Streptomycin
C. Penicillin
D.Biseptol
E.5-NOK
What is the treatment of patients with a plague:
A.Immediately after hospitalization
B. *Immediately after hospitalization, carrying out only material for research
C. After raising of final diagnosis
D.After laboratory and instrumental diagnostics
E.All answers are faithful
How many pandemics of plague was in history of mankind?
A.*Three
B. Four
C. One
D.Two
E.Five
Especially dangerous for surroundings are patients with:
A.Skin form of plague
B. *Pulmonary form of plague
C. Skin-bubonic form of plague
849.
850.
851.
852.
853.
854.
855.
856.
D.Bubonic form of plague
E.Septic form of plague
Risk group of plague infection the most frequent is:
A.Doctors
B. *Hunters
C. Alcoholic
D.Drug users
E.Prostitutes
Who is the reservoir of causative agent of plague in nature?
A.Birds
B. Insects
C. Fresh-water fish
D.*Rodents
E.Cattle
What is the susceptibility of human to plague?
A.Non susceptible
B. 50 %
C. *Almost 100 %
D.10 %
E.70 %
The causative agent of plague is:
A.*Yersinia pestis
B. Yersinia enterocolitica
C. Yersinia pseudotuberculosis
D.Bac. anthracis
E.Pseudomonas mallei
The duration of incubation period at plague is:
A.2-6 h
B. *2-6 d
C. 10-15 d
D.17-21 d
E.1-6 w
To the localized forms of plague belong:
A.Secondary-septic
B. Primary-septic
C. *Skin
D.Primary-pulmonary
E.Intestinal
To the localized forms of plague belong:
A.Intestinal
B. Primary-septic
C. Secondary-septic
D.Primary-pulmonary
E.*Skin-bubonic
To the localized forms of plague belong:
A.Intestinal
857.
858.
859.
860.
861.
862.
863.
864.
B. Primary-septic
C. Secondary-septic
D.Primary-pulmonary
E.*Bubonic
To the internal-disseminated forms of plague belong:
A.*Primary-septic
B. Bubonic
C. Secondary-pulmonary
D.Primary-pulmonary
E.Intestinal
Choose the specific treatment of tetanus.
A.Antibiotics
B. *Serum
C. Anticonvulsant medicine
D.Cardiac preparations
E.Desintoxication therapy
The exciter of tetanus is:
A.*Clostridia
B. Escherichia
C. Candida albicans
D.Neisseria
E.Gonococcus
For the exciter of tetanus characteristic such properties, except:
A.Formation of exotoxins
B. Ability to propagate in anaerobic conditions
C. Formation of spores
D.*Formation of gametes
The best terms of tetanus exciter cultivation:
A.*Anaerobic conditions
B. Oxygen supply
C. Presence of animal albumen in nutritive medium
D.Low temperature
E.1 % peptone water
Vegetative form of exciter of tetanus is destroyed in such terms, except for:
A.At a temperature of 100 °C
B. *At room temperature
C. Under action of carbolic acid
D.Under the action of oxygen
Who is the source of tetanus?
A.Sick person
B. Rodents
C. *Soil
D.Insects
E.Cattle
The spores of tetanus are saved:
A.After boiling during 1 hour
B. Under act of dry air at the temperature of 115 degrees C
C. *In soil during many years
D.In 1 % solution of formalin during 6 hours
865.
Tetanus toxin consists of all units among the listed below, except:
A.Tetanospasmin
B. Tetanolysin
C. Exotoxin
D.Low-molecular fraction
E.*Enterotoxin
866.
What is the receptivity of population to the tetanus?
A.0 %
B. 50 %
C. *Almost 100 %
D.10 %
867.
Causing of tetanus are:
A.*C. tetani
B. E. coli
C. Candida
D.Epstein-Barr virus
E.Hemolytic streptococcus group A
868.
Duration of the latent period in case of tetanus:
A.1-6 hours
B. 1-4 days
C. 5-14 days
D.*1-6 weeks.
E.1-6 months
869.
How long does the incubation period of tetanus last?
A.1-5 days
B. 5-10 days
C. 3-5 days
D.*5-14 days
E.15-20 days
870.
Tetanus might appear in case of:
A.*Trauma
B. Mosquito bite
C. Usage of stranger clothes
D.Contact with the sick people
E.Drink the water with poor quality
871.
Tetanus might appear in case of:
A.*Dog bite
B. Mosquito bite
C. Usage of stranger clothes
D.Contact with the sick people
E.Drink the water with poor quality
872.
What is the medical tactic development of the severe tetanus after criminal
abortion?
A.Anticonvulsant preparations
B. Revision of the uterus cavity
C. Analgesic therapy
D.Antibiotics
E.*All answers are correct
873.
Choose dose of the specific treatment for patients with tetanus.
A.600 units/kg of antytetanus serum
B. 900 units/kg of antytetanus Ig
C. 500 units/kg of antytetanus Ig
D.900 units/kg of antytetanus serum
E.*500 units/kg of antytetanus serum
874.
What is the first aid preparation for the patient with tetanus?
A.Glucocorticoids
B. Analgetics
C. *Anticonvulsant medicine
D.Surgical treatment of the wound
E.Oxygen therapy
875.
Among the listed below choose the complication of the tetanus, which is not
early:
A.Tracheobronchitis
B. *Contracture of muscles and joints
C. Asphyxia
D.Myocarditis
E.Pneumonia
876.
Among the listed below choose the complication of the tetanus, which is not
early:
A.Tracheobronchitis
B. *Compressive deformation of the spine
C. Asphyxia
D.Myocarditis
E.Pneumonia
877.
Specify the measures of urgent prophylaxis of anthrax.
A.Anti-anthrax immunoglobulin
B. *Penicillinum or tetracyclinum during 5 days
C. Vaccination
D.Medical supervision
E.Biseptolum 5 days
878.
What specific test is used for anthrax diagnostic?
A.Compliment fixation test
B. Indirect hemaglutination test
C. *Coetaneous test with antraxin
D.Hemaglutination test
E.RIFA with anthrax antigen
879.
What anthrax prophylactic measures are entertained by farm workers?
A.Vitamin therapy
B. Immunization by inactivated vaccine
880.
881.
882.
883.
884.
885.
886.
887.
C. Formulated vaccine
D.*Immunization by live vaccine
E.Antibiotic therapy
The etiological factor of anthrax is:
A.Salmonella thyphi
B. Erysipelothrix rhysiopothiac
C. *Bacillus anthracis
D.Rickettsiosis sibirica
E.Toxocara canis
The source of infection of anthrax is more frequent than all:
A.People
B. Birds
C. *Home animals
D.Rodents
E.Fly
Mechanism of transmission of anthrax are:
A.Contact
B. Alimentary
C. Air-droplets’
D.Transmissiv
E.*All above it
What organ damaged more frequent than all in patients with anthrax?
A.*Skin
B. Lights
C. Gastrointestinal tract
D.Lymphatic system
E.Nervous system
The basic clinical display of a skin form of anthrax is:
A.Hyperemic of skins
B. Vesiculs
C. *Ulcer
D.Phlegmon
E.Abscess
For anthrax most characteristically:
A.Change of stool
B. Icterus of skin
C. Catarrhal phenomena
D.Meningeal phenomena
E.*Change of skin
For a skin form of anthrax the most characteristically:
A.Hyperemia
B. Painful carbuncle
C. *Not painful carbuncle
D.Painful noodles
E.Vesicles and bulls
For anthrax carbuncle the most characteristically:
888.
889.
890.
891.
892.
893.
894.
A.Ulcer with a festering bottom, roller on periphery and insignificant area of edema
B. Ulcer with hyperemia on periphery without an edema
C. *Ulcer with a black scab, black color, second vesicles and area of edema around of
ulcer
D.Ulcer with a festering bottom, roller on periphery, second vesicles and area of
edema
E.Ulcer with serosis-hemorrhagic exudates, painful, with the area of edema around of
ulcer
Symptom of Stefansky – it is:
A.Enantema on a soft palate
B. Enantema on a conjunctiva
C. Shaking of tongue at an attempt to put out a tongue
D.*Shaking of edema like to jelly at pattering a hammer in the area of edema
E.Painful of stomach in a right iliac area
For the pulmonary form of anthrax characteristically:
A.*Foamy sputum with blood
B. Glassy sputum with blood
C. Foamy sputum without blood
D.Foamy green sputum
E.Like to «ferruginous» sputum
With what diseases it is necessary to differentiate anthrax:
A.Leptospirozis
B. Typhoid fever
C. Dermatitis
D.*Carbuncle
E.Meningococcal infection
What material is necessary take for diagnosis of anthrax:
A.Spinal liquid
B. Urine
C. Saliva
D.*Content of carbuncle
E.Nose swab
The diagnostic reaction of anthrax is:
A.Rayt‘s reaction
B. Vidal‘s reaction
C. *Reaction of term precipitation of Askoly
D.Paul-Bunnel‘s reaction
E.Reaction of agglutination-lysis
The diagnostic endermic reaction of anthrax take:
A.*Antraksin
B. Dizenterin
C. Ornitin
D.Malein
E.Brucellin
For treatment of anthrax is:
A.Sulfanilamids
895.
896.
897.
898.
899.
900.
901.
902.
B. Nitrofurans
C. Hormones
D.Antiviral facilities
E.*Antibiotics
It is necessary to appoint for successful treatment of anthrax:
A.*Antyanthrax immunoglobulin and penicillin
B. Antyanthrax immunoglobulin and prednizolon
C. Antyanthrax immunoglobulin and vyrolex
D.Antyanthrax immunoglobulin and vermox
To what group of infections does the rabies belong?
A.*Zoonosis
B. Anthroponosis
C. Capronosis
D.Anthropozoonosis
E.Caprozoonosis
The basic reservoir of rhabdovirus is:
A.Pisces
B. Reptiles
C. Birds
D.Weed-eaters
E.*Carnivores
Rhabdovirus from an organism of the patient or animal is revealed to the flow:
A.Last 20 days of latent period and during all the illness
B. *Last 7-10 days of latent period and during all the illness
C. Last 7-10 days of latent period
D.Last 7-10 days of latent period and at the beginning of illness
E.During all the illness
You may be infected a rhabdovirus in case of:
A.*Bite +salivation to the skin by an animal
B. Infected meal
C. Infected water
D.Contact with the infected air
E.Bite with the infected insect
What is the sensitivity to the rabies?
A.45 %
B. 25 %
C. 85 %
D.*100 %
E.10 %
What is the mechanism of transmission of rabies?
A.Transmissive
B. Fecally-oral
C. Air drop
D.*Wound
E.Domestic contact
What is the main mechanism of transmission of rabies?
903.
904.
905.
906.
907.
908.
909.
A.*Airborne
B. Alimentary
C. Contact
D.Transmisiv
E.Vertical
Mechanism of transmission of rabies are often:
A.*Air
B. Contact
C. Transmissiv
D.Fecal-oral
E.Transplacental
What is the entrance for the rabies?
A.*Damaged skin and mucous tissues
B. Respiratory tracts
C. Family ways
D.Gastrointestinal tract
E.Blood
Causing of rabies are:
A.C. tetani
B. E. coli
C. Candida
D.Epstein-Barr virus
E.*Rabdovirus
Duration of the latent period in case of tetanus:
A.1-6 hours
B. 7-14 days
C. *7 days – 1 ear
D.1-6 weeks.
E.1-6 months
What periods of rabies do you know?
A.Incubation, depressions, excitation
B. *Incubation, depressions, excitation, paralytic
C. Depression, excitation, paralytic
D.Incubation, excitation, paralytic
E.Incubation, depressions, paralytic
For rabies the source of infection can be a dog bite in all the cases, except for:
A.With rabies
B. Suspicion on rabies
C. Vagrant
D.*Month prior to the disease
E.Last 10 days before the disease
In rabies the source of the virus can be:
A.Wild animals
B. Home animals
C. Bats
D.Rodents
910.
911.
912.
913.
914.
915.
916.
917.
E.*All the answers are correct
What is the duration of the prodromal period for the rabies?
A.*1-3 days
B. Up to 1 day
C. 4-7 days
D.3-4 days
E.3-5 days
The first symptom of prodromal period of rabies is:
A.Cough
B. Nausea
C. Vomiting
D.Diarrhea
E.*Slight swelling and erethema of the scar
The first symptom of prodromal period of rabies is:
A.Cough
B. Nausea
C. Vomiting
D.*Neurological pains in motion nervous barrels, the nearest to the place of bite
E.Diarrhea
The first symptom of prodromal period of rabies is:
A.Cough
B. Nausea
C. Vomiting
D.*Apathy and depression
E.Diarrhea
Most characteristic symptoms of the rabies are:
A.*Paroxysm of hydrophobia
B. Apathy and depression
C. Neuralgic pains on motion nervous barrels, the nearest to the place of bite
D.Dyspepsia disorders
E.Catarrhal phenomen
Most characteristic symptoms of the rabies are:
A.*Paroxysm of hydrophobia
B. Paroxysm of aerophobia
C. Paroxysm of fotophobia
D.Paroxysm of akuzophobia
E.Paroxysm of soilphobia
What is the duration of excitation period of rabies?
A.7-10 days
B. 24 hours
C. *2-3 days, sometimes to 6 days
D.Not more than 2 days
E.Up to 6 hours
What temperature of the body is typical for the paralytic period?
A.*Hyperpyrexia
B. Hypothermia
C. High
D.Normal
E.Subfebril
918.
Who of the listed below persons must take the conditional course of inoculations
against rabies?
A.*A teenager bitten by a dog which is on a leash, not instilled
B. Man bitten by a fox which perished
C. A child, scratched by a squirrel which disappeared in-field
D.A woman, bitten by a cat ill with rabies
E.Man, who had a meal of undercooked of animal with rabies
919.
When from the beginning of vaccination an antibodies to the rhabdovirus
appear?
A.In a week
B. *In 2 weeks
C. In a month
D.After half of year
E.Don’t produced
920.
For what infectious pathology is characterized Babesh-Negri‘ bodies?
A.Poisoning mushrooms
B. Meningo-encefalit
C. Poliomyelitis
D.*Rabies
E.Tetanus
921.
Dog bite man on foot. What kind of specific prophylaxis should be conducted for
this patient?
A.Human rabies immunoglobulin 6 doses of antirabies vaccine
B. 12 doses of antirabies vaccine
C. Human rabies immunoglobulin and 6 doses of antirabies vaccine
D.Human rabies immunoglobulin and 21 dose of antirabies vaccine
E.*6 doses of antirabies vaccine
922.
What kind of specific prophylaxis should be conducted for patient with bitten
foot?
A.Human rabies immunoglobulin
B. *Vaccine antirabies
C. Vaccine antirabies and rabies immunoglobulin
D.Human rabies immunoglobulin and serum
E.Human rabies serum
923.
How are the little bodies named (in patients with rabies)?
A.Lorin-Epshteyn
B. Blyumberg
C. Murson
D.Rozenberg
E.*Babesh-Negri
924.
Where are the little bodies of Babesh-Negri?
A.In lungs
B. In liver
925.
926.
927.
928.
929.
930.
931.
932.
C. In a spinal cord
D.*In neurons
E.In blood
Rabies might appear in case of:
A.*Dog bite
B. Mosquito bite
C. Usage of stranger clothes
D.Contact with the sick people
E.Drink the water with poor quality
Select the correct rabies vaccination scheme:
A.0, 3, 6, 14, 20, 90 days
B. 0, 4, 7, 14, 50, 90 days
C. 0, 5, 8, 14, 30, 100 days
D.*0, 3, 7, 14, 30, 90 days
E.0, 1, 5, 13, 60, 90 days
The exciter of tetanus is:
A.*Clostridia
B. Escherichia
C. Candida albicans
D.Neisseria
E.Gonococcus
For the exciter of tetanus characteristic such properties, except:
A.Formation of exotoxins
B. Ability to propagate in anaerobic conditions
C. Formation of spores
D.*Formation of gametes
E.Gram positive
The best terms of tetanus exciter cultivation:
A.*Anaerobic conditions
B. Oxygen supply
C. Presence of animal albumen in nutritive medium
D.Low temperature
E.1 % peptone water
Vegetative form of exciter of tetanus is destroyed in such terms, except for:
A.At a temperature of 100 °C
B. *At room temperature
C. Under action of carbolic acid
D.Under the action of oxygen
E.Under action of antibiotics
Who is the source of tetanus?
A.Sick person
B. Rodents
C. *Soil
D.Bacteriocarrier
E.Sick person and bacteriocarrier
The spores of tetanus are saved:
933.
934.
935.
936.
937.
938.
939.
A.After boiling during 1 hour
B. Under act of dry air at the temperature of 115 degrees C
C. *In soil during many years
D.In 1 % solution of formalin during 6 hours
E.All answers are correct
Tetanus toxin consists of all units among the listed below, except:
A.Tetanospasmin
B. Tetanolysin
C. Exotoxin
D.Low-molecular fraction
E.*Enterotoxin
Mechanism of transmission in case of tetanus are:
A.Intra muscular conduction
B. *Contact
C. Insect conduction
D.Faecally-oral
E.Vertical conduction
What is the receptivity of population to the tetanus?
A.0 %
B. 50 %
C. *Almost 100 %
D.10 %
E.70 %
Causing of tetanus are:
A.*C. tetani
B. E. coli
C. Candida
D.Epstein-Barr virus
E.Hemolytic streptococcus group A
Duration of the latent period in case of tetanus:
A.1-6 hours
B. 1-4 days
C. *5-14 days
D.1-6 weeks.
E.1-6 months
How long does the incubation period of tetanus last?
A.1-5 days
B. 5-10 days
C. 3-5 days
D.*5-14 days
E.15-20 days
Tetanus might appear in case of:
A.*Trauma
B. Mosquito bite
C. Usage of stranger clothes
D.Contact with the sick people
E.Drink the water with poor quality
940.
Tetanus might appear in case of:
A.*Dog bite
B. Mosquito bite
C. Usage of stranger clothes
D.Contact with the sick people
E.Drink the water with poor quality
941.
What is the medical tactic development of the severe tetanus after criminal
abortion?
A.Anticonvulsant preparations
B. Revision of the uterus cavity
C. Analgesic therapy
D.Antibiotics
E.*All answers are correct
942.
What measures should be taken in relation to contact persons in case of tetanus?
A.Vaccination
B. Isolation of contacts
C. Chemoprophylaxis
D.Laboratory inspection
E.*They need no measures
943.
Among the listed below what preparations are not etiological for tetanus?
A.AC-anatoxin
B. Medical horse serum
C. Human immunoprotein
D.*Anticonvulsant preparations
E.Penicillin
944.
Choose dose of the specific treatment for patients with tetanus.
A.500 international units of antytetanus Ig
B. 500 international units of antytetanus serum
C. *900 international units of antytetanus Ig
D.900 international units of antytetanus serum
E.900 units/kg of antytetanus serum
945.
Which early complications occurs in tetanus?
A.Tracheobronchitis
B. Asphyxia
C. Myocarditis
D.Pneumonia
E.*All the above
946.
Which late complications occurs in tetanus?
A.Contracture of muscles and joints
B. Compressive deformation of the spine
C. Asthenic syndrome
D.Chronic heterospecific diseases of lungs
E.*All the above
947.
What is the duration of outpatient supervision for patients, recovered of tetanus?
A.*2 years
B. 3 months
C. 1 month
D.For the decreed groups of population for life time
E.There is no such supervision at all
948.
Urgent immuno prophylactic of tetanus in the case of trauma should be
conducted in such period:
A.25 days from the moment of trauma
B. 30 days from the moment of trauma
C. In the first 10 days from the moment of trauma
D.*At once after the trauma
E.Not mentioned
949.
Among the listed below people who should receive an immediate prophylactic of
the tetanus in form of AC-anatoxin and AC IP injections after trauma?
A.Man of 40 years, in anamnesis with 1 inoculation one year ago
B. Pregnant woman of 30 years, in the second half of pregnancy
C. Child, 7 months, instilled according to a calendar
D.*Retire man of 57 years, who is not instilled
E.Child of 6 years, instilled according to a calendar
950.
In case of tetanus the epidemiological measures are directed on:
A.Elimination of the source of tetanus
B. Treatment of the source of tetanus
C. *Specific prophylaxis
D.Medicines prophylactics
E.Nothing should be performed
951.
At what infectious disease does conduct the spasm almost always commences in
the muscles of the neck and jaw. causing closure of the jaws?
A.Poisoning mushrooms
B. Meningoencefalitis
C. Poliomyelitis
D.Rabies
E.*Tetanus
952.
For what disease is characterized this symptom (the generalized spasm of soft
muscles, flexion of the arms and extension of the legs)?
A.Poliomyelitis
B. Brucellosis
C. Pseudo tuberculosis
D.*Tetanus
E.Hydrophobia
953.
For what disease is characterized opistotonus?
A.Poliomyelitis
B. Brucellosis
C. Pseudo tuberculosis
D.*Tetanus
E.Hydrophobia
954.
For what disease is characterized rizos sardonicus?
A.Poliomyelitis
955.
956.
957.
958.
959.
960.
961.
B. Brucellosis
C. Pseudo tuberculosis
D.*Tetanus
E.Hydrophobia
For what disease is characterized lockjaw?
A.Poliomyelitis
B. Brucellosis
C. Pseudo tuberculosis
D.*Tetanus
E.Hydrophobia
what is name of symptom characterized to the patient with tetanus?
A.*Lorin-Epshteyn
B. Blyumberg
C. Murson
D.Rozenberg
Name the HIV infection high risk groups :
A.Homo- and bisexual, prostitutes and other persons who conduct disorderly sexual
life
B. Drug addicts who enter drugs parenterally
C. Only recipeint of blood, its preparations, sperm and organs
D.Only patients with venereal diseases and parenteral viral hepatitis and from the
HIV infected mothers
E.*All the above
What is the most effective methods of HIV prevention:
A.Vaccination and immunoprotein
B. Chemoprophylactic
C. solation of patients
D.*Safe sex and prevention of drug addiction
E.Disinfection
How many types of HIV are known?
A.One
B. *Two
C. Three
D.Four
E.Five
When HIV/AIDS agent was discovered?
A.1981
B. 1982
C. *1983
D.2002
E.2003
Name the main specific method of HIV diagnosis?
A.RPGA
B. PLR
C. *IFA and ELISA
D.Bioassey
E.RIA
962.
Name the most dangerous parenteral way of infection of HIV/AIDS?
A.*Infusion of donor blood and its preparations
B. Transplantation of organs
C. Injections of medications
D.Diagnostic manipulations
E.Cosmetic manipulations
963.
Name the source of exciter of HIV infection/AIDS?
A.*Human
B. Animals
C. Poultries
D.Amphibious
E.Fish
964.
What is the basic way of transmission of HIV infection:
A.Air-born
B. Alimentary
C. *Parententeral
D.Transmissive
E.Water
965.
What humans cell of body is a target of HIV?
A.Erythrocytes
B. Neutrophyl leucocytes
C. Monocytes
D.T-killer-lymphocytes
E.*T-cell helpers
966.
What cellular receptors of human attract HIV?
A.*CD4
B. CD8
C. CD95
D.CD40
E.CD3
967.
What clinical features of sarcoma Kaposhi in patients with AIDS?
A.Strike the persons of young and middle age
B. Primary elements appear on a head and trunk
C. Elements with necrosis and ulceration
D.Metastasis in internal organs and high lethality
E.*All the above
968.
What family of viruses the exciter of HIV/AIDS belong to?
A.Orto- and paramyxovirus
B. Rabdovirus
C. *Retrovirus
D.Herpesvirus
E.Reovirus
969.
What group of infectious diseases an exciter of HIV infection/AIDS belong to
according to L. Gromashevsky classification?
A.Intestinal infection
B. Infections of respiratory tract
C. Blood infection
D.*Infection of external covers
E.Transmissive
970.
What group of infectious diseases the exciter of HIV/AIDS belong to?
A.*Antroponozis
B. Zoonosis
C. Sapronosis
D.Saprozoonosis
E.Zooantroponosis
971.
What sexual contact are the most dangerous in relation to an infection with HIV?
A.Vaginal
B. *Anal
C. Oral
D.Lesbian
E.Artificial impregnation
972.
Intravenous introduction of drugs, transfusion of blood or blood products,
because of the risk of transmitting BICH should be conducted, except for:
A. For health reasons
B. By decision of the consilium
C. Agreement of the patient or his relatives
D. A careful selection of donors
E.* There are no restrictions
973.
What dose of antiretroviral drugs for HIV prevention after contact of person with
blood and body fluids?
A.600-800 mg / day
B. 700-800 mg / day
C. * 800-1000 mg / day
D.. 1000-1100 mg / day
E.. 1100-1200 mg / day
974.
How long held antiretroviral prophylaxis regimen after contact with blood and
other body fluids?
A.1 Week
B. B. 2 weeks
C. * 1 month
D. 3 months
E. 6 months
975.
HIV-infected person is dangerous for others:
A. Only in symptomatic period
B. Only in the stage of acute infection
C. Only in the stage of asymptomatic infection
D. Only in the terminal stage
E.. * During Lifetime
976.
Select an indication for post-exposure prophylaxis of HIV:
A. * Medical accident with HIV-infected patient
B. HIV-infected pregnant
C. HIV infection
D. AIDS
E. All the above listed
977.
Select an indication for post-exposure prophylaxis of HIV:
A. AIDS
B. HIV-infected pregnant
C. HIV infection
D. * Childbirth by HIV-infected mother
E. All the above listed
978.
Select an indication for post-exposure prophylaxis of HIV:
A. Medical accident with HIV-infected patient
B. Delivery a baby by HIV-infected mother
C. Rape
D. Transfused
E. * All the above listed
979.
Select an indication for post-exposure prophylaxis of HIV:
A. AIDS
B. HIV-infected pregnant
C. HIV infection
D. * Rape
E. All the above listed
980.
Select an indication for post-exposure prophylaxis of HIV:
A.. * Medical accident with HIV-infected patients
B. HIV-infected pregnant
C. HIV infection
D. AIDS
E.All the above listed
981.
Select an indication for post-exposure prophylaxis of HIV:
A.AIDS
B. HIV-infected pregnant
C. HIV infection
D.* Childbirth from HIV-infected mother
E.All the above listed
982.
Select an HAART drugs used in the treatment of patients with AIDS except:
A.. * Selection of the drug according to the antibiogram
B. . Nucleoside reverse transcriptase inhibitors HIV
C. . NNRTIs
D.. protease Inhibitors
E.. integrase inhibitors
983.
Terms of prophylaxis with antiretroviral drugs after exposure with blood and
body fluids?
A. * No later than 72 hours
B. During a week
C. If the infection is confirmed
D.. Dont perform
E.. To seropositive persons
984.
If the specific markers HIV identified the victim as a result of an accident during
the medical examination in the first 5 days after the accident, it means:
A. Occupational exposure
B. The patient is in the incubation period of HIV infection
C. * The patient was HIV-positive before the accident
D. Seroconversion after crash
E. Does not mean anything
985.
Increased risk of infection HIV, except:
A. Gays
B. Addicts
C. . Patients after transplantation of organs and tissues
D. Paramedics
E. * Donors of blood and organs
986.
Increased risk of HIV infection , except:
A. Homosexuals
B. Addicts
C. . sex partners of HIV infected
D. paramedics
E.* Persons living in the same apartment
987.
Increased risk of infection HIV, except:
A. Persons entering into promiscuity
B. Addicts
C. Hemophiliacs
D. Children from HIV-infected mothers
E. * Donors of blood and organs
988.
Increased risk of infection HIV, except:
A. Homosexuals
B. Addicts
C. sex partners of HIV infected
D. paramedics
E. * Persons living in the same apartment
989.
Increased risk of infection BICH:
A. Homosexuals
B. Addicts
C. Hemophiliacs
D. Children from HIV iinfected mothers
E. * all the above listed
990.
For the diagnosis of Kaposi's sarcoma in AIDS should be:
A. The presence of tumor formation or brown cherry color of the skin or mucous
membranes
B. Affects young men
C. Frequent ulceration, bleeding
D. histologic confirmation
E. * all the above listed
991.
To perform highly active antiretroviral therapy, select the correct combination of
drugs:
A. 2 NRTIs 2 SP
B. * 2 NRTIs + 1 NNRTI
C. 1 + 2 NRTIs, NNRTIs
D. 1 NRTI + 1 PI 2 NNRTI
E. 1 2 NRTI PI
992.
To perform highly active antiretroviral therapy, select the combination of drugs:
A. 1 2 NRTI PI
B. 1 + 2 NRTIs, NNRTIs
C. * 3 NRTIs
D. 1 NRTI + 1 PI 2 NNRTI
E. 2 NNRTI 2 SP
993.
For post-exposure prophylaxis of HIV infection select the correct combination
of drugs:
A. 1 2 NRTI PI
B. 1 + 2 NRTIs, NNRTIs
C. * 2 NRTIs + 1 NNRTI
D. 1 NRTI + 1 PI 2 NNRTI
E. 2 NNRTI 2 SP
994.
For post-exposure prophylaxis of HIV iinfection select the correct combination
of drugs:
A. 1 2 NRTI PI
B. 1 + 2 NRTIs, NNRTIs
C. * 2 NRTIs + 1 NNRTI
D. 1 NRTI + 1 PI 2 NNRTI
E. 2 NNRTI 2 SP
995.
What is characteristic of Kaposhi's sarcoma in AIDS except:
A. common malignancy
B. Affects young men
C. frequent bleeding
D.* Localization only on legs and feet
E. frequently ulcerate
996.
What is characteristic of Kaposhi's sarcoma in AIDS except:
A. Affects young people
B. * It affects only older men
C. widespread localization
D.malignancy
E. frequently ulcerate
997.
What is characteristic of Kaposhi's sarcoma in AIDS , except:
A. * The current benign
B. Affects young men
C. frequent bleeding
D. Visceral
E. frequently ulcerate
998.
HIV contamination may be considered professional if specific markers of the
virus identified the victim as a result of an accident during the medical examination at such
times, except:
A. * The first 5 days after the accident
B. 1 month
C. 3 months
D. after 6 months
E. 1 year
999.
The most important clinical criteria of AIDS should include:
A. Significant weight loss (10 %) if it continues over a month
B. Duration of fever, if it continues over a month
C. Pillar, diarrhea, if it lasts more than a month
D. Lymphadenopathy than 3 months
E.*All the above
1000.
The most important clinical criteria of AIDS must include the following, except:
A. Significant weight loss (10 %) if it continues over a month
B. Duration of fever, if it continues over a month
C. Pillar, diarrhea, if it lasts more than a month
D.* Rush on the skin, if it lasts more than a month
E.Lymphadenopathy longer than 3 months.
1001.
. The most important clinical criteria of AIDS should include:
A.Significant weight loss (10 %) if it continues over a month
B. Duration of fever, if it continues over a month
C. Pillar, diarrhea, if it lasts more than a month
D. * Lymphadenopathy than 3 months
E. All the above
1002.
How to decide the child's immunizations in case of AIDS ?
A.Vaccination against diphtheria and tetanus
B. . Vaccination against polio
C. Vaccination against tuberculosis
D. Vaccination against polio and tuberculosis
E. * Vaccination do not conduct
1003.
How to decide on the child's immunizations with established HIV infection?
A.* Vaccination against diphtheria and tetanus
B. Vaccination against polio
C. Vaccination against tuberculosis
D. Vaccination against polio and tuberculosis
E. Vaccination not conduct
1004.
What antiretroviral drugs may prescribed for HIV-infected pregnant women in
order to reduce the risk of infection of the fetus?
A. Zidovudine and lopinavir
B. * Zidovudine and Viramune
C. . Timazid and Retrovir
D.AZT and Retrovir
E.. Zidovudine, Retrovir and timazid
1005.
What antiretroviral drugs may prescribed for HIV-infected pregnant women in
order to reduce the risk of infection of the fetus?
A. * Zidovudine and lopinavir
B. Retrovir and Viramune
C. Timazid and Retrovir
D.AZT and Retrovir
E.Zidovudine, Retrovir and timazid
1006.
What antiretroviral drugs may prescribed for HIV-infected pregnant women in
order to reduce the risk of infection of the fetus?
A.Zidovudine and lopinavir
B. * Zidovudine and Nevirapine
C. Timazid and Retrovir
D.AZT and Retrovir
E.Zidovudine, Retrovir and timazid
1007.
What additional testing it is necessary to patients with infectious mononucleosis?
A.* IFA for BICH infection, bakobsledovanie of diphtheria
B. IFA for BICH infection, tularemia on bakobsledovanie
C. Bacteriological tests of diphtheria and typhoid fever
D. Reaction Burne and Wright-Heddlsona
E. The reaction of Paul Bunnelya and lymph node puncture
1008.
What immunological changes are identified in patients with HIV infection?
A. Is a polyclonal B-cell inhibition, increases the formation of autoantibodies and
immune complexes
B. Is a polyclonal B-cell activation, reduced the formation of autoantibodies and
immune complexes
C. Is a polyclonal B-cell inhibition, decreases the formation of autoantibodies and
immune complexes
D.* There is a polyclonal B-cell activation, increases the formation of autoantibodies
and immune complexes
E.Is a polyclonal B-cell activation, increases the formation of autoantibodies and
decreases the formation of immune complexes
1009.
What laboratory findings are not typical for AIDS-related complex?
A.* Decreasing of the level of circulating immune complexes
B. The aspect ratio of 1.0 CD4/CD8
C. Anemia
D.Leukopenia
E.Thrombocytopenia
1010.
What laboratory findings are not typical for AIDS-related complex?
A.Increasing levels of circulating immune complexes
B. The aspect ratio of 1.0 CD4/CD8
C. Anemia
D.* Leukocytosis
E.Thrombocytopenia
1011.
What laboratory findings are not typical for AIDS-related complex?
A. * Decrease of immunoglobulins A is the G.
B. Decreasing CD4/CD8 ratio below 1.0
C. Anemiya
D.Leukopenia
E.Increase levels of circulating immune complexes
1012.
What laboratory findings are not typical for AIDS-related complex
A.* Decrease of immunoglobulins A is the G.
B. . Decreasing CD4/CD8 ratio below 1.0
C. Anemiya
D. Leukopenia
E. Increased levels of circulating immune complexes
1013.
What laboratory findings are not typical for AIDS-related complex?
A.Increased levels of circulating immune complexes
B. The aspect ratio of 1.0 CD4/CD8
C. Anemiya
D. * Leukocytosis
E. Thrombocytopenia
1014.
What laboratory findings are not typical for AIDS-related complex?
A. * Decrease in the level of circulating immune complexes
B. The aspect ratio of 1.0 CD4/CD8
C. Anemiya
D. leukopenia
E. thrombocytopenia
1015.
What antiretroviral drug exposure prophylaxis is carried out after contact with
blood and other body fluids?
A.azidothymidine
B. nevirapine
C. * Azidothymidine + nevirapine
D.AZT + saquinavir
E.Іfavіrents
1016.
What term is usually defined by quantitative HIV RNA levels in blood plasma?
A. viral strain
B. the number of viral
C. * Viral load
D. viral factor
E. Viral RNA levels
1017.
What is the most common term of seroconversion in patients with HIV
infection?
A. 1 Week
B. 1 month
C. * 3 months
D. 1 year
E. indefinitely
1018.
What is the maximum duration of the incubation period in HIV infection?
A. 1 month
B. 3 months
C. 1 year
D. * 5-6 years
E. indefinitely
1019.
What is the minimum duration of the incubation period in HIV infection?
A. * 1 week
B. 1 month
C. 3 months
D. 1 year
E. indefinitely
1020.
What stage of the life cycle of the virus suppressed antiretroviral drugs, called
protease inhibitors (PI)?
A. Entry of HIV into cells
B. reverse Transcription
C. integration
D.Transcription
E.* Build and branch
1021.
Clinical manifestations of acute retroviral syndrome, except:
A.Fever
B. various rashes
C. Influenzalike syndrome
D.Diarrhea
E.. Bleeding
1022.
Clinical manifestations of acute retroviral syndrome, except:
A. meningeal syndrome
B. various rashes
C. Influenzalike syndrome
D. diarrhea
E. * Jaundice
1023.
When AIDS can be diagnose?
A. Only when the number of CD4-lymphocytes less than 500 in 1 ml of blood
B. Only when the number of CD4-l lymphocytes less than 400 in 1 ml of blood
C. Only when the number of CD4-lymphocytes less than 300 in 1 ml of blood
D.* Only when the number of CD4- lymphocytes less than 200 in 1 ml of blood
E. Only when the number of CD4- lymphocytes less than 100 in 1 ml of blood
1024.
When prescribed antiretrovirals to a HIV-infected pregnant women in need of
ART for their own health in order to reduce the risk of infection of the fetus?
A.* Regardless of the duration of pregnancy and childbirth
B. Only during childbirth
C. C 20 weeks of pregnancy and during labor
D. C 14 weeks
E. C 24 weeks of pregnancy and during labor
1025.
When prescribed antiretrovirals HIV-infected pregnant women that do not need
ART for their own health in order to reduce the risk of infection of the fetus?
A. throughout pregnancy and during labor
B. Only during childbirth
C. 20 weeks of pregnancy and during labor
D.14 weeks
E.* C 24 weeks of pregnancy and during labor
1026.
When prescribed antiretrovirals HIV-infected pregnant women in need of ART
for their own health in order to reduce the risk of infection of the fetus?
A.. * Regardless of the duration of pregnancy and childbirth
B. Only during childbirth
C. C 20 weeks of pregnancy and during labor
D. C 14 weeks
E. C 24 weeks of pregnancy and during labor
1027.
When administered antiretroviral drugs to HIV-infected pregnant women to
reduce the risk of infection of the fetus?
A.Throughout pregnancy and during labor
B. Only during childbirth
C. From 25 weeks of pregnancy and during labor
D. With 14 weeks of pregnancy
E.* From 28 weeks of pregnancy and during labor
1028.
When antiviral therapy for children born from HIV-infected women should
starting?
A.* In the first hour after birth, 12.8
B. On the second day
C. Do not spend
D.After a month
E.After diagnosis SPIDA.
1029.
When the planned caesarean section for HIV-infected pregnant women reduce
the risk of infection of the fetus?
A. In 36 weeks
B. In 37 weeks
C. * In 38 weeks
D. In 39 weeks
E. In 40 weeks
1030.
What cells is the main target for HIV?
A.T-suppressor
B. T-killers
C. * T-helper cells
D.D-cells
E.0-cells
1031.
What stage of HIV life cycle nucleoside reverse transcriptase inhibitors perform
action (NRTIs)?
A.HIV penetration into cells
B. reverse Transcription
C. integration
D.transcription
E.* Replacement of the nucleoside
1032.
What stage of the life cycle is HIV zidovudine may destroy?
A.HIV penetration into cells
B. Reverse transcription
C. integration
D.transcription
E.* Replacement of the nucleoside
1033.
What stage of the life cycle is HIV non-nucleoside reverse transcriptase
inhibitors (NNRTIs) may destroy?
A.HIV penetration into cells
B. * Reverse Transcription
C. Integration
D.Transcription
E.Broadcasting
1034.
What stage of the life cycle is HIV zidovudine nucleoside reverse transcriptase
inhibitors (NRTIs) may destroy?
A.HIV penetration into cells
B. Reverse Transcription
C. Integration
D.Transcription
E.* Replacement of the nucleoside
1035.
What is acute retroviral syndrome clinically:
A.Fever
B. Lymphadenopathy
C. Enlargement of the liver
D.Meningeal syndrome
E.* All the above
1036.
What cells of the immune system, which contain CD4 molecules on their
surfaces may infected by HIV.
A.Monocytes, macrophages
B. Macrophages, T-lymphocytes
C. T-lymphocytes, macrophages, cells of the central nervous system
D.* T-lymphocytes, cells of the CNS, monocytes, macrophages
E.Monocytes, macrophages, T-lymphocytes
1037.
What cells of the immune system, which contain CD4 molecules on their
surfaces may infected by HIV.
A.Monocytes
B. Macrophages
C. T-lymphocyte
D.Microglia
E.* all the above
1038.
What cells of the immune system, which contain CD4 molecules on their
surfaces may infected by HIV.
A.Monocytes
B. Macrophages
C. T-lymphocyte
D.Microglia
E.* all the above
1039.
HIV-positive, practical healthy baby of first year was allowed of such preventive
vaccines, except:
A.Diphtheria Toxoid
B. Tetany toxoid
C. * oral polio
D.inactivated polio
E.pertussis
1040.
HIV-positive, practical healthy baby of first year was allowed of such preventive
vaccines, except:
A.Diphtheria Toxoid
B. Tetany toxoid
C. * tuberculosis
D.inactivated polio
E.pertussis
1041.
HIV-positive, practical healthy baby of first year was allowed of such preventive
vaccines, except:
A.Diphtheria Toxoid
B. Tetany toxoid
C. * Triple vaccine against measles, mumps, rubella
D.Inactivated polio
E.pertussis
1042.
HIV-positive, practical healthy baby of first year was allowed of such preventive
vaccines, except:
A.Diphtheria Toxoid
B. Tetany toxoid
C. * measles
D.inactivated polio
E.pertussis
1043.
HIV-positive, practical healthy baby of first year was allowed of such preventive
vaccines, except:
A.Diphtheria Toxoid
B. Tetany toxoid
C. * rubella
D.inactivated polio
E.pertussis
1044.
HIV-positive, practical healthy baby of first year was allowed of such preventive
vaccines, except:
A.Diphtheria Toxoid
B. Tetany toxoid
C. * mumps
D.inactivated polio
E.pertussis
1045.
What is not typical for the IV clinical stage of AIDS, according to clinical stage
classification, developed by experts of WHO(2006)?
A.Cachexia
B. PCP
C. Cerebral toxoplasmosis
D.Extrapulmonary cryptococcosis
E.* Cryptosporidiosis with diarrhea less than 1 month
1046.
According to clinical stage classification developed by experts of WHO (2006)
what is not typical for the IV clinical stage of AIDS?
A.Cachexia associated with HIV infection
B. PCP
C. Cerebral toxoplasmosis
D.Extrapulmonary cryptococcosis
E.* Diarrhea lasting less than 1 month
1047.
According to clinical stage classification developed by experts of WHO (2006)
what is not typical for the II clinical stage of AIDS?
A.Loss of body weight less than 10 % from the initial
B. Minimal damage of the skin and mucous membranes (seborrheic dermatitis,
pruritus, fungal nail infections)
C. Episode of herpes zoster during last five years
D.Recurrent upper respiratory tract infections (bacterial sinusitis)
E.* Tuberculosis of the lungs, which has evolved over the year preceding the
examination
1048.
According to clinical stage classification developed by experts of WHO (2006)
what is not typical for the II clinical stage of AIDS?
A.* Weight loss of more than 10 % from the initial
B. Persistent generalized lymphadenopathy
C. Episode of herpes zoster during last five years
D.Recurrent upper respiratory tract infections
E.Minimal mucosal lesions (recurrent ulcers of the oral mucosa, angular cheilitis)
1049.
According to clinical stage classification developed by experts of WHO (2006)
what is not typical for the II clinical stage of AIDS?
A.Loss of body weight less than 10 % from the initial
B. * Unmotivated chronic diarrhea that lasts more than 1 month
C. Episode of herpes zoster during last five years
D.Recurrent upper respiratory tract infections
E.Minimal mucosal lesions (recurrent ulcers of the oral mucosa, angular cheilitis)
1050.
According to clinical stage classification developed by experts of WHO (2006)
what is not typical for the IV clinical stage of AIDS?
A.Loss of body weight less than 10 % from the initial
B. * Multiple lesions of the skin and mucous membranes
C. Episode of herpes zoster
D.Recurrent upper respiratory tract infections
E.Minimal mucosal lesions (recurrent ulcers of the oral mucosa, angular cheilitis)
1051.
According to clinical stage classification developed by experts of WHO (2006)
what is not typical for the IV clinical stage of AIDS?
A.Loss of body weight less than 10 % of the initial
B. * Sarcoma Kaposhi
C. Episode of herpes zoster in the past five years
D.Recurrent upper respiratory tract infections
E.Minimal mucosal lesions (recurrent ulcers of the oral mucosa, angular cheilitis)
1052.
According to clinical stage classification developed by experts of WHO (2006)
what is not typical for the II clinical stage of AIDS?
A.Loss of body weight less than 10 % from the original
B. Minimal damage to the skin and mucous membranes (seborrheic dermatitis,
pruritus, fungal nail infections)
C. Episode of herpes zoster
D.Recurrent upper respiratory tract infections (eg, bacterial sinusitis)
E.* Tuberculosis of lungs, which has evolved over the year preceding the
examination
1053.
According to clinical stage classification developed by experts of WHO (2006)
what is not typical for the II clinical stage of AIDS?
A.Loss of body weight less than 10 % from the initial
B. * Multiple lesions of the skin and mucous membranes
C. Episode of herpes zoster in the past five years
D.Recurrent upper respiratory tract infections
E.Minimal mucosal lesions (recurrent ulcers of the oral mucosa, angular cheilitis)
1054.
According to clinical stage classification developed by experts of WHO (2006)
what is not typical for the II clinical stage of AIDS?
A.* Weight loss of more than 10 % from the initial
B. Persistent generalized lymphadenopathy
C. Episode of herpes zoster in the past five years
D.Recurrent upper respiratory tract infections
E.Minimal mucosal lesions (recurrent ulcers of the oral mucosa, angular cheilitis)
1055.
According to clinical stage classification developed by experts of WHO (2006)
what is not typical for the II clinical stage of AIDS?
A.Loss of body weight less than 10 % from the initial
B. * Unmotivated chronic diarrhea that lasts more than 1 month
C. Episode of herpes zoster in the past five years
D.Recurrent upper respiratory tract infections
E.Minimal mucosal lesions (recurrent ulcers of the oral mucosa, angular cheilitis)
1056.
According to clinical stage classification developed by experts of WHO (2006)
what is not typical for the II clinical stage of AIDS?
A.Loss of body weight less than 10 % of the initial
B. * sarcoma Kaposhi
C. Episode of herpes zoster in the past five years
D.Recurrent upper respiratory tract infections
E.Minimal mucosal lesions (recurrent ulcers of the oral mucosa, angular cheilitis)
1057.
According to clinical stage classification developed by experts of WHO (2006)
what is not typical for the III clinical stage of AIDS?
A.Weight loss of more than 10 %
B. Unmotivated chronic diarrhea that lasts more than 1 month
C. Unexplained fever that lasts more than 1 month (permanently or temporarily)
D.* Episods of upper respiratory tract infection ( bacterial sinusitis)
E.Leukoplakia of the oral mucosa
1058.
According to clinical stage classification developed by experts of WHO (2006)
what is not typical for the III clinical stage of AIDS?
A.Lymphadenopathy more than 3 months
B. Unmotivated chronic diarrhea that lasts more than 1 month
C. Unexplained fever that lasts more than 1 month (permanently or temporarily)
D.* Cachexia
E.Cytomegalovirus chorioretinitis
1059.
According to clinical stage classification developed by experts of WHO (2006)
what is not typical for the IV clinical stage of AIDS?
A.Weight loss of more than 10 % of the initial
B. Unmotivated chronic diarrhea that lasts more than 1 month
C. Unexplained fever that lasts more than 1 month (permanently or temporarily)
D.* Single lymph nodes in one anatomical region, acute pain
E.Cytomegalovirus chorioretinitis
1060.
According to clinical stage classification developed by experts of WHO (2006)
what is not typical for the III clinical stage of AIDS?
A.Weight loss of more than 10 % of the initial
B. Unmotivated chronic diarrhea that lasts more than 1 month
C. Unexplained fever that lasts more than 1 month (permanently or temporarily)
D.* Single upper respiratory tract infection (eg, bacterial sinusitis)
E.Hairy leukoplakia of the oral mucos
1061.
According to clinical stage classification developed by experts of WHO (2006)
what is not typical for the IV clinical stage of AIDS?
A.Cachexia
B. PCP
C. Cerebral toxoplasmosis
D.Carkoma sarcoma
E.* All the above
1062.
Experts of WHO believe suspicious according to AIDS lymph nodes:
A.3 or more nodes in more than two anatomical and topographical groups (except
inguinal), measuring more than 2 cm in diameter, which extends more than 1
month.
B. 3 or more nodes in more than two anatomical and topographical groups (except
inguinal), larger than 1 cm in diameter, for more than 2 months.
C. * 2 or more nodes in more than two anatomical and topographical groups (except
inguinal), larger than 1 cm in diameter, which lasts more than 3 months.
D.2 or more units in more than two anatomical topographic groups (except inguinal),
larger than 2 cm in diameter, which extends over 2 months.
E.2 or more units in more than two anatomical topographic groups (except inguinal),
larger than 1 cm in diameter, which continued for over 2 months.
1063.
Immediately after contact with blood and other biological liquids it is necessary
to wash the dirty areas of skin with water and soap and to begin a postcontact prophylaxis,
antiretroviral preparations not later than:
A.* 24-36 hrs
B. 36-48 hrs
C. 48-60 hrs
D.60-72 hrs
E.72-86 hrs
1064.
What antiretroviral preparation is taken as post contact prophylaxis after a
contact with blood and other biological fluids?
A.* Azidotimidin
B. Nevirapin
C. Indinavir
D.Saqvinavir
E.Ifavirent
1065.
When do the plan caesarian section will conduct to HIV infected pregnant with
the purpose for decreasing of the risk of infecting the fetus?
A.In 36 weeks
B. In 37 weeks
C. * In 38 weeks
D.In 39 weeks
E.In 40 weeks
1066.
After a car accident patient has been taken to hospital in critical condition,
caused by shock, bleeding. Before blood transfusion the doctor should consider:
A.Health reasons
B. in order to prevent HIV transmission The conclusion of doctors consilium
C. Agree of the patient (or his family)
D.The results of a survey of emergency donor about HIV (IHA)
E.* All the above
1067.
Immediately after contact with blood or other body fluids person should wash
exposed skin with soap and water, and contaminated mucous membranes - with clean
water. When is the post-exposure prophylaxis with antiretroviral drugs should start?
A.*No later than 72 hours
B. B for a week
C. If the infection is confirmed
D.Not available
E.Seropositive persons
1068.
Immediately after contact with blood or other body fluids person should wash
exposed skin with soap and water, and contaminated mucous membranes - with clean
water. How long is conducted post-exposure prophylaxis?
A.During a week
B. *4 weeks
C. 3 months
D.Until the end of the observation period
E.Not available
1069.
At what dose of antiretroviral drug prophylaxis is carried out after contact with
blood and other body fluids?
A.600-800 mg
B. 700-800 mg
C. * 800-1000 mg
D.1000-1100 mg
E.1100-1200 mg
1070.
How long is the antiretroviral prophylaxis after contact with blood and other
body fluids perfomed?
A.1 Week
B. 2 weeks
C. * 1 month
D.3 months
E.6 months
1071.
The epidemic outbreak rationally organize inspection of the immune system.
Using the IHA to identify nonimmune individuals to diphtheria in a few hours. What is the
minimum protective titer?
A.1:10
B. 1:20
C. * 1:40
D.1:80
E.1:160
1072.
The most important clinical criteria for AIDS must include the following, except:
A.Significant weight loss (10%) if it continues over a month
B. Duration of fever, if it continues over a month
C. Persistent diarrhea, if it lasts more than a month
D.* A rash on the skin, if it continues over a month
E.Lymphadenopathy than 3 mesyatseB.
1073.
How to decide on the child's immunizations with unclear HIV status to HIVinfected mothers?
A.Vaccination against diphtheria and tetanus
B. Polio vaccination
C. Vaccination against tuberculosis
D.Vaccination against polio and tuberculosis
E.* Vaccination is not performed
1074.
What additional testing is necessary to patients with infectious mononucleosis?
A.*ELISA for HIV, bacteriology of diphtheria
B. ELISA for HIV, bacteriology for tularemia
C. bacteriology of diphtheria and typhoid fever
D.Reaction Burne and Wright-Heddlsona
E.The reaction of Paul Bunnelya and lymph node puncture
1075.
What immunological changes are identified in a patient with HIV infection?
A.Is a polyclonal B-cell inhibition, increases the formation of autoantibodies and
immune complexes
B. Is a polyclonal B-cell activation and decreases the formation of autoantibodies and
immune complexes
C. Is a polyclonal B-cell inhibition, decreases the formation of autoantibodies and
immune complexes
D.* There is a polyclonal B-cell activation, increases the formation of autoantibodies
and immune complexes
E.Is a polyclonal B-cell activation, increases the formation of autoantibodies and
decreases the formation of immune complexes
1076.
What laboratory findings are not typical of Sneed-related complex?
A.Decreasing the amount of immunoglobulin A is the G.
B. * Reducing the ratio below 1.0 CD4/CD8
C. Anemia
D.Leukopenia
E.Increased levels of circulating immune complexes
1077.
Which antiretroviral drug prophylaxis is carried out after contact with blood and
other body fluids?
A.Azidothymidine
B. Nevirapine
C. * Azidothymidine + nevirapine
D.AZT + saquinavir
E.Іfavіrents
1078.
What term is usually defined by quantitative HIV RNA levels in blood plasma?
A.Viral strain
B. The number of viral
C. * Viral load
D.Viral factor
E.Viral RNA levels
1079.
What are the characteristics of plaque in diphtheria?
A.Single-sided, gray-white, on the surface ulcer crater
B. * The gray-white, dense, with sharp edges and glossy surface
C. Yellow-white, brittle, located perilakunarno
D.Sided, yellow-white, in the gaps
E.White, brittle, easily removed with a spatula
1080.
What stage of the life cycle of the virus are suppressed by antiretroviral drugs,
called protease inhibitors (PI)?
A.Entry of HIV into cells
B. reverse Transcription
C. integration
D.transcription
E.* Build and branch
1081.
When you can make a diagnosis of AIDS?
A.Only when the content of CD4-lymphocytes less than 500 in 1 ml of blood
B. Only when the content of CD4- lymphocytes less than 400 in 1 ml of blood
C. Only when the content of CD4- lymphocytes less than 300 in 1 ml of blood
D.* Only when the content of CD4- lymphocytes less than 200 in 1 ml of blood
E.Only when the content of CD4- lymphocytes less than 100 in 1 ml of blood
1082.
When are antiretroviral drugs administered to HIV-infected pregnant women to
reduce the risk of infection of the fetus?
A.Throughout pregnancy and during labor
B. Only during childbirth
C. From 25 weeks of pregnancy and during labor
D.With 14 weeks of pregnancy
E.* From 28 weeks of pregnancy and during labor
1083.
When is antiviral therapy started for children born to HIV-infected women?
A.* In the first hour after birth, 12.8
B. On the second day
C. Do not spend
D.After a month
E.After diagnosis SPIDA.
1084.
When is the planned caesarean section for HIV-infected pregnant women to
reduce the risk of infection of the fetus?
A.In 36 weeks
B. In 37 weeks
C. * At 38 weeks
D.In 39 weeks
E.In 40 weeks
1085.
Who is the primary target for HIV?
A.T-suppressor
B. T-killers
C. * T-helper cells
D.D-cells
E.0-cells
1086.
Violation of which stage of the life cycle are caused by nucleoside reverse
transcriptase inhibitors (NRTIs)?
A.Entry of HIV into cells
B. reverse Transcription
C. integration
D.transcription
E.* Replacement of the nucleoside
1087.
Violation of which stage of the life cycle of HIV is zidovudine?
A.Entry of HIV into cells
B. reverse Transcription
C. integration
D.transcription
E.* Replacement of the nucleoside
1088.
Violation of which stage of the life cycle are caused by non-nucleoside reverse
transcriptase inhibitors (NNRTIs)?
A.Entry of HIV into cells
B. * Reverse Transcription
C. integration
D.transcription
E.Translation
1089.
Complication in the 4-5th week of diphtheria:
A.Encephalitis
B. Bulbar disorders, pancreatitis, hepatitis
C. * Polyradiculitis, myocarditis
D.Nephrosonephritis
E.Stenosing laryngotracheitis
1090.
Complication that often develops in the first week of diphtheria oropharynx:
A.Polyradiculitis
B. Asphyxia
C. Failure of adrenal glands
D.Hepatosplenomegaly
E.* Paresis of the soft palate
1091.
Particularly high titre diphtheria antitoxic antibodies in moderate titer of antitetanus antibodies indicates:
A.Tetanus
B. Diphtheria
C. Carriering of Corynebacterium diphtheria
D.Immunity to diphtheria and to the formation of bacteria
E.* Nothing
1092.
Before revaccination against diphtheria in adults, they recommended:
A.* Identify the antibody titer
B. Preventive antibiotics
C. Proactively assign antihistamines
D.Five years after the last booster
E.10 years after vaccination
1093.
List all the cells of the immune system, which contain CD4 molecules on their
surfaces that are infected with HIV.
A.Monocytes, macrophages
B. Macrophages, T-lymphocytes
C. T-lymphocytes, macrophages, cells of the central nervous system
D.* T-lymphocytes, cells of the CNS, monocytes, macrophages
1094.
How to prevent occupational HIV infection?
A.Local wound treatment
B. * Post eccident HAART
C. Laboratory testing for HIV
D.Register the fact of the accident in a special register
E.All the above
1095.
What preparations are necessary for prevention of occupational HIV infection?
A.Local wound treatment
B. * Antiretrovirus drugs
C. Antibiotics
D.Specific immunoglobulin
E.All the above
1096.
During assistance nurse accidentally pricked her finger with a contaminated
needle. What is prevention of disease?
A.The combination of NRTIs + PI
B. * Combination NRTI + NNRTI
C. Interferons
D.Specific immunoglobulin
E.Initial debridement
1097.
During assistance nurse accidentally pricked her finger with a contaminated
needle . For the recognition of occupational HIV infection?
A.Seroconversion after crash
B. HIV asymptomatic infection in accident
C. Occupational exposure
D.* Infection occurred before the accident
E.All the above
1098.
During assistance nurse accidentally pricked her finger with a contaminated
needle . For the recognition of occupational HIV infection :
A.Confirmation of HIV
B. Accounting the fact of the accident in a special register
C. Negative results of laboratory examination in the first 5 days after the accident
D.Positive results of laboratory testing for HIV at 1, 3 or 6 months after the accident
E.* All the above
1099.
During assistance nurse accidentally pricked her finger with a contaminated
needle . For the recognition of occupational HIV infection :
A.Confirmation of HIV
B. Statement of victim
C. * Negative results of laboratory examination in the first 5 days after the accident
and positive 1, 3 or 6 months after the accident
D.Availability of health book in the affected
E.Full-time work regimen
1100.
A child of HIV-positive without clinical and laboratory signs of the disease.
How to solve the problem of vaccination against polio?
A.* Conduct, as it is provided in the Calendar routine immunization
B. Vaccination dont perform
C. Hold until the results of a survey on child markers HIV
D.Not available as prohibit the use of live vaccines
E.Show all of the above is true
1101.
A child of HIV-positive without clinical and laboratory signs of the disease.
How to solve the problem of vaccination against polio?
A.Not available as prohibit the use of live vaccines
B. Hold, as it is provided according the Calendar routine immunization
C. Postponed until the results of child markers HIV
D.* Replace the live vaccine to inactivated
E.Vaccination dont perform
1102.
A child of HIV-infected mother, was born apparently healthy. When you can
confirm HIV-negative status of the child?
A.Immediately after birth if there are no clinical manifestations
B. After 3 months in the absence of HIV markers
C. After 6 months in the absence of the child HIV markers
D.* After 18 months in the absence of the child markers HIV
E.Never, since the HIV transmission from mother occurs in 100% of cases
1103.
A child of HIV-infected mother, was born apparently healthy. How to solve the
problem of vaccination against tuberculosis?
A.Hold, as it is provided in the Calendar routine immunization
B. Performed if there is no clinical and laboratory signs of disease
C. Postponed until the results of a survey on child markers HIV
D.* Not available as prohibit the use of live vaccines
E.Vaccination dont perform
1104.
A child of HIV-infected mother, was born prematurely, with clinical signs of
AIDS. How to solve the question of vaccination (tuberculosis, hepatitis B)?
A.Hold, as it is provided in the Calendar routine immunization
B. Performed if there is no clinical and laboratory signs of disease
C. Postponed until the results of a survey on child markers HIV
D.Not available as prohibit the use of live vaccines
E.* Vaccination dont perform
1105.
Immediately after contact with blood and other body fluids should be washed
exposed skin with soap and water, and contaminated mucous membranes - with clean
water. When is the post-exposure prophylaxis with antiretroviral drugs?
A.* No later than 72 hours
B. During the week
C. If the infection is confirmed
D.Not available
E.Seropositive persons
1106.
In a different places of settlement a few cases of cholera was found. Who in the
focus of cholera will be send in a hospital?
A.Carriers
B. Persons contact with the patient
C. *Patients with cholera
D.Persons with dysfunction of alimentary tract
E.Persons with hyperthermia
1107.
Diagnosed a patient: chronic hepatitis in the stage of integration. What markers
will be in patient in this stage disease?
A.HBeAg
B. Antibodies to HBeAg
C. DNA OF HBV
D.Viral DNA-polimerase
E.*HBsAg, anti-НBе
1108.
As etiotropic therapy of acute and chronic viral hepatitis B utillize:
A.Corticosteroid
B. Immunomodulators
C. Cytostatics
D.Antibiotics
E.*Antiviral
1109.
Direct bilirubin is increased , in urine there is significant increase of bilirubin and
urobilin, increasing of stercobilin of excrements. What is the type of icterus?
A.Haemolitic
B. *Parenhimatous
C. Transport
D.Extraliver
E.Mechanical
1110.
On the average 15 to 30 % of all population of the planet suffer from some
pathology of liver. Prevalence of hepatitis and cirrhosis in the European countries is about
1 % of adults. Annually in the world there are about 2 million people with acute viral
hepatitis. What % of all cases will develop chronic form.
A.100 %
B. 50 %
C. 25 %
D.*10 %
E.1 %
1111.
When sick people get epidemic typhus infection, which period affects more?
A. Over the past 2 days, the incubation period and 2-3 days after lowering
temperature
B. All hectic period and 2-3 days after lowering temperature
C. 2-3 days after lowering temperature
D. *Over the past 2 days, the incubation period, all febrile period and 2-3 days after
lowering temperature
E.Over the past 2 days, the incubation period and the hectic period
1112.
In which period the maximal growth of infection occurs during epidemic typhus?
A.
At the incubation period
B.
*At the 1th week of illness
C.
At the 2nd week of illness
D.
At the 3rd week of illness
E.At the time of recovery
1113.
Often, in patient with epidemic typhus arise transition petehies in the
conjunctivA. What term is used to describe this?
A.
Symptom of Heller
B.
Conjunctivitis
C.
Symptom of Govorova-Godele
D.
*Symptom of Zorohovich-Chiari-Avtsyna
E.Enantema Rosenberg‘s
1114.
In the family of the patient with epidemic typhus were lice in the children. With
the help of any of these measures could prevent the subsequent spread of the disease?
A.
*Monitoring and complete sanitation of contact in the centre
B.
The use of chemoprophylaxis
C.
The use of antibiotics
D.
Isolation contact
E.Check-up
1115.
When can you stop the etiotropic medication treatment of the patient with
epidemic typhus?
A.
Immediately after the normalization of body temperature
B.
After the normalization of the liver and spleen
C.
*After a 2-day normal body temperature
D.
After the disappearance of roseola
E.Within 10 days after the disappearance of roseola
1116.
Often, in patient with epidemic typhus arise transition petehies in the
conjunctivA. What term did it call?
A.Symptom of Heller
B. Conjunctivitis
C. Symptom of Govorov-Godele
D.*Symptom of Zorohovich-Chiari-Avtsyna
E.Enantema Rosenberg‘s
1117.
Often, in patient with epidemic typhus arises petehies on mucosal soft palate.
What term did it call?
A.Symptom of Heller
B. Conjunctivitis
C. Symptom of Govorova-Godele
D.Symptom of Zorohovich-Chiari
E.*Enantema Rosenberg‘s
1118.
Often, in patient with epidemic typhus is tongue‘s tremor when protrusion that
sticked on the lower teeth. What term did it call?
A.Symptom of Heller
B. Conjunctivitis
C. *Symptom of Govorova-Godele
D.Symptom of Zorohovich-Kiari
E.Enantema Rosenberg‘s
1119.
In the typical form of typhoid fever, the body temperature rises progressively
from day to day 39-40 °C at the end of the 1st week, and during the next 10-14 days it gets
back approximately to this level, and then becomes remittent and, gradually goes down to
the norm. What is such temperature curve called?
A.*The temperature curve as Vunderlihs
B. The temperature curve as Botkin
C. Temperature curve as Kildushevsky
D.Temperature curve as Ellers
E.Intermedium temperature curve
1120.
In the typical form of typhoid fever the body temperature rises to 39-40 °C. The
temperature curve looks like 2 waves (during 3-4 weeks of disease). What is such
temperature curve called?
A.The temperature curve as Vunderlihs
B. *The temperature curve as Botkin
C. Temperature curve as Kildushevsky
D.Temperature curve as Ellers
E.Intermedium temperature curve
1121.
In the typical form of typhoid fever, the body temperature rises pregressively
from day to day to 39-40 °C at the end of the 1st week, and then gradually during 2-3 week
it goes down to the norm. What is such temperature curve called?
A.The temperature curve as Vunderlihs
B. The temperature curve as Botkin
C. *Temperature curve as Kildushevsky
D.Temperature curve as Ellers
E.Intermedium temperature curve
1122.
One methods of diagnostics of typhoid fever and paratyphoid fever is the
selection of hemoculture. This is done during the fever period blood is taken from the vein
on bilious bulione or Rappaport‘s media in correlation 1:10 is made. What volume of blood
must be taken on the 2nd week of disease?
A.5 ml of blood
B. 10 ml of blood
C. *15 ml of blood
D.20 ml of blood
E.25 ml of blood
1123.
One methods of diagnostics of typhoid fever and paratyphoid fever is the
selection of hemoculture. This is done during the fever period blood is taken from the vein
bilious bulione or Rappaport‘s media in correlation 1:10 is made. What volume of blood
must be taken on the 1st week of disease?
A.0,5 ml of blood
B. *10 ml of blood
C. 15 ml of blood
D.20 ml of blood
E.25 ml of blood
1124.
One methods of diagnostics of typhoid fever and paratyphoid fever is the
selection of hemoculture. This is done during the fever period blood is taken from the vein
bilious bulione or Rappaport‘s media in correlation 1:10 is made. What volume of blood
must be taken on the 3nd week of disease?
A.5 ml of blood
B. 10 ml of blood
C. 15 ml of blood
D.*20 ml of blood
E.25 ml of blood
1125.
When sick people gets epidemic typhus infection, which period affects more?
A.Over the past 2 days, the incubation period and 2-3 days after lowering temperature
B. All hectic period and 2-3 days after lowering temperature
C. 2-3 days after lowering temperature
D.*Over the past 2 days, the incubation period, all febrile period and 2-3 days after
lowering temperature
E.Over the past 2 days, the incubation period and the hectic period
1126.
On which period the maximal growth of infection occurs during epidemic typhus
disease?
A.At the incubation period
B. *At the 1th week of illness
C. At the 2nd week of illness
D.At the 3rd week of illness
E.At the time of recovery
1127.
One of methods of diagnostics of typhoid fever and paratyphoid fever is the
selection of hemoculture. For that in a fever period sowing of blood from a vein on bilious
bulione or Rappaport‘s media in correlation 1:10 is made. What volume of blood must be
taken on the 2nd week of disease?
A.5 ml of blood
B. 10 ml of blood
C. *15 ml of blood
D.20 ml of blood
E.25 ml of blood
1128.
One of methods of diagnostics of typhoid fever and paratyphoid fever is the
selection of hemoculture. For that in a fever period sowing of blood from a vein on bilious
bulione or Rappaport‘s media in correlation 1:10 is made. What volume of blood must be
taken on the 1st week of disease?
A.0,5 ml of blood
B. *10 ml of blood
C. 15 ml of blood
D.20 ml of blood
E.25 ml of blood
1129.
One of methods of diagnostics of typhoid fever and paratyphoid fever is the
selection of hemoculture. For that in a fever period sowing of blood from a vein on bilious
bulione or Rappaport‘s media in correlation 1:10 is made. What volume of blood must be
taken on the 3nd week of disease?
A.5 ml of blood
B. 10 ml of blood
C. 15 ml of blood
D.*20 ml of blood
E.25 ml of blood
1130.
What is etiology of acute primary tonsillitis?
A.Influenza virus
B. *β-hemolitic streptococci
C. N. meningitidis
D.M. tuberculosis
E.K. pneumonia
1131.
What group of lymph nodes more often change in acute tonsillitis?
A.Anterior cervical
B. *Submandibular
C. Posterior cervical
D.Retroauricular
E.Occipital
1132.
What is most usual complication of acute tonsillitis?
A.Otitis
B. Sinusitis
C. *Abscess
D.Frontitis
E.Pneumonia
1133.
Differential diagnosis of acute tonsillitis should perform with?
A.Infectious mononucleosis
B. Diphtheria
C. Scarlet fever
D.Adenoviral infection
E.*All the above
1134.
Which group of antibiotics is most effective in treatment of tonsillitis?
A.*Penicillin
B. Tetracycline
C. Cephalexin
D. Sulfalen
E.Ampicillin
1135.
What is the duration of incubation period of tonsillitis?
A.*Few hours – two days
B. Few days – one week
C. Few weeks – one month
D. 1 – 10 hours
E. 1- 10 days
1136.
In tonsillitis streptococcus usually cultivated from:
A.Blood
B. Urine
C. Feces
D.*Mucous
E.Saliva
1137.
What group of infectious diseases diphtheria belong to according to L.
Gromashevsky classification?
A.Intestinal infection
B. *Infection of respiratory tract
C. Blood infection
D. Infection of external covers
E.Transmissive
1138.
What is the agent of diphtheria?
A.Influenza virus
B. β-hemolitic streptococci
C. N. meningitidis
D. *B. Leffleri
E.K. pneumonia
1139.
Diphtheria is transmitted by:
A.Food
B. Water
C. *Air
D.Blood
E.Milk
1140.
What rear form of diphtheria localization do you know?
A. Laryng
B. Trachea
C. Bronch
D.*Eye
E.Nose
1141.
What early complication more often may occur in diphtheria?
A.Pneumonia
B. *Myocarditis
C. Gastritis
D.Otitis
E.Frontitis
1142.
What is the agent of infectious mononucleosis?
A.*Epstein-Barr virus
B. β-hemolitic streptococci
C. N. meningitidis
D. B. Leffleri
E.K. pneumonia
1143.
Which of clinical symptoms are more typical for infectious mononucleosis?
A.Fever, hepatomegaly, abdominal pain
B. *Polyadenitis, hepatomegaly, splenomegaly, fever, tonsillitis
C. Fever, adenitis, abdominal pain
D. Dry cough, pain in the throat, high temperature
E.Dysphagia, dyspnoe, wet cough
1144.
Specific blood changes in influenza:
A.Erythrocytopenia, lymphopenia, monocytosis
B. Decreasing of hemoglobin, leucocytosis
C. Decreasing of ESR, monocytosis, leucocytosis
D.Leucocytosis, increasing of ESR, thrombocytopenia
E.*Leucopenia, lymphocytosis, increasing of ESR
1145.
What is forms of adenoviral infection, except:
A. Acute respiratory disease
B. Pneumonia
C. Pharyngoconjuctival fever
D.Conjuctivatis
E.*Adenitis
1146.
What group of infectious diseases adenoviral infection belong to according to L.
Gromashevsky classification?
A.Intestinal infection
B. Infection of respiratory tract
C. Blood infection
D.*Infection of external covers
E.Transmissive
1147.
Duration of incubation period of adenoviral infection:
A.2-5 days
B. 5-7 days
C. 1-2 days
D.7-10 days
E.10-15 days
1148.
Differential diagnosis of adenoviral infection should perform with?
A.Infectious mononucleosis
B. Diphtheria
C. Scarlet fever
D.Tonsillitis
E.*All the above
1149.
What is the agent of meningococcal disease?
A.Epstein-Barr virus
B. β-hemolitic streptococci
C. *N. meningitidis
D. B. Leffleri
E.K. pneumonia
1150.
What group of infectious diseases meningococcal disease belong to according to
L. Gromashevsky classification?
A.Intestinal infection
B. Infection of respiratory tract
C. Blood infection
D.*Infection of external covers
E.Transmissive
1151.
Which of this symptoms are often present in patients with meningococcol
disease?
A. Algor, high temperatura, headache
B. Profuse watery diarrhea, vomiting, dehydration, muscular cramps
C. Abdominal pain, diarrhea, constipation, flatulance
D.Headache, dry cough, algor
E.*Sore throat, fever, headache, stiff neck, vomiting, confusion
1152.
What laboratory test is most helpful in diagnosis of meningococcal disease?
A.Serological
B. Bacteriological
C. *Lumbar puncture
D.Biopsy
E.X-rays examination
1153.
Who is the source of meningococcal disease?
A.Birds
B. Cattle
C. Fish
D.*Human
E.Pets
1154.
Name the agent of epidemic typhus?
A. Entamoeba
B. Shigella
C. Salmonella
D.*Rickettsia
E.Esherihia
1155.
What group of infectious diseases epidemic typhus belong to according to L.
Gromashevsky classification?
A.Intestinal infection
B. Infection of respiratory tract
C. *Blood infection
D.Infection of external covers
E.Transmissive
1156.
In what days of epidemic typhus the rash appears?
A.2-3
B. *4-5
C. 5-7
D.7-10
E.10-15
1157.
Since what period the serological investigation for epidemic typhus diagnosis
can be used?
A.3-5
B. *5-7
C. 7-10
D.10-15
E.15-20
1158.
Most effective etiotropical drug in treatment of epidemic typhus?
A. Penicillin
B. *Tetracycline
C. Cephalexin
D. Sulfalen
E.Ampicillin
1159.
What is the agent of malaria?
A. Plasmodium
B. Shigella
C. Salmonella
D.*Rickettsia
E.Esherihia
1160.
What group of infectious diseases malaria belong to according to L.
Gromashevsky classification?
A.Intestinal infection
B. Infections of respiratory tract
C. *Blood infection
D.Infection of external covers
E.Transmissive
1161.
What is specific prophylaxis and supression therapy in malaria:
A.*Chloroquine
B. Cephalexin
C. Vormil
D.Decametoxin
E.Laferon
1162.
What material should be taken for revealing of Plasmodium malaria?
A.Urine
B. Feces
C. Saliva
D.*Blood
E.Sputum
1163.
Transmissive factor of malaria is:
A.Water
B. Air
C. *Blood
D.Food
E.Saliva
1164.
Y. pestis is transmitted by:
A. Fly
B. *Flea
C. Tick
D.Lice
E.Bug
1165.
What is the duration of incubation period of plague?
A. 3 – 7 days
B. 2 – 12 days
C. *2 – 10 days
D. 1 – 8 days
E. 3 -15 days
1166.
What form of plague is highly fatal?
A.Skin
B. Bubonic
C. Intestinal
D.*Pneumonic
E.Septic
1167.
What form of plague is most contagious?
A.Skin
B. Bubonic
C. Intestinal
D.*Pneumonic
E.Septic
1168.
What is the main etiotropical drug of plague treatment?
A.Penicillin
B. Amoxicillin
C. *Streptomycin
D.Biseptol
E.Cefazolin
1169.
What group of infectious diseases plague belong to according to L.
Gromashevsky classification?
A.Intestinal
B. Respiratory tract
C. *Blood
D.Infection of external covers
E.Transmissive
1170.
1171.
1172.
Drugs of choice for the treatment of amoebae cyst carrier:
A.Monomycinum
B. Delagilum
C. Tetracyclin
D.* Furamid
E.Ursosan
Principles of treatment of patients with shigellosis.
A.Diet
B. Antibacterial drugs
C. Correction of dysbacteriosis
D.Detoxication therapy
E.*All the above
Largest nematode parasite is:
A.Ascaris lumbricoides
B.Necator americanus
C. Ancilostoma duodenum
D.*Dracunculus medinensis
1173.
1174.
1175.
1176.
E.Trichinella spiralis
Which of the following nematodes is ovoviviparous?
A.Ascaris lumbricoides
B. *Strongiloides stercoralis
C. Ancilostoma duodenum
D.Dracunculus medinensis
E.Trichinella spiralis
Which of the following nematodeslays unsegmented eggs?
A.Ascaris lumbricoides
B. Strongiloides stercoralis
C. Ancilostoma duodenum
D.*Trichuris trichiura
E.Trichinella spiralis
Which of the following nematodes is ovoviviparous?
A.Ascaris lumbricoides
B. *Strongiloides stercoralis
C. Ancilostoma duodenum
D.Dracunculus medinensis
E.Trichinella spiralis
Best site for taking biopsy for diagnosis of trichinellosis is:
A.*Deltoid muscle
B. Diaphragm
C. Pectoralis major
D.Liver
E.Spleen
1177.
Rectal prolapse is seen in infection with:
A.Ascaris lumbricoides
B. Strongiloides stercoralis
C. Ancilostoma duodenum
D.Dracunculus medinensis
E.*Tricuris trichiura
1178.
Disseminated systemic infection in AIDS patients is seen with:
A.Ascaris lumbricoides
B. *Strongiloides stercoralis
C. Ancilostoma duodenum
D.Dracunculus medinensis
E.Trichinella spiralis
1179.
“Larva currens” is the name given to the migranting larvae of:
A.Ascaris lumbricoides
B. *Strongiloides stercoralis
C. Ancilostoma duodenum
D.Dracunculus medinensis
E.Trichinella spiralis
1180.
Which of the following nematodes does not pass through lungs during its life
cycle?
A.Ascaris lumbricoides
B. Strongiloides stercoralis
C. *Tricuris trichiura
D.Dracunculus medinensis
E.Trichinella spiralis
1181.
All the following parasites may cause B12 deficiency anaemia:
A.*Diphyllobothrium latum
B. Strongiloides stercoralis
C. Tricuris trichiura
D.Dracunculus medinensis
E.Trichinella spiralis
1182.
Eggs are passed in sputum in case of infection with:
A.Clonorchis sinensis
B.*Paragonimus westermani
C. Ascaris lumbricoides
D.Strongiloides stercoralis
E.Trichinella spiralis
1183.
Which of the following is not a hepatic trematode?
A.Fasciola hepatica
B.*Fasciolopsis buski
C. Clonorchis sinensis
D.Schistosoma haematobium
E.All of the above
1184.
Largest trematode is:
A.Fasciola hepatica
B.*Fasciolopsis buski
C. Clonorchis sinensis
D.Schistosoma haematobium
E.Opisthorchis felineus
1185.
Gynaecophoric canal is seen in case of male worm of:
A.Ascaris lumbricoides
B.Fasciolopsis buski
C. Clonorchis sinensis
D.*Schistosoma mansoni
E.Opisthorchis felineus
1186.
Pseudotubercurcles may be formed around the eggs of:
A.*Schistosoma haematobium
B.Fasciolopsis buski
C. Clonorchis sinensis
D.Schistosoma mansoni
E.Opisthorchis felineus
1187.
Carcinoma of urinary bladder is associated with which of the following
parasites?
A.Schistosoma japonicum
B.*Schistosoma haematobium
C. Clonorchis sinensis
D.Schistosoma mansoni
E.Opisthorchis felineus
1188.
Flame cells are the organ of excretion in:
A.*Trematodes
B. Nematodes
C. Cestodes
D.Pectodes
E.All of the above
1189.
Convulsive seizures may be seen in infection with:
A.Trypanosoma brucei gambiense
B.Negleria fowleri
C. *Paragonimus westermani
D.Schistosoma mansoni
E.Opisthorchis felineus
1190.
Cercaria is the infective stage of:
A.Schistosoma japonicum
B.*Schistosoma haematobium
C. Clonorchis sinensis
D.Schistosoma mansoni
E.Opisthorchis felineus
1191.
Which of the following acts as the main reservoir of Balantidium coli infection
in human beings?
A.Man
B.Monkey
C. * Pig
D.Cow
E.Dog
1192.
Which of the following is the largest protozoal parasite inhabiting the large
intestine of man?
A.Entamoeba histolitica
B.Entamoeba coli
C. *Balantidium coli
D.Giardia lamblia
E.All of the above
1193.
Which of the following is characteristic of Balantidium coli trophozoite?
A.Body covered with short cilia
B.Two nuclei
C. Two contractive vacuoles
D.Numerous food vacuoles
E.All of the above
1194.
Toxoplasma gondii lives inside the:
A.Lumen of small intestine
B. Lumen of large intestine
C. *Reticuloendothelial cells
D.Red blood cells
E.White blood cells
1195.
During which trimestr pregnancy infection with Toxoplasma gondii is more
severy?
A.*First
B. Second
C. Third
D.Second and third
E.First and second
1196.
Commonest manifestation of postnatally acquired infection with Toxoplasma
gondii is:
A.* Lymphadenopathy
B. Pneumonitis
C. Myocarditis
D.Meningoencephalitis
E.Bronchitis
1197.
Drug used for the treatment of toxoplasmosis is:
A.Pyrimethamine
B. Sulphadiazine
C. Spiramycin
D.Clindamycin
E.*All of the above
1198.
Which is the infective form of the malaria parasite?
A.Trophosoite
B.Schizont
C. Merozoite
D.*Sporozoite
E.All of the above
1199.
Which of the following phases of malaria parasites brings on a clinical attack of
malaria?
A.Primary exoerythrocytic schizogony
B.*Erythrocytic schizogony
C. Gametogony
D.Sporogony
E.Secondary exoerythrocytic schizogony
1200.
Resting stage of the malaria parasite is known as:
A.Trophosoite
B.Schizont
C. *Hypnozoite
D.Merozoite
E.Sporozoite
1.
2.
3.
4.
5.
Ситуаційні Задачі
For a man 25 years old, half-year ago there was a positive reaction on protien. Last 3
months are complaint with general weakness, fatigueability, somnolence, chest pain,
during last 2 weeks developed anxiety, fear, depression, disorders of memory and aphasia,
untidiness appeared 5 days ago. Set a diagnosis.
a. Dementia of AIDS
b. * Patient has еncephalopathy (AIDS-related complex)
c. Organic psychosis
d. Anxiously depressed syndrome for HIV infection
e. Somatoform depression
For a man 30 years old, it was discovered positive reaction on HIV half-year ago. Last 3
months complaints of general weakness, fatigueability, somnolence, chest pain. Last 2
weeks with anxiety, fear, depression, disorders of memory and aphasia, untidiness
appeared 5 days ago. What is the treatment?
a. Acylovir
b. Cerebroprotector
c. Antidepressants
d. Neuroleptic
e. * Zidovudin, didanosin
For a patient 35 years old, after 4-months of treatment by isoniaside - аdynamia, icterus,
pain in right hypochondric appeared. The liver is megascopic. In blood activity of
enzymes of AlAT is increased in three times, AsAT in two times. Bilirubin of blood - 122
mcmol/l (conjugated - 82, unconjugated - 40). НBs-аntigen is not found out. What is the
diagnos?
a. Calculary cholecystitis
b. Hepatocirrhosis
c. Acute viral hepatitis
d. Chronic active hepatitis
e. * Toxic hepatitis
HIV positive patient, 28 years old, drug addict, complaints of dyspnea, unproductive
cough, fever with 37,5 °C for 2 months.Objectively: skin is pale, hyposthenic breathing in
lungs, especially in lower lobe, short breathing 24 per min. Roentgenologic chromophilic
bilateral infiltrations as “wings of butterfly”. What is the most effective treatment:
a. Antibacterial preparations. Inhibitors of transcriptase
b. Inhibitors of proteases. Inhibitors of transcriptase
c. Transplantation of marrow. Inhibitors of transcriptase
d. Antiviral gamma-globulin. Vitamins of A, C groups. Inhibitors of proteases
e. * Inhibitors of proteases. Inhibitors of transcriptase. Antibacterial drugs
In 2 months after returning from India, where she often drank unboiled water; the 23-yearold pregnant woman felt nauseated, strong general weakness, head acke appeared. Later
the temperature of body rose to 38,6 °C, which stuck to within a week. icterus appeared on
the 6th day, the general condition continued to worsen. On the 12th day of illness the
general condition became severe. EuphoriA. Vomiting at night. Complete fastidium.
Bright icterus, signs of hemorragic syndrome, tachycardiA. BP 110/60 mm Hg, body
temperature 37,8 °C. The liver is insignificantly megascopic, soft, painfull, spleen +2 cm
There is neutrophilic leykocytosis in general blood analysis. General bilirubinum of blood
up to 570, to the line - 300, activity of ALAT is enhanceable in 100 times, timol test of 26
units., urea - 2,1 mmol/l. With most probability for a patient:
6. Man 30 years old, drug addict, takes drugs intravenously. Has been taking drugs for 12
years Complaints of weakness, moderate icterus, weight in right subcostal region. The
state became gradually worse. Biochemical indexes: general bilirubin 28,2 mcmol/l;
ALAT 1,0, ASAT 0,8 . Will you define a diagnostic method which is expedient to conduct
for establishment of etiologic diagnosis?
a. Biopsy of liver
b. * Polymerase chain reaction (PCR)
c. Enzymes of liver
d. Computer tomography
e. Immunological tests
7. On a background of prolonged treatment for an HIV patient, appeared ulcer on the mucous
of mouth. During objective examination, the doctor found erosions on the mucous,
hyperemic and filling out mucous of oral cavity, tongue without raid, with a smooth
surface. About what complication is possible to think?
a. Leptospirosis
b. Acute herpes
c. Stevens-Johnson syndrome
d. * Candidos stomatitis
e. Layel syndrome
8. Patient P., 21 years old, complaints of diarhhea that lasts for one and a half months,
changes sometimes, on emptying there are admixtures of blood and mucus, 13 kg loss of
body mass, weakness, subfebrile temperature of body, recurrent herpes. It is discovered
generalised lymphadenopathy, increase of liver size by 2 cm. Blood test: Er 4,4*1012 g/l,
Hb 115 g/l, ESR - 15 mm/hr, L 10,0*109 g/l, е 2 %, b 6 %, n 61 %, s 17 %, m 3 %,
atypical mononuclears 6 %. What is most credible diagnosis?
a. Shigellosis
b. * AIDS
c. Infectious mononucleosis
d. Salmonellosis
e. Ameobiasis
9. Sick O., 25 years old, complaints of considerable fatigue, fever up to 39 °C profuse
sweating, lethargy. Objectively: all groups of peripheral lymphatic node are enlarged,
candidiasis of oral cavity, herpetic pouring out with megascopic on lips. In the global
analysis of blood: leucocytes 3,3*109 g/l, ESR – 15 mm/hr, B-lymphocyte 12 %, a decline
of CD4 to 600/mm. What is most credible diagnosis is?
a. Herpetic infection
b. Malaria
c. Urogenital chlamydia
d. * HIV/AIDS
e. Acute brucellosis
10. Student L, 20 years old, complaints of considerable fatigue, fever up to 39 °C profuse
sweating, lethargy. Objectively: all groups of peripheral lymphatic node are enlarged,
candidiasis of oral cavity, herpetic pouring out with megascopic on lips. In the global
analysis of blood: leucocytes of 6,2*109 g/l, ESR 15 mm/hr, B-lymp 12 %. What is most
credible diagnosis is?
a. Megakaryoblastoma
b. Malaria
c. * HIV/AIDS
d. Urogenital chlamydiasis
e. Brucellosis
11. Patient T., 35 years old, appeared to the doctor on the 8th day of gradual development of
illness with complaints of general weakness, rapid fatigue, dark color of urine. In the
morning noticed an icterus. On examination temperature of body 36,8 °C. Found out the
increasing of liver +3 sm. The changes of what laboratory index most informing at this
illness?
a. *AlAT
b. Hemodiastases
c. Protrombin index
d. Cholesterol
e. Alkaline phosphatase
12. The sick complaints of a prolonged cough, more than half-year, rising temperature of body
to 38 °C, enlargement of peripheral lymphnode, frequent herpetic wide-spread pouring out
with considerable lowering of body mass. In іmmunogram correlation of T-helper to Tsuppression is 0,3. It takes place because the infestant:
a. Infects cells with the receptors of CD22
b. Induces proliferation of Т-helpers
c. Induces proliferation of T-suppressors
d. Infects cells from receptor CD8
e. * Infects cells from receptor CD4
13. HIV-positive patients, 28 years old, a former injecting drug user, complained of shortness
of breath, nonproductive cough, temperature rise up to 37,5 °C for 2 months. Objective:
pale skin, lungs auscultated respiratory depression, especially in lowback, dyspnea 24 for
1 min. Radiographically bilateral infiltrates in the form of “butterfly wings”. Choose the
most effective approaches to treatment:
a. Antbacterial drugs. Transcriptase inhibitors
b. Protease inhibitors. Transcriptase inhibitor
c. Bone marrow transplant. Transcriptase inhibitors
d. Antiviral gamma globulin. Vitamins B, C. Protease Inhibitors
E.* Protease inhibitors. Transcriptase inhibitors. Antibacterial drugs
14. During influenza epidemic, a patient in the hospital arrived 43 years with complaints of
fever above 38 °C, malaise, in the analysis of blood on the outpatient phase of the survey
neutrophilic leukocytosis 12.0 * 109 g/L, a shift to the left, ESR 50 mm / hour. As it
turned out, a fever with a few interruptions lasted for about 3 months, then lost 13 kg in
body weight with normal appetite. Catarrhal symptoms are minor. Liver to 2 cm below the
costal arch. Other abnormalities are not detected. What kind of disease should think of
first?
a. Flu
b. Pneumonia
c. Chronic hepatitis in the acute stage
d. HIV infection
e. *AIDS
15. Patient was taken to a hospital after car accident in critical condition, caused by shock,
bleeding. Before emergency blood transfusion in order to prevent HIV transmission
everybody should consider:
a. Health reasons
b. The conclusion of the panel of doctors
c. Agree of the patient (or his family)
d. The results of a survey of emergency donor HIV (IHA)
e. *All the above listed
16. 33 years patient comes in with complaints of fever above 38 °C, and general malaise. As it
turned out, a fever with a few interruptions lasted for about 2 months, then lost of body
weight to 16 kg. Periodically bowel dysfunction. Palpable enlarged lymph nodes in the
axillary regions on both sides of the neck. Other abnormalities are not detected. What
changes would expect to find in the study of blood?
a. Anemia
b. Leukopenia
c. Hypolymphemia
d. Thrombocytopenia
e. *All the above listed
17. 33 years patient with complaints of fever above 38 °C, and general malaise. As it turned
out, a fever with a few interruptions lasted for about 2 months, then lost of body weight to
16 kg. Periodically bowel dysfunction. Palpable enlarged lymph nodes in the axillary
regions on both sides of the neck. Other abnormalities are not detected. What changes
would expect to find in the study of blood?
a. Leukopenia
b. Decrease the number of CD4
c. Decrease immunoregulatory index
d. Increase ESR
e. *All the above listed
18. 40 years sick with complaints of general weakness, sweating, wasting a 12 kg, often longterm diarrhea and recurrent respiratory infections. Objectively: generalized
lymphadenopathy, oral candidiasis, fecal mucus and blood. B blood reduced the number
of T-helper cells, T4/T8 less than 0.5. What is the most likely diagnosis?
a. Dysentery
b. Amebiaz
c. *AIDS
d. Tuberculosis
e. Cytomegalovirus infection
19. Student 20 years during the month has been concerned a significant fatigue, increased
body temperature 39 °C, profuse perspiration, emaciation. Objective: increasing of all
peripheral lymph nodes, oral candidiasis, herpes on lips. General blood analysis:
leukocytes 3.3 x 109 / l, ESR 15 mm/hour, B-lymphocytes 12%. What additional test is
helpful in diagnosis?
a. Biological assay in guinea pigs
b. Determination of the number of T-lymphocytes
c. *Determination by IFA antibodies in response to antigens or RIA HIV
d. Burne test
e. Leukocyte migration inhibition test
20. Patient 18 years intravenous drugs user, complaints of a cough lasting more than 4
months, Increasing of body temperature to 38 °C, treatment of “interstitial pneumonia” is
not effective. There have been several episodes of widespread herpes, decreasing of body
weight. The study will identify all immunograms authentic:
a. Increasing of T4-helper lymphocytes
b. Increase immunoregulatory index T4/T8
c. *Decrease the number of T4-helper lymphocytes
d. Increase the absolute number of T lymphocytes
e. Improving performance of delayed-type hypersensitivity
21. Patient 18 years intravenous drugs user, complaints of a cough lasting more than 4
months, Increasing of body temperature to 38 °C, treatment of “interstitial pneumonia” is
not effective. There have been several episodes of widespread herpes, decreasing of body
weight. The study will identify all immunograms authentic:
a. Increasing of T4-helper lymphocytes
b. Increase immunoregulatory index T4/T8
c. *Reducing the number of T4-helper lymphocytes
d. Increase the absolute number of T lymphocytes
e. Improving performance of delayed-type hypersensitivity
22. Patient 23 years old, fell ill at the end of summer, when the temperature rose to 37.2 °C,
mild headache, and weakness. Up to 7 days of illness treated at the outpatient ARI.
Condition worsened, admitted to hospital. The body temperature of 40 °C, a pale, severe
weakness. Pulse 96 per 1 min, blood pressure 110 and 70 mm Hg. Tongue with imprints
of teeth, the abdomen is soft, swollen, hepatosplenomegaly. Constipation. There is
congestion and hyperplasia of the tonsils, on the surface of the right - the ulcer. What
disease we may think about?
a. Vincent-Simanovsky tonsillitis
b. Herpangina
c. *Necrotizing tonsillitis
d. Kissing disease
e. Angina Duge
23. Patient 30 years after exposure to the earnings to consult a dermatologist complaining of
rashes around the skin surface. Over the last three months have seen a sharp emaciation,
weakness, constant low-grade fever. Presens what disease we will exam the patient?
a. Syphilis
b. TB infection
c. *AIDS
d. CMV infection
e. Malignant neoplasms of the skin
24. Patient B., 28, a prostitute, 2 years living in the Middle East. Admitted to hospital with
complaints of weakness, cough, sputum gray, burning in the chest, shortness of breath,
night chills, fever, sweating, pain in the joints. Patients lost of weight near 12 kg during 5
months, decreased appetite. On examination: the soft palate, temples, back of the throat
covered by white raids. Lymphadenopathy. Hepatosplenomegaly. In the blood: anemia,
leukocytosis, shift formula to myelocytes, lymphopenia, ESR 60 mm/hour. Your
diagnosis?
a. Visceral leishmaniasis
b. *HIV infection. Pneumocyst pneumoniA. Esophageal candidiasis
c. Brucellosis
d. Lymphogranulomatosis
e. Esophageal carcinoma
25. Patient K., 29 years old, not vaccinated, complaints of the increasing temperature to 37,2
°C, headache, weakness, pain in the throat which increase with swallowing, acute disease
began two days ago. Objectively: pale skin. Pulse 110 for 1 min. Heart sounds are
muffled, especially the first one. Gentle systolic murmur. AP 100 and 65 mm Hg. Tonsils,
soft palate, uvula swollen. On the surface of the tonsils, more to the right, pale-gray film,
which spreads their borders, removed hard, dense. The regional lymph nodes are enlarged.
Subcutaneous edema, which reaches to the middle of the neck. Specify the most likely
diagnosis:
a. *Diphtheria oropharynx
b. Angina
c. Kissing disease
d. Paratonsillar abscess
e. Acute leukemia
26. A patient, 16 y.o., complaints of general weakness, fever, sore throat. Objectively:
oropharyngeal mucosa bright red, the tonsils - white raids are removed easily, an
increasing of all groups of lymph nodes, 1-3 cm in diameter, firm, elastic
maloboleznennye, not welded together. The liver is enlarged by 3 cm, the spleen - 1 cm in
blood leukocytosis, atypical mononuclear cells – 20 %. What is previous diagnosis?
a. *Infectious mononucleosis
b. Acute lymphocytic leukemia
c. Acute streptococcal tonsillitis
d. Diphtheria
e. Adenovirul infection
27. Patient G., 24, complaines of sore throat, fatigue. Temperature 38 °C. Pulse 96 per minute,
flushing of the skin, hyperemia of mucous membranes of oropharynx. Enlargment of
tonsills, friable. Palpable enlargement submandibular lymph nodes. Spleen + 1 cm. What
method can confirm the diagnosis?
a. Paul-Bunnel, Wright-Heddlson, Hoff-Bauer reactions
b. *Paul-Bunnel, Lovrika-Volner, Hoff-Bauer reactions
c. Paul-Bunnel, Wright-Heddlson, Lovrika-Volner reactions
d. Paul-Bunnel, Burne, Hoff-Bauer reactions
e. Paul-Bunnel, Hoff-Bauer, Wright-Heddlson, Lovrika-Volner reactions
28. A patient 15 years old, was admitted to the hospital on the third day of illness with
complaints of pain in the throat when swallowing, fever. The disease is linked to the use
of raw molokA. On examination: body temperature of 38 °C, hyperemic pharynx,
enlargement of right tonsill, swollen, with necrotising layering grayish-white color. Right
neck lymph node on the size like a hen’s egg, mild painful, with clear contours, the skin
over it is not changed. What is preliminary diagnosis?
a. Lacunar tonsillitis
b. Diphtheria of oropharynx
c. Vincent-Simanovsky tonsillitis
d. Duge-Strumpell tonsillitis
e. *Tonsilar-bubonic form of tularemia
29. The patient became ill 17 years after acute hypothermia: 39,5 ° C fever, headache and
muscle pain, sharp pain in the throat when swallowing, aching pain. Hypertrophy of
tonsils, bright red, in the gaps of the purulent deposit. Diagnosed as lacunar tonsillitis.
What is most effective laboratory method of investigation in this case?
a. Biochemical blood
b. Microscopic examination of the pus gaps
c. Immunological study of blood
d. Biological research method
e. *Bacteriological study of the content of gaps
30. 28 years old patient complaints of increasing of lymph nodes during six months,
progressive weakness, sweating, recurrent fevers to 38 °C. During several years has used
drugs intravenously. Malnutrition, widespread seborrheic dermatitis, painless palpable
enlarged cervical, axillary and inguinal lymph nodes of up to 2-2.5 cm What research is to
assign at first?
a. Lymph node biopsy
b. Blood cultures
c. *Blood test for antibodies to HIV
d. Sternum puncture
e. Examination of blood for antibodies to the Epstein-Barr virus
31. A patient 30 years old, was taken to the infectious department by ambulance in serious
condition. On examination: the temperature of 38,9 °C, the patient is exhausted, lethargic,
pale skin, in the peripheral vein - injection marks. In the mouth - a lot of carious teeth,
white attacks on the mucosA. By an increase in the axillary, inguinal, supraclavicular
lymph nodes, not painful, not welded to the underlying fiber. In the lungs - breathing
weakened, respiratory rate 30 per minute., Pulse 92 / Min., AD 100 and 65 mm Hg. Heart
sound muffled, rhythmic, hepatosplenomegaly. In history - injecting drug user for 3 years.
What is your diagnosis?
a. Toxic hepatitis
b. Sepsis
c. Lymphogranulomatosis
d. Brucellosis
e. *AIDS
32. A patient B., 18 years old, was admitted to hospital with complaints of headache,
weakness, fever up to 37,5-38,2 °C for 6 days, the pain in the throat. Objective: increasing
of all lymph nodes, 1-3 cm in diameter, flexible, not painfull, not welded together. The
liver is enlarged by 3 cm, the spleen - 1 cm in blood leukocytosis, virotsity - 15%. Likely
diagnosis?
a. Tonsillitis
b. Adenovirus infection
c. *Infectious mononucleosis
d. Diphtheria
e. Acute lymphocytic leukemia
33. Patient B., 22 years old, desperately ill, mild pain in the throat during swallowing, nasal
voice. Objectively: the left tonsil and the arc is filmy coating. Diagnosed with lacunar
tonsillitis, appointed penicillin and rinse 2% soda solution. Till the second day covers
spread on the soft palate and uvulA. Swelling of the neck on to the collar bone, nasal
voice. Muffled heart sounds, pulse 90 per minute. AP 95 and 65 mm Hg. What is the
most expedient treatment in this case?
a. Penicillin and diphtheria toxoid
b. Rinse 2% soda solution in combination with erythromycin peroral
c. Autohaemotherapy, hot compress and quartz tube
d. Disclosure and drainage paratonsillar fiber
e. *Antitoxic diphtheria serum and benzylpenicillin
34. Patient B., 34 years old, complained of fever, headache, aching joints, fever, sore throat,
worse when swallowing. Objectively: flushing of the oropharynx, tonsils hyperemic,
hypertrophied, on both - the necrotic areas of dark gray color, which was formed after the
delamination defect deep mucosa with bumpy bottom, enlarged and painful
submandibular lymph nodes. Put the diagnosis?
a. Diphtheria
b. Tonsillitis
c. Lacunar tonsillitis
d. Vincent's angina
e. *Necrotizing tonsillitis
35. C. become sick gradually. There was general weakness, fatigue, sore throat, abdominal
pain, and nauseA. Hospitalized on the 5th day of illness. Examination: body temperature
38 ° C, increased cervical, and submandibular lymph nodes. Skin or eyes subikterichny.
There have been isolated maculo-papular rash all over body. The deposits on the tonsils,
white-yellow, tongue coated with white covering, moderately distended abdomen,
hepatosplenomegaly. In blood leukocytosis, neutrophilic left shift, atypical mononuclear
cells 10 %, 10 % plasma cells. What is the preliminary diagnosis?
a. Viral hepatitis A
b. *Infectious mononucleosis
c. Typhus, paratyphoid diseases
d. Yersiniosis
e. Lacunar tonsillitis
36. The patient, age 20, complained of a sore throat when swallowing, weakness and headache
on the 2nd day of the disease. OBJECTIVE: Temperature 39,0 ° C, the pulse 110 for 1
min, oropharyngeal mucosa hyperemic, tonsils enlarged in size, loose, covered with
plaque that is easily removed, the surface after removal of the plaque does not bleed. What
kind of disease is necessary to think about?
a. Tularemia
b. Diphtheria oropharynx
c. -Vincent Angina Simanovsky
d. Kissing disease
e. * Acute streptococcal tonsillitis
37. In infectious hospital, patients who present with acute onset of disease, temperature of
39,9 °C, mild sore throat, swelling, and redness with slight cyanosis of the mucous
membranes of the oropharynx, the tonsils are dense, shiny, grayish deposits in the form of
a continuous film, which hard to remove, exposing a bleeding surface. Submandibular
lymph nodes were moderately enlarged. The patient should have what immediately
performed:
a. Swabs from the tonsils, nose or other areas to identify diphtheria bacilli
(culture)
b. ELISA
c. * Microscopy (Neisser staining)
d. Blood culture
e. Serology (RPHA with diphtheria diagnostic tools)
38. HIV-positive patient, 28 years old, a former injecting drug user, complained of shortness
of breath, nonproductive cough, temperature rise to 37.5 °C for 2 months. OBJECTIVE:
pale skin, lungs auscultated respiratory depression, especially in the lower parts of the
posterolateral, dyspnea 24 for 1 min. Radiographically bilateral infiltrates in the form of
"butterfly wings." Choose the most effective approaches to treatment:
a. Antibacterials. Transcriptase inhibitors
b. Protease inhibitors. Transcriptase inhibitors
c. Bone marrow transplant. Transcriptase inhibitors
d. Antiviral gamma globulin. Vitamins B, C. Protease inhibitors
e. * Protease inhibitors. Transcriptase inhibitors. Antibacterials
39. The 40 year-old driver is sick for two months - general weakness, sweating, lost 12 kg,
often long-term diarrhea and recurrent respiratory infections. OBJECTIVE: generalized
lymphadenopathy, oral candidiasis, fecal mucus and blood. In the blood, reducing the
number of T-helper cells, T4/T8 less than 0.5. What is the most likely diagnosis?
a. Dysentery
b. Amebiasis
c. * AIDS
d. Tuberculosis
e. Cytomegalovirus infection
40. Gynecologist, during the operation emergency of a young woman accidentally hurt
himself with a scalpel. After 3 days, a reply came from the laboratory with positive
antibodies to HIV in the blood of a patient taken before surgery. Priority actions in respect
of the operated physician:
a. Debridement
b. Immediate laboratory testing for HIV
c. Post-exposure prophylaxis with antiretroviral drugs by the results of
laboratory testing for HIV
d. * post-exposure prophylaxis with antiretroviral drugs
e. Observation
41. Girl 8 years old, was admitted to hospital with complaints of general weakness, fever,
sore throat. OBJECTIVE: oropharyngeal mucosa is bright red, the tonsils have white
plaques which are removed easily, shows an increase in all groups of lymph nodes, 1-3 cm
in diameter, firm, elastic slightly painful, not grouped together. The liver is increased by 3
cm, the spleen 1 cm in the blood - leukocytosis, plasma cells 20%. Likely diagnosis?
a. Acute lymphocytic leukemia
b. * Infectious mononucleosis
c. Angina
d. Diphtheria
e. Adenovirus infection
42. Female 24 years old, went to the doctor due to prolonged fever, night sweats. Over the
past three months lost 7 kg. Was promiscuity. An objective study found an increase in all
groups of lymph nodes, hepatoileal syndrome. Leukocytes-. 2.2 *109 / l. What disease
should be suspected?
a. Hroniosepsis
b. Lymphogranulomatosis
c. Tuberculosis
d. Kissing disease
e. * HIV infection
43. Woman, 28 years old, complaining of general weakness, headache, fever up to 39-40 ° C,
sore throat for 3-days. Her husband has strep throat. OBJECTIVE: pale skin, lips cyanotic.
Hyperemia of oropharynx with cyanosis, swelling of tongue, palatine arches, tonsils. On
the surface of the tonsils - solid thick white with pearl shade plaque, which are difficult to
remove with a spatula, after removing plaque, mucosa bleeds. Enlarged submandibular
lymph nodes. Swelling of the neck. TachycardiA. AP 105 and 65 mm Hg.. What is the
most likely diagnosis?
a. Acute leukemia
b. Angina
c. Kissing disease
d. * Diphtheria oropharynx
e. Adenovirus infection
44. Patient N., 23, was hospitalized in infectious department and diagnosed with
oropharyngeal diphtheriA. On examination, the tonsils of the throat have solid thick white
with pearl shade plaque, which are difficult to remove with a spatula, after removing
plaque, mucosa bleeds. When can the patient be discharged?
a. After the disappearance of clinical symptoms
b. After the disappearance of clinical symptoms and get a negative culture
results mucus tonsils and nasal diphtheria at two-day intervals
c. After the disappearance of clinical symptoms and get negative results of two
bacteriology mucus tonsils and nasal diphtheria at a three-day intervals
d. After the disappearance of clinical symptoms of diphtheria and get three
negative results of bacteriological studies of mucus from the nose to the
tonsils and diphtheria three-day intervals
e. * After the disappearance of clinical symptoms of diphtheria and get negative
results of two bacteriology mucus tonsils and nasal diphtheria at two-day
intervals
45. Patient K., 40 years old, complained of fever, night sweats, rapid weight loss, muscle
aches, sore throat, joint pain, photophobia, recurrent diarrheA. On examination revealed
generalized lymphadenopathy. Most likely the following diseases:
a. Lymphogranulomatosis
b. Tumor of the digestive system
c. * AIDS
d. Tuberculosis
e. Salmonellosis
46. Patient received a blood transfusion from an unverified voluntary donor. Remains of
blood sent to the station of blood transfusion, where the examination of the donor
antibodies to HIV. Your actions for recipient:
a. Observation
b. laboratory testing for HIV
c. Emergency antibiotic
d. * Post-exposure prophylaxis with antiretroviral drugs
e. all the above
47. After months of unsuccessful treatment for pneumonia in a 7-month-old baby antibodies
to HIV was revealed. During pregnancy, the mother of the child was examined twice by
HIV with negative results, the last at 24 weeks of pregnancy. Childbirth was without
complications. Baby breastfeeding. What is the most likely route of HIV infection of the
child:
a. HIV-infected mother
b. Vaginall birth
c. Breast-feeding
d. Parental interventions after birth
e. * All the above
48. After months of unsuccessful treatment for pneumonia in a 7-month-old baby antibodies
to HIV was revealed. During pregnancy, the mother of the child was examined twice by
HIV with negative results, the last at 24 weeks of pregnancy. Childbirth was without
complications. Baby breastfeeding. What is the most likely route of HIV infection of the
child:
a. * HIV infection of the mother
b. Laboratory examination of the child
c. Parental interventions during treatment of pneumonia
d. AIDS of the father
e. All variants are possible
49. . 7 -month old child with pneumonia had antibodies to HIV. During pregnancy, the mother
of the child was examined twice by HIV with negative results, the last at 24 weeks of
pregnancy. What is possible reasons of HIV of the mother:
a. * HIV infection during pregnancy
b. Presence of AIDS of the father
c. Infection during care of a sick child
d. Contamination during breast-feeding
e. All variants are possible
50. 7-month old child with pneumonia had antibodies to HIV. During pregnancy, the mother
of the child was examined twice by HIV with negative results, the last at 24 weeks of
pregnancy. What terms of final investigation of mother for presence of HIV infection:
a. Negative screening result in the first trimester
b. Negative screening result in the II trimester
c. * A negative screening result after 28 weeks
d. Negative screening result in I and II trimesters
e. All variants are possible
51. . Patient 37 years old, slower muscle and motor responses during 3 monthes have been
developed, impaired memory for names, addresses, disrupting cognitive function, there
was stiffness, drowsiness, sloppiness, indifference to others and to his state. Loss of
weight 12 kg. Intermittent fever. Generalized lymphadenopathy. What is clinical
diagnosis.
a. Tuberculosis
b. Herpes encephalitis
c. * AIDS
d. Syphilis
e. Lymphocytic Leukemia
52. 12 years old boy with catarrhal symptoms observed an increase of all groups of lymph
nodes, injection of sclera, hyperplasia of the tonsills with white loose build-up on them in
the form of islands, there are single roseolous-papular rash, increasing of liver and spleen.
What additional research should be appointed?
a. Ultrasound of the abdomen
b. Puncture of the lymph nodes followed by microscopy
c. * Blood on atypical mononuclear
d. Blood culture
e. RPHA with influenza viruses
53. 96. 30 years old person, six months ago, was found positive for HIV. 3 months complaints
of general weakness, fatigue, drowsiness, headache. 2 Weeks - anxiety, fearfulness,
depression. 5 days ago, there were memory disorders and aphasia, tremor, impaired fine
movements, myoclonus, sloppiness. What additional tests need to be carried out for the
diagnosis of the nervous system?
a. Ganciclovir
b. Cerebroprotector
c. Antidepressants
d. Antipsychotics
e. * Zidovudine, didanosine
54. 97. 30 years old person, six months ago, was found positive for HIV. 3 months complaints
of general weakness, fatigue, drowsiness, headache. 2 Weeks - anxiety, fearfulness,
depression. 5 days ago, there were memory disorders and aphasia, tremor, impaired fine
movements, myoclonus, sloppiness. What additional tests need to be carried out for the
diagnosis of the nervous system?
a. Ultrasound
b. * Computer Tomography
c. Doppler
d. Blood analysis for HIV
e. Pathophisiology study
55. . The patient has been diagnosed with herpes zoster. A patient suffering from childhood
diabetes, father and grandfather - asthmA. A few days ago, suffered food poisoning. 7
days took analgin due to headaches. Antibodies against HIVwas find. What factor has the
greatest pathogenetic significance?
a. *HIV-relative immune deficiency
b. Food poisoning
c. An allergic disease in relatives
d. Diabetes
e. Analgesics
56. Sick C., 20 years old, was admitted to the hospital with complaints of the frequent
emptying without stomach-ache, vomiting without nausea, pain in calf muscle.
Objectively: Temperature of body is 36,2 oC. Skin is pale, cold, tongue is dry, voice is
hoarsed. The stomach is pulled is not painful at palpation. Emptying resemble with “ricewater”. Are there what terms of discharge of patients from the hospital at such illness?
a. A.*Triple negative results of bacteriological examination of excrements
b. Double negative results of bacteriological examination of excrements
c. Single negative result of bacteriological examination of excrements
d. Single negative results of bacteriological examination of excrements
and urine
e. Double negative results of bacteriological examination of excrements and
urine
57. A sick 18 years, hospitalized in an infectious department with diagnosis of cholera,
dehydration of IV degree. What measures are possible primarily?
a. Oral rehydration by glucose solutions
b. Tetracycline
c. *Intravenous stream introduction of salt solutions
d. Proceeding in the normal microflora of intestine
e. Intravenous stream introduction of sodium chloride solution
58. Patient B., 20 years, became ill acutely. The unexpected severe diarrhea appeared with
frequent vomiting, with plenty of vomits. The patient arrived from a South-east Asia
countries. He has temperature 36,1 oC during 3 weeks. Abdomen is pulled, not painful.
Stool is a rice-water. What most reliable changes in blood will be present?
a. Increasing amount of erytrocytes, leucocytes, relative closeness of plasma of
blood, hyperkalemia, metabolic acidosis
b. Decreasing of amount of erytrocytes, leucocytes, relative closeness of plasma
of blood, hypokalemia, metabolic acidosis
c. Decreasing of amount of erytrocytes, leucocytes, increase of relative
closeness of plasma of blood, hyperkalemia, metabolic acidosis
d. *Decreasing amounts of erytrocytes, leucocytes, relative closeness of plasma
of blood, metabolic hypokalemic acidosis
e. Increasing amount of erytrocytes, leucocytes, relative closeness of plasma of
blood, hypokalemia, metabolic alkalosis
59. A sick, 29 years, emptying watery stool repeatedly, frequent vomiting. Objectively: total
cyanosis, dryness of mucous membrane, turgor of skin is decreased Temperature of body
35,2 oC. Pulse in radial artery is not determined. Tachypnea, musle spasm, urination is
absent for 6 hours. What is the state of the patient?
a. Dehydration of IV degree
b. Dehydration of I degree
c. Infectious-toxic shock
d. Anaphylactic shock
e. *Uncompensated hypovolemic shock
60. Patient with complaints of a general weakness, diarrhoea appeared to the doctor.
Objectively: temperature of body 36,4 oC, skin covers clean, acrocyanosis, tongue is dry,
stomach is soft, not painful, emptying is abundant, watery, with the supernatant flakes of
white color, odourless and admixtures. For clarification of diagnosis culture of stool was
made on ResselyA. What is the color of culture chang?
a. From blue to green
b. From yellow to green
c. From green to yellow
d. From yellow to blue
e. *From blue to yellow
61. For a patient E., 37 years old, a disease began rapidly, 6 hours the frequent liquid
emptying appeared onto the plain rise of normal temperature of body, vomit joined then.
At an inspection: aphonia, eyes are hollow, pulse frequent, threadlike, low blood pressure
cramps appeared in lower extremities. Liver and spleen are not enlarged. Choose
preparations for starting etiotropic therapy.
a. Tetracyclin, erythromycin, levomycetin, ciprofloxacin or imodium
b. Erythromycin, levomycetin, benzylpenicillin or imodium
c. Tetracyclin, erythromycin, levomycetin, benzylpenicillin or ofloxacin
d. Tetracyclin, erythromycin, levomycetin, or bifi-form
e. *Levomycetin, erythromycin or ciprofloxacin
62. Citizen of Pakistan, 30 years became ill rapidly with frequent watery stool which appears
like a rice water. Objectively: temperature of body 35.4 oC, skin is cold, acrocynosis,
elasticity of skin and turgor is decreased. By what method may estimate the degree of
dehydration?
a. To examine an eyeballs
b. To check central venous pressure
c. To define the level of urea and creatinine in blood
d. *To check the loss of blood plasma
e. To check the pressure of blood
63. For a patient E., 37 years, body weight of 70 kg, frequent liquid emptying appeared with
rise of body temperature, frequent vomiting joined in later. At an inspection: aphonia,
eyes are hollow, pulse with frequent threadlike, blood pressure is not determined,
tachypnea, total cramps. Liver and spleen are not enlarged. What volume of salt solutions
must be entered for primary rehydration?
a. *7 L
b. 3,5 L
c. 5 L
d. 10 L
e. 2 L
64. Sick C., 23 years, ill from 3 days after returning from IndiA. The disease has begun with
the liquid emptying which looks like a watery, after wards joined with multiple vomiting,
expressed weakness and cramps. The state progressively got worse and was admitted
within 12 hour in an infectious hospital in a grave condition. The eyes of person is
sharped, skin is cold, acrocynosis temperature of body 35,5 oC. Aphonia, cramps of hands
and feet. The fold of skin falls out through abdomen. stomach is pulled in, no pain at
palpation. Blood pressure is 70 and 30 mm of Hg. Pulse on radial artery is not determined,
urination and emptying are not good. What is the most appropriate diagnosis?
a. Intestinal echerihiosis
b. Salmonellosis
c. Shigellosis
d. *Cholera
e. Amebiasis, intestinal form
65. Patient with cholera on a background treatment has signs of hyperkalemiA. What solution
must be applied for futher rehydration therapy?
a. Neohemodez
b. *Disalt
c. Chlosalt
d. Polyhybrid
e. Lactosalt
66. Sick I., 25 years, returned from rest in Egypt. Early in the morning diarrhea began.
Emptying each 1-1,5 hrs, abundant watery, without mucus and blood. Abundant vomiting
also appeared. The temperature of body at first rise to 37,3 oC, afterwards became 35,5
oC, stomach-aches negative. Delivered in an infectious department. What is first aid?
a. Treatment with sorbents
b. Polyhybrid solution intravenous
c. 5 % solution of glucose intravenous
d. Fresh-frozen plasma intravenous
e. *Salt solutions intravenous
67. Sick, 20 year old, apeared to the hospital with complaints offrequent emptying without
stomach-ache, vomit without nausea, pain in calve. Objectively: temperature of body –
36,2 oC. A skin is pale and cold, a tongue is dry, voice getting hoarse. A stomach is pulled
in, painless, emptying like a “rice-water”. What are the criterias for discharging of the
patient from the hospital?
a. Double negative bacteriological examination of excrements
b. Single negative bacteriological examination of excrements
c. *Triple negative bacteriological examination of excrements
d. Negative bacteriological examination of excrement and urine
e. Double negative bacteriological examination of excrement and urine
68. For a patient, the disease begun rapidly with a chill, increase of temperature to 39 oC,
vomitting, pain in epigastrium, diarrhea with the watery stinking emptying. 6 hours before
the disease he ate a raw egg, potato with the braised meat and drink juice. What exciter
did cause the similar state probably?
a. Shigella
b. Collibacillus
c. Campylobacter
d. *Salmonella
e. Citrobacter
69. Patient C., 17 years old, worker in vegetable garden. Became sick 2 days ago. The disease
is related to use of meal of dirty root crop. Disease begun with chill, fever of 38.1 oC, had
pain in abdomen and in the muscles of all groups and joints, weakness, nauseA. Emptying
is liquid, viscid, with a strong smell, ordinary color. At objective examination: sclera,
conjunctivitis, hyperemia of soft palate, “raspberry” tongue. At palpation of abdomen –
moderate pain in epigastrium and in right iliac areA. What will be the diagnosis?
a. Cholera
b. Dysentery
c. Salmonellosis
d. *Scarlet fever
e. Viral hepatitis
70. Patient with cholera has bradycardia, low blood pressure, weakness. What is most
important factor in the given clinical situation?
a. . Hypocalcemia
b. Hypopotassium
c. Hyponatremia
d. Hypernatremia
e. *Hyperpotassium
71. Patient E, 47 years, became ill in 4 days after returning from Egypt. A disease begun with
the liquid emptying which look like a watery kind afterwards, joined with frequent
vomiting, expressed weakness and fatigue. The state progressively got worse and within
18 hrs was admitted in an infectious department in a very grave condition. The lines of
person are strained, skin is cold, cyanosis, temperature of body 35.5 oC. Aphonia, cramps
of hands and feet. The skin fold falls out through the abdomen. A stomach is pulled; no
pain at palpation. Blood pressure is 70 and 30 mm of Hg. The pulse on radial artery is not
determined, urination and emptying normal. Weight of patient at admission to the hospital
was 60 kg. What is the degree of dehydration of the patient?
a. I
b. II
c. *IV
d. III
e. It is not
72. Sick X., which returned from vacations from Turkey within 5 hrs in the morning, diarrhea
began. Emptying each 1-1.5 hrs, with abundant watery without mucus and blood. In future
bloodless watery, abundant vomitting appeared through 12 hrs. The temperature of body
at first rise to 37.5 oC afterwards became 35.7 oC is delivered in an infectious department.
What disease did you suspect?
a. Echeriosis
b. Food poisoning|
c. *Cholera
d. Salmonellosis
e. Balantidiasis
73. Sick P., 25 years old, presents with frequent vomitting. Objectively: dryness of skin and
mucous membranes, brief cramps in gastrocnemius muscles, the temperature of body is
normal, voice is hoarsed, moderate tachycardia and hypotension. The compensated
metabolic acidosis is marked. About what degree of dehydration is it possible to think?
a. IV
b. II
c. *III
d. I
e. There is no dehydration
74. An unconscious patient is delivered in the intensive department. Pale dark circles around
eyes. Skin is cold with sticky sweat. Temperature of body 35,6 oC. Pulse 140/min and
weak. Blood pressure is 40 /0 mm of Hg. Tongue is dry. Emptying is involuntary and
“watery”, vomited twice. What infusion must be given as intensive therapy.
a. Albumin
b. Rheopoliglykin
c. 5 % glucose solution
d. *Polyionic salt solutions
e. 10 % glucose solution
75. A farmer O., 50 years old, hospitalized in a moderate condition with complaints of dryness
in mouth, multiple vomitings, pain in the epigastriums and frequent watery stool. First aid
to the patient is?
a. Hypersaturated oxygen
b. Transfusion of fresh-frozen plasma
c. Tetracyclin
d. *Intravenous introduction of salt solutions
e. Introduction of polyhybrid
76. Patient P, 35 years old, became ill within 2 days after returning from IndiA. A disease
begun with appearance of liquid diarrhea which resembled a watery appearance,
afterwards joined with multiple vomit, expressed weakness and fatigue. State
progressively got worse and within 12 hrs delivered in an infectious department in a grave
condition. The skin is cold, cyanosis, temperature of body 35.5 oC. Aphonia, cramps of
hands and feet. A skin fold falls out from abdomen. The stomach is pulled in, no pain at
palpation. Blood pressure is 70 and 30 mm of Hg. A pulse on a radial artery is not
determined, urination and emptying are normal. Weight of patient at admission to the
hospital was 80 kg. What volume of infusion solutions is needed for primary rehydration?
a. 10 liters
b. *8 liters
c. 4 liters
d. 6 liters
e. 2 liters
77. A patient L., who returned from Crimea, developed diarrhea at 5AM. Bowel movements
are each 1-1.5 hrs, watery, without mucus and blood. In 12 hrs a single episode of
vomiting developed. The temperature of body at first rise to 37.3 oC, stomach-aches is
present. he was examined by the doctor of first-aid and delivered to an infectious isolation
with the diagnosis of acute intestinal infection.Which disease is most probable for the
patient?
a. Intestinal echerihiosis
b. . Salmonellosis
c. Echeriosis
d. Food poisoning
e. *Cholera
78. The ill patient in severe state was delivered to infectious department with no
consciousness. Pale dark circles around eyes. Skin is cold, covered with sticky sweat.
Temperature of body 35.6 oC. Pulse 140/min and weak.Blood pressure is 40 and 0 m of
Hg. Tongue is dry. Emptying is involuntary and watery. Three times vomited like
“fountain”.What is the state of the patient?
a. Collapse
b. Infectious toxic shock
c. *Dehydrational shock
d. Cereblral edema
e. Intoxicated encephalopathy
79. Patient T., 22 years old, appealed to the hospital with complaints of the frequent stool
without the abdominal pain, vomits without nausea, pain in calves. Objectively: Т-36,2
°C. A skin is pale, cold, tongue dry, voice getting hoarse. An abdomen is pulled in,
painless. Emptying remind a “rice-water”. What terms of extract of patients from a
hospital at this illness?
a. Non-permanent negative bacteriological research of excrement
b. Double negative bacteriological research of excrement
c. Double negative bacteriological research of excrement and urine
d. Non-permanent negative bacteriological research of excrement and urine
e. *Triple negative bacteriological research of excrement
80. Patient N, 13 years old, hospitalized in the infectious department with a diagnosis of
cholera, severe dehydration of Ш-ІV stage. What measures are primary?
a. Setting of etiotropic antibacterial preparations
b. Stream intravenous introduction of salts solutions
c. Peroral rehydratation with glucose-salts solutions
d. Renewal of normal microflora of intestine
e. *Setting of enzyme preparations
81. A patient J., 23 years old, became ill suddenly. Profuse diarrhea with frequent and large
amount vomits. A patient arrived from one of countries of south-east Asia, where was
near 3 weeks. T 36,1 °C. An abdomen is pulled in, painless. The stool has the appearance
of rice-water. What most reliable changes will be in a blood?
a. Decrease amount of erythrocytes, leucocytes, relative tightness of plasma of
blood, hypopotassemia, metabolic acidosis.
b. *Increase amount of erythrocytes, leucocytes, relative tightness of plasma of
blood, hypopotassemia, metabolic acidosis.
c. Decrease amount of erythrocytes, leucocytes, increase of relative tightness of
plasma of blood, hyperpotassemia, metabolic acidosis.
d. Increase amount of erythrocytes, leucocytes, relative tightness of plasma of
blood, hyperpotassemia, metabolic acidosis.
e. Increase amount of erythrocytes, leucocytes, relative tightness of plasma of
blood, hipopotassemia, metabolic alkalosis.
82. Patient C, 17 years old, became ill suddenly. Profuse diarrhea with frequent vomits
without nosier. A patient arrived from south-east AsiA. Temperature – 36,1 °C. An
abdomen is pulled in, painless. The stool has the appearance of rice-water. What diagnosis
is most reliable?
a. *Cholera
b. Dysentery
c. Salmonellosis
d. Esheryhiosis
e. Rotavirus gastroenteritis
83. To the internist appealed patient with complaints of weakness, diarrheA. Rested on a
south, where the cases of diarrhea were present. Objectively: t-36,4 °C, skin covers are
clean, acrocyanosis, tongue is dry, abdomen is soft, painless, emptying abundant, watery,
with the flakes of white color floating on a surface, without odor and admixtures. For
clarification of diagnosis sowing on the Ressels medium was made. How will the color of
medium change?
a. From yellow to blue
b. From yellow to green
c. From green to yellow
d. *From blue to yellow
e. From blue to green
84. The disease began sharply, 6 hours ago at a normal temperature appeared frequent liquid
emptying, then vomit joined. At the inspection: voice is soundless, eyes reddish, pulse
frequent, arterial pressure low, urine is not present, cramps appeared in lower extremities.
The heart and lungs without changes. A liver and spleen are not enlarged. Choose
preparations of ethiotropic therapy which can be used:
a. Tetracycline, erythromycin, levomycetine, gentamycine, ofloxacine
b. Erythromycine, evomycetine, gentamycine, ofloxacine, ciprofloxacin,
imodium
c. Tetracycline, erythromycine, levomycetine, benzyl-penicillin sodium salt,
ofloxacine
d. *Erythromycine, levomycetine, gentamycine, ofloxacine, ciprofloxacin
e. Erythromycine, gentamycine, ofloxacine, ciprofloxacine, imodium
85. The disease began sharply from diarrhea, that was accompanied by an abdominal pain.
The act of defecation brought facilitation. Emptying of green color, abundant, foamy, with
a strong unpleasant smell. Temperature of body is subfebrile. It is found out hyperemia
and graininess of soft palate. In blood: leucopenia, eozinophiliA. Violations of waterelectrolyte balance are moderate.Your diagnosis:
a. Cholera
b. Toxic food-borne infection
c. Salmonelliosis
d. Dysentery
e. *Rotaviral gastroenteritis
86. The seventh pandemic of cholera is caused by V. cholera El Tor. It begans in 1961 on the
Sulawesi island. However, to the epidemic of cholera arose up only in the countries of the
third world. It is known that cholera is classic bacterial infection with the fecal-oral
mechanism of transmission with the certain infectious dose of exciter. What is the basic
factor of risk, that is instrumental in the such uneven division of morbidity on countries?
a. Biological properties of exciter
b. Climate of country
c. *Social-economic conditions of population of country
d. Immune status of population
e. High development of industry and contamination of surrounding environment
87. Patient C., 36 years old, periodically goes on a business trip in Egypt, desperately ill.
Complaints of the increasing body temperature up to 39,4 °C with chills and sweating,
pain in the right under rib, emaciation. A slight jaundice. Increased liver density, painful.
In blood neutrophils leukocytosis, increased ESR. When USD revealed multiple liver
abscesses. What kind of illness need to think.
a. Legionellosis
b. Echinococcosis
c. . Ascariasis
d. Liver cancer with metastases
e. *Amoebiasis
88. Student A., 22 years old, ill for 3 weeks, a month after returning from Ethiopia: appeared
abdominal pain, liquid excrement, abundant, up to 10 times per day, in the form of
“raspberry jelly”, joined cramping in the abdomen, more right, growing in the bowel.
Over time the disease has lost 6 kg body weight. Your diagnosis.
a. *Intestinal amoebiasis
b. Shigellosis
c. Salmonellosis
d. Tumor bowel
e. Intestinal yersiniosis
89. Patient K., 40 years old, had complained of fever, night sweating, a significant decrease in
body weight, pain in muscles, throat, joints, photophobia, recurrent diarrheA. When
inspection revealed a generalized lymphadenopathy. What is the most reliable diagnosis.
a. *AIDS
b. Lymphogranulomatos
c. The tumor of the digestive system
d. Tuberculosis
e. Salmonellosis
90. Patient C., 35 years old, the disease has started severe chills, raising the temperature to 39
°C, vomiting, pain in epigastric region, diarrhea with water-smelly bowel movement. Over
6 hours after eating raw eggs, potatoes with braised meat, drank juice. What kind of agent
is likely to cause this condition.
a. *Salmonella
b. Vibrio cholera
c. Enteric stick
d. Campylobacter
e. Shigella
91. Patient C., 17 years old, working vegetable base. Acutely ill, 2 days ago. His illness is
linked to eating dirty roots. The disease started with chills, body temperature increase to
38,1 °C, headache, pain in muscles and joints of all groups, weakness, nausea, Cramping
in the stomach around the navel, in epigastrium, diarrhea up to 5 times. Excrements liquid,
viscous, bed smell, normal colour. Objective inspection: scleritis, conjunctivitis,
hyperemia of soft palate, “raspberry” tongue. In palpation of abdomen - the moderate
morbidity in epigastrium. What is diagnosis.
a. *Yersiniosis
b. Dysentery
c. Salmonellosis
d. Cholera
e. Viral hepatitis
92. Patient 25 years, received complaints of double vision of the eyes, a decline of view,
shortness of breath. Before the disease eat mushrooms home preservation. Objective:
pallor, wide pupils with a weak reaction to light, dry mouth, a violation of swallowing,
flatulence, delay of stool. What is preliminary diagnosis.
a. Leptospirosis
b. Yersiniosis
c. *Botulism
d. Giardiasis
e. Salmonellosis
93. Patient C., 22 years old, hospitalized in an infectious department complaining of chills,
temperature increase to 38,5 °C, vomiting, pain in epigastrii, frequent stool. 7 hours before
the disease, ate raw eggs, potatoes with braised meat, drink tomato juice. What kind of
organism most likely to cause disease.
a. Enterovirus
b. *Salmonella
c. Enteric stick
d. Staphylococcus
e. perfringens
94. Emergency ambulance delivered in infectious hospital girl N., 17 years old, complaining
of headaches, expressed general weakness, pain in epigastrium, repeated vomiting,
diarrhea to 8 times a day greenish color. She was used 2 raw chicken eggs. Body
temperature 39 °C, the tongue covered with white coat, moderately cramps, pain in
abdomen and sigmoid areA. The most likely diagnosis.
a. Acute appendicitis
b. Crohn’s disease
c. Shigellosis
d. Viral gastroenteritis
e. *Salmonellosis
95. Patient L., 32 years old, was admitted to the infectious hospital complaining of chills,
headache, expressed general weakness, cutting pain in epigastric, diarrheA. The
temperature of 39 °C, repeated vomiting. Excrement abundant, greenish color, without
pathological impurities. Over 4 hours to illnesses used the meat salad in the factory
canteen. Which drugs will be most effective for the treatment of this patient.
a. Antibotulism serum
b. Salt and colloid solutions
c. Sulfanilamides
d. *Antibiotics
e. Enterosorbent
96. Patient B., 55 years old, was hospitalized in an infectious hospital with complaints of
frequent vomiting, liquid excrements, abdominal pain, mainly in the right iliac area and
epigastrium. Pale skin, dry tongue with a touch of gray, hypotension, tachycardiA. The
doctor was suspected salmonellA. What laboratory test is need to perform.
a. General blood analysis
b. Parasytoscopy of blood smear
c. Microscopy of stool
d. *Coproculture
e. Biologic test
97. Patient C., 30 years old, was hospitalized with a preliminary diagnosis of salmonellosis.
Body temperature – 40 °C, repeated vomiting, profused diarrheA. Hypotension (BP 55/30
mm Hg), expressed tachycardia (pulse to 120/min), face pale, pain mainly in right iliac
area and epigastrium, hepatosplenomegaly were found. What treatment tactic will be most
effective in this case.
a. Glucocorticoids, crystalloid solutions, antibiotics, diuretics
b. Cardiac glycosides, colloid solutions, antibiotics
c. Sorbents, diuretics, desintoxication
d. Dopamine, colloid solutions, antibacterial
e. *Glucocorticoids, crystalloid solutions, antibiotics
98. Patient F., 25 years old, was hospitalized in the infectious hospital with complaints of
frequent vomiting, liquid stool, abdominal pain in epigastrium. Pale skin, dry tongue with
a grey cover, hypotension, tachycardiA. The doctor was suspected salmonellA. What
serologic study is need to perform.
a. Microprecipitation reaction
b. RA (Widal)
c. *RIGA with Salmonella diagnosticum
d. RIGA with Shigella diagnosticum and RA (Widal) with paired serum
e. RKC
99. Patient N., 22 years old, was admitted to the infectious office complaining of chills,
headache, expressed general weakness, cutting pain in sigmoid area, diarrheA. The
temperature of the body of 39,3 °C, vomiting. Faces watery. Patient was used the salad
with sausage and eggs. For further diagnosis doctor was prescribed RIGA with salmonella
diagnosticums. When it’s necessary to take blood for the investigation.
a. At the 1st and 2nd days of illness
b. Once in recovering period
c. *At the end of the 1st week and after 7-10 days
d. At the end of the 1st week and after 3-4 days
e. Once in acute period
100.
Patient B., 32 years old, an employ of poultry was hospitalized with complaints of
pain in abdomen, mostly in sigmoid area, fever up to 38,8 °C, nausea, vomiting, diarrhea
with dark-green colour of stool, without any pathological changes. Skin and mucous
membranes are dry, moderate tension in epigastric region and pain in the right iliac areA.
The most likely diagnosis.
a. Acute appendicitis
b. Cholera
c. Disbacteriosis
d. *Salmonellosis
e. Rotavirus gastroenteritis
101.
Three patients were delivered with symptoms of fever, diarrhea and vomiting.
Excrements dark-green, without any pathological changes. All patients together took part
in the preparation of food and have used eggs, meat salad. The most likely diagnosis.
a. Cholera
b. Botulism
c. Dysbacteriosis
d. *Salmonellosis
e. Shigellosis
102.
Patient L., 33 years old, was admitted to the hospital with the complaints of
untolarable pain in the abdomen, mostly in sigmoid region, fever up to 38,8 ?C, nausea,
vomiting , diarrhea of dark-green colour in every1-1,5 hour, without admixture of blood
and mucus. The condition progressively deteriorated, increasing hypotension, paleness,
weak cardiac sounds, decreased intestinal peristalsis with moderate tension in epigastric
region and pain in the right iliac region. What tactics of treatment?
a. Glucocorticoids, infusion of salt solutions, antibiotics intravenously, diuretic
b. Cardiac glycosides, infusion colloidal solution, antibacterial drugs
c. Enterosorbents, diuretic, desintoxication therapy
d. Mezaton 1% / in drops, infusion of salt solutions, antibacterial drugs orally
e. *Glucocorticoids, infusion of salt solutions, antibiotics intravenously
103.
Patient 45 years old, became ill within 10 hours after consumption of grinded meat.
Complaints of vomiting, pain in epigastric region, diarrhea of green color, increasing of
temperature up to to 39 °C. During objective examination revealed that: patients general
condition is sever, pain in the muscles of the lower limbs, pale face, acrocyanosis. Pulse
130 per min,. B.P 70/40 mm Hg. weak cardiac sounds, dry tongue, brown-whitecovering .
abdomen cramps, painful iliac and epigastric regions. What is the most likely diagnosis.
a. Shigellosis
b. Cholera
c. *Salmonellosis
d. Viral gastroenteritis
e. Eshericiosis
104.
Five patients during 15 hours were admited In a hospital. All of them have similiar
complaints of mild diarrhea and vomiting developed ,diplopia, midriasis, visual
disturbance, difficult swallowing, dry mouth, difficulty in breathing. All patients together
joined in a dinner party, used different dishes, including meat, salad, canned mushroom.
The most likely diagnosis.
a. Cholera
b. *Botulism
c. Thypus
d. Salmonellosis
e. Rotaviral gastroenteritis
105.
Patient G., 24 years old, 6 hours after taking his breakfast with boiled eggs, a cup
of coffee appeared chill, fever up to 38,8 °C, pain in left inguinal area, nausea,vomiting,
rumbling in the stomach, then frequent diarrhea, feces copious, watery, mudy-green. What
is the most likely diagnosis.
a. Cholera
b. Botulism
c. *Salmonellosis
d. Abdominal thypoid
e. Shigellosis
106.
Patient, who was arrived from the Crimea, diarrhea appeared in an intervel of 1-1.5
hour. with copious watery increments, without admixtures of mucous and blood. He was
admtted in an infectious dipartment with the diagnosis of “acute intestinal infection”.
What laboratory tests are necessary to confirm the etoilogical diagnosis.
a. General blood analysis
b. Bacteriological blood analysis
c. Microscopical analysis of stool
d. *Coprogram
e. The biological examination in mice
107.
Patient C. with complaints of diarrhea with mucous, stabing pain in epigastric area
, rumbling in the stomach, fever. According to his anamnesis he took preserved milk,
eggs, salad of fresh vegetables. What is the disease most likely.
a. Shigellosis
b. Yersiniosis
c. *Salmonellosis
d. Cholera
e. Botulism
108.
Patient B., 38 years old, came to the admission department with the complaints of
fever up to 38 °C, vomiting 4 times in a day, diarrhea up to 6 times in a day. Durin
objective examination revealed that B.P 125/75 mm Hg, pulse 80 per min, abdomen is
soft,painful in the right iliac and epigastric area .According to anamnesis he took salad of
boiled eggs and sausage. What treatment should be started first.
a. Antibotulism serum
b. Intravenous antibiotics
c. Washing of the stomach and intestine, rehydration therapy , glucocorticoids
d. *Gastric lavage and washing of intestine, rehydraton therapy enterosorbents
e. Treatment after getting of the laboratory test results
109.
Patient 27 years old, complaints of headaches, weakness, pain in epigastric area,
vomiting, diarrhea (9 times in a day; huge excrement, greenish colour). Before the
appearance of disease he consumed raw eggs. Objectively: body temperature 38,8 °C.
Tongue is white, pain in epigastric and umbilical region. The most likely diagnosis.
a. *Salmonellosis
b. Cholera
c. Dysentery
d. Viral gastroenteritis
e. Acute appendicitis
110.
Patient K., 30 years old, came with complaints of nausea, vomiting, pain in
epigastrium and paraumbilical areA. High body temperature, pale skin, dry. tongue
covered with whitish-gray coating. quick pulse, low blood pressure. 2 hours before the
onset of symptoms of the disease he ate in cafe. What is the diagnosis most likely.
a. Cholera
b. Shigellosis
c. Thypoid fever
d. *Salmonellosis
e. Meningitis
111.
Disease started acutely with the complaints of watery diarrhea, vomiting, cramps in
the lower extremities. Objectively: slow voice, shunken eyes, quick pulse, low blood
pressure, decrease of urination, weak cardiac sounds. In liver and spleen no changes. Put
the correct diagnosis.
a. Salmonellosis
b. Dysentery
c. Food poisoning
d. *Cholera
e. Typhoid fever
112.
A Pakistani, 30 years old, severly ill: with the complaints of frequent diarrhea like
rice water. Objectively: body temperature 35,4 °C, skin of peripheral parts of the body
cold to the touch, acrocyanosis, xerostomia, sharply decreased skin elasticity and turgor.
What should be done to assess the degree of dehydration.
a. Measurement of central venous pressure
b. Determination of urea and creatinine in blood
c. Funduscopy
d. Plain X-ray film of abdomen
e. *Determination of specific gravity of blood plasma
113.
Patient N., 30 years old, works in a restaurant, admitted to the hospital complaining
of unbearable pain in the abdomen, increase of temperature 38,8 °C, nausea, vomiting,
diarrhea of dark-green colour, He took eggs, mushrooms, canned food. Objective state:
general condition is moderately sever, skin and mucous membranes are dry, pain in
epigastric region. Name the disease.
a. Shigellosis
b. Yersiniosis
c. *Salmonellosis
d. Cholera
e. Rotaviral infection
114.
Patient 35 years old with complaints of increasing of temperature to 39 °C with
chill, vomiting, pain in epigastric region, diarrhea with foul smell, abdominal cramps. 6
hours before onset of the disease ate raw eggs, fried potatoes with grinded meat, drank
juice. To clarify the diagnosis of serological diagnostic methods often used RNGA with
the salmonella groupdiagnostic tools and RA (Vidal test) . When blood should take for
the diagnostic procedures?
a. In the first day of illness
b. At the end of the first month
c. In 1st week in 3-4 days
d. *At the end of the 1st week from 7-10 days
e. During admission to the hospital
115.
A patient was hospitalised after 4 hours of consumption of raw eggs, complaining
frequent vomiting, abdominal pain, mostly in the right iliac area and umbilical areA.
Objectively pale skin, dry tongue with gray coating, hypotension, tachycardiA. What tests
should be carried out to confirm the diagnosis.
a. General blood analysis
b. Parasitological examinations
c. Microscopic examination of stool
d. Biological test
e. *Coproculture
116.
Patient G., 22 years old, was hospitalised in an infectious department complaining
of chills, stabing pain in epigastrium, diarrhea, temperature increased up to 39,6 °C,
repeated vomiting. Feces copious, watery, without pathological changes. 4 hours before
the onset of symptoms ate the meat, salad. What is the most appropriate drug to be
prescribe.
a. Antidiarrheal drugs
b. Flouroquinolone group of antibiotics
c. Sulfanilamide group of antibiotics
d. Desintoxication therapy
e. *Enterosorbents
117.
Patient K., 40 years old, was hospitalised with the diagnosis of intestinal infection.
Complaints of general weakness, headache and diarrheA. Symptomes appeared suddenly
2 hours after consumption of cake with custard. Suddenly there were chills, nausea,
profuse repeated vomiting, frequent diarrhea, abdominal pain and short-term loss of
consciousness, temperature increased to 38,6 °C. The preliminary dianosis?
a. Salmonellosis, localized form, gastroenteritis type
b. *Salmonellosis, localized form gastroenterocolitis type
c. Salmonellosis, localized form enterocolitis type
d. Salmonellosis, generalized form
e. Salmonellosis, nosoparasitic
118.
Patient K., 40 years old, hospitalised with the diagnosis of salmonellosis,
with symptoms of rise of body temperature to 40 °C, repeated vomiting, diarrheA.
Objectively: AP 60/20 mm Hg, pulse 120/minute, pale face, increased intestinal
peristalsis, enlarged liver and spleen. What is the best treatment.
a. *Glucocorticoids, desintoxication therapy, antibacterial drugs
b. Diuretic, desintoxication therapy, antibacterial drugs
c. Enterorsorbents, diuretic, detsintoxication therapy
d. Glucocorticoids, diuretic, desintoxication therapy, antibacterial drugs
e. Glucocorticoids, diuretic, antibacterial drugs
119.
Patient P., 36 years old, a farmer is seriously ill, with high temperature, having
chills, headache, flatulence, nauseA. At present he is complaining of severe abdominal
pain, diarrhea up to 15 times a day stool is liquid, smelly, with impurities mucous and
blood. His eyes are sunken. The tongue is covered with white coating. On palpationpainful colon, increased liver. In blood – neutropil leukocytosis with a shift formula the
left anemiA. On rectomanoscopy deep ulcers with irregular edges are observed, including
along the folds. What is the diagnosis.
a. Shigellosis
b. *Balantidiasis
c. Nonspecific ulcerous colitis
d. Amoebiasis
e. Colon tumor
120.
A 29 years old patient T. works on a pig farm. Shi was hospitalized with symptoms
of balantidiasis. What would be the drug of choice.
a. Benzylpenicillin
b. Gentamycin
c. Chloramphenicol
d. *Monomycin
e. Timogen
121.
A 63 years old villager,is gravely ill: increased body temperature, headache,
nausea, vomiting, abdominal pain, diarrheA. The patient was not seeking mediacal
attention, and was taking levomycetin. The condition had not improved, the frequency of
bowel movements was 8-12 times a day. At present he is complaining of severe
abdominal pain, diarrhea, general weakness. The stool is liquid, smelly, with mixture of
puss and blood. The patient is exosted his eyes are sunken. His tongue is covered with
white coating. In blood analys is leukocytosis and anemia observed. On rectomanoscopy
detected ulcers with jagged edges. What is the most likely diagnosis.
a. *Balantiasis
b. Shigellosis
c. Nonspecific ulcerative colitis
d. Rotavirus gastroenteritis
e. Colon tumor
122.
Patient C., 35 years old, the disease has started from severe chills, raising the
temperature to 39 oC, vomiting, pain in epigastric region, diarrhea with water-smelly
bowel movement. Over 6 hours after eating raw eggs, potatoes with braised meat, drank
juice. What kind of agent is likely to cause this condition.
a. *Salmonella
b. Vibrio cholera
c. Enteric stick
d. Campylobacter
e. Shigella
123.
Patient C., 17 years old, working at vegetable base. Acutely ill, 2 days ago. His
illness is linked to eating dirty roots. The disease started with chills, body temperature
increase to 38,1 oC, headache, pain in muscles and joints of all groups, weakness, nausea,
Cramping in the stomach around the navel, in epigastrium, diarrhea up to 5 times.
Excrements liquid, viscous, bed smell, normal colour. Objective inspection: scleritis,
conjunctivitis, hyperemia of soft palate, “raspberry” tongue. In palpation of abdomen - the
moderate morbidity in epigastrium. What is diagnosis.
a. *Yersiniosis
b. Dysentery
c. Salmonellosis
d. Cholera
e. Viral hepatitis
124.
Patient 25 years, received complaints of double vision of the eyes, a decline of
view, shortness of breath. Before the disease eat mushrooms home preservation.
Objective: pallor, wide pupils with a weak reaction to light, dry mouth, a violation of
swallowing, flatulence, delay of stool. What is preliminary diagnosis.
a. Leptospirosis
b. Yersiniosis
c. *Botulism
d. Giardiasis
e. Salmonellosis
125.
Patient C., 22 years old, hospitalized in an infectious department complaining of
chills, temperature increase to 38,5 oC, vomiting, pain in epigastrii, frequent stool. 7 hours
before the disease, ate raw eggs, potatoes with braised meat, drink tomato juice. What
kind of organism most likely to cause disease.
a. Enterovirus
b. *Salmonella
c. Enteric stick
d. Staphylococcus
e. Cl. perfringens
126.
Emergency ambulance delivered in infectious hospital girl N., 17 years old,
complaining of headaches, expressed general weakness, pain in epigastrium, repeated
vomiting, diarrhea to 8 times a day greenish color. She was used 2 raw chicken eggs.
Body temperature 39 °C, the tongue covered with white coat, moderately cramps, pain in
abdomen and sigmoid areA. What is the most likely diagnosis.
a. Acute appendicitis
b. Crohn’s disease
c. Shigellosis
d. Viral gastroenteritis
e. *Salmonellosis
127.
Patient L., 32 years old, was admitted to the infectious office complaining of chills,
headache, expressed general weakness, cutting pain in epigastrii, diarrheA. The
temperature of 39 °C, repeated vomiting. Excrement abundant, greenish color, without
pathological impurities. Over 4 hours to illnesses used the meat salad in the factory
canteen. Which drugs will be most effective for the treatment of this patient.
a. Antibotulism serum
b. Salt and colloid solutions
c. Sulfanilamides
d. *Antibiotics
e. Enterosorbent
128.
Patient B., 55 years old, was hospitalized in an infectious hospital with complaints
on frequent vomiting, liquid excrements, abdominal pain, mainly in the right iliac area and
epigastrium. Pale skin, dry tongue with a touch of gray, hypotension, tachycardiA. The
doctor was suspected salmonellA. What laboratory test is need to perform.
a. General blood analysis
b. Parasytoscopy of blood smear
c. Microscopy of stool
d. *Coproculture
e. Biologic test
129.
Patient C., 30 years old, was hospitalized with a preliminary diagnosis of
salmonellosis. Body temperature – 40 °C, repeated vomiting, profused diarrheA.
Hypotension (BP 55/30 mm Hg), expressed tachycardia (pulse to 120/min), face pale, pain
mainly in right iliac area and epigastrium, hepatosplenomegaly were found. What
treatment tactic will be most effective in this case.
a. Glucocorticoids, crystalloid solutions, antibiotics, diuretics
b. Cardiac glycosides, colloid solutions, antibiotics
c. Sorbents, diuretics, desintoxication
d. Dopamine, colloid solutions, antibacterial
e. *Glucocorticoids, crystalloid solutions, antibiotics
130.
Patient F., 25 years old, was hospitalized in the infectious hospital with complaints
of frequent vomiting, liquid stool, abdominal pain in epigastrium. Pale skin, dry tongue
with a grey cover, hypotension, tachycardiA. The doctor was suspected salmonellA. What
serologic study is need to perform.
a. Microprecipitation reaction
b. RA (Widal)
c. *RIGA with Salmonella diagnosticum
d. RIGA with Shigella diagnosticum and RA (Widal) with paired serum
e. RKC
131.
Patient N., 22 years old, was admitted to the infectious office complaining of chills,
headache, expressed general weakness, cutting pain in sigmoid area, diarrheA. The
temperature of the body of 39,3 °C, vomiting. Faces watery. Patient was used the salad
with sausage and eggs. For further diagnosis doctor was prescribed RIGA with salmonella
diagnosticums. When it’s necessary to take blood for the investigation.
a. At the 1st and 2nd days of illness
b. Once in recovering period
c. *At the end of the 1st week and after 7-10 days
d. At the end of the 1st week and after 3-4 days
e. Once in acute period
132.
Patient B., 32 years old, an employ of poultry was hospitalized with complaints of
pain in abdomen, mostly in sigmoid area, fever up to 38,8 oC, nausea, vomiting, diarrhea
with dark-green colour of stool, without any pathological changes. Skin and mucous
membranes are dry, moderate tension in epigastric region and pain in the right iliac areA.
The most likely diagnosis.
a. Acute appendicitis
b. Cholera
c. Disbacteriosis
d. *Salmonellosis
e. Rotavirus gastroenteritis
133.
Patient L., 33 years old, was admitted to the hospital with the complaints of
untolarable pain in the abdomen, mostly in sigmoid region, fever up to 38,8 oC, nausea,
vomiting , diarrhea of dark-green colour in every1-1,5 hour, without admixture of blood
and mucus. The condition progressively deteriorated, increasing hypotension, paleness,
weak cardiac sounds, decreased intestinal peristalsis with moderate tension in epigastric
region and pain in the right iliac region. What tactics of treatment?
a. Glucocorticoids, infusion of salt solutions, antibiotics intravenously, diuretic
b. Cardiac glycosides, infusion colloidal solution, antibacterial drugs
c. Enterosorbents, diuretic, desintoxication therapy
d. Mezaton 1% / in drops, infusion of salt solutions, antibacterial drugs orally
e. *Glucocorticoids, infusion of salt solutions, antibiotics intravenously
134.
Patient 45 years old, became ill within 10 hours after consumption of grinded meat.
Complaints of vomiting, pain in epigastric region, diarrhea of green color, increasing of
temperature up to to 39 °C. During objective examination revealed that: patients general
condition is sever, pain in the muscles of the lower limbs, pale face, acrocyanosis. Pulse
130 per min,. B.P 70/40 mm Hg. weak cardiac sounds, dry tongue, brown-whitecovering .
abdomen cramps, painful iliac and epigastric regions. What is the most likely diagnosis.
a. Shigellosis
b. Cholera
c. *Salmonellosis
d. Viral gastroenteritis
e. Eshericiosis
135.
Patient U., who was arrived from the Crimea, diarrhea appeared in an intervel of 11.5 hour. with copious watery increments, without admixtures of mucous and blood. He
was admtted in an infectious dipartment with the diagnosis of “acute intestinal infection”.
What laboratory tests are necessary to confirm the etoilogical diagnosis.
a. General blood analysis
b. Bacteriological blood analysis
c. Microscopical analysis of stool
d. *Coprogram
e. The biological examination in mice
136.
Patient C with complaints of diarrhea with mucous, stabing pain in epigastric area ,
rumbling in the stomach, fever. According to his anamnesis he took preserved milk, eggs,
salad of fresh vegetables. What is the disease most likely.
a. Shigellosis
b. Yersiniosis
c. *Salmonellosis
d. Cholera
e. Botulism
137.
Patient B., 38 years old, came to the admission department with the complaints of
fever up to 38 °C, vomiting 4 times in a day, diarrhea up to 6 times in a day. Durin
objective examination revealed that B.P 125/75 mm Hg, pulse 80 per min, abdomen is
soft,painful in the right iliac and epigastric area .According to anamnesis he took salad of
boiled eggs and sausage. What treatment should be started first.
a. Antibotulism serum
b. Intravenous antibiotics
c. Washing of the stomach and intestine, rehydration therapy , glucocorticoids
d. *Gastric lavage and washing of intestine, rehydraton therapy enterosorbents
e. Treatment after getting of the laboratory test results
138.
Patient 27 years old, complaints of headaches, weakness, pain in epigastric area,
vomiting, diarrhea (9 times in a day; huge excrement, greenish colour). Before the
appearance of disease he consumed raw eggs. Objectively: body temperature 38,8 °C.
Tongue is white, pain in epigastric and umbilical region. The most likely diagnosis.
a. *Salmonellosis
b. Cholera
c. Dysentery
d. Viral gastroenteritis
e. Acute appendicitis
139.
Patient K., 30 years old, came with complaints of nausea, vomiting, pain in
epigastrium and paraumbilical areA. High body temperature, pale skin, dry. tongue
covered with whitish-gray coating. quick pulse, low blood pressure. 2 hours before the
onset of symptoms of the disease he ate in cafe. What is the diagnosis most likely.
a. Cholera
b. Shigellosis
c. Thypoid fever
d. *Salmonellosis
e. Meningitis
140.
Disease started acutely with the complaints of watery diarrhea, vomiting, cramps in
the lower extremities. Objectively: slow voice, shunken eyes, quick pulse, low blood
pressure, decrease of urination, weak cardiac sounds. In liver and spleen no changes. Put
the correct diagnosis.
a. Salmonellosis
b. Dysentery
c. Food poisoning
d. *Cholera
e. Typhoid fever
141.
Patient 35 years old with complaints of increasing of temperature to 39 °C with
chill, vomiting, pain in epigastric region, diarrhea with foul smell, abdominal cramps. 6
hours before onset of the disease ate raw eggs, fried potatoes with grinded meat, drank
juice. To clarify the diagnosis serological diagnostic methods often used. When blood
should be taken for the diagnostic procedures?
a. In the first day of illness
b. At the end of the first month
c. In 1st week in 3-4 days
d. *At the end of the 1st week from 7-10 days
e. During admission to the hospital
142.
A patient was hospitalised after 4 hours of consumption of raw eggs, complaining
frequent vomiting, abdominal pain, mostly in the right iliac area and umbilical areA.
Objectively pale skin, dry tongue with gray coating, hypotension, tachycardiA. What tests
should be carried out to confirm the diagnosis.
a. General blood analysis
b. Parasitological examinations
c. Microscopic examination of stool
d. Biological test
e. *Coproculture
143.
Patient G., 22 years old, was hospitalised in an infectious department complaining
of chills, stabing pain in epigastrium, diarrhea, temperature increased up to 39,6 °C,
repeated vomiting. Feces copious, watery, without pathological changes. 4 hours before
the onset of symptoms ate the meat, salad. What is the most appropriate drug to be
prescribe.
a. Antidiarrheal drugs
b. Flouroquinolone group of antibiotics
c. Sulfanilamide group of antibiotics
d. Desintoxication therapy
e. *Enterosorbents
144.
Patient K., 40 years old, was hospitalised with the diagnosis of intestinal infection.
Complaints of general weakness, headache and diarrheA. Symptomes appeared suddenly
2 hours after consumption of cake with custard. Suddenly there were chills, nausea,
profuse repeated vomiting, frequent diarrhea, abdominal pain and short-term loss of
consciousness, temperature increased to 38,6 °C. The preliminary dianosis?
a. Salmonellosis, localized form, gastroenteritis type
b. *Salmonellosis, localized form gastroenterocolitis type
c. Salmonellosis, localized form enterocolitis type
d. Salmonellosis, generalized form
e. Salmonellosis, nosoparasitic
145.
Patient K., 40 years old, hospitalised with the diagnosis of salmonellosis, with
symptoms of rise of body temperature to 40 °C, repeated vomiting, diarrheA. Objectively:
AP 60/20 mm Hg, pulse 120/minute, pale face, increased intestinal peristalsis, enlarged
liver and spleen. What is the best treatment.
a. *Glucocorticoids, desintoxication therapy, antibacterial drugs
b. Diuretic, desintoxication therapy, antibacterial drugs
c. Enterorsorbents, diuretic, detsintoxication therapy
d. Glucocorticoids, diuretic, desintoxication therapy, antibacterial drugs
146.
Young farmer was diagnosed with balantidiasis. Drugs of choice would be all
except:
a. Monomycin
b. Ampicillin
c. Aminarson
d. *Gentamicin
e.
Metronidazole
147.
A patient complaints of severe abdominal pain, smelly watery diarrheA. with
content of blood, . What kind of disease might be considered in the first place.
a. Amoebiasis
b. Rotaviral gastroenteritis
c. Giardiasis
d. *Balantidiasis
e. Cholera
148.
Patient P., 36 years old, a farmer is seriously ill, with high temperature, having
chills, headache, flatulence, nauseA. At present he is complaining of severe abdominal
pain, diarrhea up to 15 times a day stool is liquid, smelly, with impurities mucous and
blood. His eyes are sunken. The tongue is covered with white coating. On palpationpainful colon, increased liver. In blood – neutropil leukocytosis with a shift formula the
left anemiA. On rectomanoscopy deep ulcers with irregular edges are observed, including
along the folds. What is the diagnosis.
a. Shigellosis
b. *Balantidiasis
c. Nonspecific ulcerous colitis
d. Amoebiasis
e. Colon tumor
149.
A 29 years old patient T. works on a pig farm. She was hospitalized with
symptoms of balantidiasis. What would be the drug of choice.
a. Benzylpenicillin
b. Gentamycin
c. Chloramphenicol
d. *Monomycin
e. Timogen
150.
A 63 years old villager,is gravely ill: increased body temperature, headache,
nausea, vomiting, abdominal pain, diarrheA. The patient was not seeking mediacal
attention, and was taking levomycetin. The condition had not improved, the frequency of
bowel movements was 8-12 times a day. At present he is complaining of severe
abdominal pain, diarrhea, general weakness. The stool is liquid, smelly, with mixture of
puss and blood. The patient is exosted his eyes are sunken. His tongue is covered with
white coating. In blood analys is leukocytosis and anemia observed. On rectomanoscopy
detected ulcers with jagged edges. What is the most likely diagnosis.
a. *Balantidiasis
b. Shigellosis
c. Nonspecific ulcerative colitis
d. Rotavirus gastroenteritis
e. Colon tumor
151.
Patient admitted in hospital with complaints of abdominal pain, increased body
temperature, diarrheA. Stool is liquid, smelly, with mixture of mucus and blood. What
kind of illness need to think about in the first place?
a. *Balantidiasis
b. Rotavirus gastroenteritis
c. Food poisoning
d. Amoebiasis
e. Giardiasis
152.
Patient S., 53, a resident of the village, heavelly ill – common symptoms of
intoxication, intestinal dysfunction excrements are smelly, with mucous and blood.
Abdomen on palpation is soft, but painful along spazmotic sygmoid bovell. On
rectoromanoscopy - ulcers in diameter up to 10-20 mm along the folds of mucous
membrane, swelling, moist edge, surrounded by a zone of flushing, the bottom is covered
hyperemia and necrotic masses. What kind of illness need to think about?
a. *Balantidiasis
b. Shigellosis
c. Nonspecific ulcerous colitis
d. Amoebiasis
e. Colon tumor
153.
Student U. from India, 22, a month after returning home develops – liquid stool,
weak abdominal pain. Over time of the disease has lost 6 kg. The next day heavy diarrhea
10 times a day, stool is like «raspberry jelly». Cramping in the abdomen growing in the
bowel. The most likely diagnosis is:
a. Bowel tumor
b. Salmonellosis
c. Shigellosis
d. *Amoebiasis
e. HIV/AIDS
154.
Patient F., 25 years old, was hospitalized complaining of diarrhea – 10-15 times
per day. A month ago was returned from Central AsiA. The general condition of relatively
satisfactory. The temperature of the body subfebrile. Abdomen soft. Stool is liquid, highlevel clear scanty mucus and blood. What is the most likely diagnosis:
a. *Amoebiasis
b. Shigellosis
c. Salmonellosis
d. Tumor of bowel
e. Enterohemorrhagic escherihiosis
155.
Patients with аmebiasis has been hospitalized. For specific therapy are used socalled, indirect and аmebiostic tissue, and preparations with universal action. What group
does delagin belong to?
a. *Tissue аmoebiostic
b. Indirect аmoebiostic
c. Lines of аmoebiostic
d. Preparations with universal action
e. Does not belong to anyone
156.
A student from Africa, 22 years old, liquid stool and weak stomach-aches appeared
a month after returning from home. Lost of weight is 6 kg. Diarrhoe is abundant 10 times
on a day, like “raspberry jelly”. Stomach-aches increase during defecation. What is most
probable diagnosis?
a. Tumor of intestine
b. Salmonellosis
c. Shigellosis
d. *Amoebiasis
e. HIV/AIDS
157.
Patient O., 25 years old, hospitalized with complaints off diarrhea up to 10-15
times a day. Month ago he returned from Middle AsiA. The general condition is relatively
satisfactory. The temperature of body is subfebrile.A stomach is soft, pain after motion of
colon. Emptying is liquid with the considerable admixtures of glassy mucous and blood.
What is most probable diagnosis?
a. *Amoebiasis
b. Shigellosis
c. Salmonellosis
d. Tumor of intestine
e. Enterohaemorrhagic esherihiosis
158.
Patient U., 36 years old, farmer, became ill gradually. Indispositions, tormina,
diarrhea (stool was 8-12 times per day). He was in one of countries with a hot climate
during last 2 month. Temperature – 36,9 °C. At palpation – pain in iliac areas. Stool is
liquid, with the admixtures of glassy mucus, look like to „raspberry jelly”. What is
primery diagnosis?
a. *Amoebiasis
b. Shigellosis
c. Salmonellosis
d. Tumor of intestine
e. Enterohaemorrhagic esherihiosis
159.
Patient C., 46 years old, periodically visited Egypt for a business purpose, became
acutely ill. Complaints of a fever of 38 °C, with chills and shevering, pain in right
hypochorium. Insignificant icterus.Liver is enlarged,some hyperechogeninic area . In
blood neutrophilic leucocytosis, increased ESR. Purulent abscesses of liver were found
out during ultrasound examination . What is previous diagnosis?
a. Legionellosis
b. Echinnococcosis
c. Ascaridosis
d. Cancer of liver with metastases
e. *Amebiasis
160.
Student, 22 years old fell ill within a month after return from EthiopiA.Dull
stomach-ache appeared and liquid emptying. Emptying is abundant up to 10 times a day,
as “raspberry jelly” excrement, stomach-ache that increase during defecation. In times of
illness lost 6 kg of body weight. Your diagnosis will be?
a. *Intestinal аmoebiasis
b.
Shigellosis
c.
Salmonellosis
d.
Tumor of intestine
e.
Intestinal esheriosis
161.
Patient O., 47 years old, became ill acutely. Periodically visits Uzbekistan for a
business purpose. Complaints of a fever of 39,4 °C with chills and shivering, and pain in
right hypochorium, cough with sputum. Skin with an earthy tint. Signs of rightsided
pneumoniA.The liver and spleen are enlarged.In blood neutrophilic leucocytosis. On Xray infiltration of pulmonary tissues is found, at ultrasound abdominal examination
numerous abscesses are present in a liver. What disease is it necessary to think about?
a. Echeriosis
b. Alveolitis
c. *Amebiasis
d. Cancer of liver is with metastases in lungs
e. Legionellosis
162.
Sick E., 21 years old, complaints of nausea, insignificant dull or rarely aggressive
pain in a stomach. Appetite is decreased. Periodically presence of flatulence, grumbling in
a stomach and semi-fluid emptying. Sick during about 6 month. The diagnosis of chronic
cholecystitis was proposed. In examination pallor of skin, lowering of mass of body and
signs of vegetative dystonia are found.On palpation pain in еpigastric area also found.
What disease you will think about?
a. *Giardiasis
b. Salmonellosis
c. Amebiasis
d. Balantidiasis
e. Intestinal esherihiosis
163.
Diagnosis of giardiasis is based on the discovering of cyst in fresh incandescence
and vegetative forms in duodenal content. At diarhoea in incandescence can appear
trophozoite on faecal microscopic examination and preparations with solution of Lugola
and еоsin. Examination is more expedient to conduct 3-5 times at intervals 1-2 days. How
to increase frequency of findings?
a. *By application of the formalin-еpiniphrine enriching methods
b. Cultivation in thermostat
c. Cultivation in an anaerobic chamber
d. By the method of floatation in bilious clear soup
e. To sow on a nutrient medium
164.
A patient complaints of severe abdominal pain, smelly watery diarrheA. with
content of blood. What kind of disease might be considered in the first place.
a. Amoebiasis
b. Rotaviral gastroenteritis
c. Giardiasis
d. *Balantidiasis
e. Cholera
165.
Patient P., 36 years old, a farmer is seriously ill, with high temperature, having
chills, headache, flatulence, nauseA. At present he is complaining of severe abdominal
pain, diarrhea up to 15 times a day stool is liquid, smelly, with impurities mucous and
blood. His eyes are sunken. The tongue is covered with white coating. On palpationpainful colon, increased liver. In blood – neutrophil leukocytosis with a shift formula the
left with anemiA. On rectomanoscopy deep ulcers with irregular edges are observed,
including along the folds. What is the diagnosis.
a. Shigellosis
b. *Balantidiasis
c. Nonspecific ulcerous colitis
d. Amoebiasis
e. Colon tumor
166.
A 63 years old villager, is gravely ill: increased body temperature, headache,
nausea, vomiting, abdominal pain, diarrheA. The patient was taking levomycetin. The
condition had not improved, the frequency of bowel movements was 8-12 times a day. At
present he is complaining of severe abdominal pain, diarrhea, general weakness. The stool
is liquid, smelly, with mixture of puss and blood. The patient is exosted his eyes are
sunken. His tongue is covered with white coating. In blood analys is leukocytosis and
anemia observed. On rectomanoscopy detected ulcers with jagged edges. What is the most
likely diagnosis.
a. Colon tumor
b. Shigellosis
c. Nonspecific ulcerative colitis
d. Rotavirus gastroenteritis
e. *Balantidiasis
167.
A patient was admitted in hospital with complaints of abdominal pain, increased
body temperature, diarrheA. Stool is liquid, smelly, with mixture of mucus and blood.
What kind of illness need to think about in the first place?
a. *Balantidiasis
b. Rotavirus gastroenteritis
c. Food poisoning
d. Amoebiasis
e. Giardiasis
168.
A patient was admited in permanent establishment with complaints of stomachache, fever and diarrheA. Stool is liquid, stinking, with the admixtures of pus and blood.
About what disease it is necessary to think about?
a. Food poisoning
b. Rotaviral gastroenteritis
c. *Balantidiasis
d. Amebiosis
e. Giardiasis
169.
Patient F, 53 years, habitant of a village, diseased acutely,the symptoms of general
intoxication, dysfunction of bowels appeared, emptying became semi-fluid with the
admixtures of pus and blood. The stomach at palpation is soft, morbid along belly and
ascending colon with spasm. At rectoscopy are found ulcers with diameter upto 10-20
mm, located along the folds of mucus membrane, have fillings out, sharp edges places,
surrounded by the area of hyperemia, bottom is covered by pus and necrotizing masses.
What disease it is needed to think about?
a. *Balantidiasis
b. Shigellosis
c. Idiopatic ulcerous colitis
d. Amebiasis
e. Intestinal cancer
170.
A villager, was admited with complaints of severe pain in a stomach, and diarrheA.
The stool is liquid with mixtures of blood. About what disease it is possible to think
about?
a. Amebiosis
b. Rotavirus gastroenteritis
c. Gisrdiasis
d. *Balantidiasis
e. Cholera
171.
Sick P, 36 years, farmer, became acutely ill when the temperature of body rised.
There were a chills, pain in chest, flatulence and nauseA. In the moment of admition
grumbles about terminal diarrhoea with 15 times day. The stool is liquid, sticky with the
admixtures of pus and blood. Tongue is covered with whitecovering. Stomach is
distended, painfu colon, enlarged liver is palpable. In blood neutrophilic leucocytosis with
the shift of formula to the left and anemiA. At a rectoscopy, deep ulcers with unequal,
sharped areas which are placed along the folds was found. What disease will you think
about?
a. Shigellosis
b. Ulcerous colitis
c. *Balantidiasis
d. Amebiasis
e. Cancer of colon
172.
Patient O., 29 years old, farmer came to a doctor with the signs of balantidiasis and
was hospitalized. What is the best etiotropic drug:
a. Benzylpenicillin
b. Gentamycin
c. Laevomycitin
d. *Monomycin
e. Timogen
173.
63 years old patient became ill suddenly. The temperature of body raised, the
stomach-ache with nausea and vomiting and diarrhea were observed. The patient’s
condition did not improve even after giving him Levomycitin for long time. The diarrhea
had proceeded again 8-12 times a day and became liquid. On admition the patient
complaints of diarrhea and general weakness.The excrement is liquid stinking with the
admixtures of pus and blood. The persons face is emaciated, eyes are hollow. Tongue is
covered by white stratification. The stomach is distended, peristalsis after bowel
movments. In blood leucocytosis and anaemic. At a rectoscopy ulcers with sharped,
uneven edges was found. What is the most probable diagnosis?
a. *Amebiasis
b. Shigellosis
c. Idiopatic ulcerative colitis
d. Rotavirus gastroenteritis
e. Cancer of colon
174.
Sick E., 21 years old, complaints of nausea, insignificant dull or rarely aggressive
pain in a stomach. Appetite is decreased. Periodically presence of flatulence, grumbling in
a stomach and semi-fluid emptying. Sick during about 6 month. The diagnosis of chronic
cholecystitis was proposed. In examination pallor of skin, lowering of mass of body and
signs of vegetative dystonia are found.On palpation pain in еpigastric area also found.
What disease you will think about?
a. *Giardiasis
b. Salmonellosis
c. Amebiasis
d. Balantidiasis
e. Intestinal єcheriosis
175.
Diagnosis of giardiasis is based on the discovering of cyst in fresh incandescence
and vegetative forms in duodenal content. Trophozoites can be find during faecal
microscopic examination. Examination is more informative if it conducts 3-5 times with
1-2 days intervals. How is possible to increase the frequency of findings?
a. *By application of the formalin-еpiniphrine enriching method
b. Cultivation in thermostat
c. Cultivation in an anaerobic chamber
d. By the method of floatation in bilious clear soup
e. To sow on a nutrient medium
176.
Veterinarian 57 years old, on the 3rd day of disease appealed to the doctor with
complaints of headache, high temperature, pain in gastrocnemius muscles, icterus, dark
urine and diminishing of its amount. Objectively: temperature of body is 38 °C, sclerotic
big peteachia on overhead part of thorax, hepatospleenomegaly. Most reliable preliminary
diagnosis will be:
a. *Leptospirosis
b. Brucellosis
c. Viral hepatitis
d. Pseudotuberculosis
e. Trichinosis
177.
A sick P., 40 in 2 weeks after eating of uncooked pork, purchased at the elemental
market, has sudden rise of temperature upto 40 °C, myalgias appeared, oedematous,
papular rash on extremities and trunk and dry cough. Accepted aspirin. General blood test:
leucocytes– 12*109 /l , eosinophilia– 40 %. What disease it is possible to think about?
a. *Trichinosis
b. Ascaridosis
c. Leptospirosis
d. Teniosis
e. Allergic reaction
178.
Patient M., 32 years old, during a month has disturbing cough, increasing of
temperature up to 38.0 °C. In anamnesis ascaridosis was found. On x-ray was found
separate homogeneous infiltration without clear contour of bronchus. In blood test –
еоsinophils 55 %. What is most probable diagnosis?
a. *Eosinophil infiltration (Lefler syndrome)
b. Plague
c. Chronic bronchitis, acute phase
d. Acquired pneumonia
e. Tumour of lungs
179.
Woman 24, complaints of heaviness in the area of liver during 3 days, with an
irradiation in the back. Signs of icterus are not present. Last menstruation – 5 weeks ago.
Appointed ultrasonic research of abdominal region and small pelvis was, found marked
progress uterogestation. Expansion of common biliary tract, inflammation of gall-bladder,
bulge of its its wall, presence of crystal sediment, extended loops of small intestines are
visualized. Investigation of colonoscopy found additional exogenic structures as a
“ribbon”, which is displaced in intestine and changes form without distal acoustic shade.
Most probable diagnosis is:
a. *Ascaridosis
b. Partial intestinal uncommunicating
c. Acute calculary cholecystitis
d. Choledocholithiasis
e. Cholecystitis
180.
Sick, 37 years, acted on stationary treatment with complaints of bad feel,
headache, increase of temperature upto to 39.5 °С, edema, pains in muscles of eyes,
tongues, lower extremities. 2 weeks prior to beginning of disease used raw pork fat.
Objectively: edema present. In a blood test: Leucocytes – 18.5*109 L, eosynophils – 22 %,
basophils – 2 %, neutrophils – 56 %, stab neutrophils – 16 %, monocytes – 4 %. It is no
pathological changes in urine. What is most probable diagnosis?
a. Leptospirosis
b. Acute nephritis
c. *Trichinosis
d. Quinke’s edema
e. Dermatomyositis
181.
Patient 35 years old, was entered to the clinic with complaints of grumbling in a
stomach, propensity to diarrhea, that are already present during 2 months. After the last
two weeks at the act of defecation noticed moving snatches of flat struture. Has habit to
use in a meal the raw stuffing from meat of home animal with spices. At colonoscopy
there are numerous lateral folds which fills almost all segment.What is the drug of choice
for treatment?
a. Mebendazol
b. Combantrin
c. Albendazol
d. *Phenasalum
e. Medamin
182.
Patient A., 17 years old, complaints of headache, stomachache, nausea,
enhanceable fatigue ability, insignificant pain and sometimes diarrheA. He was sick
during 4 months. Day prior he was addressed to the hospital in incandescence noticed a
round worm length 15cm. What will be the best treatment for him?
a. Levomycitin
b. *Decaris
c. Akrikhin
d. Ftalazol
e. Furazolidon
183.
Patient, 27 years old, complaints of itching in perianal area, which is present in
evening before sleep. It continues for 1-3 days and disappears independently, but
reconvolense afterwards. Most probable diagnose will be:
a. Trichinosis
b. *Enterobiosis
c. Ascaridosis
d. Helminthosis
e. Cystitis
184.
Patient, 27 years, complaints of weakness, irritation, pain of muscles, fever, dry
cough, pain in thorax, shortness of breath. In lung - dry and moist rales and wheezes are
determined without dulling of percutory sound. In sputum are еosinophil, crystals of
Charkot-Leyden with admixtures of blood. On the repeated X-ray of lungs shows plural
еosinophilic infiltrations of different sizes which change a form and localization (Leffler
syndrome).
a. Tuberculosis of lungs
b. Acute pneumonia
c. Bronchitis
d. AIDS
e. *Ascaridosis
185.
Patient A, 29 years old, complaints of the decline of appetite, nausea, vomiting,
stomach-ache, diarrhea, pain in the chest, weakness, dizziness, parahypnosis.There is
present hypochromic anaemiA. Periodically used raw and half cooked raw pork. What
most probable diagnosis?
a. Teniarinhosis
b. Difilobotriosis
c. *Teniosis
d. Trichinellosis
e. Ascaridosis
186.
Patient, 37 years old, complaints of pain in the chest, nausea, vomitting, periodic
attacks of Grand-mal epilepsy and memory became worse. Papule rash in a skin. What
most probable diagnosis?
a. Teniarinhosis
b. Difilobotriosis
c. Teniosis
d. Trichinellosis
e. *Ascaridosis
187.
Patient, 27 years, complaints of nausea, heaviness in epigastrium, stomach-ache,
diarhoea and general weakness. An increase in appetite, mass of body is decreased. In
blood moderate anaemia and high eosinophiliA. Noticed creeping out of segments with
excrement. She likes to use raw meat. What is the most probable diagnosis?
a. *Teniarinhosis
b. Difilobotriosis
c. Teniosis
d. Trichinellosis
e. Ascaridosis
188.
Patient 30 years old, complaints of weakness, increasing of temperature up to 39.2
°C, pain in muscles, fatigue, edema present. 2 weeks ago he was used pork with the flab of
meat. Edema is present,an enlarged liver is palpable. The BP is 90/60 mm of Hg. In blood
analysis: leucocytes – 12,4*109, eos – 19 %, young neutrophills – 2 %, neutrophills – 59
%, lymphocytes– 16 %, mon – 4 %. Specify the most probable diagnosis.
a. Leptospirosis
b. Flu
c. *Trichinosis
d. Typhoid fever
e. Spotted fever
189.
Sick, 45 years old was sick for 17days, lives in a village. The sick grumbles about
strong stomach-ache, frequent 15 times and more on a day emptying, nausea, vomit,
headache and fever. Objectively: person is pale, eyes are hollow, the turgor of skin is
sharply decreased.Tongue is dry assessed white coating, stomach is sharply blown away,
at palpation pain takes place in the area of colons. Liver is moderately enlarged. At
hemanalysis leucocytosis 16*109 L, eosinophills 1 %, ESR – 7mm/hr. Change of
leucocyte formula takes place. Emptying with addition of bloods, has a pungel smell. At
rectoscopy at the area sigmoid and bowels was found deep ulcers with undetermined
edges. What most probable pathology which predetermines such picture?
a. Amebiasis
b. *Balantidiasis
c. Nonspecific ulcerous colitis
d. Tumour of rectum
e. Food poisoning
190.
A sick child 8 years old complaints of itching in the perinium areA. Child creaks
teeth during sleeping, sleeps bad. Objectively: sufficient nourishment, pallor of skin,
tongue is insignificantly covered with white patches, stomach is soft, accessible for
palpationis painless. The lowering of haemoglobin level takes place. What is most
probable pathology which predetermines such picture?
a. *Enterobiasis
b. Salmonellosis
c. Ascaridosis
d. Giardiasis
e. Trichinosis
191.
A sick 26 years appeared to the therapeutic department with complaints of the
itching of skin, weakness, liquid emptying diarhoea 1-2 times a day. Objectively:
sufficient nourishment. On the skin has pouring out red color as “hives” which has linear
character. In blood eosinophilic reaction of blood – 28 %. Lives in rural area What is the
most probable pathology which predetermines such picture?
a. Salmonellosis
b. *Strongyloidosis
c. Food poisoning
d. Giardiasis
e. Trihocephallosis
192.
A sick 20 years arrived from Western Siberia complaint of pain in the area of liver
and gall-bladder, bitter taste in mouth. Objectively: abdomen is soft, accessible for
palpation, at palpation pain takes place in an area of bilious system. At blood analysis
еosinophills – 23 %. She had eaten fish. What is the most probable pathology which
predetermines such picture?
a. *Opistorhosis
b. Ascaridosis
c. Trihocephallosis
d. Giardiasis
e. Trichinosis
193.
Patient 28 years old, complaints of an increased appetite, weakness, decreased
physical activity tolerance, pain in stomach. Objectively: colour of skin is pale, signs of
meteorism,and glossitis. In blood thereis diminished amount of red cells, leucocytes,
thrombocyte and hypochromic anaemiA. It is known from epideminological anamnesis,
that the patient had spent 4th months on Volga river and had eaten the salted fish and
caviar. What is the most probable pathology which predetermines such picture?
a. Teniosis
b. Anaemia
c. *Difilobotriosis
d. Opisthorchosis
e. Helminthosis
194.
A 5 years old girl complaints of headache, decreased appetite, weakness, nausea,
vomiting, bitter taste in mouth, stomach-ache and periodic diarrheA. Objectively:
decreased nourishment status, on her tongue there is white coating observed. The stomach
is soft, accessible for palpation, and painful in the area of gall-bladder. No Change in
blood analysis was found. According to the mother the same symptoms were observed
with her son a month ago. What is the most probable pathology which predetermines such
picture?
a. Amebiasis
b. Balantidiasis
c. Intestinal trichomonosis
d. *Giardiasis
e. Dysbacteriosis
195.
A 23 years old patient complaints of weakness, nausea, periodic presence of
segments of helminth in the stool. In anamnesis the patient had eaten undercooked meat.
Faeces were sent for microscopic examination. The bovin solitaire was found in the stool.
What would be the drug of choice?
a. *Biltritsid
b. Pyrantelum
c. Piperazinum
d. Decaris
e. Fazizhin
196.
A 24 years old engineer from Donetsk, has spent one month in India where he
drunk unboiled water. After arriving home he has become ill. He appeared to the doctor
with complaints of fever, weakness, pain in stomach, diarrhea - 12-15 times a day with
mucous and blood (like raspberry jelly stool). Objectively: the state is relatively
satisfactory, appetite became worse, tongue is coated with white patches.On deep
palpation of abdomen patient complaints of pain especially in his right half and
hypochodrium areA. Liver and spleen are not changed. No change was found in blood
analysis. On rectoscopy clear mucous and hyperemia of mucous membrane in rectum with
ulceration in sigmoid colon were found. The stool test gave the growth of pathogenic
florA. What most probable pathology which predetermines such picture?
a. Strongyloidosis
b. *Amebiasis
c. Ulcerative colitis
d. Balantidiasis
e. Food poisoning
197.
Patient K., 30 years old, came with complaints of nausea, vomiting, pain in
epigastrium and paraumbilical areA. High body temperature, pale skin, dry. tongue
covered with whitish-gray coating. Quick pulse, low blood pressure. 2 hours before the
onset of symptoms of the disease he ate in cafe. What is the diagnosis most likely.
a. Cholera
b. Shigellosis
c. Thypoid fever
d. *Salmonellosis
e. Meningitis
198.
Disease started acutely with the complaints of watery diarrhea, vomiting, cramps in
the lower extremities. Objectively: slow voice, shunken eyes, quick pulse, low blood
pressure, decrease of urination, weak cardiac sounds. In liver and spleen no changes. Put
the correct diagnosis.
a. Salmonellosis
b. Dysentery
c. Food poisoning
d. *Cholera
e. Typhoid fever
199.
A Pakistani, 30 years old, severly ill: with the complaints of frequent diarrhea like
rice water. Objectively: body temperature 35,4 °C, skin of peripheral parts of the body
cold to the touch, acrocyanosis, xerostomia, sharply decreased skin elasticity and turgor.
What should be done to assess the degree of dehydration.
a. Measurement of central venous pressure
b. Determination of urea and creatinine in blood
c. *Funduscopy
d. Plain X-ray film of abdomen
e. Determination of specific gravity of blood plasma
200.
Patient N., 30 years old, works in a restaurant, admitted to the hospital complaining
of unbearable pain in the abdomen, increase of temperature 38,8 °C, nausea, vomiting,
diarrhea of dark-green colour, He took eggs, mushrooms, canned food. Objective state:
general condition is moderately sever, skin and mucous membranes are dry, pain in
epigastric region. Name the disease.
a. Shigellosis
b. Yersiniosis
c. *Salmonellosis
d. Cholera
e. Rotaviral infection
201.
Patient 35 years old with complaints of increasing of temperature to 39 °C with
chill, vomiting, pain in epigastric region, diarrhea with foul smell, abdominal cramps. 6
hours before onset of the disease ate raw eggs, fried potatoes with grinded meat, drank
juice. To clarify the diagnosis of serological diagnostic methods often used RNGA with
the salmonella groupdiagnostic tools and RA (Vidal test) . When blood should take for
the diagnostic procedures?
a. In the first day of illness
b. At the end of the first month
c. In 1st week in 3-4 days
d. *At the end of the 1st week from 7-10 days
e. During admission to the hospital
202.
A patient was hospitalised after 4 hours of consumption of raw eggs, complaining
frequent vomiting, abdominal pain, mostly in the right iliac area and umbilical areA.
Objectively pale skin, dry tongue with gray coating, hypotension, tachycardiA. What tests
should be carried out to confirm the diagnosis.
a. General blood analysis
b. Parasitological examinations
c. Microscopic examination of stool
d. Biological test
e. *Coproculture
203.
Patient G., 22 years old, was hospitalised in an infectious department complaining
of chills, stabing pain in epigastrium, diarrhea, temperature increased up to 39,6 °C,
repeated vomiting. Feces copious, watery, without pathological changes. 4 hours before
the onset of symptoms ate the meat, salad. What is the most appropriate drug to be
prescribe.
a. Antidiarrheal drugs
b. Flouroquinolone group of antibiotics
c. Sulfanilamide group of antibiotics
d. Desintoxication therapy
e. *Enterosorbents
204.
Patient K., 40 years old, was hospitalised with the diagnosis of intestinal infection.
Complaints of general weakness, headache and diarrheA. Symptomes appeared suddenly
2 hours after consumption of cake with custard. Suddenly there were chills, nausea,
profuse repeated vomiting, frequent diarrhea, abdominal pain and short-term loss of
consciousness, temperature increased to 38,6 °C. The preliminary dianosis?
a. Salmonellosis, localized form, gastroenteritis type
b. *Salmonellosis, localized form gastroenterocolitis type
c. Salmonellosis, localized form enterocolitis type
d. Salmonellosis, generalized form
e. Salmonellosis, nosoparasitic
205.
Patient K., 40 years old, hospitalised with the diagnosis of salmonellosis, with
symptoms of rise of body temperature to 40 °C, repeated vomiting, diarrheA. Objectively:
AP 60/20 mm Hg, pulse 120/minute, pale face, increased intestinal peristalsis, enlarged
liver and spleen. What is the best treatment.
a. *Glucocorticoids, desintoxication, antibacterial therapy
b. Diuretic, desintoxication therapy, antibacterial drugs
c. Enterorsorbents, diuretic, detsintoxication therapy
d. Glucocorticoids, diuretic, desintoxication therapy, antibacterial drugs
e. Glucocorticoids, diuretic, antibacterial drugs
206.
Patient P., 36 years old, a farmer is seriously ill, with high temperature, having
chills, headache, flatulence, nauseA. At present he is complaining of severe abdominal
pain, diarrhea up to 15 times a day stool is liquid, smelly, with impurities mucous and
blood. His eyes are sunken. The tongue is covered with white coating. On palpationpainful colon, increased liver. In blood – neutropil leukocytosis with a shift formula the
left anemiA. On rectomanoscopy deep ulcers with irregular edges are observed, including
along the folds. What is the diagnosis.
a. Shigellosis
b. *Balantidiasis
c. Nonspecific ulcerous colitis
d. Amoebiasis
e. Colon tumor
207.
A 29 years old patient T. Has been working on a pig farm. She was hospitalized
with symptoms of balantidiasis. What would be the drug of choice.
a. Benzylpenicillin
b. Gentamycin
c. Chloramphenicol
d. *Monomycin
e. Timogen
208.
Patient C., 35 years old, the disease has started severe chills, raising the
temperature to 39 oC, vomiting, pain in epigastric region, diarrhea with water-smelly
bowel movement. Over 6 hours after eating raw eggs, potatoes with braised meat, drank
juice. What kind of agent is likely to cause this condition.
a. *Salmonella
b. Vibrio cholera
c. Enteric stick
d. Campylobacter
e. Shigella
209.
Patient 25 years, received complaints of double vision of the eyes, a decline of
view, shortness of breath. Before the disease eat mushrooms home preservation.
Objective: pallor, wide pupils with a weak reaction to light, dry mouth, a violation of
swallowing, flatulence, delay of stool. What is preliminary diagnosis.
a. Leptospirosis
b. Yersinioz
c. *Botulism
d. Giardiasis
e. Salmonellosis
210.
Patient C., 22 years old, hospitalized in an infectious department complaining of
chills, temperature increase to 38,5 oC, vomiting, pain in epigastrii, frequent stool. 7 hours
before the disease, ate raw eggs, potatoes with braised meat, drink tomato juice. What
kind of organism most likely to cause disease.
a. Enterovirus
b. *Salmonella
c. Enteric stick
d. Staphylococcus
e. C. perfringens
211.
Emergency ambulance delivered in infectious hospital girl N., 17 years old,
complaining of headaches, expressed general weakness, pain in epigastrium, repeated
vomiting, diarrhea to 8 times a day greenish color. She was used 2 raw chicken eggs.
Body temperature 39 °C, the tongue covered with white coat, moderately cramps, pain in
abdomen and sigmoid areA. What is the most likely diagnosis.
a. Acute appendicitis
b. Crohn’s disease
c. Shigellosis
d. Viral gastroenteritis
e. *Salmonellosis
212.
Patient L., 32 years old, was admitted to the infectious office complaining of chills,
headache, expressed general weakness, cutting pain in epigastrii, diarrheA. The
temperature of 39 °C, repeated vomiting. Excrement abundant, greenish color, without
pathological impurities. Over 4 hours to illnesses used the meat salad in the factory
canteen. Which drugs will be most effective for the treatment of this patient.
a. Antibotulism serum
b. Salt and colloid solutions
c. Sulfanilamides
d. *Antibiotics
e. Enterosorbent
213.
Patient B., 55 years old, was hospitalized in an infectious hospital with complaints
of frequent vomiting, liquid excrements, abdominal pain, mainly in the right iliac area and
epigastrium. Pale skin, dry tongue with a touch of gray, hypotension, tachycardiA. The
doctor was suspected salmonellA. What laboratory test is need to perform.
a. General blood analysis
b. Parasytoscopy of blood smear
c. Microscopy of stool
d. *Coproculture
e. Biological test
214.
Patient C., 30 years old, was hospitalized with a preliminary diagnosis of
salmonellosis. Body temperature – 40 °C, repeated vomiting, profused diarrheA.
Hypotension (BP 55/30 mm Hg), expressed tachycardia (pulse to 120/min), face pale, pain
mainly in right iliac area and epigastrium, hepatosplenomegaly were found. What
treatment tactic will be most effective in this case.
a. Glucocorticoids, crystalloid solutions, antibiotics, diuretics
b. Cardiac glycosides, colloid solutions, antibiotics
c. Sorbents, diuretics, desintoxication
d. Dopamine, colloid solutions, antibacterial
e. *Glucocorticoids, crystalloid solutions, antibiotics
215.
Patient F., 25 years old, was hospitalized in the infectious hospital with complaints
of frequent vomiting, liquid stool, abdominal pain in epigastrium. Pale skin, dry tongue
with a grey cover, hypotension, tachycardiA. The doctor was suspected salmonellA. What
serologic study is need to perform.
a. Microprecipitation reaction
b. RA (Widal)
c. *RIGA with Salmonella diagnosticum
d. RIGA with Shigella diagnosticum and RA (Widal) with paired serum
e. RKC
216.
Patient N., 22 years old, was admitted to the infectious office complaining of chills,
headache, expressed general weakness, cutting pain in sigmoid area, diarrheA. The
temperature of the body of 39,3 °C, vomiting. Faces watery. Patient was used the salad
with sausage and eggs. For further diagnosis doctor was prescribed RIGA with salmonella
diagnosticums. When it’s necessary to take blood for the investigation.
a. At the 1st and 2nd days of illness
b. Once in recovering period
c. *At the end of the 1st week and after 7-10 days
d. At the end of the 1st week and after 3-4 days
e. Once in acute period
217.
Patient B., 32 years old, an employ of poultry was hospitalized with complaints of
pain in abdomen, mostly in sigmoid area, fever up to 38,8 oC, nausea, vomiting, diarrhea
with dark-green colour of stool, without any pathological changes. Skin and mucous
membranes are dry, moderate tension in epigastric region and pain in the right iliac areA.
The most likely diagnosis.
a. Acute appendicitis
b. Cholera
c. Dysbacteriosis
d. *Salmonellosis
e. Rotavirus gastroenteritis
218.
3 patients were delivered with symptoms of fever, diarrhea and vomiting.
Excrements dark-green, without any pathological changes. All patients together took part
in the preparation of food and have used eggs, meat salad. The most likely diagnosis.
a. Cholera
b. Botulism
c. Dysbacteriosis
d. *Salmonellosis
e. Shigellosis
219.
Patient L., 33 years old, was admitted to the hospital with the complaints of
untolarable pain in the abdomen, mostly in sigmoid region, fever up to 38,8 oC, nausea,
vomiting , diarrhea of dark-green colour in every1-1,5 hour, without admixture of blood
and mucus. The condition progressively deteriorated, increasing hypotension, paleness,
weak cardiac sounds, decreased intestinal peristalsis with moderate tension in epigastric
region and pain in the right iliac region. What tactics of treatment?
a. Glucocorticoids, infusion of salt solutions, antibiotics intravenously, diuretic
b. Cardiac glycosides, infusion colloidal solution, antibacterial drugs
c. Enterosorbents, diuretic, desintoxication therapy
d. Mezaton 1% / in drops, infusion of salt solutions, antibacterial drugs orally
e. *Glucocorticoids, infusion of salt solutions, antibiotics intravenously
220.
Patient 45 years old, became ill within 10 hours after consumption of grinded meat.
Complaints of vomiting, pain in epigastric region, diarrhea of green color, increasing of
temperature up to to 39 °C. During objective examination revealed that: patients general
condition is sever, pain in the muscles of the lower limbs, pale face, acrocyanosis. Pulse
130 per min,. B.P 70/40 mm Hg. weak cardiac sounds, dry tongue, brown-whitecovering .
abdomen cramps, painful iliac and epigastric regions. What is the most likely diagnosis.
a. Shigellosis
b. Cholera
c. *Salmonellosis
d. Viral gastroenteritis
e. Eshericiosis
221.
5 patients during 15 hours were admited In a hospital. All of them have similiar
complaints of mild diarrhea and vomiting developed ,diplopia, midriasis, visual
disturbance, difficult swallowing, dry mouth, difficulty in breathing. All patients together
joined in a dinner party, used different dishes, including meat, salad, canned mushroom.
The most likely diagnosis.
a. Cholera
b. *Botulism
c. Typhoid fever
d. Salmonellosis
e. Rotaviral gastroenteritis
222.
Patient G., 24 years old, 6 hours after taking his breakfast with boiled eggs, a cup
of coffee appeared chill, fever up to 38,8 °C, pain in left inguinal area, nausea,vomiting,
rumbling in the stomach, then frequent diarrhea, feces copious, watery, mudy-green. What
is the most likely diagnosis.
a. Cholera
b. Botulism
c. *Salmonellosis
d. Typhoid fever
e. Shigellosis
223.
Patient U., who was arrived from the Crimea, diarrhea appeared in an intervel of 11.5 hour. with copious watery increments, without admixtures of mucous and blood. He
was admitted in an infectious dipartment with the diagnosis of “acute intestinal infection”.
What laboratory tests are necessary to confirm the etoilogical diagnosis.
a. General blood analysis
b. Bacteriological blood analysis
c. Microscopical analysis of stool
d. *Coprogram
e. The biological examination in mice
224.
Patient C with complaints of diarrhea with mucous, stabing pain in epigastric area,
rumbling in the stomach, fever. According to his anamnesis he took preserved milk, eggs,
salad of fresh vegetables. What is the disease most likely.
a. Shigellosis
b. Yersiniosis
c. *Salmonellosis
d. Cholera
e. Botulism
225.
Patient B., 38 years old, came to the admission department with the complaints of
fever up to 38 °C, vomiting 4 times in a day, diarrhea up to 6 times in a day. Durin
objective examination revealed that BP 125/75 mm Hg, pulse 80 per min, abdomen is
soft,painful in the right iliac and epigastric area .According to anamnesis he took salad of
boiled eggs and sausage. What treatment should be started first.
a. Antibotulism serum
b. Intravenous antibiotics
c. Washing of the stomach and intestine, rehydration therapy , glucocorticoids
d. *Gastric lavage and washing of intestine, rehydraton therapy enterosorbents
e. Treatment after getting of the laboratory test results
226.
Patient 27 years old, complaints of headaches, weakness, pain in epigastric area,
vomiting, diarrhea (9 times in a day; huge excrement, greenish colour). Before the
appearance of disease he consumed raw eggs. Objectively: body temperature 38,8 °C.
Tongue is white, pain in epigastric and umbilical region. The most likely diagnosis.
a. *Salmonellosis
b. Cholera
c. Dysentery
d. Viral gastroenteritis
e. Acute appendicitis
227.
Patient K., 30 years old, came with complaints of nausea, vomiting, pain in
epigastrium and paraumbilical areA. High body temperature, pale skin, dry. tongue
covered with whitish-gray coating. quick pulse, low blood pressure. 2 hours before the
onset of symptoms of the disease he ate in cafe. What is the diagnosis most likely.
a. Cholera
b. Shigellosis
c. Thypoid fever
d. *Salmonellosis
e. Meningitis
228.
Disease started acutely with the complaints of watery diarrhea, vomiting, cramps in
the lower extremities. Objectively: slow voice, shunken eyes, quick pulse, low blood
pressure, decrease of urination, weak cardiac sounds. In liver and spleen no changes. Put
the correct diagnosis.
a. Salmonellosis
b. Dysentery
c. Food poisoning
d. *Cholera
e. Typhoid fever
229.
A Pakistani, 30 years old, severly ill: with the complaints of frequent diarrhea like
rice water. Objectively: body temperature 35,4 °C, skin of peripheral parts of the body
cold to the touch, acrocyanosis, xerostomia, sharply decreased skin elasticity and turgor.
What should be done to assess the degree of dehydration.
a. Measurement of central venous pressure
b. Determination of urea and creatinine in blood
c. Funduscopy
d. X-rays examination of abdomen
e. *Determination of specific gravity of blood plasma
230.
Patient N., 30 years old, works in a restaurant, admitted to the hospital complaining
of unbearable pain in the abdomen, increase of temperature 38,8 oC, nausea, vomiting,
diarrhea of dark-green colour, He took eggs, mushrooms, canned food. Objective state:
general condition is moderately sever, skin and mucous membranes are dry, pain in
epigastric region. Name the disease.
a. Shigellosis
b. Yersiniosis
c. *Salmonellosis
d. Cholera
e. Rotaviral infection
231.
Patient 35 years old with complaints of increasing of temperature to 39 °C with
chill, vomiting, pain in epigastric region, diarrhea with foul smell, abdominal cramps. 6
hours before onset of the disease ate raw eggs, fried potatoes with grinded meat, drank
juice. To clarify the diagnosis serological diagnostic methods often used. When blood
should be taken for the diagnostic procedures?
a. In the first day of illness
b. At the end of the first month
c. In 1st week in 3-4 days
d. *At the end of the 1st week from 7-10 days
e. During admission to the hospital
232.
A patient 20 years old, had treated himself concerning an acute respiratory disease
for 5 days, but marked no displays of respirator syndrome. Last 2 days temperature has
been normal, appetite disappeared, however, appeared pain in epigastrium with nausea,
and urine turned dark. About what illness is it possible to think?
a. Hepatitis B
b. Infectious mononucleosis
c. Pseudotuberculosis
d. Leptospirosis
e. *Hepatitis A
233.
A patient 28 years old, an injection drug addict, complaints of dull pain in right
subcostal region, weakness, decline of appetite, pain in joints. The symptoms have been
present for 2 weeks. At examination: the icterus of skin and scleras observed. The liver
and spleen are enlarged. Urine is dark, excrements are discoloured. What is preliminary
diagnosis?
a. Hepatitis A
b. *Hepatitis B
c. Chronic cholecystitis
d. Toxic hepatitis
e. HIV-infection
234.
A patient 25 years old got sick suddenly after chills and temperature increase up to
38,0 °C. There was vomiting 2 times. Moderate pharyngeal pain appeared at swallowing
with stomach discomfort. Signs of bursitis observed on knee and elbow joints. The light
icterus of sclera and skins of joints on 4th day, on the skin of lateral surfaces of trunk,
forehead and lower extremities. The small bright red papular rash appeared, more
concentrated in natural folds.Liver is enlarged, the tongue is raspberry like appearence.
Preliminary diagnosis would be:
a. Hepatitis A
b. Hepatitis B
c. *Pseudotuberculosis
d. Scarlet fever
e. Infectious mononucleosis
235.
A 19 years old patient was diagnosed with hepatitis B. After violation of diet and
nervous stress the condition of patient got worse: increasing of icterus intensity , pulse
110/min, BP 80/50 mm Hg. «Coffee-grounds» vomiting . Decreasing of the liver size,
pain during palpation. What complication would you think about?
a. Infectious-toxic shock
b. Acute kidneys insufficiency
c. Acute adrenal insufficiency
d. Hemolitic icterus
e. *Acute hepatic insufficiency
236.
A patient C., 43 years old, has been treated for 5years . During the last
hospitalization a liver-biopsy was conducted. Histologicaly there is bulb dystrophy and
necrosis of hepatocytes, with leucocyts infiltration, and pericellular fibrosis. What
diagnosis is most credible in this case?
a. *Alcohol liver cirrhosis
b. Viral hepatitis B
c. Viral hepatitis C
d. Cryptogenic hepatitis
e. Fatty dystrophy of liver
237.
Patient B., 23 years old. Objectively: skin is yellow, icterus of sclerA. Pulse
66/min, BP 120/80. Indexes of AlAT, AsAT, and Tymol tests are normal. General
billirubin is 34 mmol/l due to indirect fraction. Specify the most credible diagnosis.
a. Chronic toxic hepatitis
b. Chronic cryptogenic hepatitis
c. Chronic hepatitis C
d. *Zhilber’s syndrome
e. Chronic B hepatitis
238.
A nurse got sick on 16.09: general weakness, nausea, vomiting, pain in joints
appeared. 21.09 urine turned dark, and skin yellow. 21.09 she was hospitalized.
Objectively: general status worse, temperature 38,2 °C, expressed skin and mucous
membranes jaundice. Liver palpated 3 cm below the costal arc. 26.09 stomach-aches and
increased sleepiness with disorientation in time and place were appeared. 27.09
haematomas in the places of injections also appeared. Hepatic breath was noticed. Light
edema of feet and lumbar region were observed. The liver was not palpable. Development
of what syndrome can be?
a. Meningoencephalitis
b. *Hepatic encephalopathy
c. Acute nephrosonephritis
d. Pancreatitis
e. IDS syndrom
239.
A patient B., 52 years old, for 3 month complaints of nausea, periodic vomiting,
swelling of stomach, weakness, loss of weight up to 12 kg, consistency of stool is
chainging. During some days consciousness is entangled, somnolence, allolalia, general
weakness are expressed. Temperature of body 37,4 °C. Icterusis on the skin. Tremor of
brushes and nystagmus is marked. Pupils are narrowed, with a slow reaction on light,
increase of tendon reflexes. Ps 112/min, unrhythmical. BP 90/65 mm Hg. A liver is dense
+2. Laboratory tests: Hb 86 g/L, Leuc 4,2*109/L, ESR 18 mm/hour, glucose 3,4 mmol/l,
general bilirubin 56,7 mm/L, albumen 52 g/L, K+ 3,2 mmol/l, AsAT 0,62; AlAT 0,84;
prothrombin index-58 %; рН blood 7,3. What is previous diagnosis?
a. Hepatocirrhosis
b. Vilsona-Konovalov disease
c. Cancer of liver
d. *Endogenous hepatic encephalopathy
e. Meygs disease
240.
A patient, 17 years, complaints of a weakness, worsening of appetite, nausea,
painfull in right subcostal area, dull ache in the body, dark color of urine, temperature to
37,8 °C. She stood in contact with sick on icterusis 1,5 months ago. Objectively: yellow
colour of the skin and visible mucous membranes. The liver is increased to 1,5 cm.
Unsignificant enlargment of spleen. What is your preliminary diagnosis?
a. Viral hepatitis B
b. Infectious mononucleosis
c. *Viral hepatitis A
d. Leptospirosis
e. Pseudotuberculosis
241.
Before the appearance of jaundice in a patient 16 years old, during 2 days there
were an increasing of the body temperature (38,5 °C), headache, dull ache disturbed
whole body. Name the variant of pre-icteric period of viral hepatitis for this patient?
a. *Influenza-like
b. Astenovegetative
c. Artralgic
d. Dyspeptic
e. Allergic
242.
A patient T., 28 years, appilled due to worsening of common status at the seventh
day to the infectious department concerning viral hepatitis. Nausea and vomiting were
present, hepatic smell appeared, increasing of the liver sizes. What biochemical index
does allow to suspect complication in the patient?
a. Increasing activity of AlAT
b. Increasing of bilirubin, increasing activity of AlAT
c. Increasing timol test
d. Decreasing of protrombin, increasing of bilirubin
e. *Decreasing of AlAT
243.
A sick P., 54 years old, complaints of dull pain in right subcostal area, bad taste in
the mouth, itching of the skin, increasing of abdomen. After the liver puncture fatty
dystrophy of hepatocytes, eccentric placing of veins in a hepatic lobule were found. What
laboratory index are most characteristic in this case?
a. Alkaline phosphotase
b. Hemodiastase
c. *Level of transaminases
d. Glucose of blood
e. Creatinphosphokinase
244.
Direct bilirubin is increased , in urine there is significant increase of bilirubin and
urobilin, increasing of stercobilin of excrements. What is the type of icterus?
a. Haemolitic
b. *Parenhimatous
c. Transport
d. Extraliver
e. Mechanical
245.
On the average 15 to 30 % of all population of the planet suffer from some
pathology of liver. Prevalence of hepatitis and cirrhosis in the European countries is about
1 % of adults. Annually in the world there are about 2 million people with acute viral
hepatitis. What % of all cases will develop chronic form.
a. 100 %
b. 50 %
c. 25 %
d. *10 %
e. 1 %
246.
A patient, 24, complaints of pains in right subcostal area, increasing after-meal,
nausea, increase temperature of body to 37,7 °C, icterus, pains in large joints. He is ill
from 8 months. Suffers a nonspecific ulcerative colitis. Hepatosplenomegaly. ESR 47
mm/hr, general bilirubin level is 86,1mmol/lt, direct-42,3 mmol/lt. In blood found out
antibodies against smooth muscles. General albumen of 62 gram/lt, albumin б. 40 %,
globulin. 60 %, gamma globulins 38 %. Not found out the markers of viral hepatitis. On
USD diameter of portal vein is 1 cm What will be your diagnosis?
a. Primary biliary cirrhosis
b. Zhilber syndrome
c. Viral hepatitis
d. Hemochromatosis
e. *Autoimmune hepatitis
247.
Woman of 22 years old, in the 7th month of pregnancy felt ill acutely in 3 weeks
after arrival from Turkmenistan. An icterus, untraceable vomiting, pain in right subcostal
region, hemmorhaeges on the skin, appeared after three-day fever. Most probable
diagnosis will be:
a. Acute fatty hepatosis of pregnancy
b. Cholestatic hepatosis of pregnant
c. *Viral hepatitis of A
d. Viral hepatitis B
e. Acute sepsis
248.
Weakness appeared in a patient, decrease in an appetite, began pains in the joints
of extremities, felling of weight in right subcostal region, vomiting. Urine became dark in
12 days, and in a day – sclera and skin turned yellow. It was discovered in an
immunogram: antuIgG -HAV (+), HBsAg (+), antiIgМ -НBcorAg (+), anti-НCV (–), antiНDV (–). To the patient 3 months back was extracted a tooth. Most probable diagnosis?
a. *Viral hepatitis B
b. Viral hepatitis A
c. Viral hepatitis C
d. Viral hepatitis D
e. Viral hepatitis E
249.
A patient 42 years, radiologist, entered surgical department with bleeding from the
mouth, was admitted to the gastroenterological departments . He considered himself
healthy. 20 years ago had haemotransfusion. Fibrogastroduodenoscopy shows venous
dilation of the oesophagus of the III degree. Most probable etiologic reason of disease:
a. Autoantibodies against hepatocytes
b. Viral hepatitis A
c. X-ray irradiation
d. *Viral of hepatitis B
e. Defficiency of antitrypsin
250.
A patient during half of the year got frequent parenterally injections concerning
bronchial asthmA. There was decrease in appetite gradually, started to feel weakness,
arthralgia, dark coloured urine, the icterus of skin appeared. Objectively: temperature of
37 °C, pulse 68/min BP 115/70 mm of Hg Liver +4 cm, spleen +1 cm, skin and sclera is
yellow coloured In the general blood analysis: amount of leucocytes: 3,6 109,among them
52 % lymphocytes, ESR 6 mm/hr, activity of ALAT is increased in 10 times. which
information will be more credible than all in blood of patient?
a. Anti-HBs antibodies
b. Anti-HAV IgM antibodies
c. *Anti-HBV antibodies
d. Anti-HCV IgG antibodies
e. Anti-HEV IgM antibodies
251.
Patient 34 years old complaints of fatigue, decreasing of appetite, nausea, feeling
of weight in a epigastric region, bitter taste in to the mouth. Objectively: Pulse is 76/min ,
temperature 37,2 °C. Skin is pale, with single vascular “asterisks” present on thorax.
Liver is enlarged with 2 cm below the costal arch . In blood: bilirubinn :36 mmol/lt ,
ASAT :2,5 mmol/lt, ALAT :2,8 mmol. HBeAg, anti-HBc (–), HBsAg and anti-HBe (+).
What will be the diagnosis of the patient?
a. *Chronic hepatitis, phase of integration, activity is poorly expressed
b. Chronic hepatitis, phase of replication, activity is poorly expressed
c. Chronic hepatitis, phase of integration, high activity
d. Chronic hepatitis, phase of replication, activity is poorly expressed
e. Chronic hepatitis, phase of replication, moderate activity
252.
A patient of 34 years complaints of fatigue, decreasing of appetite, nausea, feeling
of weight in a epigastric region, bitter taste in the mouth. Objective examination : Pulse
76/min, temperature 37,2 °C. Skin is pale, single vascular star-like rash on the thorax.
Liver is 2 cm below the costal arch. In blood: bilirubin level is 36 mmol/lt, ASAT 2,5,
ALAT 2,8 . Anti-HBc (–), HBsAg and HBeAg (+). What therapeutic tactic is expedient in
this case?
a. Therapy by hepatoprotectors
b. *Antiviral therapy by lamivudine
c. Antiviral therapy by ribavirin
d. Therapy by corticosteriods
e. Therapy by immunostimulators
253.
Man 30 years, drug addict, takes drugs intravenously. Has been taking drugs for 12
years Complaints of weakness, moderate icterus, weight in right subcostal region. The
state was worsened gradually. Biochemical indexes: general bilirubin 28,2 mmol/lt;
ALAT 1,0, ASAT 0,8 . Will you define a diagnostic method which it is expedient to
conduct for establishment of etiologic diagnosis?
a. Biopsy of liver
b. *Polymerase chain reaction (PCR)
c. Enzymes of liver
d. Computer tomography
e. Immunological tests
254.
Patient B., 51 years, a lot of years used an alcohol. Complaints of nausea, liquid
stools, icterus of skin. Objectively: skin and sclera is yellow coloured, atrophy of muscles,
subfebrile temperature. Liver 3 cm below the costal arch, painful on palpation What
method of diagnostics most informing in this case?
a. Activity of cytolysis (AsAT, AlAT)
b. Proteinogram
c. *Biopsy of liver
d. Markers of viral hepatitis
e. Violation of cellular immunity (T4,T8)
255.
Patient of B., 64 years old, has been sick with chronic hepatitis. Complaints of pain
in the liver, growth retardation. Objectively: his skin is dry, erythemic, atrophy of
muscles, telangiectasis on shoulders, hyperemia of hands, abdomen is enlarged. The liver
below the costal arc on 3 cm, painful, dense, spleen on 1 cm below the costal arch,
positive symptom of fluctuation. Laboratory investigation: hypergammaglobulinaemia,
increasing of AsAT activity, AlAT is highly increased. What drug of choice for the
patient?
a. Lactulose
b. *Hepatoprotectors
c. Antibiotics
d. Glucocorticoids
e. Alpha interferon
256.
A patient 48 years old, complaints of attacks pains in right subcostal area after the
physical loading. Periodically marks more light excrement, darkening of urine.
Objectively: skin and mucous membranes high icteric. General bilirubin 36,8 mm/L,
direct fraction - 26,4. Ultrasound of gall-bladder: thickness of wall 4 mm, there is a lot of
bile in the ducts. It is necessary to prescribe with a lytolitic purpose:
a. Choleretics
b. *Ursofalk
c. Cholekinetics
d. Spasmolytics
e. Cytostatics
257.
During annual inspection increasing of a liver to 4 cm of 23 years old patient was
found, increasing of bilirubin level in 2 times, AlAT in 2,5 time. He has been often sick
with genital herpes. It was discovered RNA of hepatitis C virus. What is etiologic
treatment
a. *Interferons
b. Essencial phosphotides
c. Choleretics
d. Cholekinetics
e. Ursodesoksihole acids
258.
A woman of 22 years old, on the 7th month of pregnancy, fell ill suddenly after 3
weeks arrival from Turkmenistan. An icterus, incessant vomiting, pain in right subcostal
area, skin haemorrhages, were appeared after 7-daily fever. What is most credible
diagnosis:
a. *Hepatitis A
b. Hepatitis B
c. Acute fatty hepatosis of pregnant
d. Cholestatic hepatosis of pregnant
e. Cholecystitis
259.
A patient 28 years old, injection drug addict, complaints of dull pain in the right
subcostal area, weakness, decline of appetite, pain in joints, which disturbs during 2
weeks. At examination: icterus of the skin and sclera, enlargement of the liver and spleen,
dark urine, excrement is discoloured. What is preliminary diagnosis?
a. HIV-infecion
b. Hepatitis B
c. Hepatitis C
d. *Toxic hepatitis
e. Hepatitis A
260.
At inspection of a 8 weeks term pregnant woman HBsAg was found. Level of
bilirubin of blood and activity of ALAT were normal. What is necessary to do?
a. *To save pregnancy and conduct the inoculation to newborn against hepatitis
B
b. Termination of pregnancy
c. Termination of pregnancy and conduct treatment by interferon
d. To save pregnancy and conduct treatment by lamivudin
e. To save pregnancy and ultrasonic inspection of the fetus
261.
Student 20 years, treated oneself on an occasion of ARVI (increasing of
temperature to 38,2 °C during 3 days). He complaints of worsening of appetite, increasing
fatigue at a normal temperature and absence of the catarrhal phenomena of upper
respiratory tracts. A doctor found out an increasing and moderate sickliness of liver. There
were cases of hepatitis A in a student’s group. What method of investigation will allow?
a. Ultrasound scanning of the liver
b. Determination of bilirubin level of the blood
c. Determining the amount of beta-lipoproteins
d. *Determination of activity of аminotransferases of the blood
e. Immunofluorescent research of the nasal smears
262.
A patient I., 25 years old, appealed to the internist with complaints of a general
weakness, loss of appetite, pain in the right hypochodrium. Treated himself on an
occasion of ARVI ambulatory 3 days. Became acute worsening of fealling: nausea, pain in
right hypochondrium, ochrodermia of sclera, darkening of urine, appeared. Objectively:
breathing superficial, cardiac tone is rhythmic. Abdomen is soft, painfull in right
hypochondrium, a liver is megascopic, a spleen not palpable. What is most credible
diagnosis?
a. *Hepatitis A
b. Leptospirosis
c. Cholecystitis
d. Influenza
e. Pseudotuberculosis
263.
A patient 42 years old, complaints of dull pain in a right hypochondrium,
weakness, decline of appetite, itching of skin, icterus. Disease began 1 month ago from
protracted аrthralgia, disgust for a meal, strong weakness. An icterus which now increased
considerably appeared three weeks ago. Temperature 36,0 °C. Pulse 56 per 1 min. A
tongue is assessed with white cover. A stomach is soft, sickly in right hypochondrium. A
liver +3 cm, spleen is not megascopic. Common analysis of blood: leukopenia, relative
lymphomonocytosis, ESR 3 mm/hour. What diagnosis is most credible?
a. Cancer of head of a pancreas
b. *Hepatitis B
c. Hepatitis A
d. Cirrhosis of liver
e. Chronic cholecystitis
264.
For a patient 35 years after 4-months of treatment by isoniaside - аdynemia,
icterus, pain in right hypochondric were appeared. A liver is megascopic. In blood activity
of enzymes of AlAT is enhanceable in three times, AsAT in two times. Bilirubin of blood
of 122 gm/ml (conjugated – 82, unconjugated – 40). НBs-аntigen is not found out. What
is the diagnos?
a. Calculary cholecystitis
b. Hepatocirrhosis
c. Acute viral hepatitis
d. Chronic active hepatitis
e. *Toxic hepatitis
265.
A patient 75 years old. Complaints of a subfibrile temperature, general weakness,
pharyngalgia, conjunctivitis. A child in family an acute adenoviral disease is ill. A patient
considers itself a patient the second day. At examination discovered sign of acute
pharyngitis. Lymphatic nodes are megascopic: neck front and back, arm-pits and inguinal,
to 1 cm in a diameter, soft, not soldered between itself and with a surrounding tissue.
Hyperemia of pharynx and tonsills. Wheezes are not present in lungs. Breathing clean.
Tones of heart are muffled. BP - 140/80 mm Hg, Ps - 80 per 1 min. Stomach soft.
Palpatory- megascopic liver and spleen, 3 cm below the costal arc, soft, painless. Choose
the most credible diagnosis:
a. Flu, to middle weight
b. Limfogranulomatosis
c. *Adenoviral infection
d. Infectious mononucleosis
e. Hepatitis A
266.
A patient 35 years old, complaints of aching pain in right hypocostal area, nausea,
decline of appetite. Beginning of disease binds to appendicitis. After it in 2 months an
icterus appeared first. Treated oneself in an infectious department. In 1 began to notice
aching pain in right hypocostal area, in analyses is an increase of level of bilirubin. Your
diagnosis?
a. Calculary cholecystitis
b. Zhil'ber disease
c. Acute viral hepatitis
d. Chronic cholangitis
e. *Chronic hepatitis
267.
The patient T., 35 years, operating trained nurse, appealed to the doctor on the 8th
day of gradual development of illness with complaints of a general weakness, rapid
fatigueability, dark color of urine. In the morning noticed the icterus. On examination
temperature of body 36,8 °C. Found out the increase of liver -+3 sm The changes of what
laboratory index most informing at this illness?
a. *AlAT
b. Hemodiastases
c. Protrombin index
d. Cholesterol
e. Alkaline phosphatase
268.
23-years old patient during 6 months gets diabetes mellitus in a policlinic the
injections of insulin. A weakness, arthalgia, grew gradually, an appetite disappeared, then
dull pain appeared in an epigastrium after-meal. In 2 weeks from the beginning of illness
noticed the dark color of urine, and afterwards – icterus on a background which the
general state continues to be worsened. Temperature of body 36,5°C, Ps 58 per 1 min
Liver +5 sm, spleen +1 sm What from hemanalysis is it needed to appoint for
confirmation of diagnosis?
a. Activity of alkaline phosphatase
b. Activity of lactatdehydrogenase
c. *Activity of ALAT
d. Activity of creatinphosphokinase
e. Activity of amylase
269.
42-years old patient got blood transfusion two months ago. A weakness was
gradually increased, an appetite was worsened, arthalgia, small nausea appeared. On a
12th day noticed an insignificant icterus, appealed to the doctor. At examination is the
general state fully satisfactory. The temperature of body is normal, Ps 60 per 1 min, BP
100/70 mm Hg.A liver and spleen is moderate megascopic, a stomach at palpatory is not
sickly. Anti-HCV IGM is found in blood. What from changes in the global analysis of
blood most probably?
a. Neutrocytosis
b. *Leykopeniya
c. Lymphopenia
d. Speed-up ESR
e. Aneozinofiliya
270.
A 17-years-old patient during 4 days suddenly had headackes, myalgias, fever of
permanent type. From a 5th day is a normal temperature of body, the general state was
considerably improved, an appetite appeared, nausea, general weakness, disappeared, but
an icterus became noticeable. The general state is satisfactory, liver +3 cm, spleen +1 cm,
Ps 56 per 1 mins Peripheral lymphatic knots, amygdales are not megascopic, hyperemia of
pharynx is not present. At research of global analysis of blood is leykopenia with a
relative lymphocytosis, 3 % virocytis, ESR normal. Activity of ALAT of blood is increase
in 5 times. What diagnos is most credible?
a. *Hepatitis A
b. Infectious mnonucleosis
c. Citomegaloviral infection
d. Hepatitis B
e. Hepatitis C
271.
On the 15th day of illness in a 17-years-old patient with hepatitis anti-HBs is
found in blood. The condition of patient was considerably worsened the day before. There
was excitation, a nose-bleed, a hemorragic rash appeared on a skin, diminished and
became sickly at palpatory liver. Ps 106 per 1 min, BP of 110/ 70 mm Hg, the temperature
of body subfebrile. What changes of blood indexes is most credible?
a. Growth of alkalinephosphatase
b. Growth of Fibrinogenum
c. Growth of indirect bilirubinum
d. *Decline of protrombin index
e. Growth of albumin
272.
25-years old woman during a year got numerous injections concerning bronchial
asthmA. An appetite disappeared gradually, a weakness, arthalgia grew, urine became
dark in two weeks, through three – an icterus on a background which the general
condition continues to be worsened appeared. The temperature of body is normal, Ps 62
per 1 min. Moderate sickliness in the area of pancreas, positive symptom of
Voskresensky, liver +4 cm, spleen +2 cm. What markers of viral hepatitis more credible
will be positive for a patient?
a. *Anti-HBc IGM
b. ANTI-HEV IGM
c. ANTI-CMV IGM
d. HBsAg
e. ANTI-HAV IGM
273.
26-years old man during 6 days marks gettings up temperatures with a strong chill.
6 months ago there were alike attacks of fever during a trip to Afghanistan, from where
returned 4 months ago. Temperature of body 36,4°C, Ps 94 per 1 min. The moderate
increasing of spleen and liver is marked. A diuresis suffices, without pathological
changes. What disease can be suspected for a patient?
a. Leptospirosis
b. Hemorragic fever
c. *Malaria
d. Viral hepatitis
e. Sepsis
274.
A 16-years-old patient in a month after returning from Crimea had headackes,
myalgias appear suddenly, strong weakness with a fever to 39°C during 3 days. The
general condition was considerably improved farther, a bad appetite and nausea, dull
pain, was saved only in right hypocostal area, became dark urine, an excrements became
white. On a 6th day a moderate icterus appeared. Liver +4 cm, spleen +1 cm. In the
general blood analysis is leykopenia, relative limphomonocytosis, ESR 4 mm/hour.
General bilirubin of blood 89 mmol/L, the direct prevails, activity of ALAT is increase in
4 times, ASAT – in 3 times. What is most credible diagnoses?
a. Hepatitis B
b. *Hepatitis A
c. Opisthorchiasis
d. Hemolitic icterus
e. Acute cholecystitis
275.
Patient A., 19 years, became sick acutely – the temperature of body rose to 39 °C,
a weakness, nausea, appeared. Vomiting, a stomach-ache are not present. A temperature
reposed on high numbers 2 days, then went down to normal. Urine became dark on the 6th
day of illness, subicterus noticed on 5th. To this time condition of patient was improved,
nausea, weakness, disappeared. Objectively: moderate icterus of skin and sclera, a rash is
not present. Pulse 66 per 1 min, BP 110/70 mm Hg. Abdomen is soft, painless in all of
parts, liver + 2,5 cm, the spleen was palpitated. What is the reason of changes in the preicteric period of the disease?
a. *Toxicemiya
b. Bacterialemiya
c. Pancreatitis
d. Viremia
e. Cholecystitis
276.
40-years old patient during 2th days marks absence of appetite, nausea, general
weakness. A year ago carried acute hepatitis B, avoided a clinical supervision.
Objectively: the temperature of body is normal, skin and sclera are icteric, liver + 3 cm. A
spleen is not megascopic. Urine is moderate dark colouring. ALAT of blood 4,0. What is
most credible diagnosis?
a. Chronic viral hepatitis, minimum activity
b. Chronic cholecystitis
c. *Chronic viral hepatitis, moderate activity
d. Hepatocirrhosis
e. Chronic viral hepatitis, high activity
277.
A man, 37 years, injection drug addict, alcoholic, complaints of a general
weakness, dull pain in right hypocostal area, increasing of abdomen, shortness of breath.
He was sick by chronic hepatitis during 10 years. Objectively: temperature of body
37,1°C, an expressed icterus. Liver +6 cm. Free liquor in abdomen. Positive symptom of
fluctuation. What complication present?
a. Exacerbation of chronic hepatitis C
b. Acute hepatic insufficiency
c. Hepatocarcinoma
d. Exacerbation of chronic hepatitis B
e. *Hepatocirrhosis
278.
A patient is 59 years, suffers from chronic viral hepatitis with development of
cirrhosis, hospitalized with a diagnosis: “bleeding from the veins of esophagus”. Acute
pallor of skin covers. BP 80/40 mm Hg, pulse 100 per 1 min, the temperature of body is
normal. On a front abdominal wall a venous net is extended. What preparation is it
necessary to begin therapy from?
a. Albumen
b. Plasma
c. Neogemodez
d. *Blood
e. Glucose
279.
Patient 20 years, treated oneself on an occasion ARVI during 5 days, but marked
no displays of respirator syndrome. Last a temperature is normal 2 days, an appetite
disappeared however, pain appeared in an epigastrium, nausea, urine had darked. About
what illness is it possible to think?
a. Acute hepatitis B
b. Infectious mononucleosisз
c. Pseudotuberculosis
d. Leptospirosis
e. *Hepatitis A
280.
In 2 months after returning from India, where often drank unboiled water, the 23years-old pregnant nauseated, strong general weakness, head pain, later the temperature of
body rose to 38,6°, which stuck to within a week. An icterus appeared on a 6th day, the
general state continued to be worsened. On the 12th day of illness the general state heavy.
EuphoriA. Vomiting at night. Complete fastidium. Bright icterus, signs of hemorragic
syndrome, tachycardiA. BP 110/60 mm Hg, temperature of body of 37,8°C. A liver is
insignificantly megascopic, soft, painfull, spleen +2 sm There is neutrophilic leykocytosis
in the global analysis of blood. General bilirubinum of blood of 570, to the line – 300,
activity of ALAT is enhanceable in 100 times, timol test of 26 units., urea – 2,1 mmol/l.
With most probability for a patient:
a. *Hepatitis A
b. Malignant icterus of pregnant
c. Typhoid
d. Mechanical icterus
e. Leptospirosis
281.
A patient is 25 years, appealed to the internist with complaints of a general
weakness, worsening of appetite, feeling of weight in right hypocostal areA. Treated
oneself on an occasion ARVI ambulatory 3 days. A feel was worsened, nausea, pain in
right hypocostal arrea, ochrodermia of sclerotica, darkening of urine, appeared.
Objectively: breathing normal, cardiac tones are rhythmic. A stomach is soft, sickly in
right hypocostal area, a liver is megascopic, a spleen is not megascopic. What is the most
credible diagnosis?
a. *Hepatitis A
b. Leptospirosis
c. Calculary cholecystitis
d. Flu
e. Pseudotuberculosis
282.
A patient 20 years old, had treated himself concerning an acute respiratory disease
for 5 days, but marked no displays of respirator syndrome. Last 2 days temperature has
been normal, appetite disappeared, however, appeared pain in epigastrium with nausea,
and urine turned dark. About what illness is it possible to think?
a. Hepatitis B
b. Infectious mononucleosis
c. Pseudotuberculosis
d. Leptospirosis
e. *Hepatitis A
283.
A patient 28 years old, an injection drug addict, complaints of dull pain in right
subcostal region, weakness, decline of appetite, pain in joints. The symptoms have been
present for 2 weeks. At examination: the icterus of skin and scleras observed. The liver
and spleen are enlarged. Urine is dark, excrements are discoloured. What is preliminary
diagnosis?
a. Hepatitis A
b. *Hepatitis B
c. Chronic cholecystitis
d. Toxic hepatitis
e. HIV-infection
284.
A patient 25 years old got sick suddenly after chills and temperature increase up to
38,0 °C. There was vomiting 2 times. Moderate pharyngeal pain appeared at swallowing.
with stomach discomfort. Signs of bursitis observed on knee and elbow joints. The light
icterus of sclera and skins of joints on 4th day, on the skin of lateral surfaces of trunk,
forehead and lower extremities. The small bright red papular rash appeared, more
concentrated in natural folds.Liver is enlarged, the tongue is raspberry like appearence.
Preliminary diagnosis would be:
a. Hepatitis A
b. Hepatitis B
c. *Pseudotuberculosis
d. Scarlet fever
e. Infectious mononucleosis
285.
A 19 years old patient was diagnosed with hepatitis B. After violation of diet and
nervous stress the state of patient got worse: intensity of ichterus was increased, pulse
110/min, BP 80/50 mm Hg. Also there was vomiting by «coffee-grounds». The size of
liver has decreased, the liver on palpation was painful. What complication would you
think about?
a. Infectious-toxic shock
b. Acute kidneys insufficiency
c. Acute adrenal insufficiency
d. Hemolitic icterus
e. *Acute hepatic insufficiency
286.
A patient C., 43 years old, has been treated for 5years . During the last
hospitalization a liver-biopsy was conducted. Histologicaly there is bulb dystrophy and
necrosis of hepatocytes, with leucocyts infiltration, and pericellular fibrosis. What
diagnosis is most credible in this case?
a. *Alcoholic liver cirrhosis
b. Viral hepatitis B
c. Viral hepatitis C
d. Cryptogenic hepatitis
e. Fatty dystrophy of liver
287.
Patient B., 23 years old. Objectively: skin is yellow, icterus of sclerA. Pulse
66/min, BP 120/80 . Indexes of AlAT, AsAT, and Tymol tests are normal. General
billirubin is 34 mmol/l due to indirect fraction. Specify the most credible diagnosis.
a. Chronic toxic hepatitis
b. Chronic cryptogenic hepatitis
c. Chronic hepatitis C
d. *Zhilber’s syndrome
e. Chronic B hepatitis
288.
A trained nurse got sick on 16.09: general weakness, nausea, vomiting, pain in
joints appeared. 21.09 urine turned dark, and skin yellow. 21.09 she was hospitalized.
Objectively: general status satisfactory, temperature 38,2 °C, expressed skin and mucous
membranes jaundice. Liver palpated 3 cm below the costal arc. 26.09 stomach-aches and
increased sleepiness with disorientation in time and place were appeared. 27.09
haematomas in the places of injections also appeared. Hepatic breath was noticed. Light
edema of feet and lumbar region were observed. The liver was not palpable. Development
of what syndrome can be forecast?
a. Meningoencephalitis
b. Hepatic encephalopathy
c. Acute nephrosonephritis
d. Pancreatitis
e. *ID syndrom
289.
A patient B., 52 years old, for 3 month complaints of nausea, periodic vomiting,
swelling of stomach, weakness, loss of weight up to 12 kg, consistency of stool is
chainging. During some days consciousness is entangled, somnolence, allolalia, general
weakness are expressed. Temperature of body 37,4 °C. Icterusis on the skin. Tremor of
brushes and nystagmus is marked. Pupils are narrowed, with a slow reaction on light,
increase of tendon reflexes. Ps 112/min, unrhythmical. BP 90/65 mm Hg. A liver is dense
+2. Laboratory tests: Hb 86 g/L, Leuc 4,2*109/L, ESR 18 mm/hour, glucose 3,4 mmol/l,
general bilirubin 56,7 mm/L, albumen 52 g/L, K+ 3,2 mmol/l, AsAT 0,62; AlAT 0,84;
prothrombin index-58 %; рН blood 7,3. What is previous diagnosis?
a. Hepatocirrhosis
b. Vilson-Konovalov disease
c. Cancer of liver
d. *Endogenous hepatic encephalopathy
e. Meygs disease
290.
A patient, 17 years, complaints of a weakness, worsening of appetite, nausea,
painfull in right subcostal area, dull ache in the body, dark color of urine, temperature to
37,8 °C. She stood in contact with sick on icterusis 1,5 months ago. Objectively: yellow
colour of the skin and visible mucous membranes. The liver is increased to 1,5 cm.
Unsignificant enlargment of spleen. What is your preliminary diagnosis?
a. Viral hepatitis B
b. Infectious mononucleosis
c. *Viral hepatitis A
d. Leptospirosis
e. Pseudotuberculosis
291.
Before the appearance of jaundice in a patient 16 years old, during 2 days there
were an increasing of the body temperature (38,5 °C), headache, dull ache disturbed
whole body. Name the variant of pre-icteric period of viral hepatitis for this patient?
a. *Influenza-like
b. Astenovegetative
c. Artralgic
d. Dyspeptic
e. Allergic
292.
A patient T., 28 years, appilled due to worsening of common status at the seventh
day to the infectious department concerning viral hepatitis. Nausea and vomiting were
present, hepatic smell appeared, increasing of the liver sizes. What biochemical index
does allow to suspect complication in the patient?
a. Increasing activity of AlAT
b. Increasing of bilirubin, increasing activity of AlAT
c. Increasing timol test
d. *Decreasing of protrombin, increasing of bilirubin
e. Decreasing of AlAT
293.
A sick P., 54 years old, complaints of dull pain in right subcostal area, bad taste in
the mouth, itching of the skin, increasing of abdomen. After the liver puncture fatty
dystrophy of hepatocytes, eccentric placing of veins in a hepatic lobule were found. What
laboratory index are most characteristic in this case?
a. Alkaline phosphotase
b. Hemodiastase
c. *Level of transaminases
d. Glucose of blood
e. Creatinphosphokinase
294.
A patient, 24, complaints of pains in right subcostal area, increasing after-meal,
nausea, increase temperature of body to 37,7 °C, icterus, pains in large joints. He is ill
from 8 months. Suffers a nonspecific ulcerative colitis. Hepatosplenomegaly. ESR 47
mm/hr, general bilirubin level is 86,1mmol/lt, direct-42,3 mmol/lt. In blood found out
antibodies against smooth muscles. General albumen of 62 gram/lt, albumin б. 40 %,
globulin. 60 %, gamma globulins 38 %. Not found out the markers of viral hepatitis. On
USD diameter of portal vein is 1 cm What will be your diagnosis?
a. Primary biliary cirrhosis
b. Zhilber syndrome
c. Viral hepatitis
d. Hemochromatosis
e. *Autoimmune hepatitis
295.
Woman of 22 years old, in the 7th month of pregnancy felt ill acutely in 3 weeks
after arrival from Turkmenistan. An icterus, untraceable vomiting, pain in right subcostal
region,, hemmorhaeges on the skin, appeared after three-day fever. Most probable
diagnosis will be:
a. Acute fatty hepatosis of pregnancy
b. Cholestatic hepatosis of pregnant
c. *Viral hepatitis of A
d. Viral hepatitis B
e. Acute sepsis
296.
Weakness appeared in a patient, decrease in an appetite, began pains in the joints
of extremities, felling of weight in right subcostal region, vomiting. Urine became dark in
12 days, and in a day – sclera and skin turned yellow. It was discovered in an
immunogram: antuIgG -HAV (+), HBsAg (+), antiIgМ -НBcorAg (+), anti-НCV (–), antiНDV (–). To the patient 3 months back was extracted a tooth. Most probable diagnosis?
a. *Viral hepatitis B
b. Viral hepatitis A
c. Viral hepatitis C
d. Viral hepatitis D
e. Viral hepatitis E
a.
b.
c.
d.
e.
297.
A patient 42 years, radiologist, entered surgical department with bleeding from the
mouth, was admitted to the gastroenterological departments . He considered himself
healthy. 20 years ago had haemotransfusion. Fibrogastroduodenoscopy shows venous
dilation of the oesophagus of the III degree. Most probable etiologic reason of disease:
Autoantibodies against hepatocytes
Viral hepatitis A
X-ray irradiation
*Viral of hepatitis B
Defficiency of antitrypsin
298.
A patient during half of the year got frequent parenterally injections concerning
bronchial asthmA. There was decrease in appetite gradually, started to feel weakness,
arthralgia, dark coloured urine, the icterus of skin appeared. Objectively: temperature of
37 °C, pulse 68/min BP 115/70 mm of Hg Liver +4 cm, spleen +1 cm, skin and sclera is
yellow coloured In the general blood analysis: amount of leucocytes: 3,6 109,among them
52 % lymphocytes, ESR 6 mm/hr, activity of ALAT is increased in 10 times. which
information will be more credible than all in blood of patient?
a. Anti-HBs antibodies
b. Anti-HAV IgM antibodies
c. *Anti-HBV antibodies
d. Anti-HCV IgG antibodies
e. Anti-HEV IgM antibodies
299.
Patient 34years old complaints , about fatigue, decreasing of appetite, nausea,
feeling of weight in a epigastric region, bitter taste in to the mouth. Objectively: Pulse is
76/min , temperature 37,2 -°C. Skin is pale, with single vascular “asterisks” present on
thorax. Liver is enlarged with 2 cm below the costal arch . In blood: bilirubinn :36
mmol/lt , ASAT :2,5 mmol/lt, ALAT :2,8 mmol. HBeAg, anti-HBc (–), HBsAg and antiHBe (+). What will be the diagnosis of the patient?
a. *Chronic hepatitis B, phase of integration, activity is poorly expressed
b. Chronic hepatitis B, phase of replication, activity is poorly expressed
c. Chronic hepatitis B phase of integration, high activity
d. Chronic hepatitis C, phase of replication, activity is poorly expressed
e. Chronic hepatitis C phase of replication, moderate activity
300.
A patient of 34 years complaints of fatigue, decreasing of appetite, nausea, feeling
of weight in a epigastric region, bitter taste in the mouth. Objective examination : Pulse
76/min, temperature 37,2 °C. Skin is pale, single vascular star-like rash on the thorax.
Liver is 2 cm below the costal arch. In blood: bilirubin level is 36 mmol/lt, ASAT 2,5,
ALAT 2,8 . Anti-HBc (–), HBsAg and HBeAg (+). What therapeutic tactic is expedient in
this case?
a. Therapy by hepato-protectors
b. *Antiviral therapy by lamivudine
c. Antiviral therapy by ribavirin
d. Therapy by corticosteriods
e. Therapy by immunostimulators
301.
Man 30 years, drug addict, takes drugs intravenously. Has been taking drugs for 12
years Complaints of weakness, moderate icterus, pain in right subcostal region. The state
was worsened gradually. Biochemical indexes: general bilirubin 28,2 mmol/lt; ALAT 1,0,
ASAT 0,8 . Will you define a diagnostic method which it is expedient to conduct for
establishment of etiologic diagnosis?
a. Biopsy of liver
b. *Polymerase chain reaction (PCR)
c. Enzymes of liver
d. Computer tomography
e. Immunological tests
302.
Patient B., 51 years, a lot of years used an alcohol. Complaints of nausea, liquid
stools, icterus of skin. Objectively: skin and sclera is yellow coloured, atrophy of muscles,
subfebrile temperature. Liver 3 cm below the costal arch, painful on palpation What
method of diagnostics most informing in this case?
a. Activity of cytolysis (AsAT, AlAT)
b. Proteinogram
c. *Biopsy of liver
d. Markers of viral hepatitis
e. Violation of cellular immunity (T4,T8)
303.
Patient of B., 64 years old, has been sick with chronic hepatitis. Complaints of pain
in the liver, growth retardation. Objectively: his skin is dry, erythemic, atrophy of
muscles, telangiectasis on shoulders, hyperemia of hands, abdomen is enlarged, look like
“jelly-fish head”. The liver below the costal arc on 3 cm, painful, dense, spleen on 1 cm
below the costal arch, positive symptom of fluctuation. Laboratory investigation:
hypergammaglobulinaemia, increasing of AsAT activity, AlAT is highly increased. What
drug of choice for the patient?
a. Lactulose
b. *Hepatoprotectors
c. Antibiotics
d. Glucocorticoids
e. Alpha interferon
304.
A patient 48 years old, complaints of attacks pains in right subcostal area after the
physical loading. Periodically marks more light excrement, darkening of urine.
Objectively: skin and mucous membranes high icteric. General bilirubin 36,8 mm/L,
direct fraction - 26,4. Ultrasound of gall-bladder: thickness of wall 4 mm, there is a lot of
bile in the ducts. It is necessary to prescribe with a lytolitic purpose:
a. Choleretics
b. *Ursofalk
c. Cholekinetics
d. Spasmolytics
e. Cytostatics
305.
During annual inspection increasing of a liver to 4 cm of 23 years old patient was
found, increasing of bilirubin level in 2 times, AlAT in 2,5 time. He has been often sick
with genital herpes. It was discovered RNA of hepatitis C virus. What is etiologic
treatment
a. *Interferons
b. Essencial phosphotides
c. Choleretics
d. Cholekinetics
e. Ursodesoksihole acids
306.
A patient, 19 years old, with hepatitis B. After violation of diet and nervous stress
the state of the patient became worse: increasing of intensity of jaundice, pulse 110 per 1
min, BP 80/50 mm Hg, vomiting with “coffee-grounds”, decreasing of the liver size. What
complication is it possible to think about?
a. *Acute liver insufficiency
b. Infectious-toxic shock
c. Acute kidney insufficiency
d. Acute extrarenal insufficiency
e. Hemolytic
307.
A woman of 22 years old, on the 7th month of pregnancy, fell ill suddenly after 3
weeks arrival from Turkmenistan. An icterus, incessant vomiting, pain in right subcostal
area, skin haemorrhages, were appeared after 7-daily fever. What is most credible
diagnosis:
a. *Hepatitis A
b. Hepatitis B
c. Acute fatty hepatosis of pregnant
d. Cholestatic hepatosis of pregnant
e. Cholecystitis
308.
A patient, 25 years old, fell ill suddenly: after a chill temperature rose up to 38,0
°C, 2 times vomiting, moderate pharyngalgias at swallowing, pain in the stomach, knee
and elbow joints. Non intensive jaundice of scleras and skin at 4th day of disease were
appiared, urine became dark; on the lateral surfaces of trunk, overhead and lower
extremities small spot bright red rash appeared. Liver is increased, tongue is assessed,
“raspberry”, increased lymphonodes. What is preliminary diagnosis?
a. Hepatitis A
b. *Pseudotuberculosis
c. Allergodermia
d. Scarlet fever
e. Infectious mononucleosis
309.
A patient 21 years old, complaints of increasing of temperature to 39,2 °C,
weakness, headache, pharyngalgia, pain in the muscles, joints and stomach, nausea,
vomiting once. On the 3rd day stomach-ache increased and began to be expressly
localized in the right iliac areA. The patient was hospitalised in a surgical department with
a diagnosis “appendicitis”. At examination: hyperemia of mucous of nasopharynges,
subicteroses, liver +2. At operation changed appendix and packages of mesenterial
lymphonodes has been found. What is preliminary diagnosis:
a. Typhoid fever
b. Adenoviral infection
c. Hepatitis A
d. *Pseudotuberculosis
e. Enteroviral infection
310.
Animal technician 57 years, on the 3th day of illness appealed to the doctor with
complaints of headacke, high temperature, pain in gastrocnemius muscles, dark urine and
diminishing of its amount. Objectively: temperature 38,1 °C, injection of the sclera
vessels, petechial rash on upper part of the thorax, hepatosplenomegaly. What is most
credible preliminary diagnosis?
a. Pseudotuberculosis
b. Brucellosis
c. Viral hepatitis
d. *Leptospirosis
e. Influenza
311.
A patient 28 years old, injection drug addict, complaints of dull pain in the right
subcostal area, weakness, decline of appetite, pain in joints, which disturbs during 2
weeks. At examination: icterus of the skin and sclera, enlargement of the liver and spleen,
dark urine, excrement is discoloured. What is preliminary diagnosis?
a. HIV-infecion
b. Hepatitis B
c. Hepatitis C
d. *Toxic hepatitis
e. Hepatitis A
312.
At inspection of a 8 weeks term pregnant woman HBsAg was found. Level of
bilirubin of blood and activity of ALAT were normal. What is necessary to do?
a. *To save pregnancy and conduct the inoculation to newborn against hepatitis
B
b. Termination of pregnancy
c. Termination of pregnancy and conduct treatment by interferon
d. To save pregnancy and conduct treatment by lamivudin
e. To save pregnancy and ultrasonic inspection of the fetus
313.
A patient I., 26 years old, delivered in a hospital on the 4th day of disease with
complaints of fever, headache, pain in gastrocnemius muscles. Works as a specialist in
land-reclamation. Has a lot of sexual contacts. Objectively: temperature – 39,7 °C. Severe
common condition. Expressed icterosis of skins and sclerA. Hemorrhages in conjunctiva
and sclerA. There is a hemorragic rash on a skin. A liver increased on 3 see below the
costal arc, edge of spleen, the Day's diuresis 300 ml. The etiologic factor of disease most
for certain is:
a. Rickettsia
b. Virus
c. *Leptospira
d. Spirocheta
e. Chlamidia
314.
Patient 43, miner, on the 7th day of disease complaints of acute weakness, high
temperature, pain in the muscles of feet and back, icterus, dark color of urine, headache.
Fell ill sharply from a chill, temperature 40,2 °, there was a nose-bleed. A diuresis 200 ml.
What is a credible diagnosis?
a. Sepsis
b. Typhoid
c. Viral hepatitis
d. *Leptospirosis
e. Malaria
315.
36 years old man, during 8 days marks increasing of temperature with a strong
chill, which end with abundant sweating and repeat oneself regularly in a day. There were
alike attacks of fever in Afghanistan, from where returned 4 months ago. Temperature
39,2 °, pulse 94 after 1 mins A stomach during palpation is not sickly. There is a moderate
increase of spleen and liver, insignificant icterus. What diagnosis is most credible?
a. Viral hepatitis
b. Typhoid
c. Sepsis
d. *Malaria
e. Tuberculosis
316.
Patient 40 years old, in a week after returning from Ethiopia on a motherland
disturb the periodic attacks of fever. The icterus of sclera and skin covers,
hepatospleenomegaly, is marked. What from the transferred diagnoses most credible?
a. Sepsis
b. *Malaria
c. Hepatitis A
d. Hemolitic anaemia
e. Leptospirosis
317.
Patient 45 years, hospitalized on the 7th day of fever. Objectively: temperature
39,8 °C, somnolence (at night insomnia), dormancy, аdynemia, pallor of skin covers,
pulse 78 per 1 min, BP 105/70 mm of Hg. The tangue is thickly assessed with a grey
cover with the imprints of a teeth. A stomach is swollen, liver and spleen megascopic, in a
right iliac area rumbling and hyperemia of skin. Stool is absent 2 days. What investigation
is it neccessary to perform for clarification of diagnosis?
a. *Bacteriologic examination of blood
b. Spinal puncture
c. Myelogram
d. Colonoscopy
e. Analysis on the markers of viral hepatitis
318.
Patient 52 years old, suffering sharply from a chill and headache, fever to 40,5 °C.
On a 3rd day of disease nausea, vomiting, dark urine appeA. On a 4th day temperature
falls down to 37,2 °C, but an icterus appeared and the amount of urine decreased to 600
ml. What disease such symptoms develop from?
a. Sepsis
b. Hemorrhagic fever with a kidney syndrome
c. Hepatitis A
d. *Leptospirosis
e. Acute glomerulonephritis
319.
A patient, 35 years, suffering sharply, complaints of a headache, pain in the
muscles of lower extremities, increasing of temperature to 39,3 °C. Objectively on the 4th
day of disease: the state is severy, hyperemic face, icterosis of skin and sclerA.
Enlargment of liver and spleen. A diuresis is mionectic. What is most credible diagnosis?
a. Trichinosis
b. Hepatitis A
c. Yersiniosis
d. Infectious mononucleus
e. *Leptospirosis
320.
Student 20 years, treated oneself on an occasion of ARVI (increasing of
temperature to 38,2 °C during 3 days). He complaints of worsening of appetite, increasing
fatigue at a normal temperature and absence of the catarrhal phenomena of upper
respiratory tracts. A doctor found out an increasing and moderate sickliness of liver. There
were cases of hepatitis A in a student’s group. What method of investigation will allow?
a. Ultrasound scanning of the liver
b. Determination of bilirubin level of the blood
c. Determining the amount of beta-lipoproteins
d. *Determination of activity of аminotransferases of the blood
e. Immunofluorescent research of the nasal smears
321.
For a patient, workwoman of a pig farm, on a background of complete health a
chill appeared suddenly, a temperature rose to 39,9 °C, there was a headache, nauseA. On
the next day marked pains in the muscles of lower extremities, nose-bleeding began. On
the 3rd day of illness, state became more severy. Face is hyperemic, scleritis, hyperemic
scleritis. Liver +3 cm. Daily diuresis 700 ml. What is previous diagnosis?
a. Yersiniosis
b. Hepatitis A
c. Hemorrhagic fever with a kidney syndrome
d. Flu
e. *Leptospirosis
322.
A patient Z., 33 years old, miner, entered clinic of infectious diseases on the 7th
day of disease with complaints of a acute weakness, high temperature, pain in the muscles
of feet and back, icterus, dark color of urine, headache. Became sick sharply from a chill,
temperature rises up to 40,1 °C. On a 4th day there is an icterus, nose-bleeding,
hemorrhages in sclerA. Duration of fever 6 days. Diuresis is 200 ml. What is credible
diagnosis?
a. Typhoid fever
b. *Leptospirosis
c. Hepatitis A
d. Sepsis
e. Influenza
323.
A patient is disturbed by attacks of fever which repeated every third day. The
icterus of sclera and skin, hepatospleenomegaly were marked. What is most credible
diagnosis?
a. Viral hepatitis
b. Sepsis
c. *Malaria
d. Influenza
e. Leptospirosis
324.
A patient Y., 25 years old, entered infectious department on the 3rd day of disease
with complaints of headache, pain in the back, gastrocnemius muscles, high fever, chill.
State is moderate. Icteruses of the scleras. Mucous membrane of soft palate is hyperemic.
Tongue is dry, assessed with brown cover. The abdomen is swollen. Liver +2 cm, spleen
is not megascopic. Painfull muscles, especially gastrocnemius. Urine is dark, excrements
ordinary color. What is the most credible diagnosis?
a. Infectious moneuclious
b. Hepatitis A
c. Malaria
d. *Leptospirosis
e. Yersiniosis
325.
On the 3rd day of illness the sick is delivered in a severy condition with complaints
of a suddenly arising up high temperature, headache, repeated nose-bleed, pains in
gastrocnemius muscles. Objectively: moderate icterus of sclera and skins,
hepatospleenomegaly, оliguriA. What is most credible diagnosis?
a. *Leptospirosis
b. Viral hepatitis
c. Influenza
d. Infectious mononucleosis
e. Malaria
326.
A patient S., 45 years old, suffering suddenly from appearance of chill and
increasing of temperature to 39,2 °C. In the evening pain appeared in a stomach and
gastrocnemius muscles. In 2 days noticed the ochrodermia of skin and sclerA.
Objectively: the state is severy, temperature 39,9 °C. The tongue is covered. Moderate
jaundice of the skin and sclerA. There is plural petachiae on a trunk. Superficial breathing
20 times per 1 min, pulse 102 per 1 min, AP 100/60 mm of Hg. A stomach is soft, sickly
in epigastrium, a liver on 3 cm comes from a costal arc. Daily diuresis 300 mm, urine is
sad-coloured. What is preliminary diagnosis?
a. Sepsis
b. *Leptospirosis
c. Influenza
d. Hepatitis B
e. Infectious mononucleus
327.
A patient I., 25 years old, appealed to the internist with complaints of a general
weakness, loss of appetite, pain in the right hypochodrium. Treated himself on an
occasion of ARVI ambulatory 3 days. Became acute worsening of fealling: nausea, pain in
right hypochondrium, ochrodermia of sclera, darkening of urine, appeared. Objectively:
breathing superficial, cardiac tone is rhythmic. Abdomen is soft, painfull in right
hypochondrium, a liver is megascopic, a spleen not palpable. What is most credible
diagnosis?
a. *Hepatitis A
b. Leptospirosis
c. Cholecystitis
d. Influenza
e. Pseudotuberculosis
328.
A patient 42 years old, complaints of dull pain in a right hypochondrium,
weakness, decline of appetite, itching of skin, icterus. Disease began 1 month ago from
protracted аrthralgia, disgust for a meal, strong weakness. An icterus which now increased
considerably appeared three weeks ago. Temperature 36,0 °C. Pulse 56 per 1 min. A
tongue is assessed with white cover. A stomach is soft, sickly in right hypochondrium. A
liver +3 cm, spleen is not megascopic. Common analysis of blood: leukopenia, relative
lymphomonocytosis, ESR 3 mm/hour. What diagnosis is most credible?
a. Cancer of head of a pancreas
b. *Hepatitis B
c. Hepatitis A
d. Cirrhosis of liver
e. Chronic cholecystitis
329.
For a patient 35 years after 4-months of treatment by isoniaside - аdynemia,
icterus, pain in right hypochondric were appeared. A liver is megascopic. In blood activity
of enzymes of AlAT is enhanceable in three times, AsAT in two times. Bilirubin of blood
of 122 gm/ml (conjugated – 82, unconjugated – 40). НBs-аntigen is not found out. What
is the diagnos?
a. Calculary cholecystitis
b. Hepatocirrhosis
c. Acute viral hepatitis
d. Chronic active hepatitis
e. *Toxic hepatitis
330.
A patient is 35 years, grumbles about aching pain in right hypocostal area, nausea,
decline of appetite. Beginning of disease binds to appendicitis. After it in 2 months an
icterus appeared first. Treated oneself in an infectious department. In 1 began to notice
aching pain in right hypocostal area, in analyses is an increase of level of bilirubin. Your
diagnosis?
a. Calculary cholecystitis
b. Illness of Zhil'bera
c. Sharp viral hepatitis
d. Chronic cholangitis
e. *Chronic hepatitis
331.
The patient T., 35 years, operating trained nurse, appealed to the doctor on the 8th
day of gradual development of illness with complaints of a general weakness, rapid
fatigueability, dark color of urine. In the morning noticed the icterus. On examination
temperature of body 36,8 °C. Found out the increase of liver -+3 sm The changes of what
laboratory index most informing at this illness?
a. *AlAt
b. Hemodiastases
c. Protrombin index
d. Cholesterol
e. Alkaline phosphatase
332.
Diagnosed a patient: chronic hepatitis in the stage of integration. What markers
will be in patient in this stage disease?
a. HBeAg
b. Antibodies to HBeAg
c. DNA OF HBV
d. Viral DNA-polimerase
e. *HBsAg, anti-НBе
333.
23-years old patient during 6 months gets diabetes mellitus in a policlinic the
injections of insulin. A weakness, arthalgia, grew gradually, an appetite disappeared, then
dull pain appeared in an epigastrium after-meal. In 2 weeks from the beginning of illness
noticed the dark color of urine, and afterwards – icterus on a background which the
general state continues to be worsened. Temperature of body 36,5°C, Ps 58 per 1 min
Liver +5 sm, spleen +1 sm What from hemanalysis is it needed to appoint for
confirmation of diagnosis?
a. Activity of alkaline phosphatase
b. Activity of lactatdehydrogenase
c. *Activity of ALAT
d. Activity of creatinphosphokinase
e. Activity of amylase
334.
42-years old patient got blood transfusion two months ago. A weakness was
gradually increased, an appetite was worsened, arthalgia, small nausea appeared. On a
12th day noticed an insignificant icterus, appealed to the doctor. At examination is the
general state fully satisfactory. The temperature of body is normal, Ps 60 per 1 min, BP
100/70 mm Hg.A liver and spleen is moderate megascopic, a stomach at palpatory is not
sickly. Anti-HCV IGM is found in blood. What from changes in the global analysis of
blood most probably?
a. Neutrocytosis
b. *Leykopeniya
c. Lymphopenia
d. Speed-up ESR
e. Aneozinofiliya
335.
A 17-years-old patient during 4 days suddenly had head pains, myalgias, fever of
permanent type. From a 5th day is a normal temperature of body, the general state was
considerably improved, an appetite appeared, nausea, general weakness, disappeared, but
an icterus became noticeable. The general state is satisfactory, liver +3 sm, spleen +1 sm,
Ps 56 per 1 mins Peripheral lymphatic knots, amygdales are not megascopic, hyperemia of
pharynx is not present. At research of global analysis of blood is leykopenia with a
relative lymphocytosis, 3 % virocytis, ESR normal. Activity of ALAT of blood is
enhanceable in 5 times. What from diagnoses most credible for a patient?
a. *Hepatitis A
b. Infectious mnonucleosis
c. Citomegaloviral infection
d. Hepatitis B
e. Hepatitis C
336.
On the 15th day of illness for a 17-years-old patient by hepatitis In anti-HBs is
found in blood. The state of patient was considerably worsened the day before. There was
excitation, there was a nose-bleed, a hemorragic rash appeared on a skin, diminished and
became sickly at palpatory liver. Ps 106 per 1 min, BP of 110/ 70 mm Hg, the temperature
of body subfebrile. What changes of indexes blood will a patient have most credible?
a. Growth of alkalinephosphatase
b. Growth of Fibrinogenum
c. Growth of indirect bilirubinum
d. *Decline of protrombin index
e. Growth of albumin
337.
25-years old woman during a year got numerous injections concerning bronchial
asthmA. An appetite disappeared gradually, a weakness, arthalgia grew, urine became
dark in two weeks, through three – an icterus on a background which the general state
continues to be worsened appeared. The temperature of body is normal, Ps 62 after 1 mins
Moderate sickliness in the area of pancreas, positive Voskresensky symptom, liver +4 cm,
spleen +2 cm What from the markers of viral hepatitis more credible will be positive for a
patient?
a. *Anti-HBc IGM
b. ANTI-HEV IGM
c. ANTI-CMV IGM
d. HBsAg
e. ANTI-HAV IGM
338.
26-years old man during 6 days marks the 6-8-sentinel gettings up temperatures
with a strong chill, which end with abundant then and repeat oneself regularly in a day. 6
months ago there were alike attacks of fever during a stay in Afghanistan, from where
returned 4 months back. Temperature of body 36,4°C, Ps 94 per 1 min. The moderate
increase of spleen and liver is marked. A diuresis suffices, wetting without pathological
changes. What disease can be suspected for a patient?
a. Leptospirosis
b. Hemorragic fever
c. *Malaria
d. Viral hepatitis
e. Sepsis
339.
40-years old patient during 2th days marks absence of appetite, nausea, general
weakness. A year ago was treated due to acute hepatitis B, avoided a clinical supervision.
Objectively: the temperature of body is normal, icterus of skin and sclera, increasing of
liver 3 cm. A spleen is not megascopic. Urine moderatory dark colouring. ALAT of blood
4,0. What is most credible diagnosis?
a. Chronic viral hepatitis, minimum activity
b. Chronic cholecystitis
c. *Chronic viral hepatitis, moderate activity
d. Hepatocirrhosis
e. Chronic viral hepatitis, high activity
340.
A man, 37 years, injection drug addict, practises upon an alcohol, grumbles about a
general weakness, dull pain in right hypocostal area, increase of stomach, shortness of
breath. It is ill chronic hepatitis on an extent 10 years. Objectively: temperature of body
37,1°C, an icterus is expressed. Liver +6 sm. In an abdominal region is a free liquid.
Positive symptom of fluctuation. What complication did arise up for a patient?
a. Intensifying of chronic hepatitis C
b. Acutehepatic insufficiency
c. Hepatocarcinoma
d. Intensifying of chronic hepatitis B
e. *Hepatocirrhosis
341.
A patient is 59 years, suffers chronic viral hepatitis with development of cirrhosis,
hospitalized with a diagnosis: “bleeding from the veins of gullet”. Sharp pallor of skin
covers. BP 80/40 mm Hg, pulse 100 per 1 min, the temperature of body is normal. On a
front abdominal wall a venous net is extended. What preparation is it necessary to begin
therapy from?
a. Albumen
b. Plasma
c. Neogemodez
d. *Blood
e. Glucose
342.
24 years old patien is bothered by pharyngalgia, general weakness. Objectively:
temperature 38,0 °C. pulse 96 per min, hyperemia of otopharynx, enlargment of tonsills.
Neck and submandibular lymphatic nodes are palpated, spleenomegaly present.
Polymorhyc spots on the skin of trunk. What disease is most probable?
a. Scarlet fever
b. Acute respiratory disease
c. Follicle tonsillitis
d. Typhoid fever
e. *Infectious mononucleosis
343.
17 years old patient has a temperature of 38,2 °C, generalized lymphadenopathy
(neck lymphatic nodes which are located along m. sternocleidomastoideus, mild icterus,
hepatospleenomegaly. What is the preliminary diagnosis?
a. Tuberculosis of lymphatic nodes
b. Bacterial tonsillitis
c. Diphtheria
d. *Infectious mononucleosis
e. Megacaryoblastoma
344.
Sick, 24 years old, is bothered by pharyngalgia, general weakness. Objectively:
temperature – 38 °C. Pulse – 96 per min., increasing and huperemia of tonsills. Also
increasing of neck and submandibular lymphatic nodes, spleenomegaly. What disease is
most probable?
a. Measles
b. Follicular tonsillitis
c. Typhoid fever
d. *Infectious mononucleosis
e. Acute respiratory infection
345.
Sick girl 8 years, entered permanent establishment with complaints of a general
weakness, increasing of temperature, pharyngalgiA. Objectively: the mucous of
otopharynx is bright red, on tonsills are white raids, is taken off easily. Increasing of all
groups of lymphonodes, 1-3 cm in a diameter, dense, elastic, littlesickly, not soldered
between itself. A liver is megascopic, spleen – on +1 cm. Leycocytosis, plasmatic cells –
20 %. What is the possible diagnosis?
a. Acute lympholeycosis
b. *Infectious mononucleosis
c. Tonsillitis
d. Diphtheria
e. Adenoviral infection
346.
At a child with the catarrhal phenomena increasing of submandibular and back
neck lymphatic nodes, hyperplasia of tonsills, presence of the single roseol-papular rash,
increasing of liver it was found. What is the possible diagnosis.
a. *Infectious mononucleosis
b. Adenoviral infection
c. Scarlet fever
d. Measles
e. Flu
347.
In a policlinic 18 years old patient complaints of a moderate pharyngalgia,
headacke, general weakness, high temperature of 38,9 °C. He was sick during three days.
Objectively it was found: increasing of back neck, and inguinal lymphatic nodes,
hepatospleenomegaly, subicterus of scleras and skin, changes in a throat, characteristic for
a quinsy. Lymphomonocytosis in a general blood analysis. What is the possible diagnosis?
a. *Infectious mononucleosis
b. Follicular tonsillitis
c. Adenoviral infection
d. Flu
e. Viral hepatitis
348.
In a policlinic the patient C., 18 years old, appealed with complaints of a moderate
pharyngalgia, head pain, general weakness fervescence, to 38,9 °C. It is ill already three
days. Objectively: it is found out megascopic back neck, and inguinal lymphatic knots,
hepatospleenomegaly, subicterus scleroticas and skin, changes in a throat, characteristic
for a quinsy. At the hemanalysis – lymphomonocytosis. What are the principles of
treatment?
a. Antibiotics, hepar protective preparations, antihistaminic
b. Antiherpetic preparations, hepar protective preparations, antihistaminic
c. *Antibiotics, preparations of interferon, hepar protective preparations
d. Antibiotics, Antiherpetic preparations , antihistaminic
e. Antibiotics, preparations of interferon, vitamins
349.
At a patient, 17 years: quinsy, temperature 38,2 °C, generilized lymphadenopathy
(the first multiplied neck lymphatic nodeswhich are located along m.
sternocleidomastoideus), small icterus hepatospleenomegaly. What exciter causes this
disease?
a. Herpesvirus I type
b. Herpesvirus ІІ type
c. Herpesvirus ІІІ type
d. *Herpesvirus ІV type
e. Herpesvirus V type
350.
Sick, in 24, disturbs a pharyngalgia, general weakness. Objectively: temperature –
38 °C. Pulse 96 shots per min, dermahemia mucus of otopharynx bloodshot, tonsills are
megascopic in sizes, loose. Palpated megascopic back neck and submandibular lymphatic
knots., a spleen comes forward from under the edge of costal arc of to 1 sm. What
methods of diagnostics is it possible to confirm the diagnosis?
a. Reaction of Paul-Bynnel, Right-Heddlson, Goffa-Bauer
b. *Reaction of Paul-Bynnel, Lovrik-Volner, Goffa-Bauer
c. Reaction of Paul-Bynnel, Right-Heddlson, Lovrik-Volner
d. Reaction of Paul-Bynnel, Goffa-Bauer
e. Reaction of Paul-Bynnel, Goffa-Bauer, Right-Heddlson, Lovrik-Volner
351.
Patient P, 17 years old, has got ill gradually. General weakness, fatigue, painful
throat, and nausea were marked. He was hospitalized on the 5th day of illness.
Objectively: temperature is 38 °C. Objectively: skin and sclera are yellowish, neck,
anticubital and subclavian lymph nodes are enlarged. Not numerous mculo-papular
elements of rash on a trunk are found. The tongue is covered with white coat. The tonsils
covered with white yellowish patches.On palpation of stomach enlarged spleen and liver
are found. In blood there is leucocytosis with neutophylic shift, atypical mononuclears-10
%, plasmatic cells- 10 %. What would be the diagnosis?
a. Scarlet fever
b. *Infectious mononucleosis
c. Typhoid fever
d. Iersiniosis
e. Lacunar tonsilitis
352.
The boy 12 years old, with catarrhal phenomena seen an increase of all the lymph
nodes, sclera, hyperplasia of tonsils white layers on them as islands, existing single
maculo-papular rash, increased liver and spleen size. What additional research should be
done?
a. Reaction of passive heamaglutination(RPG) with influenza viruses
b. Ultrasound of the abdomen
c. Puncture of the lymph nodes, with following microscopy
d. Inoculation of blood
e. *Blood test for atypical mononuclears
353.
Sick C., 17 years old fell ill gradually. Disturbed by general weakness, fatigue, sore
throat, abdominal pain, nauseA. Hospitalized at 5th day of illness. Examination: body
temperature 38,0 °C, increasing of the neck, shaped inframaxillary lymph nodes. Skin and
eye sclera were subicteric. Macula-papular rash on the trunk. Raids on tonsils thick, white,
yellow, white furred tongue stomach is moderately swollen, hepatosplenomegaly. In blood
analyses: leukocytosis, microphage with the left shift, atypical mononuclear – 10 %,
plasma cells – 10 %. What is the preliminary diagnosis?
a. *Infectious mononucleosis
b. Yersyniosis
c. Lacunar tonsillitis
d. Viral hepatitis A
e. Typho-paratyphoid disease
354.
Patient 18 years, entered permanent establishment with complaints of head pain,
general weakness increase of temperatures, to 37,5–38,2 °C during 6 days, pharyngalgiA.
Objectively: all groups of lymphonoduses, 1-3 cm in a diameter, elastic, are megascopic,
little sickly, not soldered between itself. A liver is megascopic on 3 sm, spleen – on 1 sm.
In a blood is leycocytosis, plasmatic mews – 15 %. What group of herpes does the exciter
of this illness belong to?
a. Alpha
b. Beta
c. *Gamma
d. Delta
e. Teta
355.
Patient A., 18 years old, is complaining of headache, weakness, high temperature,
sore throat. Objectively: general lymphadenopaty,with lymph nodes 1-3 cm in diameter,
dense, elastic not painful and not combined between themselves. Hepatospleenomegaly
was noticed. In blood there is leucocytosis, monocytes – 15 %. What is the diagnosis?
a. *Infectious mononucleosis
b. Adenoviral infection
c. Tonsillitis
d. Diphtheria
e. Acute leukemia
356.
An 18 years old patient, entered the hospital with complaints of headache, general
weakness, raising the temperature to 37,5-38,0 °C for 6 days, a sore throat. Objective
examination: increasing of all lymph nodes, 1-3 cm in diameter, flexible, megaloblastic
not soliter together. Increasing of the liver size till 3 cm, spleen – 1 cm. Common blood
analysis: leukocytosis, plasma cells – 15 %. What is the preliminary diagnosis?
a. Diphtheria
b. Adenovirul infection
c. Lacunar tonsillitis
d. *Infectious mononucleosis
e. Acute lympholucosis
357.
Patient D., 20 years old, fell ill and went to infectious polyclinic cabinet with
complaints of moderate pain in the throat, headache, general weakness, increased body
temperature of 38.9 °C. Fells bed during three days. At examination: increasing of
inguinal lymph nodes, hepatosplenomegaly, subicteric sclera and skin, changes in the
throat are typical for sore throats. In the blood – lymphogranulomatosis. The principles of
treatment are?
a. *Antibiotics, interferons, hepatoprotectors
b. Antihypertensives hepatoprotectors, antihistamines
c. Antibiotics, hepatoprotectors, antihistamines
d. Antibiotics, antihypertensives, antihistamine
e. Antibiotics, interferones, vitamins
358.
A patient D., 20 years old came to the infectious polyclinic with complaints of
moderate pain in the throat, headache, general weakness, increasing of body temperature
to 38.9 °C. He fells sick during 3 days. Objective examination: increasing inguinal lymph
nodes, hepatosplenomegaly, subicteric sclera and skin, changes in the throat are typical for
sore throats were found. Blood analysis – lymphogranulomatosis. What is the probable
diagnosis?
a. *Infectious mononucleosis
b. Tonsillitis
c. Adenovirus infection
d. Influenza
e. Viral hepatitis
359.
The patients, 20 years old, with 5 days fever, pain in the throat during swallowing,
pain in muscles. During review – there is thick, congested tonsils with purulent layers that
are easily removed, enlarged neck, and inframaxillary lymph glands, liver + 1.5 cm
possible palpation of edge of spleen. What additional research should be done?
a. Puncture of the lymph nodes, followed by microscopy
b. Ultrasound of the abdominal cavity
c. *Blood test for atypical mononuclear
d. Inoculation of blood
e. Reaction of passive heamoglutination(RPH) influenza A viruses
360.
In children with catarrhal phenomena revealed an increase of inframaxillary lymph
nodes, tonsil hyperplasia with whitish soft touch on them in the form of points and islands,
the presence of rare maculo-papular bulging out, increased liver and spleen. Identify the
likely diagnosis?
a. *Infectious mononucleosis
b. Acute leukemia
c. Measles
d. Scarlet fever
e. Chicken pox
361.
Patient A., 35 years old, came to the clinic on the third day of disease with
complaints of drowsiness, sweat, headache, fever up to 38.5 °C. Reviewing physician
found sore throat, splenomegaly, enlarged lymph nodes. Liver not enlarged. Blood
analysis – leukocytosis, lymphomonocytosis. What is the treatment for the patient?
a. *Antibiotic, drugs interferon, hepatoprotectors
b. Antihistamin, antiherpetic preparations hepatoprotectors
c. Antibiotic, hepatoprotector, antihistamine
d. Antibiotics, antihistamine, antiherpatic drugs
e. Vitamins, antibiotics, drugs interferon
362.
35 years old patient A., came to the clinic on the third day of illness with
complaints of drowsiness, sweat, headache, fever up to 38.5 °C. Reviewing physician
found sore throat, splenomegaly, enlarged lymph nodes. Liver is not enlarged. Blood
analysis – leukocytosis, lymphocytosis. What additional test should hold the patient for
infectious mononucleosis?
a. Byurne and Wright-Hadulson’s reaction
b. ELISA-test, bacteriological test for tularemia
c. Bacteriological test for diphtheria and typhoid fever
d. *ELISA-test, bacteriological test for diphtheria
e. Paul-Bunnel’s reaction and lymph node puncture
363.
Patient L., 38 years old, complaints of recurrent attacks of fever with severe chills
and heavy then. Epemiologically found that he had recently returned from IndiA. When
parazitoscopii blood found erythrocytic schizonts. Vector of the disease is:
a. Fly
b. *Mosquitoes
c. Flea
d. Bee
e. Pliers
364.
Patient G., 28 years old, sailor, joined the 18-day illness, complaining of high
fever, headache, weakness. Disease began as a chills during an hour, then the temperature
had risen to 39,8 °C. In next 5-6 days, chills repeated every day in the middle of the day,
after which the temperature rose to 40-41 °C, such attacks fever occur every other day.
Face is hyperemic, herpes, vascular injection of sclerA. Tons of the heart muted,
rhythmic, pulse 140 for 1 min, AD 140 and 60 mm RT. Art. Liver is normal, spleen +2
sm. Which method of diagnosing is the most valuable in this situation?
a. *Smear and thick drop of blood for malaria
b. An overall analysis of blood
c. Bilirubin in the blood, ALaT
d. culture of blood
e. Immunological Methods
365.
Patient G., 28 years old, sailor, has been ill in 18-day illness, complaining of high
fever, headache, weakness. Disease began as a chills during an hour, then the temperature
had risen to 39,8 °C. In next 5-6 days, chills repeated every day in the middle of the day,
after which the temperature rose to 40-41 °C, such attacks fever occur every other day.
Face is hyperemic, herpes, vascular injection of sclerA. Tons of the heart muted,
rhythmic, pulse 140 for 1 min, AD 140 and 60 mm RT. Art. Liver is normal, spleen +2
sm. What you need to start treatment?
a. *Primaquine
b. Fansidar
c. Metakelfin
d. Quinine
e. Tetracycline
366.
A citizen of Nigeria, 19 years old, came to study in Ukraine. After 10 days of
appeared a strong chill, which lasted about 1 hour and changed the feeling of heat. There
has been a headache, slice, nagging pain in the muscles. Attack of such clinical symptoms
rapid again after 2 days. OBJECTIVE: body temperature 39,6 °C, sclera subicterich,
appeared lips herpes. Spleen significantly increased, dense, the liver is a normal. What
investigation needs to be done?
a. *Microscopy of peripheral blood
b. Microscopy of centrifuge of urine
c. Lumbar puncture
d. Observation of oculi
e. USD abdomen
367.
The patient concerned about attacks of fever, which are repeated every third day.
There are jaundice of sclera and skin, hepatosplenomegali. Which of the diagnoses most
likely?
a. Sepsis
b. *Malaria
c. viral hepatitis
d. Hemolytic anemia
e. Leptospirosis
368.
Patient G., 20 years old, six months ago returned from AfricA. A month later he
introduced the three-day clinical symptoms of malariA. Is it necessary to carry out
epidemic contacts that the patient had?
a. You do not need because there is no vector
b. You need because there is vector
c. *You do not need because reconvalence not cause
d. It should be, because rekonvalence allocates cause
e. You do not need because epidemic activities inefficient
369.
A patient G., 42 years old, for a week every 48 hours had attacks of chills, which
followed by heat. Body temperature rises to 40 °C, in 3-4 hours was a severe sweat.
Worsened appetite, a common weakness. Skin pale. Hepatosplenomegaly. What are the
most effective method for verification of the diagnosis.
a. *Microscopy and a thick smear of blood drops
b. Microscopy hanging drops
c. An overall analysis of blood
d. Bacteriological method
e. Enzyme multiplied immunoassay
370.
Citizen B. travelling in the summer as a tourist to resort, disadvantaged malariA.
How to prevent malaria infection?
a. Follow the rules of personal hygiene
b. Do not drink raw water
c. Make vaccine
d. *Hold chemoprophylaxis
e. Conduct heat treatment of food
371.
Patient A., 25, treated at a hospital about malariA. The latest attack was the day
before. Assign etiotropic treatment.
a. Antibiotics
b. Serum transfer
c. Delagil
d. Primaquine
e. *Delagil + Primaquine
372.
A patient 20 years old, had treated himself concerning an acute respiratory disease
for 5 days, but marked no displays of respirator syndrome. Last 2 days temperature has
been normal, appetite disappeared, however, appeared pain in epigastrium with nausea,
and urine turned dark. About what illness is it possible to think?
a. Hepatitis B
b. Infectious mononucleosis
c. Pseudotuberculosis
d. Leptospirosis
e. *Hepatitis A
373.
A patient 28 years old, an injection drug addict, complaints of dull pain in right
subcostal region, weakness, decline of appetite, pain in joints. The symptoms have been
present for 2 weeks. At examination: the icterus of skin and scleras observed. The liver
and spleen are enlarged. Urine is dark, excrements are discoloured. What is preliminary
diagnosis?
a. Hepatitis A
b. *Hepatitis B
c. Chronic cholecystitis
d. Toxic hepatitis
e. HIV-infection
374.
A patient 25 years old got sick suddenly after chills and temperature increase up to
38,0 °C. There was vomiting 2 times. Moderate pharyngeal pain appeared at swallowing.
with stomach discomfort. Signs of bursitis observed on knee and elbow joints. The light
icterus of sclera and skins of joints on 4th day, on the skin of lateral surfaces of trunk,
forehead and lower extremities. The small bright red papular rash appeared, more
concentrated in natural folds.Liver is enlarged, the tongue is raspberry like appearence.
Preliminary diagnosis would be:
a. Hepatitis A
b. . Hepatitis B
c. *Pseudotuberculosis
d. Scarlet fever
e. Infectious mononucleosis
375.
A 19 years old patient was diagnosed with hepatitis B. After violation of diet and
nervous stress the state of patient got worse: intensity of ichterus was increased, pulse
110/min, BP 80/50 mm Hg. Also there was vomiting by «coffee-grounds». The size of
liver has decreased, the liver on palpation was painful. What complication would you
think about?
a. Infectious-toxic shock
b. Acute kidneys insufficiency
c. Acute adrenal insufficiency
d. Hemolitic icterus
e. *Acute hepatic insufficiency
376.
A patient C., 43 years old, has been treated for 5years . During the last
hospitalization a liver-biopsy was conducted. Histologicaly there is bulb dystrophy and
necrosis of hepatocytes, with leucocyts infiltration, and pericellular fibrosis. What
diagnosis is most credible in this case?
a. *Alcoholic liver cirrhosis
b. Viral hepatitis B
c. Viral hepatitis C
d. Cryptogenic hepatitis
e. Fatty dystrophy of liver
377.
Patient B., 23 years old. Objectively: skin is yellow, icterus of sclerA. Pulse
66/min, BP 120/80 . Indexes of AlAT, AsAT, and Tymol tests are normal. General
billirubin is 34 mmol/l due to indirect fraction. Specify the most credible diagnosis.
a. Chronic toxic hepatitis
b. Chronic cryptogenic hepatitis
c. Chronic hepatitis C
d. *Gilbert’s syndrome
e. Chronic B hepatitis
378.
A trained nurse got sick on 16.09: general weakness, nausea, vomiting, pain in
joints appeared. 21.09 urine turned dark, and skin yellow. 21.09 she was hospitalized.
Objectively: general status satisfactory, temperature 38,2 °C, expressed skin and mucous
membranes jaundice. Liver palpated 3 cm below the costal arc. 26.09 stomach-aches and
increased sleepiness with disorientation in time and place were appeared. 27.09
haematomas in the places of injections also appeared. Hepatic breath was noticed. Light
edema of feet and lumbar region were observed. The liver was not palpable. Development
of what syndrome can be forecast?
a. Meningoencephalitis
b. *Hepatic encephalopathy
c. Acute nephrosonephritis
d. Pancreatitis
e. IDS syndrom
379.
A patient B., 52 years old, for 3 month complaints of nausea, periodic vomiting,
swelling of stomach, weakness, loss of weight up to 12 kg, consistency of stool is
chainging. During some days consciousness is entangled, somnolence, allolalia, general
weakness are expressed. Temperature of body 37,4 °C. Icterusis on the skin. Tremor of
brushes and nystagmus is marked. Pupils are narrowed, with a slow reaction on light,
increase of tendon reflexes. Ps 112/min, unrhythmical. BP 90/65 mm Hg. A liver is dense
+2. Laboratory tests: Hb 86 g/L, Leuc 4,2*109/L, ESR 18 mm/hour, glucose 3,4 mmol/l,
general bilirubin 56,7 mm/L, albumen 52 g/L, K+ 3,2 mmol/l, AsAT 0,62; AlAT 0,84;
prothrombin index-58 %; рН blood 7,3. What is previous diagnosis?
a. Hepatocirrhosis
b. Vilson-Konovalov disease
c. Cancer of liver
d. *Endogenous hepatic encephalopathy
e. Meygs disease
380.
A patient, 17 years, complaints of a weakness, worsening of appetite, nausea,
painfull in right subcostal area, dull ache in the body, dark color of urine, temperature to
37,8 °C. She stood in contact with sick on icterusis 1,5 months ago. Objectively: yellow
colour of the skin and visible mucous membranes. The liver is increased to 1,5 cm.
Unsignificant enlargment of spleen. What is your preliminary diagnosis?
a. Viral hepatitis B
b. Infectious mononucleosis
c. *Viral hepatitis A
d. Leptospirosis
e. Pseudotuberculosis
381.
Before the appearance of jaundice in a patient 16 years old, during 2 days there
were an increasing of the body temperature (38,5 °C), headache, dull ache disturbed
whole body. Name the variant of pre-icteric period of viral hepatitis for this patient?
a. *Influenza-like
b. Astenovegetative
c. Artralgic
d. Dyspeptic
e. Allergic
382.
A patient T., 28 years, appilled due to worsening of common status at the seventh
day to the infectious department concerning viral hepatitis. Nausea and vomiting were
present, hepatic smell appeared, increasing of the liver sizes. What biochemical index
does allow to suspect complication in the patient?
a. Increasing activity of AlAT
b. Increasing of bilirubin, increasing activity of AlAT
c. Increasing timol test
d. *Decreasing of protrombin, increasing of bilirubin
e. Decreasing of AlAT
383.
A sick P., 54 years old, complaints of dull pain in right subcostal area, bad taste in
the mouth, itching of the skin, increasing of abdomen. After the liver puncture fatty
dystrophy of hepatocytes, eccentric placing of veins in a hepatic lobule were found. What
laboratory index are most characteristic in this case?
a. Alkaline phosphotase
b. Hemodiastase
c. *Level of transaminases
d. Glucose of blood
e. Creatinphosphokinase
384.
Direct bilirubin is increased , in urine there is significant increase of bilirubin and
urobilin, increasing of stercobilin of excrements. What is the type of icterus?
a. Haemolitic
b. *Parenchimatous
c. Transport
d. Extraliver
e. Mechanical
385.
On the average 15 to 30 % of all population of the planet suffer from some
pathology of liver. Prevalence of hepatitis and cirrhosis in the European countries is about
1 % of adults. Annually in the world there are about 2 million people with acute viral
hepatitis. What % of all cases will develop chronic form.
a. 100 %
b. 50 %
c. 25 %
d. *10 %
e. 1 %
386.
A patient, 24, complaints of pains in right subcostal area, increasing after-meal,
nausea, increase temperature of body to 37,7 °C, icterus, pains in large joints. He is ill
from 8 months. Suffers a nonspecific ulcerative colitis. Hepatosplenomegaly. ESR 47
mm/hr, general bilirubin level is 86,1mmol/lt, direct-42,3 mmol/lt. In blood found out
antibodies against smooth muscles. General albumen of 62 gram/lt, albumin б. 40 %,
globulin. 60 %, gamma globulins 38 %. Not found out the markers of viral hepatitis. On
USD diameter of portal vein is 1 cm What will be your diagnosis?
a. Primary biliary cirrhosis
b. Gilbert syndrome
c. Viral hepatitis
d. Hemochromatosis
e. *Autoimmune hepatitis
387.
Woman of 22 years old, in the 7th month of pregnancy felt ill acutely in 3 weeks
after arrival from Turkmenistan. An icterus, untraceable vomiting, pain in right subcostal
region,, hemmorhaeges on the skin, appeared after three-day fever. Most probable
diagnosis will be:
a. Acute fatty hepatosis of pregnancy
b. Cholestatic hepatosis of pregnant
c. *Viral hepatitis of A
d. Viral hepatitis B
e. Acute sepsis
388.
Weakness appeared in a patient, decrease in an appetite, began pains in the joints
of extremities, felling of weight in right subcostal region, vomiting. Urine became dark in
12 days, and in a day – sclera and skin turned yellow. It was discovered in an
immunogram: antuIgG -HAV (+), HBsAg (+), antiIgМ -НBcorAg (+), anti-НCV (–), antiНDV (–). To the patient 3 months back was extracted a tooth. Most probable diagnosis?
a. *Viral hepatitis B
b. Viral hepatitis A
c. Viral hepatitis C
d. Viral hepatitis D
e. Viral hepatitis E
389.
A patient 42 years, radiologist, entered surgical department with bleeding from the
mouth ,was admitted to the gastroenterological departments . He considered himself
healthy. 20 years ago had haemotransfusion. Fibrogastroduodenoscopy shows venous
dilation of the oesophagus of the III degree. Most probable etiologic reason of disease:
a. Autoantibodies against hepatocytes
b. Viral hepatitis A
c. X-ray irradiation
d. *Viral hepatitis B
e. Defficiency of antitrypsin
390.
A patient during half of the year got frequent parenterally injections concerning
bronchial asthmA. There was decrease in appetite gradually, started to feel weakness,
arthralgia, dark coloured urine, the icterus of skin appeared. Objectively: temperature of
37 °C, pulse 68/min BP 115/70 mm of Hg Liver +4 cm, spleen +1 cm, skin and sclera is
yellow coloured In the general blood analysis: amount of leucocytes: 3,6 109,among them
52 % lymphocytes, ESR 6 mm/hr, activity of ALAT is increased in 10 times. which
information will be more credible than all in blood of patient?
a. Anti-HBs antibodies
b. Anti-HAV IgM antibodies
c. *Anti-HBV antibodies
d. Anti-HCV IgG antibodies
e. Anti-HEV IgM antibodies
391.
A patient of 34 years complaints of fatigue, decreasing of appetite, nausea, feeling
of weight in a epigastric region, bitter taste in the mouth. Objective examination : Pulse
76/min, temperature 37,2 °C. Skin is pale, single vascular star-like rash on the thorax.
Liver is 2 cm below the costal arch. In blood: bilirubin level is 36 mmol/lt, ASAT 2,5,
ALAT 2,8 . Anti-HBc (–), HBsAg and HBeAg (+). What therapeutic tactic is expedient in
this case?
a. Therapy by hepato-protectors
b. *Antiviral therapy by lamivudine
c. Antiviral therapy by ribavirin
d. Therapy by corticosteriods
e. Therapy by immunostimulators
392.
Man 30 years, drug addict, takes drugs intravenously. Has been taking drugs for 12
years Complaints of weakness, moderate icterus, weight in right subcostal region. The
state was worsened gradually. Biochemical indexes: general bilirubin 28,2 mmol/lt;
ALAT 1,0, ASAT 0,8 . Will you define a diagnostic method which it is expedient to
conduct for establishment of etiologic diagnosis?
a. Biopsy of liver
b. *Polymerase chain reaction (PCR)
c. Enzymes of liver
d. Computer tomography
e. Immunological tests
393.
Patient B., 51 years, a lot of years used an alcohol. Complaints of nausea, liquid
stools, icterus of skin. Objectively: skin and sclera is yellow coloured, atrophy of muscles,
subfebrile temperature. Liver 3 cm below the costal arch, painful on palpation What
method of diagnostics most informing in this case?
a. Activity of cytolysis (AsAT, AlAT)
b. Proteinogram
c. *Biopsy of liver
d. Markers of viral hepatitis
e. Violation of cellular immunity (T4,T8)
394.
Patient of B., 64 years old, has been sick with chronic hepatitis. Complaints of pain
in the liver, growth retardation. Objectively: his skin is dry, erythemic, atrophy of
muscles, telangiectasis on shoulders, hyperemia of hands, abdomen is enlarged, look like
“jelly-fish head”. The liver below the costal arc on 3 cm, painful, dense, spleen on 1 cm
below the costal arch, positive symptom of fluctuation. Laboratory investigation:
hypergammaglobulinaemia, increasing of AsAT activity, AlAT is highly increased. What
drug of choice for the patient?
a. Lactulose
b. *Hepatoprotectors
c. Antibiotics
d. Glucocorticoids
e. Alpha interferon
395.
A patient 48 years old, complaints of attacks pains in right subcostal area after the
physical loading. Periodically marks more light excrement, darkening of urine.
Objectively: skin and mucous membranes high icteric. General bilirubin 36,8 mm/L,
direct fraction - 26,4. Ultrasound of gall-bladder: thickness of wall 4 mm, there is a lot of
bile in the ducts. It is necessary to prescribe with a lytolitic purpose:
a. Choleretics
b. *Ursofalk
c. Cholekinetics
d. Spasmolytics
e. Cytostatics
396.
During annual inspection increasing of a liver to 4 cm of 23 years old patient was
found, increasing of bilirubin level in 2 times, AlAT in 2,5 time. He has been often sick
with genital herpes. It was discovered RNA of hepatitis C virus. What is etiologic
treatment
a. *Interferons
b. Essencial phosphotides
c. Choleretics
d. Cholekinetics
e. Ursodesoksihole acids
397.
A patient, 19 years old, with hepatitis B. After violation of diet and nervous stress
the state of the patient became worse: increasing of intensity of jaundice, pulse 110 per 1
min, BP 80/50 mm Hg, vomiting with “coffee-grounds”, decreasing of the liver size. What
complication is it possible to think about?
a. *Acute liver insufficiency
b. Infectious-toxic shock
c. Acute kidney insufficiency
d. Acute extrarenal insufficiency
e. Hemolytic
398.
A patient 28 years old, injection drug addict, complaints of dull pain in the right
subcostal area, weakness, decline of appetite, pain in joints, which disturbs during 2
weeks. At examination: icterus of the skin and sclera, enlargement of the liver and spleen,
dark urine, excrement is discoloured. What is preliminary diagnosis?
a. HIV-infection
b. Hepatitis B
c. Hepatitis C
d. *Toxic hepatitis
e. Hepatitis A
399.
At inspection of a 8 weeks term pregnant woman HBsAg was found. Level of
bilirubin of blood and activity of ALAT were normal. What is necessary to do?
a. *To save pregnancy and conduct the inoculation to newborn against hepatitis
B
b. Termination of pregnancy
c. Termination of pregnancy and conduct treatment by interferon
d. To save pregnancy and conduct treatment by lamivudin
e. To save pregnancy and ultrasonic inspection of the fetus
400.
Student 20 years, treated oneself on an occasion of ARVI (increasing of
temperature to 38,2 °C during 3 days). He complaints of worsening of appetite, increasing
fatigue at a normal temperature and absence of the catarrhal phenomena of upper
respiratory tracts. A doctor found out an increasing and moderate sickliness of liver. There
were cases of hepatitis A in a student’s group. What method of investigation will allow?
a. Ultrasound scanning of the liver
b. Determination of bilirubin level of the blood
c. Determining the amount of beta-lipoproteins
d. *Determination of activity of аminotransferases of the blood
e. Immunofluorescent research of the nasal smears
401.
A patient I., 25 years old, appealed to the internist with complaints of a general
weakness, loss of appetite, pain in the right hypochodrium. Treated himself on an
occasion of ARVI ambulatory 3 days. Became acute worsening of fealling: nausea, pain in
right hypochondrium, ochrodermia of sclera, darkening of urine, appeared. Objectively:
breathing superficial, cardiac tone is rhythmic. Abdomen is soft, painfull in right
hypochondrium, a liver is megascopic, a spleen not palpable. What is most credible
diagnosis?
a. *Hepatitis A
b. Leptospirosis
c. Cholecystitis
d. Influenza
e. Pseudotuberculosis
402.
A patient 42 years old, complaints of dull pain in a right hypochondrium,
weakness, decline of appetite, itching of skin, icterus. Disease began 1 month ago from
protracted аrthralgia, disgust for a meal, strong weakness. An icterus which now increased
considerably appeared three weeks ago. Temperature 36,0 °C. Pulse 56 per 1 min. A
tongue is assessed with white cover. A stomach is soft, sickly in right hypochondrium. A
liver +3 cm, spleen is not megascopic. Common analysis of blood: leukopenia, relative
lymphomonocytosis, ESR 3 mm/hour. What diagnosis is most credible?
a. Cancer of head of a pancreas
b. *Hepatitis B
c. Hepatitis A
d. Cirrhosis of liver
e. Chronic cholecystitis
403.
For a patient 35 years after 4-months of treatment by isoniaside - аdynamia,
icterus, pain in right hypochondric were appeared. A liver is megascopic. In blood activity
of enzymes of AlAT is enhanceable in three times, AsAT in two times. Bilirubin of blood
of 122 gm/ml (conjugated – 82, unconjugated – 40). НBs-аntigen is not found out. What
is the diagnos?
a. Calculary cholecystitis
b. Hepatocirrhosis
c. Acute viral hepatitis
d. Chronic active hepatitis
e. *Toxic hepatitis
404.
A patient 75 years old. Complaints of a subfibrile temperature, general weakness,
pharyngalgia, conjunctivitis. A child in family an acute adenoviral disease is ill. A patient
considers itself a patient the second day. At examination discovered sign of sharp
pharyngitis. Lymphatic nodesare megascopic: neck front and back, arm-pits and inguinal,
to 1 sm in a diameter, soft, not soldered between itself and with a surrounding cellulose. A
pharynx is bloodshot, amygdales are hypertrophied and hyperaemiA. In lights wheezes are
not present. Breathing clean. Tones of heart are muffled. BP - 140/80 mm Hg, Ps - 80 per
1 min. Stomach soft. Palpatory- megascopic liver, salient on 3 sm below than costal arc,
and spleen, soft, painless. Choose the most credible diagnosis from offered:
a. Flu, to middle weight
b. Limfogranulomatosis
c. *Adenoviral infection
d. Infectious mononucleosis
e. Hepatitis A
405.
A patient is 35 years, grumbles about aching pain in right hypocostal area, nausea,
decline of appetite. Beginning of disease binds to appendicitis. After it in 2 months an
icterus appeared first. Treated oneself in an infectious department. In 1 began to notice
aching pain in right hypocostal area, in analyses is an increase of level of bilirubin. Your
diagnosis?
a. Calculary cholecystitis
b. Zhil'ber disease
c. Sharp viral hepatitis
d. Chronic cholangitis
e. *Chronic hepatitis
406.
The patient T., 35 years, operating trained nurse, appealed to the doctor on the 8th
day of gradual development of illness with complaints of a general weakness, rapid
fatigueability, dark color of urine. In the morning noticed the icterus. On examination
temperature of body 36,8 °C. Found out the increase of liver -+3 sm The changes of what
laboratory index most informing at this illness?
a. *AlAT
b. Hemodiastases
c. Protrombin index
d. Cholesterol
e. Alkaline phosphatase
407.
23-years old patient during 6 months gets diabetes mellitus in a policlinic the
injections of insulin. A weakness, arthalgia, grew gradually, an appetite disappeared, then
dull pain appeared in an epigastrium after-meal. In 2 weeks from the beginning of illness
noticed the dark color of urine, and afterwards – icterus on a background which the
general state continues to be worsened. Temperature of body 36,5°C, Ps 58 per 1 min
Liver +5 sm, spleen +1 sm What from hemanalysis is it needed to appoint for
confirmation of diagnosis?
a. Activity of alkaline phosphatase
b. Activity of lactatdehydrogenase
c. *Activity of ALAT
d. Activity of creatinphosphokinase
e. Activity of amylase
408.
42-years old patient got blood transfusion two months ago. A weakness was
gradually increased, an appetite was worsened, arthalgia, small nausea appeared. On a
12th day noticed an insignificant icterus, appealed to the doctor. At examination is the
general state fully satisfactory. The temperature of body is normal, Ps 60 per 1 min, BP
100/70 mm Hg.A liver and spleen is moderate megascopic, a stomach at palpatory is not
sickly. Anti-HCV IGM is found in blood. What from changes in the global analysis of
blood most probably?
a. Neutrocytosis
b. *Leucopenia
c. Lymphopenia
d. Increased ESR
e. Aneozinophilia
409.
A 17-years-old patient during 4 days suddenly had head pains, myalgias, fever of
permanent type. From a 5th day is a normal temperature of body, the general state was
considerably improved, an appetite appeared, nausea, general weakness, disappeared, but
an icterus became noticeable. The general state is satisfactory, liver +3 cm, spleen +1 sm,
Ps 56 per 1 mins Peripheral lymphatic knots, amygdales are not megascopic, hyperemia of
pharynx is not present. At research of global analysis of blood is leykopenia with a
relative lymphocytosis, 3 % virocytis, ESR is normal. Activity of ALAT of blood is
enhanceable in 5 times. What from diagnoses most credible for a patient?
a. *Hepatitis A
b. Infectious mnonucleosis
c. Cytomegaloviral infection
d. Hepatitis B
e. Hepatitis C
410.
In 2 months after returning from India, where often drank unboiled water, a 23years-old expectant mother nauseated, strong general weakness, head pain, later the
temperature of body rose to 38 °, and stuck to week. An icterus appeared on a 6th day, the
general state continued to be worsened. On a 12th day illnesses are the general state
heavy. EiforiyA. At night there was vomiting. Complete fastidium. Bright icterus, signs of
hemorragic syndrome, tachycardiA. BP 110/60, temperature of body 37,8 °C. A liver is
insignificantly megascopic, soft, sickly, spleen +2 cm There is neutrophilic leycocytosis in
the global analysis of blood. General bilirubin of blood is 570 to the line 300, ALAT is
enhanceable in 100 times, timol test of 26 units., urea – 2,1 mmol/l. What is the most
probable diagnosis for this patient:
a. Malignant icterus of pregnant
b. Mechanical icterus
c. Typhoid fever
d. *Acute Hepatitis
e. Leptospirosis
411.
On the 15th day of illness for a 17-years-old patient by hepatitis In anti-HBs is
found in blood. The state of patient was considerably worsened the day before. There was
excitation, there was a nose-bleed, a hemorragic rash appeared on a skin, diminished and
became sickly at palpatory liver. Ps 106 per 1 min, BP of 110/ 70 mm Hg, the temperature
of body subfebrile. What changes of indexes blood will a patient have most credible?
a. Increasing of alkalinephosphatase
b. Increasing of fibrinogen
c. Increasing of indirect bilirubinum
d. *Decline of protrombin index
e. Increasing of albumin
412.
25-years old woman during a semiyear got numerous injections concerning
bronchial asthmA. An appetite disappeared gradually, a weakness, arthalgia grew, urine
became dark in two weeks, through three – an icterus on a background which the general
state continues to be worsened appeared. The temperature of body is normal, Ps 62 per 1
min. Moderate sickliness in the area of pancreas, positive Voskresensky symptom, liver
+4 cm, spleen +2 cm What markers of viral hepatitis will be positive in such case?
a. *Anti-HBc IGM
b. ANTI-HEV IGM
c. ANTI-CMV IGM
d. HBsAg
e. ANTI-HAV IGM
413.
26-years old man during 6 days marks the 6-8-sentinel gettings up temperatures
with a strong chill, which end with abundant then and repeat oneself regularly in a day. 6
months ago there were alike attacks of fever during a stay in Afghanistan, from where
returned 4 months back. Temperature of body 36,4 °C, Ps 94 per 1 min The moderate
increase of spleen and liver is marked. A diuresis suffices, wetting without pathological
changes. What disease can be suspected for a patient?
a. Leptospirosis
b. Hemorragic fever
c. *Malaria
d. Viral hepatitis
e. Sepsis
414.
For a 16-years-old patient in a month after return from Crimea did head pain,
myalgias appear suddenly, strong weakness with a simultaneous fervescence to 39°C,
which stuck to 3 days. The general state was considerably improved, a bad appetite and
nausea, dull pain, was saved only in right hypocostal area, became dark wetting, white is
an excrement. On a 6th day is a moderate icterus, an appetite appeared, a general
weakness diminished. Liver +4 cm, spleen +1 cm. Leykopenia in the general blood
analysis, relative limphomonocytosis, ESR 4 mm/hour. General bilirubinum of blood of
89, the direct prevails, activity of ALAT is enhanceable in 4 times, ASAT – in 3 times.
What from diagnoses most credible?
a. Hepatitis B
b. *Hepatitis A
c. Opisthorchiasis
d. Hemolitic icterus
e. Sharp calculary cholecystitis
415.
Patient of A., 19 years, had ill acutely – the temperature of body rose to 39°C, a
weakness, nausea, appeared. Vomiting and stomach-ache was not present. A temperature
reposed on high numbers 2 days, then went down to normal. Urine became dark on the 6th
day of illness, subicterus noticed on 5th. To this time the feel of patient was improved,
nausea, weakness, disappeared. Objectively: moderate icterus of skin and sclerotica, a rash
is not present. Pulse 66 per 1 min, BP 110/70 mm Hg.A stomach is soft, painless in all of
departments, a liver on 2,5 cm comes forward from under a costal arc, the edge of spleen
was palpitated. Is there what reason of fervescence in the pre-icteric period of this
disease?
a. *Toxicemia
b. Bacteriaemia
c. Pancreatitis
d. Viremia
e. Cholecystitis
416.
40-years old patient during 2th days marks absence of appetite, nausea, general
weakness. A year ago carried sharp hepatitis B, avoided a clinical supervision.
Objectively: the temperature of body is enhanceable, skin and sclerotica are icterus, the
megascopic palpatory, to the moderate closeness, sickly liver, salient from under the edge
of costal arc on 3 sm. A spleen is not megascopic. Wetting moderatory dark colouring.
The sickliness of joints is marked at motions. ALAT of blood 4,0. What most credible
diagnosis?
a. Chronic viral hepatitis, minimum activity
b. Chronic cholecystitis
c. *Chronic viral hepatitis, moderate activity
d. Hepatocirrhosis
e. Chronic viral hepatitis, high activity
417.
A man, 37 years, injection drug addict, practises upon an alcohol, grumbles about a
general weakness, dull pain in right hypocostal area, increase of stomach, shortness of
breath. It is ill chronic hepatitis on an extent 10 years. Objectively: temperature of body
37,1°C, an icterus is expressed. Liver +6 cm. In an abdominal region is a free liquid.
Positive symptom of fluctuation. What complication did arise up for a patient?
a. Intensifying of chronic hepatitis C
b. Sharp hepatic insufficiency
c. Gepatokarcinoma
d. Intensifying of chronic hepatitis B
e. *Hepatocirrhosis
418.
A patient is 59 years, suffers chronic viral hepatitis with development of cirrhosis,
hospitalized with a diagnosis: “bleeding from the veins of gullet”. Sharp pallor of skin
covers. BP 80/40 mm Hg, pulse 100 per 1 min, the temperature of body is normal. On a
front abdominal wall a venous net is extended. What preparation is it necessary to begin
therapy from?
a. Albumen
b. Plasma
c. Neogemodez
d. *Blood
e. Glucose
419.
Patient A., 37 years old, entered to infectious hospital on the third days of disease
in the severe condition. He complaints of the high fever with chills and sweat, general
weakness, pain in right under a rib. Objectively: temperature of body 41 °С, icterus of
skin, liver +2 cm, pain at palpation in abdomen, positive symptoms of Ortner and Mussy,
a spleen is normal, tachycardiA. What is the previous diagnosis?
a. Malaria
b. *Cholangitis
c. Viral hepatitis
d. Sepsis
e. Leptospirosis
420.
Patient W., 38 years old, entered to infectious hospital on the 5th days of disease in
the severe condition. He complaints of the high fever with chills and sweat, general
weakness, pain in sacrum. Objectively: temperature of body of 41 °С, tachycardia,
positive symptom of Pasternacky, liver and spleen not is normal. Previous diagnosis?
a. Malaria
b. Cholangitis
c. *Pyelonephritis
d. Sepsis
e. Leptospirosis
421.
Patient D., 39 years old, entered to infectious hospital on the second week days of
disease in the severe condition. Local habitant, nowhere arrived. She is complaints for the
high fever with chills and sweat, general weakness. Attacks of fever without correct
periodicity. Objectively: the temperature of body – 41 °С, subicterus of sclera,
tachycardia, Pasrernacky‘s symptom positive, liver + 2 cm, spleen + 2 cm. What is the
previous diagnosis?
a. Malaria
b. Cholangitis
c. Pyelonephritis
d. *Sepsis
e. Leptospirosis
422.
Patient D., 39 years old, entered to infectious hospital on the second week days of
disease in the severe condition. She is complaints for the high fever with chills and sweat,
general weakness, pharyngalgia at swallowing. Objectively: temperature of body 37,7 °С,
hyperemia of mucus of pharynx, tonsils are enlarged, loose, festering raid in lakuns,
enlarged submandibula, neck, axilars lymphonodules, icterus of sclera and skin,
bradycardia, liver + 2 cm, spleen + 1 cm. Urine is color of beer, an excrement is
discolored. What is the previous diagnosis?
a. Malaria
b. Infectious mononucleosis
c. Viral hepatitis
d. *Sepsis
e. Leptospirosis
423.
Patient of 52 years, fell ill sharply from a chill and head pain, fevers to 40°C. On a
3th day illnesses are nausea and vomiting, dark urine. On a 4th day a temperature went
down to 37 °, but an icterus appeared and the amount of urine diminished to 600 ml. What
disease is such development of symptoms characteristic for?
a. Hepatitis A
b. Hemorragic fever with a kidney syndrome
c. *Leptospirosis
d. Sepsis
e. Acute glomerulonephritis
424.
Sick, 24 years old, complaints on a pain in a throat, general weakness. Objectively:
temperature 38,0 °C. pulse 96 shots per a min., dermahemia mucus of otopharynx
bloodshot, tonsils are enlarged in sizes, loose. During palpation-enlarged posterior
cervical and submandibular lymph nodes, a spleen comes forward from under the edge of
costal arc on 1 cm. About what disease it is necessary to think?
a. Scarlet fever
b. Acute respirator disease
c. Follicle tonsillitis
d. Typhoid fever
e. *Infectious mononucleosis
425.
At a patient, 17 years: anginA. Temperature 38,2 °C, generilised lymphadenopathy
(the first multiplied neck lymphatic nodeswhich are located along m.
sternocleidomastoideus), small icterus, hepatospleenomegaly. Previous diagnosis?
a. Tuberculosis of lymphatic knots
b. Bacterial quinsy
c. Diphtheria
d. *Infectious mononucleosis
e. Megacaryoblastoma
426.
A girl, 6 years old, admitted to hospital with complaints on general weakness,
increase of temperature, pain in a throat. Objectively: observed oropharyngeal mucous
membrane is bright red, on tonsils – white covering, can take off easily and enlarged all
groups of lymph nodes 1-3 cm in a diameter, dense and elastic in consistancy, not very
painfull, are not soldered between themselves. Liver is enlarged till 3 cm, spleen - till 1
cm. In bloods - leykocytosis, plazmocytes – 20 %. What will be the credible diagnosis?
a. Acute lympholeycosis
b. *Infectious mononucleosis
c. Angina
d. Diphtheria
e. Adenoviral infection
427.
In children with catarrhal phenomena revealed an increase of inframaxillary lymph
nodes, tonsil hyperplasia with whitish soft touch on them in the form of points and islands,
the presence of rare maculo-papular bulging out, increased liver and spleen. Identify the
likely diagnosis?
a. *Infectious mononucleosis
b. Acute leukemia
c. Measles
d. Scarlet fever
e. Chicken pox
428.
At a child with the catarrhal phenomena it is found out the increase of
submandibular and back neck lymphatic knots, hyperplasia of tonsils with the magnificent
raid of gum-blush on them as points and aits, presence of the single roseol-papular
pouring out, increase of liver. Name a credible diagnosis.
a. *Infectious mononucleosis
b. Adenoviral infection
c. Scarlet fever
d. Measles
e. German measles
429.
In a policlinic the patient C., 18 years old, appealed with complaints of a moderate
pharyngalgia, head pain, general weakness fervescence, to 38,9 °C. It is ill already three
days. Objectively: it is found out megascopic back neck, and inguinal lymphatic knots,
hepatospleenomegaly, subicterus scleroticas and skin, changes in a throat, characteristic
for tonsils. At the hemanalysis – lymphomonocytosis. Credible diagnosis?
a. *Infectious mononucleosis
b. Follicle quinsy
c. Adenoviral infection
d. Flu
e. Viral hepatitis
430.
In a policlinic the patient C., 18 years old, appealed with complaints of a moderate
pharyngalgia, head pain, general weakness fervescence, to 38,9 °C. It is ill already three
days. Objectively: it is found out megascopic back neck, and inguinal lymphatic knots,
hepatospleenomegaly, subicterus scleroticas and skin, changes in a throat, characteristic
for tonsils. In general blood analysis – lymphomonocytosis. Principles of treatment?
a. Antibiotics, hepar protective preparations, antihistaminic
b. Antiherpetic preparations, hepar protective preparations, antihistaminic
c. *Antibiotics, preparations of interferon, hepar protective preparations
d. Antibiotics, Antiherpetic preparations , antihistaminic
e. Antibiotics, preparations of interferon, vitamins
431.
At a patient, 17 years: tonsils, temperature 38,2 °C, generilized lymphadenopathy
(the first multiplied neck lymphatic nodeswhich are located along m.
sternocleidomastoideus), small icterus hepatospleenomegaly. What exciter causes this
disease?
a. Herpesvirus I type
b. Herpesvirus ІІ type
c. Herpesvirus ІІІ type
d. *Herpesvirus ІV type
e. Herpesvirus V type
432.
Sick, in 24, disturbs a pharyngalgia, general weakness. Objectively: temperature –
38 °C. Pulse 96 shots per min, dermahemia mucus of otopharynx bloodshot, tonsills are
megascopic in sizes, loose. Palpated megascopic back neck and submandibular lymphatic
knots, a spleen comes forward from under the edge of costal arc of to 1 sm. What methods
of diagnostics is it possible to confirm a diagnosis by?
a. Reaction of Paul-Bynnel, Right-Heddlson, Goffa-Bauer
b. *Reaction of Paul-Bynnel, Lovrik-Volner, Goffa-Bauer
c. Reaction of Paul-Bynnel, Right-Heddlson, Lovrik-Volner
d. Reaction of Paul-Bynnel, Goffa-Bauer
e. Reaction of Paul-Bynnel, Goffa-Bauer, Right-Heddlson, Lovrik-Volner
433.
Patient P, 17 years old, has got ill gradually. General weakness, fatigue, painful
throat, and nausea were marked. He was hospitalized on the 5th day of illness.
Objectively: temperature is 38 °С. Objectively: skin and sclera are yellowish, neck,
anticubital and subclavian lymph nodes are enlarged. Not numerous maculo-papular
elements of rash on a trunk are found. The tongue is covered with white coat. The tonsils
covered with white yellowish patches.On palpation of abdomen enlarged spleen and liver
are found. In blood there is leucocytosis with neutophylic shift, atypical mononuclears-10
%, plasmatic cells- 10 %. What would be the diagnosis?
a. Scarlet fever
b. *Infectious mononucleosis
c. Typhoid fever
d. Yersiniosis
e. Lacunar tonsillitis
434.
The boy 12 years old, with catarrhal phenomena seen an increase of all the lymph
nodes, sclera, hyperplasia of tonsils white layers on them as islands, existing single
maculo-papular rash, increased liver and spleen size. What additional research should
appoint?
a. Reaction of passive heamaglutination(RPG) with influenza viruses
b. Ultrasound of the abdomen
c. Puncture of the lymph nodes, with following microscopy
d. Inoculation of blood
e. *Blood test for atypical mononuclears
435.
Patient A., 17 years old, became ill gradually. A general weakness, fatigue, pain in
throat, abdomen-ache, nausea, was marked. He was hospitalized on the 5th day of illness.
Objectively: body temperature 38 °C, increased cervical, cubital and axillary lymph
nodes. Subecteric skin and sclerA. Observed elements of spot-papul rash on a trunk. Raids
on tonsils are loose, yellow, tongue coated by white covering, a abdomen is moderately
swollen, hepatospleenomegaly. In general blood – leucocytosis, neutrophyle shift to left,
atypical mononuclears – 10 %, plasmatic cells – 10 %. What is preliminary diagnosis?
a. *Infectious mononucleosis
b. Yersyniosis
c. Lacunar tonsillitis
d. Viral hepatitis A
e. Typho-paratyphoid disease
436.
Patient 18 years, entered permanent establishment with complaints of headacke,
general weakness increase of temperatures, to 37,5–38,2 °C during 6 days, pharyngalgiA.
Objectively: all groups of lymphonoduses, 1-3 cm in a diameter, elastic, are megascopic,
little sickly, not soldered between itself. A liver is megascopic on 3 cm, spleen – on 1 cm.
In a blood is leycocytosis, placmatic mews – 15 %. What group of herpes does the exciter
of this illness belong to?
a. Alpha
b. Beta
c. *Gamma
d. Delta
e. Teta
437.
Patient A., 18 years old, is complaining about headache, weakness, high
temperature, soar throat. Objectively: general lymphadenopaty,wit lymph nodes 1-3 cm in
diameter, dense, elastic not painful and not combined between themselves.
Hepatospleenomegaly was noticed. In blood there is leucocytosis, monocytes – 15 %.
What is the diagnosis?
a. *Infectious mononucleosis
b. Adenoviral infection
c. Tonsillitis
d. Diphtheria
e. Acute leukemia
438.
An 18 years old patient, entered the hospital with complaints of headache, general
weakness, raising the temperature to 37,5-38,0 °C for 6 days, a sore throat. Objective
examination: increasing of all lymph nodes, 1-3 cm in diameter, flexible, megaloblastic
not soliter together. Increasing of the liver size till 3 cm, spleen – 1 cm. Common blood
analysis: leukocytosis, placma cells – 15 %. What is preliminary diagnosis?
a. Diphtheria
b. Adenovirul infection
c. Lacunar tonsillitis
d. *Infectious mononucleosis
e. Acute lympholucosis
439.
Patient D., 20 years old, fell ill and went to infectious polyclinic cabinet with
complaints of moderate pain in the throat, headache, general weakness, increased body
temperature of 38.9 °C. Fells bed during three days. At examination: increasing of
inguinal lymph nodes, hepatosplenomegaly, subicteric sclera and skin, changes in the
throat are typical for sore throats. In the blood – lymphogranulomatosis. The principles of
treatment are?
a. *Antibiotics, interferons, hepatoprotectors
b. Antihypertensives hepatoprotectors, antihistamines
c. Antibiotics, hepatoprotectors, antihistamines
d. Antibiotics, antihypertensives, antihistamine
e. Antibiotics, interferones, vitamins
440.
A patient D., 20 years old came to the infectious polyclinic with complaints of
moderate pain in the throat, headache, general weakness, increasing of body temperature
to 38.9 °C. He fells sick during 3 days. Objective examination: increasing inguinal lymph
nodes, hepatosplenomegaly, subicteric sclera and skin, changes in the throat are typical for
sore throats were found. Blood analysis – lymphogranulomatosis. What is probable
diagnosis?
a. *Infectious mononucleosis
b. Tonsillitis
c. Adenovirus infection
d. Influenza
e. Viral hepatitis
441.
The patients, 20 years old, with 5 days fever, pain in the throat during swallowing,
pain in muscles. During review – there is thick, congested tonsils with purulent layers that
are easily removed, enlarged neck, and inframaxillary lymph glands, liver + 1.5 cm
possible palpation of edge of spleen. What additional research should be done?
a. Puncture of the lymph nodes, followed by microscopy
b. Ultrasound of the abdominal cavity
c. *Blood test for atypical mononuclear
d. Inoculation of blood
e. Reaction of passive heamoglutination(RPH) influenza A viruses
442.
Patient A., 35 years old, came to the clinic on the third day of disease with
complaints of drowsiness, sweat, headache, fever up to 38.5 °C. Reviewing physician
found sore throat, splenomegaly, enlarged lymph nodes. Liver not enlarged. Blood
analysis – leukocytosis, lymphomonocytosis. What is treatment for the patient?
a. *Antibiotic, drugs interferon, hepatoprotectors
b. Antihistamine, antiherpetic preparations hepatoprotectors
c. Antibiotic, hepatoprotector, antihistamine
d. Antibiotics, antihistamine, antiherpatic drugs
e. Vitamins, antibiotics, drugs interferon
443.
35 years old patient A., came to the clinic on the third day of illness with
complaints of drowsiness, sweat, headache, fever up to 38.5 °C. Reviewing physician
found sore throat, splenomegaly, enlarged lymph nodes. Liver is not enlarged. Blood
analysis – leukocytosis, lymphocytosis. What additional test should hold the patient for
infectious mononucleosis?
a. Byurne and Wright-Hadulson’s reaction
b. ELISA-test, bacteriological test for tularemia
c. Bacteriological test for diphtheria and typhoid fever
d. *ELISA-test, bacteriological test for diphtheria
e. Paul-Bunnel’s reaction and lymph node puncture
444.
A female M., 24 years old came to the doctor with long-lasting fever, night sweat.
Over the past three months, weight reduced by 7 kg. Low intention for sex. Objective
examination found an increasing of all lymph nodes, hepatolienal syndrome. In the blood:
Leucocyte – 2,2?109. What disease should be suspected?
a. *HIV infection
B.
Lymphogranulomatosis
C. Tuberculosis
D.Infectious mononucleosis
E. Sepsis
445.
A patient, 13 years old, complaints on pain in a throat, body temperature rise till 38
°C. Objectively: hyperemia of skin, hyperpigmentation of oropharyngeal mucous , tonsils
are enlarged in size, marked suppuration of follicles. During palpation enlarged posterior
cervical and submandibular lymph nodes. Pulse 96 times/minute, spleen – under the edge
of costal arc on 1 cm. About what disease it is necessary to think?
a. *Infectious mononucleosis
b. Follicular tonsillitis
c. Typhoid fever
d. Measles
e. Acute respiratory disease
446.
During the examination of a patient, 17 years old, observe tonsilitis, body
temperature 38,2 °C, generalized lymphadenopathy (cervical ltmph nodes, located along
the m. sternocleidomastoideus), mild jaundice, hepatospleenomegaly. What is preliminary
diagnosis?
a. Tuberculosis of lymph nodes
b. Bacterial tonsillitis
c. Diphtheria
d. *Infectious mononucleosis
e. Lymphogranulematosis
447.
Patient C., 16 years old, consulted to the infectious hospital on the 2nd day of
diseases with complaints for a pain in throat at swallowing, increase of temperature.
Objectively: body temperature 38,6 °C, hyperemia to the pharynx, edematous and loose
tonsils, on both sides – purulent covering which can be taken off easily by spatulA. By
palpation painful enlarged submandibular lymph nodes. Pulse 114 times/minute. From
epidemic anamnesis it is clear that the same symptoms had her boyfriend. Changes in
other organs didn’t observe. What preliminary diagnosis can be suspected?
a. *Lacunar tonsillitis
b. Diphtheria of pharynx.
c. Infectious mononucleosis.
d. Influenza
e. Scarlet fever
448.
A 17 years old boy, got ill suddenly: the temperature rose up-to 40,3 °С, extremely
sharp headache, makes him to yell. The patient is agitated and vomits frequently. Tremor
of fingers of extremities is observed. The star like hemorrhagic rash of different form and
sizes appeared, mainly on buttocks, thighs, shins, and trunk. Meningeal signs are positive.
What is the most credible diagnosis?
a. Encephalitis
b. Flu with a hemorrhagic syndrome
c. * Meningococcal infection
d. Measles
e. Leptospirosis
449.
A 19-years-old patient became ill 5 days ago: subfebrile temperature, acquired
voice, dry cough weakness. After hospitalization: t - 37,1 °C, but general condition is
serious, pallor, expressed weakness, aphonia, noisy stenotic breathing, inciters indrawings
of intercostal intervals, pulse 110 per min, frequency of breathings 36 per min., cyanosys
of lips and nailes. Tonsills is covered by the greyish dense raid. Define strategy of
intensive therapy:
a. Tracheotomy, AVL, antitoxin
b. Tracheotomy, AVL, toxoid, desintoxic therapy
c. * Sedative, intubation, antitoxin, dexametazon, desintoxic therapy
d. intubation, sedative, toxoid, penicillinum, desintoxic therapy
e. sedative, oxygenotherapy, antitoxin i/v
450.
A girl, 6 years old, admitted to hospital with complaints of general weakness,
increase of temperature, pain in a throat. Objectively: observed oropharyngeal mucous
membrane is bright red, on tonsils - white covering, can take off easily and enlarged all
groups of lymph nodes 1-3 cm in a diameter, dense and elastic in consistancy, not very
painfull, are not soldered between themselves. Liver is enlarged till 3 cm, spleen - till 1
cm. In bloods - leykocytosis, plazmocytes - 20 %. What will be the credible diagnosis?
a. Acute lympholeycosis
b. *Infectious mononucleosis
c. Angina
d. Diphtheria
e. Adenoviral infection
451.
A new born child on 10th day of life became worse: Temperature 39.2 °C, frequent
vomiting, generalized cramps, violations of consciousness, spastic paresis of left
extremities. Month prior to his birth herpes virus was present in the mother, which she did
not treat. What disease is most possible?
a. * Herpetic encephalitis
b. Meningococcal meningitis
c. Subarechnoid hemorrhage
d. Cerebral abscess
e. Violation of blood cerebral circulation
452.
A patient 17 years old, became suddenly ill: .Temperature rose to 40,3 °C. Severe
headache, motive excitation, frequent vomiting, tremor of fingers of extremities.
Hemorrhagic spots of round form and different sizes, more frequently as stars, mainly on
buttocks and trunk. Meningeal signs are positive. What is the most possible diagnosis?
a. Encephalitis
b. Flu with a hemorrhagic syndrome
c. * Meningococcal infection
d. Measles
e. Leptospirosis
453.
A patient 21 years old 4th day grumbles about a general weakness, moderate
pharyngalgia fervescence, to 39 °C. Did not treat oneself. Objectively: edema and
cyanosys of mucous of otopharynx of left tonsill rough fibrinose raid which goes out on a
palatal handle and tongue. Bloodstreaks appear at the attempt of removal by his spatulA.
Submandibular lymphatic nodesare megascopic, more on the left side. What diagnosis is
most credible?
a. Paratonsill abscess
b. Lacunar quinsy
c. Quinsy of Vensan-Symanovskiy
d. * Diphtheria of otopharynx
e. Infectious mononucleosis
454.
A patient 60 years old for 2 days has disturbed severe pain in a right arm. On 3rd
day appeared blisters, pouring out as a chain on the skin of shoulder, forearm and brush.
Sensitiveness in the area of pouring out is mionectic. What disease can be diagnosed?
a. Dermatitis
b. * Herpetic ganglionitis
c. Neck-pectoral redicals
d. Psoriasis
e. Allergy
455.
A patient A., 23 years old, hospitalized on the 2th day of disease with complaints
of a weakness, pharyngalgias, that increase at swallowing, chill. State is moderate,
temperature of body - 38.3 °C, a moderate hyperemia with cyanosys of mucous of the
throat, tonsills is megascopic, covered by the pellicles raids which spread on a wall
pharynx and tongue. Increasing of neck lymphonodes. Pulse - 88 per 1 min, BP 120/60
mm of Hg. A liver and spleen are not megascopic. What is previous diagnosis:
a. * Diphtheria of oropharynx
b. Mycotic tonsillitis
c. Simanovsky-Vensent's tonsillitis
d. Infectious mononucleosis
e. Acute leucosis
456.
A patient became ill sharply. Severe pain of head, frequent vomits appeared in the
morning. Temperature of body rose to 39,9 °C. Adopted febrifuge, however much the
state got worse. Till the evening patient lost consciousness. Excited, sharply expressed
meningeal signs. What is most reliable diagnosis?
a. Status typhosus in typhoid fever
b. Viral meningoencephalitis
c. Sepsis, infectious-toxic shock
d. * Meningococcal infection, meningitis
e. Status typhosus in epidemic typhus
457.
A patient C., 25 years old, fell suddenly ill. Every morning severe headache,
frequent vomiting, temperature of the body is 39,9 °C. Adopted fatigue, then state got
much worse. In the evening lost of consciousness. Expressed muscles pains of back and
head. Positive Кеrning's symptom. Leukocytes - 18,0*109/L. What is the most reliable
diagnosis?
a. Flu
b. Epidemic typhus, typhus state
c. Viral menigoencephalitis
d. Sepsis, infectious-toxic shock
e. * Bacterial menigoencephalitis
458.
A patient came with complaints of erosions of his penis. From anamnesis frequent
appearance of similar rashes during a year is found out. Objectively: on a balanus are the
grouped blisters and erosions of polycyclic outlines, with clear margin, soft during
palpation. What is your diagnosis?
a. * Recurrent congenital herpes
b. Pemphigus vulgaris
c. Primary syphillis
d. Pyodermia
e. Scabies
459.
A patient came with complaints of sickly erosions on his penis. From anamnesis
frequent appearance of similar rashes is found out during a year. Objectively: on a balanus
are the grouped blisters and erosions, soft on palpation. What is your diagnose?
a. * Recurrent herpes of ІІ type
b. Vulvar pemphigus
c. Primary syphyllis
d. Shankoform pyoderma
e. Recurrent herpes of ІІІ type
460.
A patient D., 20 years old came to the infectious polyclinic with complaints of
moderate pain in the throat, headache, general weakness, increasing of body temperature
to 38.9 °C. He fells sick during 3 days. Objective examination: increasing inguinal lymph
nodes, hepatosplenomegaly, subicteric sclera and skin, changes in the throat are typical for
sore throats were found. Blood analysis - lymphogranulomatosis. What is probable
diagnosis?
a. * Infectious mononucleosis
b. Tonsillitis
c. Adenovirus infection
d. Influenza
e. Viral hepatitis
461.
A patient D., yesterday in the evening delivered in the hospital due to high
temperature of body, increasing pharyngalgia, pain lockjaw of masseters, nausea, vomits.
At nasopharhyngoscopy by an otolaryngologist the moderate expressed hyperemia,
edema, cyanosys of mucous membrane of left palatal tonsil, oral cavity and nasopharynx
were found out. The surface of tonsil is covered by the raid of dirty-grey color pellicle
which goes beyond his borders (in nasopharynx). Increasing of left submandibular
lymphatic node of dense consistency was also found. The edema of hypoderm of neck
reaches to the collar-bone. What is the described disease?
a. * Hypertoxic diphtheria of nasopharynx
b. Localized diphtheria of nasopharynx
c. Subtoxic diphtheria of nasopharynx
d. Widespread diphtheria of nasopharynx
e. Lacunar tonsillitis
462.
A patient has herpetic meningitis. What preparation of specific therapy for viral
neuro infection should be given?
a. * Acyclovir
b. Cefataxime
c. Ceftriaxone
d. Gentamycin
e. Furazolidon
463.
A patient is hospitalized in infectious permanent establishment: sharp beginning of
disease, temperature 39,9 °C, moderate pharyngalgia, takes place edema, insignificant
hyperemia with cyanosys of mucuses shells of otopharynx, on tonsills dense, brilliant,
greyish color the raids placed as continuous pellicle, is heavily taken off, bare a surface,
that bleeds. Submandibular lymphonoduss are moderatory megascopic. A patient must
immediately do:
a. Strokes with tonsills, nose or other areas for the exposure of diphtherial stick
b. IFA
c. * Microscopy (painting for Neyser)
d. Haemoculture
e. RDHA with a diphtherial diagnosticum
464.
A patient is sick with meningococcal meningitis. He take a massive dose of
penicillin. 4 days temperature of body 36,6-36,8 °C. Meningeal signs are negative. When
is it possible to stop the antibiotic therapy.
a. * At a cytosis in a CSF 100 and less, lymphocytes prevail
b. After 10 days from the beginning antibiotic therapy
c. After 7 days from the beginning antibiotic therapy
d. At a cytosis 100 and less, neutrophil prevail
e. From 6 days from the beginning antibiotic
465.
A patient N., 45 years old, complaints of headache, general weakness increasing of
temperature, to 37.4 °C. In 2 days pain appeared in the pectoral region of spine with an
irradiation in a right between scapular regions. After some time skin in this region turned
red as a strip from a spine to the subarmpit line, and in 2 days red knots which through the
set time grew into blisters with transparent maintenance appeared in this place. What is
your diagnosis?
a. * Herpes zoster
b. Thoracal rediculopathy
c. Neurology of intercostal nerves
d. Neurology of superscapular nerve
e. Herpetiform dermatitis
466.
A patient sharply had a chill, headacke, vomits, temperature of body up to 38,5 °C.
Till evening rigidity of muscles, Kernig symptom appeared. Herpetic blisters are marked
on mucouse of lips and nose. Neurological symptoms is not found out expressed. What
disease will you suspect?
a. Subarachnoiditis hemorrhage
b. Herpetic encephalitis
c. Abscess of brain
d. Hemorrhage in a brain
e. * Meningococcal meningitis
467.
A patient was delivered in a hospital on the 2th day of illness with the symptoms of
expressed toxicosis. Objectively: found out the severe condition, hemorrhagic rash of
wrong form of different size on a skin, hemorrhages in the scleras of both eyes.
Extremities are cold. AP 60/20 mm/hg, pulse - 120 per minute. Previous diagnosis?
a. * Meningococcemia
b. Hemorrhagic fever with a kidney syndrome
c. Leptospirosis
d. Typhoid fever
e. Pseudotuberculosis
468.
A patient with meningococcal meningitis gets penicillin during 7 days. The
temperature of body is normal 4 days. Meningeal signs are negative. When is it possible to
stop the antibiotic therapy.
a. In default of leucocytosis displacement in blood
b. * At a cytosis in a neurolymph 100 and less, lymphocyte prevail
c. At a cytosis in a neurolymph 100 and less, neutrophil prevail
d. At a cytosis in a neurolymph 150, lymphocyte prevail
e. At once immediately
469.
A patient X., 25 years old, was examined by a otolaryngologist on the 4th day of
illness. Temperature of body - 38.1 °C. Complaints of indisposition, moderate pain in the
throat. Objectively: a mouth opens fully. Mucous of a soft palate, tongue were swollen,
insignificant hyperemia with cyanosis. Increasing of tonsills, covered by the grey dense
raid. The raid is taken off hard. The edema of neck is not present. Increasing of
submandibular lymph nodes. What is most credible diagnosis?
a. Lacunar tonsillitis
b. Infectious mononucleosis
c. Simanovsky-Vensent's tonsillitis
d. * Diphtheria of oropharynx
e. Follicular tonsillitis
470.
A patient, 13 years old, complaints of pain in a throat, body temperature rise till 38
°C. Objectively: hyperemia of skin, hyperpigmentation of oropharyngeal mucous , tonsils
are enlarged in size, marked suppuration of follicles. During palpation enlarged posterior
cervical and submandibular lymph nodes. Pulse 96 times/minute, spleen - under the edge
of costal arc on 1 cm. About what disease it is necessary to think?
a. * Infectious mononucleosis
b. Follicular tonsillitis
c. Typhoid fever
d. Measles
e. Acute respiratory disease
471.
A patient, 20 years old, during few days complaints of pharyngalgias. After
supercooling the state became worse : sudden chills, increase of temperature to 40,6 °C,
headache. On skin of low extremities, trunk and buttocks there are a lot of different sizes
hemorragic spots, acrocyanosis. Consiouness is preserved. Meningeal signs are absent.
What is the previous diagnosis?
a. * Meningococcal infection
b. Flu
c. Epidemic typhus
d. Hemorrhagic fever
e. Leptospirosis
472.
A patient, 22 years old, became ill sharply. History showed fever up to 38.2 °C
with headache, repeated vomiting, olfactory and tastes hallucinations. Quickly got
complex of meningeal symptoms, pyramidal paresis. The general epileptic attack and
comatose state also developed. Neurolymph is with mixed lymphocytosis, cytochrome,
single red corpuscles. What is previous diagnosis?
a. Brain abscess
b. Subdural empyema
c. * Herpetic encephalitis
d. Tumor of brain
e. Encephalopathy
473.
A pregnant woman, 27 years (pregnancy ІІ, 8-10 weeks.), temperature of body
increased. At the inspection on a TORCH-infection antibodies are found to the herpes
virus, ІІ types of class IGM. What we must recommend to pregnant women?
a. * To cut pregnancy
b. To prolong the supervision
c. Treatment with acyclovir
d. Symptomatic treatment
e. Appoint of alpha-fetoprotein
474.
A sick 3 years old child came to the doctor with symptoms of the fever, languor,
waiver of meal. A boy is capricious, temperature of body 37.9 °C. On the mucous of soft
palate, cheeks are single vesicle, hypersalivation. What is the diagnosis?
a. * Herpetic stomatitis
b. Candidosis of oral cavity
c. Leucoplacia
d. Follicular tonsillitis
e. Lacunar tonsillitis
475.
A sick person, 65 years old, complaints of rash, pain in a subscapular region.
Objectively: on a skin surface of the subscapular region present the arcwise rose-red
filling out hearths some infiltrative, with clear scopes. On-the-spot hearths grouped
vesicles with transparent maintenance. What preparation he should take?
a. Suprastyn
b. Prednisolone
c. Biseptolum-480
d. Loratidin
e. * Laferon
476.
A woman 65 years old the disease had beginning sharply from increase of
temperature to 39.0 °C, weakness, and pain in the left part of thorax that increased with
breathing motions. On 3rd day of disease vesicular breathing appeared after motion of rib
on the left on a hyperemic background. Together with sick a grandchild lives 4 years.
What measures of prophylaxis of disease need to be adopted?
a. Vaccination
b. Final disinfection
c. Reception of specific immunoprotein
d. * Isolation of patient
e. Acyclovir administration
477.
A woman C., during 3 days complaints of a general weakness, headache increasing
of temperature to 39-40 °C, insignificant pharyngalgiA. Her husband is sick with
tonsillitis. Objectively: a skin is pale, cyanosis of lips. Hyperemia of mucous of
oropharynx, increasing of tonsils. On the spot of tonsils there are continuous dense
accented mother-of-pearl raids, which are taken off with great effort with bleading.
Increasing of submandibular lymphatic nodes. Edema of the neck. AP 105/65 mm Hg.
What is most credible diagnosis?
a. Acute leucosis
b. Lacunar tonsillitis
c. Infectious mononucleosis
d. * Diphtheria of oropharynx
e. Adenoviral infection
478.
After the disease which was accompanied by the fever and pharyngalgias, there
were an odynophagia, dysarthria, weakness and violation of motions in hands and feet,
hyporeflexia, violation of sensitiveness in extremities to the polyneurotic type. What
disease does it follow to think about above all things?
a. Neuropathy of hypoglossus
b. * Diphtherial polyneuropathy
c. Neuropathy of glossopharyngeus nerve
d. Trunk encephalitis
e. Pseudobulbar syndrome
479.
Among the students of PTU 2 cases of generalized form of meningococcal
infection are registered. What preparation does it follow to enter to the contact persons
with the purpose of urgent prophylaxis?
a. Normal immunoglobulin
b. Leukocytic interferon
c. * Meningococcal vaccine
d. Meningococcal anatoxin
e. Bacteriophage
480.
An 18 years old patient, entered the hospital with complaints of headache, general
weakness, raising the temperature to 37,5-38,0 °C for 6 days, a sore throat. Objective
examination: increasing of all lymph nodes, 1-3 cm in diameter, flexible, megaloblastic
not soliter together. Increasing of the liver size till 3 cm, spleen - 1 cm. Common blood
analysis: leukocytosis, placma cells - 15 %. What is preliminary diagnosis?
a. Diphtheria
b. Adenovirul infection
c. Lacunar tonsillitis
d. * Infectious mononucleosis
e. Acute lympholucosis
481.
A 17 years old boy, after tick bite got ill suddenly: the temperature rose up-to 40,3
°С, extremely sharp headache, makes him to yell. The patient is agitated and vomits
frequently. Tremor of fingers of extremities is observed. Meningeal signs are positive.
What is the most credible diagnosis?
a. *Viral meningoencephalitis
b. Leptospirosis
c. Measles
d. Meningococcal infection
e. Flu with a hemorrhagic syndrome
482.
A citizen of Nigeria, 19 years old, came to study in Ukraine. After 10 days a strong
chill appeared. There has been a headache, slice, nagging pain in the muscles.
OBJECTIVE: body temperature 39,6 °C, sclera subicteric, herpes of lips. Symptoms of
bronchitis were present. Spleen is significantly increased, dense, the liver is normal. What
investigation is necessary to do?
a. USI of abdomen
b. Lumbar puncture
c. *Microscopy of tissue slades
d. Microscopy of peripheral blood
e. Observation of oculi
483.
A patient B., 62 years old, became ill suddenly. Severe pain of head, frequent
vomiting appeared in the morning. Temperature of body rose to 39,9 °C. Adopted
febrifuge, however much the state got worse. Till the evening patient lost consciousness.
Excited, expressed meningeal signs present. What is most probably diagnosis?
a. Status typhosus in epidemic typhus
b. Status typhosus in typhoid fever
c. *Viral meningoencephalitis
d. Sepsis, infectious-toxic shock
e. Meningococcal infection, meningitis
484.
Patient B., 62 years old, became ill suddenly. Severe pain of head, frequent
vomiting appeared in the morning. Temperature of body rose to 39,9 °C. Adopted
febrifuge, however much the state got worse. Till the evening patient lost consciousness.
Excited, expressed meningeal signs present. What is most probably diagnosis?
a. Meningococcal infection, meningitis
b. Status typhosus in typhoid fever
c. Sepsis, infectious-toxic shock
d. Status typhosus in epidemic typhus
e. *Viral meningoencephalitis
485.
A patient became ill sharply. Severe pain of head, frequent vomits appeared in the
morning. Temperature of body rose to 39,9 °C. Adopted febrifuge, however much the
state got worse. Till the evening patient lost consciousness. Excited, sharply expressed
meningeal signs. What is most reliable diagnosis?
a. *Viral meningoencephalitis
b. Meningococcal infection, meningitis
c. Status typhosus in epidemic typhus
d. Sepsis, infectious-toxic shock
e. Status typhosus in typhoid fever
486.
A patient became sick 2 days ago suddenly. Temperature of body rose to 41°С,
headache, vomit, positive Kernig’s and Brudzinsky’s symptoms appeared. In
cerebrospinal liquid: cytosis 7 000 in 1 mkl, 90 % are lymphocytes, some increasing of
protein, sugar, chlorides. What is clinical diagnosis?
a. Endocarditis
b. *Meningeal form of viral encephalitis
c. Infectious-toxic shock
d. Meningococcemia
e. Meningicm
487.
A patient C., 25 years old, fell suddenly ill. Every morning severe headache,
frequent vomiting, temperature of the body is 39,9 °C. Adopted fatigue, then state got
much worse. In the evening lost of consciousness. Expressed muscles pains of back and
head. Positive Кеrning’s symptom. Leukocytes – 8,0*109. What is the most reliable
diagnosis?
a. Epidemic typhus, typhus state
b. Flu
c. *Viral menigoencephalitis
d. Bacterial menigoencephalitis
e. Sepsis, infectious-toxic shock
488.
A patient is disturbed by attacks of fever. The icterus of sclera and skins is
observed, hepatosplenomegaly is found on palpation. What is possible diagnosis accept?
a. Leishmaniosis
b. Leptospirosis
c. *Sepsis
d. Malaria
e. Viral hepatitis
489.
A patient sharply had a chill, head pain, vomits, temperature of body rose up to
38,5 °C. Till evening rigidity of muscles, Kernig’s symptom appeared. Herpetic blisters
are marked on mucous of lips and nose. Neurological symptoms is not found out
expressed. What disease will you suspect?
a. Herpetic encephalitis
b. Abscess of brain
c. *Meningoencephalitis
d. Hemorrhage in a brain
e. Subarachnoid hemorrhage
490.
A patient became sick 2 days ago suddenly. Temperature of body rose to 41°С,
headache, vomit, positive Kernig’s and Brudzinsky’s symptoms appeared. In
cerebrospinal liquid: cytosis 7 000 in 1 mkl, 90 % are lymphocytes, some increasing of
protein, sugar, chlorides. What is clinical diagnosis?
a. Endocarditis
b. Meningococcemia
c. Infectious-toxic shock
d. * Meningeal form of viral encephalitis
e. Meningicm
491.
A patient C., 25 years old, fell suddenly ill. Every morning severe headache,
frequent vomiting, temperature of the body is 39,9 °C. Adopted fatigue, then state got
much worse. In the evening lost of consciousness. Expressed muscles pains of back and
head. Positive Кеrning’s symptom. Leukocytes – 8,0*109. What is the most reliable
diagnosis?
a. Epidemic typhus, typhus state
b. Bacterial menigoencephalitis
c. Flu
d. Sepsis, infectious-toxic shock
e. *Viral menigoencephalitis
492.
At a patient with meningoencephalitis 44 years old, rose up general clonic-tonic
cramps, abundant sweat, hyperemia of the face, bradycardia quickly changed on
tachycardia, violation of breathing (Cheyne-Stokes type). What complication develop?
a. Swelling and edema of cerebrum
b. *Wedging of cerebrum in the cervical channel
c. Hypovolemic shock
d. Waterhause-Friedrichsen syndrome
e. Infectious toxic shock
493.
In a patient, 27 y.o., after tick biting, headache which accompanied by nausea,
repeated vomits, hyperesthesia, photophobia appear. At a review: feet are bended to the
trunk, expressed rigidity of muscles of the back of head, positive symptoms of Kerning’s,
Brodzinsky’s. It is not found out paresises. Select a basic neurological syndrome.
a. Root syndrome
b. *Meningeal syndrome
c. Syndrome of liquor hypertension
d. Vegetative crisis
e. Syndrome of liquor hypotension
494.
Patient B., 38 years, delivered by emergency doctor, anamnesis is unknown.
However, according to neighbours, patient 2 days ago was healthy. At a review:
consciousness is absent, motion of left overhead and lower extremities is absent,
increasing of muscular tone is marked in the same extremities. Periodically are clonictonic cramps in right extremities. Expressed asymmetry of person. Sharply expressed
rigidity of muscles of the back of head, positive Кеrnig symptom, positive overhead and
lower Brudzinsky symptom. Acrocyanоsis is marked, temperature of body 38,8 °С.
Breathning 36/min, unrhythmical. Vesicular breathing during auscultation. Pulse 72/min,
weak filling and tension. Heart tones are muffled, AP 80/50 mm/hg. Tongue is dry,
assessed by the white raid. Physiology sending in a norm. Your diagnosis?
a. Epidemic typhys
b. Hemorrhagic fever
c. Typhoid fever
d. *Viral meningoencephalitis
e. Leptospirosis
495.
Patient D., 30 y.o., teacher, have been examined by the doctor of first-aid and
delivered to a clinic on the 2nd day of illness. Fell ill acutely, temperature 40 °C, severy
headacke, repeated vomits, photophobiA. The general condition is severe, consciousness
is darkened. Sharply expressed rigidity of neck muscles, positive Kernig symptom. Pulse
100 min, weak; tones of heart deaf, AP 60/30 mmHg. What is your diagnosis?
a. Crimean hemorrhagic fever
b. Disease of blood
c. Flu, toxic form
d. Hemorrhagic stroke
e. *Viral meningoencephalitis
496.
Patient D., 53 years, have been examined by doctor on the 2th day of illness. Fell
ill suddenly, temperature 40 °С, acute headache, repeated vomiting, photophobiA. The
common state is severe, consciousness is darkened, expressed rigidity of cervical muscles,
positive Kernig’s symptom. Pulse 110/min, weak. BP 60/30 mm/hg. What is your
diagnosis?
a. Hemorrhagic stroke
b. Leptospirosis
c. Flu, toxic form
d. *Viral meningoencephalitis
e. Marburg hemorrhagic fever
497.
Patient K., 21 y.o., the disease began from the increasing of temperature to 39,0
°C, headache, chill, repeated vomit. Objectively: temperature of 39,3 °C, pulse 76.
Rigidity of muscles of the back of head. The tendon reflexes are expressed. Analysis of
liquor: 84 % lymphocytes, 16 % neutrophiles, liquid is turbid, flows out under the
promoted pressure. What disease is most reliable?
a. Second festering meningitis
b. *Viral meningoencephalitis
c. Meningococcal infection: serous meningitis
d. Infectious mononucleosis
e. Meningococcal infection: purulent meningitis
498.
Patient L., 38 years old, complaints of attacks of fever with chill and common
sickness. Epemiologically was found that he had recently returned from IndiA.
Leishmania donovani was found under a microscopy. What is the vector of the disease:
a. Bee
b. Pliers
c. Flea
d. *Mosquitoes
e. Fly
499.
Person 16 years entered permanent establishment on 6 day of illness. Illness began
after tick bite. Temperature 37,7 °C. The state became worse: severe head pain, frequent
vomits unconnected with a meal appeared. The common state is heavy. Pupils are
extended, photoharmose is not present. Positive meningeal symptoms. General
hyperesthesiA. Tones of heart are deaf, BP 100/50 mm/hg. What reliable diagnosis?
a. Meningococcal meningitis
b. Influenza
c. *Viral meningoencephalitis
d. Typhoid fever
e. Infectious mononucleosis
500.
Patient, 15 y.o., back from Japane, became ill 3 days ago. Complaints of headache,
temperature of 37,4 °C. After next days: headache increased, a temperature rise to 38,3
°C, repeated vomits. Objectively: a skin is pale, without rash. Mucous membrane of
posterior wall of the throat is normal. Pulse 86, satisfactory filling. Meningeal signs are
positive. What investigation is necessary to perform:
a. Intracutaneous allergic test
b. Biological test on white mice
c. *Cerebrospinal puncture
d. Hemoculture
e. Vidal test
501.
Patient, 16 y.o., during 3 days stood with high temperature at home. 10 days ago
been bited by tick. Next 2 days felt better. A temperature became normal. There was a
chill on a 6th day, a temperature rose to 40 °C again, intensive, quickly increased
headache, during 3 hours – repeated vomiting. The state is severe, consciousness is absent,
psychomotor violations expressed meningeal signs. Meningitis is suspected. What is his
etiology?
a. Tubercular
b. Meningococcal
c. Enteroviral
d. Postinfluenzal
e. *Viral meningoencephalitis
502.
Patient, 23 y.o., suddenly experienced intensive headacke, nausea, pain in a neck
and lumbar areA. Temperature of body 39,2 °C. Expressed meningeal symptoms. Light,
tactile, pain hyperesthesiA. Blood: leucocytes – 17 000 /l, ESR-29 mm/hour. What
method of investigation is most informative?
a. Echoencephalography
b. Transcranial dopplerography
c. Electroencephalography
d. *Lumbar puncture
e. Computer tomography
503.
Sick 15 year old, became ill 3 days ago. Disturbed| headache, temperature of 37,4
°C. During next days: headackeache increased, a temperature had rose to 38,3 °C,
repeated vomiting. Objectively: a skin is pale, without rash. Pulse 86/min satisfactory
filling. Meningeal signs positive. It is needed to conduct for the selection of exciter.
a. Endermic test
b. Bioassay on mice
c. *Bacteriological examination of CSF
d. Widal test
e. Cultivation on bilious clear soup
504.
Sick, 54 years old, hospitalized in an infectious department in a severy condition.
Complaint about the expressed headache, mainly in frontal and temporal areas,
superciliary arcs, origin of vomit on height of pain, pain at motions by eyeballs, in
muscles and joints. Objectively: a patient is excited, temperature of body 39 °C, BP
100/60 mm Hg. Bradycardia changed by tachycardiA. There were tonic cramps. Doubtful
meningeal signs. From anamnesis it is known that at home a woman is ill. What
preparations must be entered?
a. Aspirin, analgin, d, dimedrol
b. Lasix, analgin, ampicillin
c. Mannitol, acetophene, prednisolone
d. *Mannitol, lasix, prednisolone, euphylin, suprastin
e. Veroshpiron, euphylin, prednisolone, dimedrol
505.
Worker, 22 y.o., became ill sharply: t 39°C, severe pain of head, frequent vomits.
Objectively: the condition is severe, psychomotor excitation, moans due to the great pain
of head, expressed rigidity of neck muscles. Sharply positive symptoms of Brudzinsky’s
and Kernig’s, general hyperesthesiA. What’s necessary to do for clarification of
diagnosis?
a. Computer tomography of cerebrum
b. *General blood analysis and liquor analysis
c. X-ray of skull in two projections
d. Analysis of the excrement and urine for a pathological flora
e. General blood analysis and blood sterility
506.
Patient A., 23 years suddenly experienced intensive head pain, nausea, pain in a
neck, and lumbar. Examination is delivered in a clinic. Objectively: temperature 40,2 °C.
Expressed meningeal symptoms. Light, haptic, pain hypesthesiA. Blood: leucocytes – 5
thousand/l, ERS-19 mm/hr. A neurolymph under high pressure, turbid, lymphocytosis,
some increasing of protein, chlorides. What is your diagnosis?
a. Meningococcal meningitis
b. *Viral meningitis
c. Tubercular meningitis
d. Staphylococcus meningitis
e. Pneumoccocal meningitis
507.
In a patient, 35 y.o., came back from China one week ago, sharp beginning of
disease have happened. Hyperthermia to 39,5 °C, headache, which is accompanied with
vomits, myalgias, scleritis; rigidity of cervical muscles, Kernig’s and Brudzinsky’s
symptoms are positive, neutrophilic leukocytosis in common blood analisis. What is most
credible diagnosis?
a. Yersiniosis
b. Leptospirosis
c. *Viral meningoencephalitis
d. Thrombocytopenic purpura
e. Typhoid fever
508.
In permanent establishment on the 2nd day of illness a patient is delivered with the
symptoms of expressed toxicosis. Have been visited Japan. Objectively: the general
condition is severe, hemorrhages in sclera of both eyes. Extremities are cold. AP is 60\20
mmHg, pulse – 120. What is previous diagnosis?
a. Typhoid fever
b. Hemorrhagic fever with a renal syndrome
c. Meningococcemia
d. *Viral meningoencephalitis
e. Crimean hemorrhagic fever
509.
At a patient, 35 years, after the trip to Japan sharp beginning of disease happened.
Body temperature – 39,5 °С, head ache, which is accompanied with vomits, myalgias.
Hemorrhagic rash, that are localized on scleras. Rigidity of cervical muscles, Kernig and
Brudzinsky symptoms are expressed. Spinal liquor without changes. What is most
credible diagnosis?
a. Meningococcal infection
b. Leptospirosis
c. Epidemic typhus
d. Typhoid fever
e. * Viral meningoencephalitis
510.
For the treatment of acidosis in viral meningoencephalitis is better to use.
a. 10 % chloride solution
b. 10-20 % glucose solution
c. *4 % sodium bicarbonate solution
d. Concentrated dry placma
511.
Patient, 35 years was hospitalized with diagnosis localized diphtheria of pharynx.
What is the first dose of antitoxic antidyphtherial serum?
a. *30 000 AU
b. XX.
50 000 AU
c. 80 000 AU
d. 120 000 AU
e. AAA. 150 000 AU
512.
What laboratory examination is compulsory to do for the patient with signs of
tonsillitis?
a. Isolation of hemolytic streptococcus from the throat mucosa
b. Biochemical blood analysis
c. X-ray examination
d. *Cmear from nose and pharynx
e. Immune-enzyme analysis
513.
In preschool the registered case of diphtheriA. What from the measures adopted
below does not conduct to the contact children?
a. *Introduction of antidiphterial whey
b. Non-permanent is stroke from a pharynx and nose for the bacteriologic
examination
c. Daily is supervision during 7 days
d. Determination of titres of specific antibodies
e. At the repeated cases of disease is extraordinary revaccination diphtheria
514.
At a patient the dense darkly-grey raid covers tonsils is considerably megascopic
and spreads for their scopes. Mucus shell bloodshot accented cyanochroic, was
considerably swollen. Immediate medical measure:
a. Ultraviolet irradiation of throat
b. Punction of peritonsillar space
c. Section of peritonsillar space
d. *Introduction of antidiphterial serum
e. Compress on a neck
515.
A boy 6 years was in the close touch with a patient with diphtheriA. What
treatment-prophylactic measures need to be conducted, if vaccine anamnesis is unknown?
a. Introduction of AWDT vaccine
b. Antibacterial therapy
c. Introduction of ADT-м to the toxoid
d. *Antibacterial therapy and double introduction of ADT toxoid
e. Antibacterial therapy and introduction of immunoprotein
516.
At a girl, 22 years old, severy form of diphtheria of otopharynx have happened.
Specific treatment begun only on a 5th day from the beginning of disease. What
complication of diphtheria is potentially dangerous?
a. Stenotic laryngotracheitis
b. Pneumotorax
c. Meningoencephalitis
d. Septicopyemia
e. *Infectious-toxic shock
517.
What remedy is most effective for treatment of dyphtheria should be used
immediately?
a. Antibiotics
b. Oxygenotherapy
c. *Antitoxic antidyphtherial serum
d. Antipyretic drugs
e. Sulfanilamides
518.
At sick L, 35 years old, a diagnosis is set is diphtheria of pharynx,
noncommunicative form. What first dose of antitoxic antidiphtheria whey is it necessary
to appoint?
a. 120 thousand of AО
b. 80 thousand of AО
c. *30 thousand of AО
d. 50 thousand of AО
e. 150 thousand of AО
519.
A child 2 years carries a diphtherial croup. There was the stop of breathing on 2nd
days of whey therapy. What was the reason of asphyxia?
a. *Mechanical obturation by tapes
b. Stenosis of larynx
c. Anaphylaxis shock
d. Whey illness
e. Paresis of respiratory musculature
520.
When patient refered to the doctor with such complaints: prodromal respiratory
illness, sore throat, fever, headache, stiff neck, vomiting, confusion, irritability. What is
previous diagnosis? What main methods can confirm the diagnosis?
a. Epidemic typhys. ELISA-test
b. Hemorrhagic fever. ELISA-test
c. Leptospirosis. Lumbar puncture.
d. Typhoid fever. ELISA-test
e. *Meningococcal infection. Lumbar puncture
521.
A patient is sick with meningococcal meningitis. He take a massive dose of
penicillin. 4 days temperature of body 36,6-36,8 °C. Meningeal signs are negative. When
is it possible to stop the antibiotic therapy.
a. *At a cytosis in a CSF 100 and less, lymphocytes prevail
b. After 10 days from the beginning antibiotic therapy
c. After 7 days from the beginning antibiotic therapy
d. At a cytosis 100 and less, neutrophil prevail
e. From 6 days from the beginning antibiotic
522.
In patients with intensive head acke, nausea, pain in a neck and lumbar area,
expressed meningeal symptoms; light, tactile, pain hyperesthesia what method of
inspection is most informing?
a. *Lumbar puncture
b. Computer tomography
c. Electroencephalography
d. Transcranial dopplerography
e. Echoencephalography
523.
Among the students of school 2 cases of generalized form of meningococcal
infection are registered. What preparation does it follow to enter to the contact persons
with the purpose of urgent prophylaxis?
a. Normal immune globulin
b. Leukocytic interferon
c. *Meningococcal vaccine
d. Meningococcal anatoxin
e. Bacteriophage
524.
In the kindergarden a child had a meningococcal infection. She was immediately
hospitalized. After clinical convalescence in child sowed meningococcus. To which
category can the carrier (child) of pathogen belongs?
a. Convalescent, chronic
b. *Convalescent, acute
c. Healthy
d. Immune in vaccinated
e. Immune in those, that had infection
525.
A patient with meningococcal meningitis gets penicillin during 7 days. The
temperature of body is normal 4 days. Meningeal signs are negative. When is it possible to
stop the antibiotic therapy.
a. In default of leucocytosis displacement in blood
b. *At a cytosis in a neurolymph 100 and less, lymphocyte prevail
c. At a cytosis in a neurolymph 100 and less, neutrophil prevail
d. At a cytosis in a neurolymph 150, lymphocyte prevail
e. At once immediately
526.
At a patient the dense darkly-grey raid covers tonsills is considerably megascopic
and spreads for their scopes. Mucus shell bloodshot accented cyanochroic, was
considerably swollen. Immediate medical measure:
a. *Antidiphterial serum
b. Punction of peritonsillar space
c. Section of peritonsillar space
d. Microscopic research of stroke from under tape
e. Bacteriologic examination of stroke from under pallatum
527.
In an epidemic focusl rationally to organize verification of the state of immunity.
The Use of RIHA allows to find out persons unimmune to diphtheria during a few hours.
What minimum protective titre?
a. 1:10
b. 1:20
c. *1:40
d. 1:80
e. 1:160
528.
At a child 4 years on the third day of disease the widespread form of diphtheria of
nasopharynx is diagnosed. Preparation of specific therapy:
a. Macrolids per os
b. Penicillin i/m
c. Cortycosteroid
d. *Antidiphterial serum i/v
e. Antitoxic therapy
529.
At maintenance of call on a house a district pediatrician put to the sick 5 years old
child diagnosis “Acute lacunar tonsillitis”. Specify, who must carry out the laboratory
inspection of patient and in what terms.
a. Worker of SES upon receipt report
b. A district medical sister is at once after determination of diagnosis
c. Doctor pediatrician in 5 hours
d. *Doctor pediatrician at once after determination of diagnosis
e. District medical sister on a next day
530.
At a patient with meningococcal meningitis 44 years old, rose up general clonictonic cramps, abundant sweat, hyperemia of person, bradycardia quickly changed on
tachycardia, violation of breathing (Cheyne-Stokes type). What complication develop?
a. Infectious toxic shock
b. *Wedging of cerebrum in the cervical channel
c. Encephalitis
d. Swelling and edema of cerebrum
e. Waterhause-Friedrichsen syndrome
531.
At a patient with meningococcal meningitis, in 52, there was the syndrome of
cerebral hypotension on the 6th day of illness. Objectively: expressed toxicosis and
dehydration of organicm, meningial symptoms disappeared, muscles tones was low and
areflexia developed. What complication arose up?
a. * Infectious toxic shock
b. Wedging of cerebrum in the cervical channel
c. Encephalitis
d. D. Swelling and edema of cerebrum
e. Waterhause-Friedrichsen syndrome
532.
Patient D., 30 years, have been examined by emergency doctor and delivered in a
clinic on the 2th day of illness. Fell ill sharply, temperature 40 °С, sharp head pain,
repeated vomits, photophobiA. The common state is severe, consciousness is darkened.
Star-like shape rashes at the skin of abdomen, buttocks and lower extremities appeared.
Sharply expressed rigidity of cervical muscles, positive Kernig symptom been present.
Pulse 100/min, weak, tones of heart seak, BP 60/30 mm/hg. Your diagnosis?
a. * Meningococcal infection
b. Flu, toxic form
c. Marburg hemorragic fever
d. Leptospirosis
e. Hemorrhagic fever
533.
Patient Н., 21 year, entered hospital on the 7th day of disease with complaints of
the expressed weakness, pains in muscles and joints, head pain, nauseA. The disease
began with irritation on the throat, chill, general weakness. After 2 days the state became
worse, temperature increased to 39-40°С. Objectively: patient adynamic, consciousness is
stored. Temperature of body 37,5 °С, the state as severe, pale skin, lips and nail phalanxes
cyantic, hemorrhagic rashes on the skin of trunk and hands, the scopes of heart are
extended to the left on 1,5 cm, tones deaf, pulse 130/min, weak filling, BP 80/40 mm/hg.
Vesicular breathing. Tongue is assessed, moist, phenomena of pharyngitis present.
Inlargement of neck lymphonodes. Meningeal symptoms are not present. OliguriA. What
is previous diagnosis?
a. Hemorrhagic fever
b. Leptospirosis
c. Epidemic typhus
d. *Meningococcal infection
e. Typhoid fever
534.
Patient B., 38 years, delivered by emergency doctor, anamnesis is unknown.
However, according to neighbours, patient 2 days ago was healthy. At a review:
consciousness is absent, motion of left overhead and lower extremities is absent,
increasing of muscular tone is marked in the same extremities. Periodically are clonictonic cramps in right extremities. Expressed asymmetry of person. Sharply expressed
rigidity of muscles of the back of head, positive Кеrnig symptom, positive overhead and
lower Brudzinsky symptom. On the skin of overhead and lower extremities, buttocks,
trunk abundant hemorrhagic rashes “star shape” character with necrosis in a center.
Acrocyanоsis is marked, temperature of body 38,8 °С, hyperemia of pharynx Breathning
36/min, unrhythmical. Vesicular breathing during auscultation. Pulse 72/min, weak filling
and tension. Heart tones are muffled, AP 80/50 mm/hg. Tongue is dry, assessed by the
white raid. Physiology sending in a norm. Your diagnosis?
a. Epidemic typhys
b. Hemorrhagic fever
c. Leptospirosis
d. Typhoid fever
e. *Meningococcal infection
535.
At a patient with meningococcal nasopharynsitis in 2 days from the beginning of
illness the temperature of body rose to 41°С, head pain, vomit, positive Kernig and
Brudzinsky symptoms appeared. In cerebrospinal liquid: cytosis 15 000 in 1 mkl, 90 %
are neutrophils. What diagnosis?
a. Chronic meningococcemia
b. *Meningitis
c. Endocarditis
d. Infectious-toxic shock
e. Meningicm
536.
A patient was delivered in permanent establishment on the 2th day of illness with
the symptoms of expressed toxicosis. Objectively: found out the severe condition,
hemorrhagic rash of wrong form of different size on a skin, hemorrhages in the scleras of
both eyes. Extremities are cold. AP 60/20 mm/hg, pulse – 120 per minute. Previous
diagnosis?
a. *Meningococcemia
b. Hemorrhagic fever with a kidney syndrome
c. Leptospirosis
d. Typhoid fever
e. Pseudotuberculosis
537.
At a patient with nasopharyngitis after 2 days from the beginning of illness the
temperature of body rose up to 41°С, head pain, vomit, positive Kernig and Brudinsky
symptoms appeared. Cerebrospinal liquid: cytosis 15000 in 1 mcl, 90 % are neutrophils.
Diagnosis?
a. Chronic meningococcemia
b. Endocarditis
c. *Meningitis
d. Meningicm
e. Meningococcal nasopharyngitis
538.
A patient sharply had a chill, head pain, vomits, temperature of body rose up to
38,5 °C. Till evening rigidity of muscles, Kernig symptom appeared. Herpetic blisters are
marked on mucouse of lips and nose. Neurological symptoms is not found out expressed.
What disease will you suspect?
a. Subarachnoiditis hemorrhage
b. Herpetic encephalitis
c. Abscess of brain
d. Hemorrhage in a brain
e. *Meningococcal meningitis
539.
Patient A., 23 years suddenly experienced intensive head pain, nausea, pain in a
neck, and lumbar. Examination is delivered in a clinic. Objectively: hemorrhagic rashes
on a body. Temperature 40,2 °C. Expressed meningeal symptoms. Light, haptic, pain
hypesthesiA. Blood: leucocytes – 25 thousand/l, ERS-29 mm/hr. A neurolymph is turbid,
neutrophilic erythrocytosis, meningococci. What is your diagnosis?
a. Staphylococcus meningitis
b. Tubercular meningitis
c. *Meningococcal meningitis
d. Viral meningitis
e. Pneumoccocal meningitis
540.
Person 16 years entered permanent establishment on 6 day of illness. Illness began
from a cold and cough. Temperature 37,7 °C. The state became worse: severe head pain,
frequent vomits unconnected with a meal appeared. The common state is heavy. Pupils are
extended, photoharmose is not present. Positive meningeal symptoms. General
hyperesthesiA. Tones of heart are deaf, BP 100/50 mm/hg. What reliable diagnosis?
a. Infectious mononucleosis
b. *Meningococcal meningitis
c. Toxic food-born infection
d. Influenza
e. Typhoid fever
541.
At a patient, 35 years, acute beginning of disease happened. Symptoms of
nasopharyngitis. Body temperature – 39,5 °С, head ache, which is accompanied with
vomits, myalgias. Hemorrhagic rash, that are localized on scleras, brushes, feet, buttocks.
Rigidity of cervical muscles, Kernig and Brudzinsky symptoms are expressed.
Neutrophilic leucocytosis in common blood analisis.What is most credible diagnosis?
a. *Meningococcal infection
b. Typhoid fever
c. Yersiniosis
d. Leptospirosis
e. Epidemic typhus
542.
A 17 years old boy, got ill suddenly: the temperature rose up-to 40,3 °С, extremely
sharp headache, makes him to yell. The patient is agitated and vomits frequently. Tremor
of fingers of extremities is observed. The star like hemorrhagic rash of different form and
sizes appeared, mainly on buttocks, thighs, shins, and trunk. Meningeal signs are positive.
What is the most credible diagnosis?
a. Encephalitis
b. Flu with a hemorrhagic syndrome
c. *Meningococcal infection
d. Measles
e. Leptospirosis
543.
Patient S., 20 years old refered to the doctor with such complaints: prodromal
respiratory illness, sore throat, fever, headache, stiff neck, vomiting, confusion, irritability.
Previous diagnosis. What main methods can confirm the diagnosis?
a. Epidemic typhys. ELISA-test
b. Hemorrhagic fever. ELISA-test
c. Leptospirosis. Lumbar puncture.
d. Typhoid fever. ELISA-test
e. *Meningococcal infection. Lumbar puncture
544.
Sick, 54, hospitalized in an infectious department in a severy condition. Complaint
about the expressed headache, mainly in frontal and temporal areas, superciliary arcs,
origin of vomit on height of pain, pain at motions by eyeballs, in muscles and joints.
Objectively: a patient is excited, temperature of body 39 °C, BP 100/60 mm Hg.
Bradycardia changed by tachycardiA. There were tonic cramps. Doubtful meningeal
signs. From anamnesis it is known that at home a woman is ill. What preparations must be
entered?
a. *Mannitol, lasix, prednisolone, euphylin, suprastin
b. Mannitol, acetophene
c. Lasix, analgin, ampicillin
d. Veroshpiron, euphylin, dimedrol
e. Aspirin, analgin, dimedrol
545.
A patient, 20 years old, during few days complaints of pharyngalgias. After
supercooling the state became worse : sudden chills, increase of temperature to 40,6 °C,
headache. On skin of lower extremities trunk and buttocks there are a lot of different sizes
of hemorragic spots, acrocyanosis. Consiouness is preserved. Meningeal signs are absent.
What is the previous diagnosis?
a. *Meningococcal infection
b. Flu
c. Epidemic typhus
d. Hemorrhagic fever
e. Leptospirosis
546.
A patient C., 25 years old, fell suddenly ill. Every morning severe headache,
frequent vomiting, temperature of the body is 39,9 °C. Adopted fatigue, then state got
much worse. In the evening lost of consciousness. Expressed muscles pains of back and
head. Positive Кеrning’s symptom. Leukocytes – 18,0*109. What is the most reliable
diagnosis?
a. Flu
b. Epidemic typhus, typhus state
c. Viral menigoencephalitis
d. Sepsis, infectious-toxic shock
e. *Bacterial menigoencephalitis
547.
A patient 17 years old, became suddenly ill: .Temperature rose to 40,3 °C. Severe
headache, motive excitation, frequent vomiting, tremor of fingers of extremities.
Hemorrhagic spots of round form and different sizes, more frequently as stars, mainly on
buttocks and trunk. Meningeal signs are positive. What is the most possible diagnosis?
a. Encephalitis
b. Flu with a hemorrhagic syndrome
c. *Meningococcal infection
d. Measles
e. Leptospirosis
548.
In permanent establishment on the 2nd day of illness a patient is delivered with the
symptoms of expressed toxicosis. Objectively: the general condition is severe, on skin
hemorrhagic rash of different form and sizes, hemorrhages in sclera of both eyes.
Extremities cold. AP is 60\20 mmHg., pulse – 120. Previous diagnosis?
a. *Meningococcemia
b. Crimean hemorrhagic fever
c. Hemorrhagic fever with a renal syndrome
d. Typhoid fever
e. ECНО-eczanthema
549.
Patient, 16 y.o., during 3 days there was a increase of temperature to 38 °C,
першіння in a throat, indisposition. Next 2 days felt better. A temperature was normal.
There was a chill on a 6th day, a temperature rose to 40 °C, intensive, quickly increased
headache, through 3 hours – repeated vomits. The state is heavy, consciousness is absent,
psychomotor violations expressed meningeal signs. Meningitis is suspected. What is his
etiology?
a. Tubercular
b. Enteroviral
c. Postinfluenzal
d. * Meningococcal
e. Lymphocytic choreomeningitis
550.
Patient D., 30 y.o., teacher, examined by the doctor of first-aid and delivered to a
clinic on the 2nd day of illness. Fell ill sharply, temperature 40 °C, sharp head pain,
repeated vomits photophobiA. The general condition is severe, consciousness is darkened.
On the skin of abdomen, buttocks and lower extremities is зірчаста rash. Sharply
expressed rigidity of neck muscles, positive Kernig symptom. Pulse 100 min, weak; tones
of heart deaf, AP 60/30 mmHg. Your diagnosis?
a. Flu, toxic form
b. Crimean hemorrhagic fever
c. Disease of blood
d. Hemorrhagic stroke
e. *Meningococcal infection, meningococcemia
551.
Worker, 22 y.o., became ill acutely: t 39°C, severy headacke, frequent vomits.
Objectively: the condition is severe, psychomotor excitation, moans due to the great pain
of head, expressed rigidity of neck muscles. Sharply positive symptoms of Brudzinsky’s
and Kernig’s, general hyperesthesiA. What needs to be done for clarification of diagnosis?
a. * General blood analysis and liquor analysis
b. General blood analysis and blood sterility
c. Computer tomography of cerebrum
d. X-ray of skull in two projections
e. Analysis of the excrement and urine for a pathological flora
552.
A patient with meningococcal meningitis gets penicillin during 7 days. Last 4 days
temperature of body is normal. Meningeal signs are absent. When is it possible to abolish
an antibiotic?
a. *At cytosis in liquor 100 and less, lymphocytes prevails
b. At absence of leukocytosis and stab-nucleus shift in a blood
c. At cytosis in liquor 100 and more less, neutrophils prevails
d. At cytosis in liquor 150, lymphocytes prevails
e. At once
553.
Among the students of university 2 cases of generalized form of meningococcal
infection are registered. What preparation does it follow to enter to the contact persons
with the purpose of urgent prophylaxis?
a. Normal immunoglobulin
b. Leukocytic interferon
c. *Meningococcal vaccine
d. Meningococcal anatoxin
e. Bacteriophage
554.
Patient K., 21 y.o.., the disease began from the increase of temperature to 39,0 °C,
headache, chill, repeated vomit. Objectively: temperature of 39,3 °C, pulse 76. Rigidity of
muscles of the back of head. The tendon reflexes are expressed. Analysis of liquor: cytosis
1237 in 1 ml, from them: 84 % neutrophiles, 16 % lymphocytes, reaction to Panda ++,
albumen 0,66 g/l, liquid is turbid, flows out under the promoted pressure. Bacterioscopy
found out in liquor gram(-) cooks morphologically similar with meningococcus. What
disease is most reliable?
a. Serous meningitis
b. Infectious mononucleosis
c. *Meningococcal meningitis
d. Tuberculosis meningitis
e. Purulent meningitis
555.
In the kindergarden a child had a meningococcal infection. She was immediately
hospitalized. After clinical convalescence in child sowed meningococcus. To which
category can the carrier (child) of pathogen belongs?
a. Convalescent, chronic
b. *Convalescent, acute
c. Healthy
d. Immune in vaccinated
e. Immune in those, that had infection
556.
In a patient, 27 y.o., on the 5th day of respirator disease, there was sharp head pain,
that was accompanied by nausea, repeated vomits, hyperesthesia, photophobiA. At a
review: lies with the neglected head, feet are bended to the trunk, expressed rigidity of
muscles of the back of head, positive symptoms of Kerning’s, Brodzinsky’s. It is not
found out paresises. Select a basic neurological syndrome.
a. Meningeal syndrome
b. *Syndrome of liquor hypertension
c. Syndrome of liquor hypotension
d. Root syndrome
e. Vegetative crisis
557.
Patient, 23 y.o., suddenly experienced intensive head pain, nausea, pain in a neck
and lumbar areA. He was delivered to the clinic. Objectively: on a body hemorrhagic
rash. Temperature of body 39,2°C. Expressed meningeal symptoms. Light, tactile, pain
hyperesthesiA. Blood: leucocytes-25x10 9/l, ESR-29 mm/hour. What method of
inspection is most informing?
a. *Lumbar puncture
b. Computer tomography
c. Electroencephalography
d. Transcranial dopplerography
e. Echoencephalography
558.
Patient, 15 y.o., became ill 3 days ago. Complaints of headache, running nose, pain
in a throat, temperature of 37,4°C. Next days : headache increased, a temperature risen to
38,3°C, repeated vomits. Objectively: a skin is pale, without rash. Mucous membrane of
posterior wall of throat is moderately hyperemic, swollen. Pulse 86, satisfactory filling.
Meningeal signs are positive. What is needed to conduct the selection of pathogen:
a. * Bacteriological research of liquor
b. Biological test on mice
c. Intracutaneous test
d. Reaction of Vidala’s
e. Cmear of the blood on bilious bullion
559.
In a patient, 35 y.o., sharp beginning of disease, preceding nasopharyngitis, t°
39,5°C, headache, which is accompanied with vomits, myalgias, hemorrhagic rash, that is
localized on sclera, fists, feet, buttocks; rigidity of cervical muscles, symptoms of
Kering’s and Brudzinsky’s are positive, expressed neutrophilic leukocytosis. What is most
credible diagnosis?
a. Typhoid fever
b. Yersiniosis
c. Leptospirosis
d. *Meningococcal infection
e. Thrombocytopenic purpura
560.
Sick C., 8 years, appealed to the infectious hospital on the second day of disease
with complaints of a pharyngalgia at swallowing, increase of temperature. Objectively:
temperature 38,6 °C, hyperemia of soft palate, tonsills, filling out, loose, on both there are
festering stratifications which are taken off by a spatulA. Megascopic sickly
submandibular lymphonoduses are palpated. Pulse – 114 per a min. Present roseol-papular
pouring out on all body. Pastia‘s symptom is positive. It is known from epidemogical
anamnesis, that its comrade had alike symptoms. It is not found out the change from the
side of other organs. What previous diagnosis can be suspected?
a. Lacunar quinsy
b. Diphtheria of pharynx
c. *Scarlet fever
d. Flu
e. Infectious mononucleosis
561.
A 4 years old child complaints of: cough, temperature of body 38,1 °C. Skin
without rashes. Conjunctiva hyperemic. Exanthema on skin. On mucous of cheeks there
are points of hyperemia gum blushs. In lungs difficult breathing. What is the most
possible diagnosis?
a. Scarlet fever
b. Rubella
c. *Measles
d. Enteroviral infection
e. Flu
562.
Sick C., 8 years, appealed to the infectious hospital on the second day of disease
with complaints of a pharyngalgia at swallowing, increase of temperature. Objectively:
temperature of body – 38,6 °C, sharp hyperemia of soft palate, tonsills, filling out, loose,
on both there are festering stratifications which are taken off by a spatulA. Palpated
megascopic, sickly submandibular lymphonoduses. Pulse – 114 per min. Present roseolpapular pouring out on all body. Pastia symptom is positive. It is known from epid
anamnesis, that at its comrade were alike symptomes. It is not found out the change from
the side of other organs. What previous diagnosis can be suspected?
a. Lacunar tonsillitis
b. Diphtheria of pharynx
c. Infectious mononucleosis
d. Flu
e. *Scarlet fever
563.
Patient A. , 27 years old, was admited on the 4th day of illness with the diagnosis
of viral ARVI, and allergic dermatitis. The patient felt ill suddenly with increase of
temperature up-to 38 °C, headache, sore throat, and intensive cough. On the 3d day rash
appeared on the skin of neck and face. Patient was taking aspirin in order to decrease the
temperature. Objectively: temperature of body 38,8 °C. The face of patient looks puffy.
Signs of conjunctivitis, and renitis were observed. On the skin of neck face and chest there
was intensive papular rash, without itch. Mucous membrane of oropharynx was brightly
hyperemic. Submandibular and frontal neck lymph nodes were enlarged. The liver and
spleen were not palpable. What is the diagnosis?
a. *Measles
b. Allergic dermatitis
c. Infectious mononucleosis
d. Rubella
e. Scarlet fever
564.
A child 10 years old presents with temperature 38 °C, renitis, conjunctivitis, moist
cough. On the mucous membrane of cheeks, lips, gums there are greyish-white points,
reminding a farinA. What is the diagnosis?
a. *Measles
b. Adenoviral infection
c. URTI
d. Enteroviral infection
e. Infectious mononucleosis
565.
In a newborn, the septic state was accompanied with the increase of temperature to
40 oC which developed on 5th day, with pneumonia expressed intoxication: pallor,
vomiting, shortness of breath, disturbance, cramps. Appeared rash (blisters with
hemorrhages) on skin, mucous membrane of mouth cavity, throat and conjuctivA. The
child suffers with congenital herpes. Lungs: difficult breathing and vesicular wheezing.
Mild enlargement of liver. What is the most reliable diagnosis?
a. *Herpetic infection
b. Cytomegaloviral infection
c. Chicken pox
d. Rubella
e. AIDS
566.
A new born child on 10th day of life became worse: Temperature 39.2 °C, no
frequent vomiting, generalized cramps, violations of consciousness, spastic paresis of left
extremities. Month prior to his birth herpes virus was present in the mother, which she did
not treat. What disease is most possible?
a. *Herpetic encephalitis
b. Meningococcal meningitis
c. Subarechnoid hemorrhage
d. Cerebral abscess
e. Violation of blood cerebral circulation
567.
A patient has herpetic meningitis. What preparation of specific therapy for viral
neuro infection should be given?
a. *Acyclovir
b. Cefataxime
c. Ceftriaxone
d. Gentamycin
e. Furazolidon
568.
A patient, 22 years old, became ill sharply. History showed fever up to 38.2 °C
with headache, repeated vomiting, olfactory and tastes hallucinations. Quickly got
complex of meningeal symptoms, pyramidal paresis. The general epileptic attack and
comatose state also developed. Neurolymph is with mixed lymphocytosis, cytochrome,
single red corpuscles. What is previous diagnosis?
a. Brain abscess
b. Subdural empyema
c. *Herpetic encephalitis
d. Tumor of brain
e. Encephalopathy
569.
A boy 11 years old, complaints of sickness at mastication, increasing of
temperature to 37.1 °C, enlargment of parotid salivary glands . At the age of 8 years
carried a paraflu infection. Objectively: in the region of right parotid salivary gland
tubular sickly at palpation, a skin above it is not changed. A pharynx is moderate
hyperemic, tonsils are not coated. What is your previous diagnosis?
a. *Cytomegaloviral syaloadenitis
b. Lymphadenitis
c. Parotitis
d. Infectious mononucleosis
e. Cholylithiasis
570.
At junior nurse, who works in child’s infectious department, herpes simplex was
found. What should manager of department must do?
a. *Create a quarantine in the department
b. To appoint an immunoprotein to the children
c. Discharge all children from the department
d. To appoint immunomodulators with a prophylactic purpose
e. To inspect a junior nurse on a staphylococcus
571.
A woman 65 years old the disease had beginning sharply from increase of
temperature to 39.0 °C, weakness, and pain in the left part of thorax that increased with
breathing motions. On 3rd day of disease vesicular breathing appeared after motion of rib
on the left on a hyperemic background. Together with sick a grandchild lives 4 years.
What measures of prophylaxis of disease need to be adopted?
a. Vaccination
b. Final disinfection
c. Reception of specific immunoprotein
d. *Isolation of patient
e. Acyclovir administration
572.
Patient 60 years old complain of pains in right hand which last for 2 days. On the
3rd day appeared vesicular chain-like rash on the skin of arm, forearm and fist. Sensitivity
in the rash area is decreased. Which disease can be diagnosed?
a. Dermatitis
b. *Herpetic ganglionitis
c. Neck-thorax radiculitis
d. Psoriasis
e. Allergy
573.
A patient 60 years old for 2 days has disturbed severe pain in a right arm. On 3rd
day appeared blisters, pouring out as a chain on the skin of shoulder, forearm and brush.
Sensitiveness in the area of pouring out is mionectic. What disease can be diagnosed?
a. Dermatitis
b. *Herpetic ganglionitis
c. Neck-pectoral redicals
d. Psoriasis
e. Allergy
574.
The patient, 58 years old, was hospitalised in the infectious department with
complaints of pain in the left half of thorax, fever. At a review: temperature of body 37.5
°C, in XI-XII intercostal area the grouped shallow blisters on a hyperemic-filling
background are filled by transparent maintenance. Preparation of choice for treatment of
this patient is:
a. Suprastin
b. Prednisolone
c. Biseptolum-480
d. Semavin
e. *Laferon
575.
Patient C., 60 years old during one year has 4th relapse of Herpes zoster.
Recommended treatment and relapses prophylaxis?
a. *Valcyclovir
b. Acyclovir
c. Herpevir
d. Proteflazid
e. Cycloferon
576.
A pregnant woman, 27 years (pregnancy ІІ, 8-10 weeks.), temperature of body
increased. At the inspection on a TORCH-infection antibodies are found to the herpes
virus, ІІ types of class IGM. What we must recommend to pregnant women?
a. *To cut pregnancy
b. To prolong the supervision
c. Treatment with acyclovir
d. Symptomatic treatment
e. Appoint of alpha-fetoprotein
577.
Patient, 25 years old, came to the physician on the 3rd day of disease with
complaints of rash on the mouth, external nose and ears, which are itching and painful.
Objectively: T-37,7 °C. On the not changed skin of mouth, nose and ear auricle there are
vesicles with size of 1-2 mm grouped localization. About what disease you can think?
a. Erysipelas, bulbous form
b. Anthrax
c. *Herpetic infection
d. Eczema
e. Streptodermic infection
578.
Sick patient, 25 years old, was consulted by a doctor on the third day of illness
with complaints of pouring out on lips, wings of nose and ears, pain and swelling in the
places of these pouring out. Objectively: temperature of body – 37.7 °C, unchanged skin
of overhead lip, wings of nose, auricles, there is vesicular scars 1-2 mm with the group
location. What disease you may think about?
a. Erysipelas, bullous form
b. Anthrax
c. *Herpetic infection
d. Anthrax, skin form
e. Chicken pox
579.
Sick A., complaints of turning red rash and edema on a right cheek. During a
review: temperature of body – 38.7 °C, submandibular lymph nodes enlarged and painful,
border between turning red and healthy skin is clear, there are blisters with a dark liquid
inwardly, palpation is painful. What is your previous diagnosis?
a. *Erysipelas, hemorrhagic form
b. Anthrax, skin form
c. Herpetic infection
d. Chicken pox
e. Phlegmon of cheek
580.
In patient R., 34 years old with general intoxication and increased body
temperature up to 38 °C, appear pain in region of right auditory canal and next day
distortion of face to the left side. Review: softening of frontal and nasal-mouth skin folds,
right eyelid cleft is wider than left, mouth cavity turned left, right eyebrow doesn’t move
upwards and cheek is strenght. Right xerophthalmia, xerotomia and disordered taste
sensation on the surface of right anterior 2/3 half of tongue. Herpetic vesicles in right
exterior auditory canal and auricle. What is the most possible diagnosis?
a. Rossolimo-Melkerson-Rozental’s syndrome right side
b. Postherpetic neuralgia of I-st and II-nd branch of right trigeminal nerve
c. Postherpetic neuralgia of all 3 branches of right trigeminal nerve
d. Sluder’s syndrome right side
e. *Hant’s syndrome
581.
A 32 years old patient appealed to a doctor on a background of fever and increase
of temperature to 38.0 °C, pain in the right ear area, left side “deviation” appear during the
next day. On examination: right side cmoothed out frontal and occipital skinning folds,
right eyelids fissure is wider than left, a mouth is overtighten to the left, a right eyebrow
does not rise upwards, and a cheek “hyperemia”, xerostomia and violation of taste
receptors on front 2/3 right halves of tongue. Herpetic blisters in right external auditory
canal and auricle. What is the most possible diagnosis?
a. *Hunt’s syndrome
b. Post herpetic neuralgia of the I- and II- branches of right trifacial nerve
c. Post herpetic neuralgia of all branches of right trifacial nerve
d. Sludder’s syndrome
e. Rosolimo-Меlkerson-Rozantalia syndrome
582.
A 40 years old scientist, became ill sharply with chill, temperature of body – 39.8
°C, severe headache, vomiting, pain in muscles. Did not went to the doctor, the state had
become worse, hyperemia of sclera, appeared, on lips herpes with hemorrhagic
maintenance, the nose-bleeding, skin and sclera, became icteric, urine color is of strong
tea, diuresis 200 ml, an anacholia was not present. What is the most reliable source of
infection?
a. *Rats
b. Cats
c. Infected people
d. Bacillocarrier
e. Mosquito
583.
A patient came with complaints of erosions of his penis. From anamnesis frequent
appearance of similar rashes during a year is found out. Objectively: on a balanus are the
grouped blisters and erosions of polycyclic outlines, with clear margin, soft during
palpation. What is your diagnosis?
a. *Recurrent congenital herpes
b. Pemphigus vulgaris
c. Primary syphillis
d. Pyodermia
e. Scabies
584.
Sick 65 years old patient, complaints of pain in a subscapular region. Objectively:
on a skin surface of subscapular region the placed arcwise rose-red filling out hearths
some infiltrative, with clear scopes was present. On-the-spot hearths grouped vesicles with
transparent maintenance. What is the diagnosis?
a. Impetigo
b. Herpes simplex
c. Erysipelas
d. Allergodermia
e. *Herpes zoster
585.
A sick person, 65 years old, complaints of rash, pain in a subscapular region.
Objectively: on a skin surface of the subscapular region present the arcwise rose-red
filling out hearths some infiltrative, with clear scopes. On-the-spot hearths grouped
vesicles with transparent maintenance. What preparation he should take?
a. Suprastyn
b. Prednisolone
c. Biseptolum-480
d. Loratidin
e. *Laferon
586.
A patient N., 45 years old, complaints of headache, general weakness increasing of
temperature, to 37.4 °C. In 2 days pain appeared in the pectoral region of spine with an
irradiation in a right between scapular regions. After some time skin in this region turned
red as a strip from a spine to the subarmpit line, and in 2 days red knots which through the
set time grew into blisters with transparent maintenance appeared in this place. What is
your diagnosis?
a. *Herpes zoster
b. Thoracal rediculopathy
c. Neurology of intercostal nerves
d. Neurology of superscapular nerve
e. Herpetiform dermatitis
587.
A 37 years old patient, 2 days ago a spot on a hand appeared, which for days grew
into pustule with a black bottom, painless at touch, with the crown of daughters vesicles
on periphery. There are painless edema on a hand and shoulder. Temperature rise to 39.0
°C, dizziness appeared. Pulse – 100 beats per min, AP – 95/60 mm Hg. BR – 30 per a
minute. What is the most possible diagnosis?
a. *Anthrax
b. Plague
c. Tularemia
d. Brucellosis
e. Herpes
588.
A 70 years old sick person, after supercooling severy pain in the left half of head in
the area of forehead and left eye appeared. 3 days ago the temperature of body increased
to 37,6 °C, the blister of pouring out at the head and left overhead eyelid appeared. What
disease can be diagnosed?
a. *Herpetic ganglionitis
b. Encephalitis
c. Allergy
d. Dermatitis
e. Trifacial neuritis
589.
A sick 3 years old child came to the doctor with symptoms of the fever, languor,
waiver of meal. A boy is capricious, temperature of body 37.9 °C. On the mucus shell of
soft palate, cheeks are single vesicle hypersalivation. What is the diagnosis?
a. *Herpetic stomatitis
b. Candidosis of oral cavity
c. Leucoplacia
d. Follicular tonsillitis
e. Lacunar tonsillitis
590.
Sick, 49 years old, came to the doctor with complaints of pain. On the 3rd day he
became ill, first marked heartburn and pain in thorax, yesterday is pouring out. At
examination: temperature of body 37,8 °C, after motion of the V-VI intercostals intervals
on a hyperemic skin group of the blisters filled by transparent maintenance. What is the
diagnosis?
a. *Herpes zoster
b. Chicken pox
c. Erysipelas, erythematous-bulous form
d. Allergic dermatitis
e. Myositis
591.
Patient K., 56 years old, during last 5 days has Herpes zoster with localization of
the process on the right cheek and paraauricular region. Treating with herpevir. Today
pain in the right eye appeared. During examination present of edema of eyelid and
hyperemia of conjunctivA. What is the action of a physician?
a. Prescribing eye drops (оphtan, аlbucid and other)
b. Prolonged treatment with herpevir
c. *Immediately consultation of ophthalmologist
d. Intensify treatment with antiviral drugs
e. Prescribe warm compress
592.
To the infectious diseases department was admitted patient M. 58 years old, with
complaints of pain in left part of thorax, fever. During a review: body temperature 37,5
°C, on the level of XI–XII intercostals spaces on hyperemic-edemous shadow grouped
cmall vesicles with transparent content. Preparation of choice for treatment of this patient?
a. Suprastyn
b. Prednisolon
c. Biseptol-480
d. Cymeven
e. *Laferon
593.
A patient appealed to the doctor with complaints of difficulties in opening of the
mouth. Two weeks ago fell down and head was hurt, did not have medication. At a review
mouth opens on 1.5 cm, moderate expressed pain of muscles at the back of head. The
paralysis of muscles of person, eyeballs, are more prominent. In a temporal area dry
bloody crusts in the place of traumA. What are your diagnosis?
a. Neuritis of facial nerve
b. Throat abscess
c. *Facial paralytic stupor of Rоsе
d. Bulbar encephalitis
e. Herpetic ganglionitis of knot of trifacial
594.
A patient came with complaints of sickly erosions on his penis. From anamnesis
frequent appearance of similar rashes is found out during a year. Objectively: on a balanus
are the grouped blisters and erosions, soft on palpation. What is your diagnose?
a. *Recurrent herpes of ІІ type
b. Vulvar pemphigus
c. Primary syphyllis
d. Shankoform pyoderma
e. Recurrent herpes of ІІІ type
595.
Patient A., complaints of redness of the skin and edema on the right cheek. During
a review: body temperature 38,7 °C, enlarged and painful right submandibular lymphatic
nodes, the border between red and normal skin is sharp, present vesicles with dark
content, palpation is painful. Your primary diagnosis?
a. *Erysipelas, hemorrhagic form
b. Anthrax, skin form
c. Herpetic infection
d. Varicella (chickenpox)
e. Phlegmone of the cheek
596.
At a patient G., 41 years old, was a high temperature during 8 days, severe
headache, constipation. Objectively: temperature of body 39,5 °C, pale, languid. Pulse 82
per a min, a tongue is dry, assessed by the brown coat. An abdomen is moderato
exaggerated, painful in a right ileac areA. A liver + 2 cm. What is the most credible
diagnosis?
a. *Typhoid fever
b. Epidemic typhus
c. Flu
d. Appendicitis
e. Yersiniosis
597.
Patient L., 20 years old, had appendectomy on the 5th day of disease. During
operation was found an appendix with the signs of catarrhal inflammation and hyperplasia
of lymphatic nodes of mesentery. After a day his state became severe: temperature – 40
°C, with hallucinations, hepatosplenomegaly. Single roseols elements appeared on the
skin of abdominal on a 10th day. The tongue is great, covered with grey coating, with the
imprints of teeth, wound is in normal. In the analysis of blood is leucopenia, relative
lympho- and monocytosis. In anamnesis was contact with a patient with typhoid fever.
What is the most credible diagnosis?
a. *Typhoid fever, atypical form: appendicotyphoid
b. Epidemic typhus
c. Yersiniosis
d. Pseudotuberculosis
e. Flu
598.
Patient L., 20 years old, had appendectomy on the 5th day of disease. During
operation was found an appendix with the signs of catarrhal inflammation and hyperplasia
of lymphatic nodes of mesentery. After a day his state became severe: temperature – 40
°C, with hallucinations, hepatosplenomegaly. Single roseols elements appeared on the
skin of abdominal on a 10th day. The tongue is great, covered with grey coating, with the
imprints of teeth, wound is in normal. In the analysis of blood is leucopenia, relative
lympho- and monocytosis. In anamnesis was contact with a patient with typhoid fever. By
which method is it possible to diagnostic this disease?
a. Urinocultura
b. Coproculture
c. Bilicultura
d. *Hemocultura
e. Reaction of Vidal
599.
Pain in abdomen appeared at a patient with typhoid fever on the 19th day of
disease, and was during 4 hours. Pulse – 100 per a min, rhythmic. A tongue is dry,
assessed by wait coat. The abdomen is tense, does not take part in the act of breathing.
Stool and urine was absent. About what complication is it necessary to think?
a. Appendicitis
b. Infectiously-toxic shock
c. Bleeding
d. *Perforation
e. Urolithiasis
600.
At the patient B., 25 years old, was diagnosed typhoid fever. On the 17th day of
disease the temperature of body critically went down to the norm, a pallor color of skin
increased. Consciousness is stored. Pulse 120 per a min, rhythmic. On the top of heart is
systolic noise. Constipation. About what complication is it necessary to think?
a. *Bleeding
b. Perforation
c. Infectious-toxic shock
d. Infectious-allergic myocarditis
e. Pneumonia
601.
A sick person, 23 years old, appealed to the hospital on 6th day with gradual
development of illness and complaints of severe headache, pain at the back of his head,
sleep disturbance and fever. Objectively: Т-39,7 °C, Рs-84/min. His face is pale, tongue is
dry, near the root covered by a grey covering. The abdomen is flatulent. A percutory
sound is heard in the right iliac areA. The liver and spleen are enlarged. Which day does
rash appear on the skin in this illness?
a. On 12 day
b. On 4 days
c. On 5 day
d. On 6 day
e. *On 8 day
602.
In a 42 y.o. annual explorers the temperature of body rose to 39 °C. At reception
the patient is pale, tongue is edematous and covered by a dirty-brown raid, with clean
edges and tag, and on the sides imprints of teeth, protrusion of the tongue is impaired and
he trembles finely, he has an ammonia breath. On anterior surface of the abdomen are
found monomorphic single roseollas. In his mouth cavity are hyperplasic lymphatic
follicles of soft palate especially in front, appeared symmetric flat superficial oval form of
ulcer, diameter up to 5 mm. Such ulcers are on tonsils. In case of such disease the
convalescents are discharge after:
a. Triple bacteriological research of blood, excrement, urine, bile
b. *Triple bacteriological research of excrement, urines and once of the bile
c. Triple bacteriological research of excrement, urines and once of the blood
d. Triple bacteriological research of excrement, urine, bile
e. Triple bacteriological research of blood, urine, bile and once excrement
603.
To the reception of the infectious hospital a patient came with complaints of
high fever – 38-40 °C In 3 weeks, headache, weakness and insomniA. She didn’t get a
doctors consultation She took antipyretic drugs, seduxen. Objectively: Т 35,7 °C, Рs –
140/min., BP-80/50 mm Hg. Her general condition is severe. Skin and mucous
membranes are pale. The tongue is thickened with the imprints of teeth, with a dirtybrown covering, apex and edges of the tongue are clean. her abdomen is flatulent. her
liver and spleen are moderately enlarged. Her stool on admission was mixed with fresh
blood. Why was there a decrease of temperature and an increase in pulse?
a. Poisoning by the drugs
b. Infectious-toxic shock
c. Hemorrhoid bleeding
d. *Intestinal bleeding
e. Endometriosis of colon
604.
Patient Н., 28 years old, came to the clinic on the ninth day of illness with
complaints of increased Т to 39,0 °C, headache, general weakness, delay of voiding, sleep
disturbance. At inspection: the skin of abdomen had single roseollas, the tongue is
covered with a brown covering, Ps. 78 beats per min. and rhythmic, the liver is enlarged
to 2сm. What changes will be seen in the cardio-vascular system?
a. Bradycardia, dicrotic pulse, muffling of cardiac tones, hypotension
b. Tachycardia, dicrotic pulse, muffling of cardiac tones, hypotension
c. Tachcardia, dicrotic pulse, muffling of cardiac tones, hypertension
d. *Relative bradycardia, dicrotic pulse, muffling of cardiac tones, hypotension
e. Dicrotic pulse, muffling of cardiac tones, hypotension
605.
A 30 y/o patient is seen on the 9th day of illness. The symptoms of illness has
been building –up gradually with increase in temperature and intoxication. Roseolar rash
has appeared on his abdomen. His skin is pale, temperature is 40 °C, pulse 80 /1 min., BP
100/65 mmHg. His tongue is covered with sediments and his abdomen is swollen. Spleen
and liver are palpable. What symptom will be positive for this patient?
a. Symptom of Botkin
b. *Symptom of Padalka
c. Symptom of Kyl'dyshevsky
d. Symptom of Ortner
e. Symptom of Pasternacky
606.
A sick woman, 32 years, complaints of diarrhea, headache, severe weakness,
insomnia and a dull pain in her right iliac areA. It is the 8th day of her illness. At a
review: Т-39,8 °C, Рs – 86/min., AP – 90/60 mm Hg. Pulse is dictoric. Skin is pale. Single
roseollas are on the abdomen. Tongue is dry, assessed by the coverings, with the imprints
of teeth on a lateral surface. Soft, dulling of percutory sound is observed in the illeocaecal
area of her abdomen. Hepatosplenomegaly, positive Blumberg’s symptom, neutrophilic
leukocytosis. What changes will be at roentgenologic examination?
a. No changes
b. *Presence of air under a diaphragm
c. Presence of the exaggerated loops of intestine
d. Enlarged liver and spleen
e. Signs of impassability of intestine
607.
Patient with the diagnosis “typhoid fever” is hospitalized in an infectious
hospital. He lives in an isolated apartment with a woman and two children. What
preparations is the mean defense against typhoid fever for people in contact with the
patient?
a. Vaccine
b.
c.
d.
e.
Antibiotic
Immunoglobulin
Antitoxin
*Bacteriophage
608.
A district doctor suspected typhoid fever in a patient M., 15 y.o. To make the
patient safe as the sources of infection, which of the following measures should be taken?
except:
a. *Introduction to the patient of antityphoid monovaccine
b. Hospitalization in the infectious department during 3-6 hours
c. Introduction etiotropic antibacterial treatment
d. Conducting of controls bacteriological researches of excrement and urine (triply)
and bile before the discharge
e. Clinical supervision after the discharge during 3 months
609.
Patient 24 y.o., was hospitalized in the infectious department on the 10th day of
disease with complaints of general weakness, headache, poor appetite and cough.
Objectively: body temperature is 39,5 °C, pallor of skin. Adynamicm. Single roseolas are
present on the skin of anterior wall of the abdomen, thorax. Liver is palpable + 1 cm
below the rib angle on the midclavicular line, spleen is not significantly palpated. What is
the most probable diagnosis?
a. *Typhoid fever
b. Flu
c. Typhoid rash
d. Brucellosis
e. Pneumonia
610.
A patient A., 43 years old, is ill for 2 weeks. The disease started as an increase
in temperature to 37,2 °C, headache, decline of appetite and weakness. Then the
temperature got to 39-40 °C. Objectively: his condition is severe, he responds slowly to
questions. AdynamiA. Pulse 80 in a min. BP is 100/60 mmHg. An abdomen is painless,
flatulence, hepatosplenomegaly. Stool is of green color. How long is it necessary to look
after people who were in contact with this patient?
a. 35 days
b. 1 month
c. *21 days
d. 1 week
e. 12 days
611.
A plumber, 45 y.o., is hospitalized on the 7th day of fever. Objectively: t – 39,8
°C, somnolence (at night insomnia), adynamia, pallor of skin, Ps 78/min, BP 105/70
mmHg. Tongue is thickly assessed by the grey raid with the imprints of teeth. Palpation:
the abdomen is distended, the liver is enlarged to 2 cm, and spleen to 1 cm below costal
arc, in the right iliac area – the crepitating grumbling and hyperesthesia of skin.
Defecation is absent for 2 days. What additional research should be performed for
clarification of diagnosis?
a. Spinal puncture
b. *Bacteriological research of blood
c. Analysis of myelogram
d. Colonoscopy
e. Research of blood for the markers of viral hepatitis
612.
Patient S., 23 years old, became ill at the end of the summer, His temperature
rose to 37,2 °C, insignificant headache and weakness appeared. For 7 days of the illness
he was treated ambulatorily as ARI (acute respiratory infection). His state became worse,
he was hospitalized in the permanent establishment. Temperature – 40 °C, pale, weakness.
Pulse 96 per a min, BP 110/70 mm Hg. Tongue with the imprints of teeth, abdomen soft,
distended, hepatosplenomegaly. Stool is not present. There is hyperemia and hyperplasia
of palatine tonsils, on the surface of right palatine tonsils are ulcers. What disease is
comes to your mind?
a. Angina of Simonovsky
b. Herpes angina
c. Ulcerative-necrotic angina
d. Infectious mononucleosis
e. *Angina of Diuge
613.
Patient L., 43 years old, entered to the clinic of infectious diseases with a
diagnosis „fever of idiopathic etiology”. He has been ill for 15 days: temperature of body
– 39,3 °C. Skin is pale. Pulse – 86 per a min, satisfactory properties. BP is 110/70 mm Hg.
Tongue is dry, assessed by the coverings, with the imprints of teeth on a lateral surface.
On a abdomen are 10-12 roseolas to 5 mm in a diameter. A liver and spleen is enlarged.
What is the most credible diagnosis?
a. Yersiniosis
b. *Typhoid fever
c. Brucellosis
d. Epidemic typhus
e. Sepsis
614.
A patient K., 26 years old, come to the permanent establishment on the 5th day
of the disease with complaints of a high temperature, chill and a dry cough. The disease
began suddenly from getting up of temperature to 38,8 °C, chill, then a dry cough. Treated
himself as ARI, took analgin and dimedrol. Objectively: moderate severity, on his
abdomen single roseollas, hepatosplenomegaly, diarrhea up to 4 times without
admixtures. The most informative methods of diagnostics for this disease are:
a. Cmear from a pharynx for a virus
b. Passive hemaglutination reaction with О-, Н- and Vi-antigens
c. Clinical blood test
d. Reaction of Vidal
e. *Hemoculture
615.
Sick person, 18 years old, became ill suddenly, when appeared chill, increased
of temperature to 39,6 °C, weakness, headache. On the 2nd day of illness appeared
diarrhea up to 10 times per day, without admixtures. A few days ago ate beef. On the 5th
day of illness appeared maculous-papulous rash on his body. Pulse was 100 per a min.,
BP 110/60 mmHg. Abdomen is soft, painful in the epigastrium and mesogastrium. Liver
+1 cm. Watery diarrhea 5 time per days. Choose the most reliable diagnosis:
a. Typhoid fever
b. Yersiniosis
c. *Paratyphoid fever B
d. Sepsis
e. Paratyphoid fever A
616.
In a patient 30 years old, on the 9th day of illness disease began gradually, with
slow progression of fever and intoxication, appeared unabundant roseollar rash on the skin
of abdomen. Skin was pale, Т-40 °C, pulse-80 per a min, BP-100/65 mm Hg. The tongue
is assessed, abdomen is distended, the spleen is enlarged, and the liver is palpable. What
symptom will be positive?
a. Botkin’s symptom
b. *Padalka’s symptom
c. Kildushevsky‘s symptom
d. Оrtner’s symptom
e. Pasternatsky‘s symptom
617.
A patient R., 23 years old, was hospitalized on the 4th day of disease, he had
contact with a patient with typhoid fever. During hospitalization body temperature – 37,8
°C, severe headache. A tongue is covered wit a yellow coverings. The abdomen is soft,
painless, rumbling in an ileocecal corner. Liver +1 cm. Defecation is absent during 3 days.
Hospitalized for an inspection on typhoid fever. What examination (laboratory diagnosis)
do you suggest for this patient?
a. Urinoculture
b. Coproculture
c. *Hemoculture
d. Biliculture
e. Positive reaction with a typhoid antigen 1:200
618.
In a patient A., 33 years old, with the diagnosis of typhoid fever, on the 5th day
of normal temperature appeared tachycardia and roseol rash on the middle and lateral
surfaces of the abdomen. About what is it necessary to consider?
a. Measles
b. Bleeding
c. Perforation
d. *Relapse of typhoid fever
e. Infectiously-allergic myocarditis
619.
In the induction centre of ian nfectious hospital a patient came in with a three
weeks history of high fever 38-40 °C, head pain, weakness, insomniA. He couldn’t speak
to the doctor, accepted antipyretics. Objectively: body temperature is 35,7 C, pulse 140
per 1 mines, BP 80/50 mm Hg. General condition is severe. Skin and mucous membranes
are pale. The tongue has teeth imprints, with a dirty-brown raid, a tag and edges of the
tongue is clean. The abdomen is swollen. The liver and spleen are enlarged. There was
defecation on reception, in the excrement was fresh blood. Why the increase in body
temperature and decrease in pulse?
a. Perforation
b. Acute poisoning medications
c. Infectiously toxic shock
d. Endometriosis of colon
e. *Bleeding
620.
The patient O., 23 years old, appealed to the hospital on the 6th day of gradual
development of illness, with complaints of severe headache (pain at the back of head)and
fever. Objectively: temperature of body 39,7 °C, pulse 84 per a min The face is pale,
tongue is dry, near a root covered a grey raid. abdomen is swollen. Dulling of percutic
sound is observed in a right iliac areA. His liver and spleen are palpated. What day does
rash appear on in this illness?
a. *On a 8th day
b. On a 2tnd day
c. On a 4th day
d. On a 5th day
e. On a 14th day
621.
A patient, 23 years old, appealed to the hospital on the 6th day of gradual
development of illness, with complaints of severe head pain at the back of his head,
parahypnosis and fever. Objectively: temperature 39,7 °C, Ps 84 per min. His face is pale,
tongue is dry, near the root is covered with a grey raid. A abdomen is swollen. Dulling of
percutory sound is observed in a right iliac areA. The liver and spleen are palpated. What
day does rash appear in this illness?
a. On a 12th day
b. On a 4th day
c. On a 5th day
d. On a 6th day
e. *On a 8th day
622.
A woman D., 41 years, complaints on diarrhea, headache, severe weakness,
insomnia, dull pain in a right iliac areA. It is a 8th day of illness. At a review: Т-39,8 °C,
Рs – 86/min., AP – 90/60 mm Hg. Pulse is dicotic. Skin is pale. Single roseollas are on the
abdomen. Tongue is dry, assessed by the coverings, with the imprints of teeth on a lateral
surface. Soft, dulling of percutory sound is determined in a illeocaecal area of abdomen.
Hepatosplenomegaly, positive Blumberg’s symptom, neutrophilic leukocytosis. What
diagnosis is most reliable?
a. *Typhoid fever
b. Epidemic typhus
c. Sepsis
d. Megakaryoblastoma
e. Tuberculosis
623.
Patient R., 35 years old, entered to the clinic on the ninth day of illness with
complaints of increased Т to 39,0 °C, headache, general weakness, delay in voiding, sleep
disturbance At a review: on the skin of abdomen are single roseollas, a tongue is assessed
by the brown covering, Ps. 78 shots in a min., rhythmic, liver is enlarged for 2сm. What is
reliable diagnosis?
a. *Typhoid fever
b. Leptospirosis
c. Brucellosis
d. Sepsis
e. Tuberculosis
624.
A vagabond 45 years old is hospitalized on the 7th day of fever. Objectively: t –
39,8 °C. He complaints of headache and insomniA. The patient is excited, talkative. Face
is hyperemic. Rash mainly on the lateral surfaces of trunk, abundant, roseollar-patechial.
Pulse is rapid. Enlarged liver and spleen. What is the preliminary diagnosis?
a. Typhoid fever
b.
c.
d.
e.
*Epidemic typhus
Paratyphoid B
Leptospirosis
Scarlet fever
625.
A patient, 24 y/o, was hospitalized in infectious department on the 10th day of
illness with complaints of weakness, headache, bad appetite and cough. Objectively:
temperature of body 39,5 °C, pallor of skin. On the abdomen, and chest some roseolas
were found. There is hard breathing over the lungs with no wheezes. RR 20 / min. Pulse
80 /1 min. The liver edge is palpable to 1 cm. below the costal arc. The spleen is not
enlarged. What is the diagnosis?
a. *Typhoid fever
b. Flu
c. Spotted fever
d. Brucellosis
e. Pneumonia
626.
Patient P., 45 years old, entered on consultation with a diagnosis „fever of
idiopathic etiology”. He has been ill 8 days. The disease developed gradually, then
appeared headache, insomnia, adynamiA. Temperature – 39,5 °C, skin is pale. Pulse – 88
per a min. The center of the tongue was covered with a thick grey coat. The abdomen is
exaggerated, grumbles in a right iliac region. The liver and spleen are enlarged on 2 cm.
What is the most probable diagnosis?
a. *Typhoid fever
b. Yersiniosis
c. Epidemic typhus
d. Adenoviral infection
e. Sepsis
627.
At the inspection of persons who had contact with a patient with typhoid fever,
stick of typhoid fever was found in urine. The reaction of Widal was negative. The patient
considers himself healthy. What is your preliminary diagnosis?
a. Typhoid fever, latent period
b. Typhoid fever, relapse
c. Transitory bacteriocarriers of stick of typhoid fever
d. *Chronic bacteriocarriers of stick of typhoid fever
e. Any of the enumerated diagnoses is possible
628.
A 39 years old train conductor, is hospitalized on the 4th day of illness with
complaints of headache, weakness, dizziness, increased perspiration, insomnia, chills.
Hyperemia of face with edema, and conjunctivitis have been observed. On a transitional
fold a conjunctiva are not numerous petechias. On the skin of trunk, thorax, abdomen,
extremities intensive roseolopetechial rash was found. On exam there was tachycardia BP
– 100/60 mm of Hg tremor of tongue were marked. The liver, and spleen, are enlarged.
Patient is constipated for 3 days. Most credible diagnosis is:
a. *Epidemic typhus
b. Typhoid fever
c. Flu
d. Meningococcemia
e. Leptospirosis
629.
A patient V., 23 years old, acted to permanent establishment on the 6th day of
the disease with complaints for a high temperature, chill, dry cough. The disease began
suddenly with a rise of temperature to 39,7 °C, chill, then dry cough. Treated oneself as
ARI, took an analgin, dimedrol. Objectively: state of moderate severity, on an abdomen
single roseollas, hepatosplenomegaly, diarrhea up to 4 times without admixtures. What is
the preliminary diagnosis?
a. Yersiniosis
b. Typhoid fever
c. Brill‘s disease
d. Meningococcemia
e. *Epidemic typhus
630.
Patient I., 28 years old, hospitalized on the 9th day of illness with complaints of
increase of temperature to 39,2 °C, headache, general weakness and absence of
defecation. There are singles roseolas on the abdomen, pulse 78 per a min, a liver + 2 cm.
What is probable diagnosis?
a. Sepsis
b. Spotted fever
c. *Typhoid fever
d. Brucellosis
e. Leptospirosis
631.
At sick, 32 years, on the 9th day of illness which began gradually, from slow
rise of fever and intoxication, appeared 3 roseols on the skin of abdomen. Objectively:
skin is pale, temperature – 40,4 °C, pulse 80 per a min, BP 100/65 mm Hg. The tongue is
assessed, an abdomen is swollen, and the spleen and liver are palpable. Inspection of
which disease is needed above all the options?
a. Spotted fever
b. *Typhoid fever
c. Measles
d. Scarlet fever
e. Sepsis
632.
A child, 8 years old, has been ill for 9 days. Complaints of weakness, headache
and insomniA. A temperature to – 38,5-39,5 °C. Skin is pale. There are 3 roseolas on the
abdomen. A tongue is assessed. Liver and spleen are enlarged. What disease is the most
probable?
a. Sepsis
b. Yersiniosis
c. Infectious mononucleosis
d. *Typhoid fever
e. Leptospirosis
633.
At sick P., 40 years old, the high temperature of the body is marked during 8
days, severe headache. Objectively: temperature – 39,5 °C, the patient is pale, languid,
adynamic. Pulse 82 per a min. Tongue is dry, assessed a brown raid, on the skin of
abdomen singles roseolas. A liver + 2 cm. What is the most probable diagnosis?
a. *Typhoid fever
b. Spotted fever
c. Sepsis
d. Tuberculosis
e. Brucellosis
634.
Patient B., 36 years old, complaints of a great headache, general weakness,
insomnia, fever to 39,7 °C. Fell ill gradually. Objectively: skin is pale, on a abdomen are
singles roseolas. A tongue with the imprints of teeth and white raid, edges and tag is clean.
Flatulence. A liver and spleen are enlarged. Dulling of percussive sound is in a right iliac
areA. Pulse 70 per a min, BP 100/60 mm Hg. In lights there are the dissipated dry
wheezes, hard breathing. Blood test: leycocytosis 3,1?109/l, RSE 25 mm/hour, eos. 0 %,
n/n 9 %, s/n 51 %, lymphs. 31 %, monocyts 5 %. What is the most credible diagnosis?
a. *Typhoid fever
b. Epidemic typhus
c. Pneumonia
d. Leptospirosis
e. Sepsis
635.
Patient K, 32 years old, of no fixed residence went to see a doctor for 5-days illness
with complaints of fever, severe headache, insomniA. On body temperature 40 °C, pulse
110/min. The patient is excited and talkative. Hyperemic, scleritis. At whole over the body
there is pink petehial rashes. Positive Govorova-Godele‘s symptom. Enlarged liver and
spleen. What do you need to find out the epidemic status?
a. *The existence of head lice
b. Use of poor food
c. Availability of parenteral interventions
d. Contact with rodents
e. Drinking water from unhygienic sources
636.
A patient L., 32 years old, who complaints of severe headache and fever, the 6-day
illness positive agglutination test with rickettsia reaction. Vector of the disease is:
a. Flea
b. Fly
c. Mosquitoes
d. Bee
e. *Lice
637.
A patient L, 72 years old, who complaints of severe headache and fever, the 6-day
illness positive agglutination test with rickettsia reaction. Past sick sick was typhus.
Vector of the disease is:
a. Flea
b. Fly
c. Mosquitoes
d. Lice
e. *Carrier does not need
638.
A patient 29 years, a few days ago, a chill, the temperature for 2-3 days increased
to 39-40 °C. There hyperemia and edema person,significant sclera like "drunk" person and
"rabbit" eyes. On the third day of illness – on mucuos of soft palate, parenthesis bright red
enantema is seen. At 3-4-day patient when trying to protrude the tongue, there was
hypermovement, tremor, rejecting it to the side. Diagnosis: epidemic typhus. Which
disease is likely in a patient?
a. *Epidemic typhus
b. Typhoid fever
c. Brill disease
d. Paratyphoid A
e. Paratyphoid B
639.
A patient at the 4-day fever has profuse rosy-petehia exanthema predominantly
located on the lateral surface of the torso and limbs flexion surfaces. Hyperemic, vascular
conjunctivitis, petehii transition of anterior fold conjunctivA. What kind of illness can
think?
a. Typhoid fever
b. *Epidemic typhus
c. Measles
d. Haemorrhagic fever with renal syndrome
e. Crimean hemorrhagic fever
640.
Patient B., complaints of headaches, delirium. When inspection: a body
temperature of 39 °C, initiated, a person hyperemic, positive symptom Govorova-Godele,
petehia rash on the trunk, limbs, tachycardia, hypotension, hepatosplenomegaly. Drunker,
drug user, living in the basement room type. What is your preliminary diagnosis?
a. AIDS infection
b. Influenza
c. Leptospirosis
d. *Epidemic typhus
e. Alcohol psychosis
641.
A patient 28 years old, of no fixed abode, hospitalized with a preliminary diagnosis
«flu», a 5-day illness appeared rosy-petehia rashes on the body and interior surfaces of the
extremities. The temperature of 41 °C, euphoria, hyperemia person, redness sclera, tongue
tremor, tachycardia, enlarged spleen; excitement. What is the likely diagnosis?
a. Typhoid fever
b. Leptospirosis
c. Alcohol deliry
d. Measles
e. *Epidemic typhus
642.
The watchman 42 years old, complaining of?desperately ill. Entered the 6-day
illness with fever 39,7 °C, severe headaches, noise in the ears, insomniA. OBJECTIVE:
instituted, euphoric, talkative. Face red, his eyes shining, sclera and conjunctiva enhanced
vascular pattern. On the inside shoulder and the side surfaces of chest rosy-petehia rash.
Found head lice and nits. Reaction Vidal 1:40. What is diagnosis?
a. Brill disease
b. Malaria
c. *Epidemic typhus
d. Typhoid fever
e. Q-fever
643.
A patient 28 years at the 4-day fever has profuse rosy-petehia exanthema
predominantly located on the lateral surface of the torso and limbs flexion surfaces. An
individual patient hyperemic expressed vessels conjunctival injection, petehies transition
to fold conjunctivA. Analysis of urine protein single hyaline and granular cylinders. What
kind of illness can think?
a. Typhoid fever
b. Haemorrhagic fever with renal syndrome
c. Crimean hemorrhagic fever
d. Measles
e. *Epidemic typhus
644.
For patients 78 years of acute illness began, which is characterized by fever with
chilling, rash dominated rosy petehia elements. The condition of the patient violated
moderately. Agent and the source of infection is not detected, but it is known that 45 years
ago patient had severe typhus. What is your diagnosis?
a. Yersiniozis
b. * Brill‘s disease
c. Typhoid fever
d. Drug allergy
e. Food Allergies
645.
The patient, a train conductor, 39 years old, hospitalized for a 4-day illness with
complaints of headache, weakness, dizziness, sweating, insomnia, fever. Hyperemic,
edema, conjunctivitis. At the transition fold conjunctiva – single petehies. At the skin
torso, chest, abdomen, limbs abundance rosy-petehia rash. TachycardiA. AD 100 and 60.
Tremor of the tough. Palpable liver, spleen. Stool arrested. What is the most likely
diagnosis?
a. Leptospirosis
b. Typhoid fever
c. Influenza
d. Meningoccemia
e. *Epidemic typhus
646.
A patient 30 years on 9th day of illness that began gradually, the slow rise of fever
and intoxication, a painless rosy rashes on the skin of the abdomen. OBJECTIVE: pale,
temperature 40 ° C, pulse 80/min, AD 100 and 65 mm RT. Art. Language is coached,
abdomen bubbles, enlarged spleen and liver. For what disease you can suggest in the first
place?
a. *Typhoid fever
b. Epidemic typhus
c. Measles
d. Scarlet fever
e. Sepsis
647.
Patient 20 years, complained about the high temperatures of up to 39 ° C, headache
in the frontal area, pain in the eyeball, photophobia, pain in muscles, dry cough. Acutely
ill the day before. Objective: condition serious, hyperaemic, eyes shining, vascular
injection sclerA. Pulse 96/min, rhythmical. Tone heart weakened. In the lungs scattered
dry bubbling rale. Faces hyperemic, granular. Meningeal symptoms are not present. Blood
tests: Leu. 3?109, eoz. 1 %, yang neu. 6 %, neu. 51 %, lymph. 35 %, mon. 7 %. What is
the most likely diagnosis?
a. Measles
b. *Influenza
c. Meningococcal disease
d. Pneumonia
e. Typhus
648.
Often, in patient with epidemic typhus arise transition petehies in the conjunctivA.
What term did it call?
a. Symptom of Heller
b. Conjunctivitis
c. Symptom of Govorova-Godele
d. *Symptom of Zorohovich-Chiari-Avtsyn
e. Enantema Rosenberg‘s
649.
In the family of the patient with epidemic typhus, were lice in the children. With
the help of any of these events could prevent the subsequent spread of the disease?
a. *Monitoring and complete sanitation of contact in the centre
b. The use of chemoprophylaxis
c. The use of antibiotics
d. Isolation contact
e. Check-up
650.
When you can stopped etiotropic medications treatment of the patient with
epidemic typhus?
a. Immediately after the normalization of body temperature
b. After the normalization of the liver and spleen
c. *After a 2-day normal body temperature
d. After the disappearance of roseola
e. Within 10 days after the disappearance of roseola
651.
Patient K., 23 years old, hospitalized on the 3rd day of illness, which was
accompanied by mild running nose, high fever to 40,2 °C, headache and hemorrhagic rash
on the skin. In the 2 hours after the introduction of penicillin blood pressure dropped to 40
and 10 mm Hg. Distal pulse and meningeal signs are not defined. What is the diagnosis in
a patient?
a. *Meningoccocemia, infectious-toxic shock
b. Epidemic Typhus, severe course
c. Measles, severe course
d. Scarlet fever, severe course
e. Flu, anaphylactic shock
652.
A patient 25 years old, who returned from the Far East, suddenly has increased
body temperature to 39 °C, a pain in the backbone, reddening skin type «hood», single
hemorrhages on the skin. After 3 days, along with declining fever, weakness, thirst,
decreased diuresis to 300 ml, decreased blood pressure. Much pronounced Pasternatskyj‘s
symptom. What is the most likely diagnosis?
a. Leptospirosis
b. Typhus
c. Haemorrhagic fever Crimean-Congo
d. Acute glomerulonephritis
e. *Haemorrhagic fever with renal syndrome
653.
Male 45 years old, fell ill after 2 weeks after returning from Afghanistan, where six
months ago underwent malariA. In return were found lice. Suddenly, there were severe
headache, weakness, body temperature 39 °C with a temporary decrease in the 4th day of
illness, followed by general weakness, much intoxication, headache, appeared on the body
abundance polymorphic rash. At the 7-day state of heavy, the phenomenon of
encephalitis, excitation, hallucinations periodically. Objective – erythematos-petehial
exanthemA. The temperature of the body 40,1 ?C, pulse was 136 for 1 min, blood
pressure 120/70 mm RT art. Moderately enlarged liver and spleen. Paradoxical ishuriyA.
In the blood analys neutrofil mild leukocytosis, eozinofilia, accelerated ESR. Which of the
studies of blood will be positive?
a. At typhoid fever
b. At the three-day malaria
c. Tropical malaria
d. *Epidemic typhus
e. At brucellosis
654.
Male 26 years old, who last month returned from Africa and was processing about
lice, sick 5 days. Home was a sudden: severe headache, weakness, body temperature 38,8
°C, which persisted all day at a constant level, but at the 4th day of the disease declined
for several hours. Then the patient condition has deteriorated significantly, a rash on the
body, delay urine. OBJECTIVE: pulse was 110 for 1 min, blood pressure 115/70 mm RT.
Art., temperature 39,7 °C. Face red, a significant injection of vascular sclera, cmall
hemorrhages in the conjunctiva, mucous membrane soft palate, abundant rosypetehialexanthema on the body. Signs interstitsial pneumonia, encephalitis. Enlarged liver
and spleen. What kind of illness can you think of?
a. *Epidemic typhus
b. Typhoid fever
c. Tropical malaria
d. Leptospirosis
e. Yellow fever
655.
Patient P., 68 years old, fell ill suddenly 7 days ago from the increase of
temperature to 39,3 °C, appearances of headache and insomniA. Objectively: excited,
inadequate. Face is hyperemic. The tongue is very dry, trembles at pulling out. On a trunk
are polymorphic rash, tachycardia, and blood pressure low. HepatosplenomegaliA. Stool
is absent. In age 10 years had epidemic typhus. What is the previous diagnosis?
a. Yersiniosis
b. Typhoid fever
c. *Brill‘s disease
d. meningococcemia
e. Sepsis
656.
Patient P., 76 years old, complianed during 7 days for the permanent increase of
temperature to 38,2-38,7 °C, headache, insomnia, horrific dreams, dry cough, myalgias,
artalgias. Polymorphic rash appeared on a trunk on the 4th day of diseases. In childhood
was ill by epidemic fever, three-day malariA. Temperature of body – 38,4 °C, pulse – 98
per a min, hepatolienal syndrome, bilateral pneumonia confirmed roentgenologic. In the
analysis of blood moderate neutrophil leycocytosis. What is the preliminary diagnosis?
a. *Brill‘s disease
b. Typhoid fever
c. Lime‘s disease
d. Malaria
e. Leptospirosis
657.
Patients D., 30 years old, became ill saddenly, when the temperature of body rose
to 40,2 °C, appeared headache, weakness, euphoria, injection of vascular sclera,
hyperemia of face, appears the positive symptom of Govorov-Godele. Reaction with
Rickettsia prowazekii 1:160, IgG – 87 %. What is your diagnosis?
a. Brill‘s disease
b. Typhoid fever
c. Paratyphoid A
d. Paratyphoid B
e. *Epidemic typhus
658.
Patients I., 78 years old, became ill saddenly, the temperature of body rose to 39,2
°C, euphoric, vessels of scleras are injection, hyperemia of face, appears enantema of
Rozenberg. Reaction with Rickettsia prowazekii 1:160, IgG – 87 %. What is your
diagnosis?
a. *Brill‘s disease
b. Typhoid fever
c. Meningococcal infection
d. Flu
e. Leptospirosis
659.
At a patient with a pediculosis the temperature of body rose to 41,6 °C, appeared
headache, euphoria, in 4th days from the beginning of illness – rozeola-petehial rash.
Reaction with Rickettsia prowazekii 1:640, IGG – 89 %. What is your diagnosis?
a. Flu
b. Typhoid fever
c. Meningococcal infection
d. *Epidemic typhus
e. Leptospirosis
660.
A patient’s temperature is 40 °C. There are also deep and unproductive cough,
photophobia, face puffiness whitish points on the mucous membrane of cheeks opposite
molar teeth. What is the most possible diagnosis?
a. Tuberculosis
b. Meningococcemia
c. *Measles
d. Enteroviral infection
e. Staphylococcus sepsis
661.
A child after consuming food in a party complaints of vomiting and diarrhea within
1-5 hours. The diagnosis is:
a. *Staphylococcus aureus
b. Streptococcus
c. Clostridium Perfringens
d. Clostridium Botulinum
e. Meningococcus
662.
Patient A., 37 years old, entered to infectious hospital on the third days of disease
in the severe condition. He complaints of the high fever with chills and sweat, general
weakness, pain in right under a rib. Objectively: temperature of body 41 °С, icterus of
skin, liver +2 cm, pain at palpation in abdomen, positive symptoms of Ortner and Mussy,
a spleen is normal, tachycardiA. What is the preliminary diagnosis?
a. Malaria
b. *Cholangitis
c. Viral hepatitis
d. Sepsis
e. Leptospirosis
663.
Patient W., 38 years old, entered to infectious hospital on the 5th days of disease in
the severe condition. He complaints of the high fever with chills and sweat, general
weakness, pain in sacrum. Objectively: temperature of body of 41 °С, tachycardia,
positive symptom of Pasternacky, liver and spleen is not normal. Preliminary diagnosis?
a. Malaria
b. Cholangitis
c. *Pyelonephritis
d. Sepsis
e. Leptospirosis
664.
Patient D., 39 years old, entered to infectious hospital on the second week of
disease in the severe condition. She complaints of high fever with chills and sweat,
general weakness, pharyngalgia at swallowing. Objectively: temperature of body 37,7 °С,
hyperemia of mucus of pharynx, tonsils are enlarged, loose, festering raid in lakuns,
enlarged submandibula, neck, axilars lymphonodules, icterus of sclera and skin,
bradycardia, liver + 2 cm, spleen + 1 cm. Urine is color of beer, an excrement is
discolored. What is the previous diagnosis?
a. Malaria
b. Infectious mononucleosis
c. Viral hepatitis
d. *Sepsis
e. Leptospirosis
665.
Patient A., 37 years old, entered to infectious hospital on the third days of disease
in the severe condition. He complaints of the high fever with chills and sweat, general
weakness, pain in right under a rib. Objectively: temperature of body 41 °С, icterus of
skin, liver + 2 cm, pain at palpation in abdomen, positive symptoms of Ortner and Mussy,
a spleen is normal, tachycardiA. What methods is it possible to confirm a previous
diagnosis by?
a. Global analysis of blood
b. *Ultrasonic research
c. Biochemical blood test
d. Hemoculture
e. Parazytoskopy of blood
666.
Patient W., 38 years old, entered the infectious hospital on the 5th days of disease
in the severe condition. He complaints of the high fever with chills and sweat, general
weakness, pain in sacrum. Objectively: temperature of body of 41 °С, tachycardia,
positive symptom of Pasternacky, liver and spleen not is normal. What is the previous
diagnosis? What methods is it possible to confirm a previous diagnosis by?
a. General blood analysis
b. Ultrasonic research
c. Biochemical blood test
d. Hemoculture
e. *Parazytoskopy of blood
667.
Patient of 52 years, fell ill sharply from a chill and head pain, fevers to 40°C. On a
3th day illnesses are nausea and vomiting, dark urine. On a 4th day a temperature went
down to 37 °, but an icterus appeared and the amount of urine diminished to 600 ml. What
disease is such development of symptoms characteristic for?
a. Hepatitis A
b. Hemorragic fever with a kidney syndrome
c. *Leptospirosis
d. Sepsis
e. Acute glomerulonephritis
668.
Animal technician 57 years, on the 3rd day of illness appealed to the doctor with
complaints of head pain, high temperature, pain in gastrocnemius muscles, cmall of the
back, icterus, dark urine and diminishing of its amount. Objectively: temperature 38,1 °C,
injection of the sclera vessels, petechial rash on upper part of the thorax,
hepatosplenomegaly. What is the most credible preliminary diagnosis?
a. Pseudotuberculosis
b. Brucellosis
c. Viral hepatitis
d. *Leptospirosis
e. Influenza
669.
A patient I., 26 years old, delivered in a hospital on the 4th day of disease with
complaints of fever, headache, pain in gastrocnemius muscles. Works as a specialist in
land-reclamation. Has a lot of sexual contacts. Objectively: temperature – 39,7 °C. Severe
common condition. Expressed icterosis of skins and sclerA. Hemorrhages in conjunctiva
and sclerA. There is a hemorragic rash on a skin. A liver increased on 3 see below the
costal arc, edge of spleen, the day's diuresis 300 ml. The etiologic factor of the disease
most probable:
a. Rikkettsia
b. Virus
c. *Leptospira
d. Spirocheta
e. Chlamidia
670.
Patient 43, miner, on the 7th day of disease grumbles about a sharp weakness, high
temperature, pain in the muscles of feet and cmall of the back, icterus, dark color of urine,
headache. Fell ill sharply is a chill, temperature 40,2 °, there was a nose-bleed. A diuresis
200 ml. Credible diagnosis?
a. Sepsis
b. Typhoid
c. Viral hepatitis
d. *Leptospirosis
e. Malaria
671.
A patient, 35 years, suffering sharply, complaints of a headache, pain in the
muscles of lower extremities, increasing of temperature to 39,3 °C. Objectively on the 4th
day of disease: the state is severy, hyperemic face, icterosis of skin and sclerA.
Enlargment of liver and spleen. A diuresis is mionectic. What is most credible diagnosis?
a. Trichinosis
b. Hepatitis A
c. Yersiniosis
d. Infectious mononucleus
e. *Leptospirosis
672.
For a patient, workwoman of a pig farm, on a background of complete health a
chill appeared suddenly, a temperature rose to 39,9 °C, there was a headache, nauseA. On
the next day marked pains in the muscles of lower extremities, nose-bleeding began. On
the 3rd day of illness, state became more severy. Face is hyperemic, scleritis, hyperemic
scleritis. Liver +3 cm. Daily diuresis 700 ml. What is the preliminary diagnosis?
a. Yersiniosis
b. Hepatitis A
c. Hemorrhagic fever with a kidney syndrome
d. Flu
e. *Leptospirosis
673.
On the 3rd day of illness the sick is delivered in a severe condition with complaints
of sudden rise of temperature, headache, repeated nose-bleed, pain in gastrocnemius
muscles. Objectively: moderate icterus of sclera and skins, hepatospleenomegaly,
оliguriA. What is most credible diagnosis?
a. *Leptospirosis
b. Viral hepatitis
c. Influenza
d. Infectious mononucleosis
e. Malaria
674.
A patient S., 45 years old, suffering suddenly from appearance of chill and
increasing of temperature to 39,2 °C. In the evening pain appeared in a abdomen and
gastrocnemius muscles. In 2 days noticed the ochrodermia of skin and sclerA.
Objectively: the state is severy, temperature 39,9 °C. The tongue is covered. Moderate
jaundice of the skin and sclerA. There is plural petachiae on a trunk. Superficial breathing
20 times per 1 min, pulse 102 per 1 min, AP 100/60 mm of Hg. A abdomen is soft, sickly
in epigastrium, a liver on 3 cm comes from a costal arc. Daily diuresis 300 mm, urine is
sad-coloured. What is your preliminary diagnosis?
a. Sepsis
b. *Leptospirosis
c. Influenza
d. Hepatitis B
e. Infectious mononucleus
675.
A patient Z., 33 years old, miner, entered clinic of infectious diseases on the 7th
day of disease with complaints of acute weakness, high temperature, pain in the muscles
of feet and back, icterus, dark color of urine, headache. Became sick sharply from a chill,
temperature rises up to 40,1 °C. On a 4th day there is an icterus, nose-bleeding,
hemorrhages in sclerA. Duration of fever 6 days. Diuresis is 200 ml. What is credible
diagnosis?
a. Typhoid fever
b. *Leptospirosis
c. Hepatitis A
d. Sepsis
e. Influenza
676.
A patient Y., 25 years old, entered infectious department on the 3rd day of disease
with complaints of headache, pain in the back, gastrocnemius muscles, high fever, chill.
State is moderate. Icteruses of the scleras. Mucous membrane of soft palate is hyperemic.
Tongue is dry, assessed with brown cover. The abdomen is swollen. Liver +2 cm, spleen
is not megascopic. Painfull muscles, especially gastrocnemius. Urine is dark, excrements
ordinary color. What is the most credible diagnosis?
a. Infectious moneuclious
b. Hepatitis A
c. Malaria
d. *Leptospirosis
e. Yersiniosis
677.
Patient A., 16 years, hospitalized in an infectious department with complaints of
absence of motions in lower extremities. A disease was begun 2 days back with the
increase of temperature to 38 °, dyarrhea is to 3-5 times per days. Melosalgias took a
place, head pain. Objectively: temperature of body 36,8 °, active motions absent in lower
extremities, in the area of defeat is areflexis, low blood pressure of muscles, a
sensitiveness is stored. Meningeal symptoms are poorly expressed. What disease is it
needed to think of?
a. Leptospirosis
b. Meningococcal infection
c. *Poliomyelitis
d. Parainfectional encephalitis
e. Tubercular meningo encephalitis
678.
For patient A., 25 years, which returned from Far East, suddenly the temperature of
body rose to 39 °, pain appeared in cmall of the back, hyperemia of person, neck,
overhead half of trunk, single haemorrages as red strips on a neck and lateral surfaces of
thorax. Nose-bleeds. Through 3 days a weakness, thirst, grew together with the decline of
fever, diminished to 300 ml diuresis, an arteriotony went down. Positive symptom of
Pasternackogo. What diagnosis is most credible?
a. Leptospirosis
b. *Hemorrhagic fever with a kidney syndrome
c. Spotted fever
d. Hemorrhagic fever Crimea-Congo
e. Acute glomerulonephritis
679.
Man 26 years, month ago returned from Africa, passed treatment concerning
pediculosis. He is ill for 5 days. Beginning was sudden: great head pain, weakness,
stationary temperature of body (38,8°C), which on a 4th day went down on a few clock.
The state of patient was considerably worsened farther, a rash appeared on a trunk,
coughing, uroschesis. Objectively: temperature of body 39,7°C, pulse 110 per 1 min, BP
115/70 mm Hg. Skin of person red, considerable injection of vessels of sclerotica, shallow
hemorrhage on a conjunctiva, mucous membrane of soft palate, abundant roseol-petechial
exanthema on a trunk. Signs of interstitial pneumonia, encephalitis. A liver and spleen is
megascopic. What disease most probably?
a. Yellow fever
b. Typhoid fever
c. Tropical malaria
d. Leptospirosis
e. * Epidemic typhus
680.
Patient A., 35 years, had ill sharply, complaints of high temperature of body, great
head pain, sickliness in gastrocnemius muscles. Objectively on the 4th day of illness: the
state is heavy, hyperemia of person, skin and sclerotica are icterus, a liver and spleen is
megascopic. OliguriyA. What diagnosis is most credible?
a. Yersiniosis
b. Viral hepatitis
c. *Leptospirosis
d. Poisoning of tetraetyl lead
e. Omsk hemorrhagic fever
681.
At one on holiday-makers ashore lake in 6 days did a temperature rise to 38,5 °C,
headacke, pain, appeared in muscles, sweating. In 3 days there was the sickly slight
swelling in an inguinal areA. At examination in an inguinal area found out a dense,
mobile, moderate sickly lymphonodus to 5 cm in a diameter. The skin above him is not
changed. What diagnosis is most credible?
a. Leptospirosis
b. Iersiniosis
c. Infectious mononucleosis
d. *Rabbit-fever
e. Acute lymphadenitis
682.
Patient of 62 years, in the past had been ill with spotted fever. Fell ill sharply: fever
39,5 °C, head pain, insomniA. On a 6th day on the skin of abdomen and lateral surfaces of
thorax roseol appeared rash. A liver and spleen, tachycardia, deafness of tones of heart,
low blood pressure is megascopic. What diagnosis is most credible?
a. *Illness of Brilla
b. Typhoid
c. Leptospirosis
d. Sepsis
e. Infectious mononucleosis
683.
Patient I., 21 years old, entered hospital on the 7th day of illness with complaints of
a sharp weakness, pains in muscles and joints, head pain, nauseA. A disease began with
sore throat, cold, general weakness. Did not measure a temperature. Last was the state
worsened 2 days, a chill appeared, high temperature 39-40°C, head pain, nauseA.
Objectively: patient of adynamic, consciousness is kept, temperature of body 37,5°C, the
state is extremely heavy, a skin is pale, lips and nail phalanxes of cianotic, on the skin of
tiptoes and hands hemorragic rash, the scopes of heart are extended to the left on 1,5 cm,
tones are deaf, pulse 130 per 1 mines, weak filling, BP 80/40 mm Hg. Breathing normal.
A tongue is assessed, moist, phenomena of pharyngitis. Neck lymphonodus are
megascopic. Meningeal signs are absent. OliguriA. What diagnosis is most credible?
a. Hemorrhagic fever
b. Leptospirosis
c. Epidemic typhus
d. Rheumatoceils
e. *Meningococcal infection. MeningococcemiA.
684.
Animal technician 57 years, on the 3rd day of illness appealed to the doctor with
complaints of head pain, high temperature, pain in gastrocnemius muscles, cmall of the
back, icterus, dark urine and diminishing of its amount. Objectively: temperature 38,2°,
injection of vessels of sclerotica, petechial rash on overhead part of thorax, megascopic
hepar and spleen. Most credible preliminary diagnosis?
a. Brucellosis
b. *Leptospirosis
c. Viral hepatitis
d. Pseudotuberculosis
e. Trichinosis
685.
Patient 43, miner, on the 7th day of disease complains of a sharp weakness, high
temperature, pain in the muscles of feet and cmall of the back, icterus, dark color of urine,
head pain. Fell ill sharply is a chill, temperature 40°, there was a nose-bleed. A diuresis
200 ml. Credible diagnosis?
a. Malaria
b. Typhoid
c. Viral hepatitis
d. Sepsis
e. *Leptospirosis
686.
Patient 35 years, which fell ill sharply, complains of headache myalgia, pain in the
muscles of lower extremities, increase of temperature to 39 °C. Objectively on the 4th day
of illness: state heavy. Face of hyperaemiA. Skin and sclera are icteric. The liver and
spleen are enlarged. A diuresis is mionectic. Most credible diagnosis?
a. Yersiniosis
b. Hepatitis A
c. *Leptospirosis
d. Infectious mononucleosis
e. Trichinosis
f. Leptospirosis
687.
For a patient, workwoman of pig farm, on a background a complete health a chill
appeared suddenly, a temperature rose to 39,9 °, there was a head myalgia, nauseA. The
next day marked pains in the muscles of lower extremities, a nose-bleed began. At the
reception of the permanent establishment, on the 3rd day of illness, state heavy. Face of
hyperaemia, scleritis, subicterus of sclerA. Liver +3cm, a diuresis 700 ml. Preliminary
diagnosis?
a. Hemorragic fever with a kidney syndrome
b. Hepatitis of A
c. Yersiniosis
d. Flu
e. *Leptospirosis
688.
The patient 33 years, miner, entered clinic of infectious diseases on the 7th day of
disease with complaints of a sharp weakness, high temperature, pain in the muscles of feet
and cmall of the back, icterus, dark color of urine, head pain. Fell ill sharply from a chill,
temperatures 40°. On a 4th day is an icterus, nose-bleed, hemorrhage in a scleroticA.
Duration of fever 6 days. A diuresis 200 ml. Credible diagnosis?
a. *Leptospirosis
b. Typhoid
c. Hepatitis A
d. Sepsis
e. Iersiniosis
689.
A patient is disturbed by attacks fevers which repeat oneself periodically every
third day. The icterus of sclerotica and skin covers, Liver and spleen are enlarged. Which
of the following diagnosis below is most correct?
a. Viral hepatitis
b. Sepsis
c. *Malaria
d. Yersiniosis
e. Leptospirosis
690.
A patient 25 years, entered infectious separation on the 3rd day of disease with
complaints of pain in gastrocnemius muscles, high fever, chill. State of middle weight.
Sclera are icterus. Mucous membrane of soft palate is hyperaemiA. A tongue is dry,
assessed a brown raid. A abdomen is swollen. Liver +2 cm A spleen is not megascopic.
Palpation of muscles, especially gastrocnemius, painful. Wetting dark. Excrement of
ordinary color. Name the most credible diagnosis:
a. Infectious mononucleosis
b. Viral hepatitis A
c. Malaria
d. *Leptospirosis
e. Yersiniosis
691.
In receiving department of infectious separation on the 3rd day of illness the sick is
delivered in a grave condition with complaints of a suddenly arising up high temperature,
head pain, repeated nose-bleed, pains in gastrocnemius muscles. Objectively: moderate
icterus of sclerotica and skins, liver and spleen are enlarged oliguriA. What is the most
credible diagnosis?
a. Infectious mononucleosis
b. Viral hepatitis
c. Yersiniosis
d. *Leptospirosis
e. Malaria
692.
Patient P., 45 years old, fell ill suddenly from appearance of chill and increase of
temperature to 39,2 °C. In the evening myalgia appeared in a abdomen and gastrocnemius
muscles. In 2 days noticed the ochrodermia of skin and scleroticA. Objectively: the state
is heavy, temperature 39,9 °C. Skin and sclerotica moderate yellow. There is plural
petechia on a trunk. Breathing is normal. FB-20/min, Pulse-102/min, BP 100/60 mm Hg.
A abdomen is soft, sickly in an epigastrium, inreasing of liver. Daily diuresis 300 ml of
urine of sad-coloured. What is preliminary diagnosis?
a. Sepsis
b. *Leptospirosis
c. Yersiniosis
d. Hepatitis B
e. Infectious mononucleosis
693.
76-years old patient during 7 days grumble about the permanent increase of
temperature to 38-38,7 °C, moderate head pain, insomnia, horrific dreams, dry cough,
myalgias, arthragiA. From the 4th day of illness – on a trunk polymorphic exanthema with
predominance of roseol elements. In childhood was ill the spotted fever, three-day
malariA. Temperature of body 38,4°, pulse 98 per 1 min, hepatolienal syndrome, bilateral
pneumonia, confirmed roentgenological. There is moderate neutrophilic leykocytosis in
the global analysis of blood, speed-up ESR. What from diagnosis is the most credible?
a. * Brill’s disease
b. Typhoid fever
c. Lime disease
d. Malaria
e. Leptospirosis
694.
Patient P., 24 years old, hospitalized in an infectious department in a severe
condition. Complaints of head acke, pain in muscles and joints, vomiting. Objectively –a
patient is excited, temperature of body 39 °C. BP 90/60 mm Hg, tachycardiA.
hyperaestesia of skin appeared. Doubtful meningial signs. It is known from anamnesis,
that lives in a mud flow, has an economy, there are rats. What preparations must be given?
a. Veroshpiron, euphyllinum, Dimedrolum
b. *Mannitol, Lasixum, prednisolon, penicillin
c. Analgin, Dimedrolum, acetophene
d. Manitol, acetophene
e. Lasixum, analgin, ampicillin
695.
Sick, 22 years old, hospitalized in an infectious separation with complaints of chill,
fever, great head pain, pain in gastrocnemius muscles. The state is severe, hyperemia of
face, skin and sclera are icteric, the iver and spleen are enlarged, oliguriA. What
preparations are routinely given in this disease?
a. Introduction of whey
b. *Antibiotics
c. Sulfanilamid preparations
d. Desintoxic therapy
e. Enterosorbtion
696.
Patient T., 40 years, hospitalized in infectious separation with complaints of chill,
fever, great head pain, pain in gastrocnemius muscles. The state is severe, hyperemia of
face, skin and sclera are icteric, the liver and spleen are enlarged. BP 60/20 mm Hg, pulse
120 per 1 min Optimum chart of treatment?
a. Hormones, desintoxical therapy, antibiotics
b. Diuretic, desintoxical therapy, antibiotics
c. Sorbtions, diuretic, desintoxical therapy
d. *Hormones, diuretic, desintoxical therapy, antibiotics
e. Hormones, diuretic, antibiotics
697.
Patient A., 57 years, animal technician, on the 3rd day of illness appealed to the
doctor with complaints of head pain, fever, pain in gastrocnemius muscles, cmall of the
back, icterus, dark urine and diminishing of its amount; temperature of body 38,2 °C ,
injection of vessels of sclerotica, petechial rash on overhead part of thorax, enlarged liver
and spleen. What preparations must be appointed above all things?
a. Intravenous introduction of salt solutions
b. Transfusion of fresh-frozen placma
c. *Setting of antibiotics
d. Hyperbaric oxygenetion
e. Haemosorbtion, sympathomimetics
698.
Patient of 43 years, miner, on the 4th day of illness complains of a sharp weakness,
high fever, onychalgias and cmall of the back, dark color of urine, head pain. Fell ill
sharply chill, temperature 40,3°, there was a nose-bleed. A diuresis 200 ml. What
preparations from the listed is necessary to use for this disease?
a. *Specific immunoprotein
b. Holinomimetics
c. Spacmolysants
d. Miorelaks
e. Vitamins of group B
699.
A patient, 44 years, entered infectious separation with a diagnosis leptospirosis. On
the 7th day of treatment his state was sharply worsened, pains appeared in cmall of the
back, somnolence, languor, cramps, head pain, a diuresis diminished to 100 ml/days. In
blood: red corpuscles 2,6 T/l, creatinine of 438 mcmoll/l, urea 13,0 mmol/l. What
complication developed in the patient?
a. Heart attack of buds
b. Acute hepatic insufficiency
c. Chronic pyelonephritis
d. Ischemic stroke
e. *Acute kidney insufficiency
700.
A patient 40 years complains of a sharp weakness, head pain, to pain in
gastrocnemius muscles, cmall of the back, insomnia, vomiting. Fell ill sharply 4 days
back, when did a chill appear, a temperature rose to 40°, myalgias, nose-bleeds, icterus.
Bathed in the river, where a lot of rodents is. The state is severe, on lips is herpes,
sclerotica injections, icterus. Hemorragic rash on a skin. Pulse 120 on 1 mines, BP 90/50
mm Hg. A liver and spleen is megascopic. The symptom of Pasternackogo is positive. For
days selected 100 ml of urine. Specify the most credible urgent state.
a. *Acute kidney insufficiency
b. Infectiously-toxic shock
c. Dehydratation shock
d. Head cerebral edema
e. Acute respiratory insufficiency
701.
A patient 42, animal technician, fell ill sharply: chill, temperature to 40°, head
pain, megalgias, is in muscles, especially gastrocnemius. Objectively: an icterus,
hemorragic rash, is expressed on a body, a liver and spleen are enlarged, positive
symptom of Pasternasky. Put a preliminary diagnosis.
a. Malaria
b. Ku-fever
c. *Leptospirosis
d. Spotted fever
e. Fever of Ebola
702.
Man 45 years, works as on a stock-raising farm a veterinary. Delivered in an
induction centre with complaints of discoloration urine, diminishing of diuresis,
fervescence to 39,3 °C, pain in muscles. It is ill during 5 days: the temperature of body
rose suddenly, great pains appeared in the muscles of shins, head pain, rash on a skin,
hemorrhage in a sclerotica, red color of urine. Pulse 56 per 1 mines, BP 90/60 mm Hg.
Uranalysis: squirrel of a 0,99 gramme/l, eras. 25-30 cylinders hyalin 8-10. Urea of blood
20,5 mmol/l. What diagnosis most credible?
a. *Leptospirosis
b. Acute glomerulonephritis
c. Urolithiasis
d. Cancer of urinary bladder
e. Rheumatoceils
703.
Sick, milkmaid diseased sharply, appeared great pain heads, broken, weakness,
profuse sweats, loss of appetite, dry cough, insomnia, myalgias, pain, in a lumbar areA. A
temperature from the first days rose to 39-40 °C. At a review it is found out hyperemia of
face, injections of sclera vessels, hyperemia of mucous of nasopharyngs. Pulse 80 per
minute. BP 90/60 mm/Hg. Lymphatic nodes are not palpable. Liver and spleen also not
palpable. About what disease is it possible to think?
a. Typhoid fever
b. Epidemic typhus
c. *Leptospirosis
d. Brucellosis
e. Flu
704.
Sick, milkmaid became sick sharply, severe head ache appeared. Weakness,
profuse sweats, loss of appetite, dry cough, insomnia, myalgias, pain in a lumbar area also
appeared. A temperature from the first days rise to 39-40 °C. During inspection hyperemia
of skin, injections of vessels of scleras, hyperemia of mucous membranes found out. Pulse
80 per min. BP is 90/60 mm/Hg. Lymphatic nodes are not palpable. Spleen and liver are
enlarged. How is it possible to confirm a credible diagnosis?
a. Virologicaly
b. *Bacteriologically
c. Exposure of exciter in emptying
d. Research of hanging drop of blood
e. Research of thick drop of blood
705.
Sick, milkmaid diseased sharply, appeared great pain heads, broken, weakness,
profuse sweats, loss of appetite, dry cough, insomnia, myalgias, pain, in a lumbar areA. A
temperature from the first days rose to 39-40 °C. At a review it is found out hyperemia
persons, injections of vessels of scleroticas, hyperemia of mucuse. Pulse 80/min. BP is
90/60 mm/Hg. Lymphatic nodes not palpable. Hepatoslpenomegaly. What etiotropic
treatment should be given?
a. Benzylpenicilline
b. *Теtracyclin
c. Acyclovir
d. Delagilum
e. Ftalazol
706.
32 years sick person, appealed to the doctor on the 7th day of illness with
complaints of high temperature, head pain, pain in muscles especially gastrocnemius. Skin
and sclera are icteric, on a skin hemorragic rash. HematuriA. 2 weeks ago rode on fishing.
Reliable diagnosis?
a. *Leptospirosis
b. Trichinosis
c. Brucellosis
d. Pyelonephritis
e. Hemorrhagic fever with a kidney syndrome
707.
45 years sick person, appealed to the hospital on the 5th day of illness. 8 days ago
he arrived from Laos with complaints of fever, headache, general weakness. Objectively:
temperature of body – 40,2 °C, skin moisture, scleroticas, subicteric, acrocyanosis, cardiac
tones, deaf, increase of liver and spleen. What complication more frequent in all develops
at the severe forms of this illness?
a. Coma
b. Edema of lungs
c. *ID-syndrome
d. Acute hepatic insufficiency
e. Acute kidney insufficiency
708.
Patient A., 35 years old, became ill suddenly, complain on the high temperature,
severe headache, pain in muscles of legs. Objectively (the 4th day of disease): the state is
severe, hyperemia of face, skin and scleras are icterus, spleen and liver are enlarged,
oligourhiA. What is the most credible diagnosis?
a. Yersiniosis
b. Viral hepatitis
c. *Leptospirosis
d. Poisoning by a tetraethyllead
e. Omsk‘s hemorrhagic fever
709.
Farmer, 57 years old, appealed to a doctor on the 3rd day of disease with
complaints on headache, high temperature, pain in the muscles, icterus of skin, dark urine
and decrease quantity of urine. Objectively: temperature – 38,2 °C, injection of scleras,
petehial rash on the overhead part of thorax, hepatosplenomegaliA. What is the most
reliable previous diagnosis?
a. Brucelosis
b. *Leptospirosis
c. Viral hepatitis
d. Yersiniosis
e. Trichinosis
710.
Chill appeared suddenly at the workwoman of pig farm, a temperature rose to 39,9
°C, there was a headache, nauseA. On the next day appeared pain in the muscles of lower
extremities, began the nose bleeding. On the 3rd day of disease, state was severe. A face is
hyperemic, scleritis, subicterous of scleras. Liver + 3 сm, diuresis – 700 ml. What is the
previous diagnosis?
a. Hemorrhagic fever with a kidney syndrome
b. Hepatitis A
c. Yersiniosis
d. Flu
e. *Leptospirosis
711.
Patient Q., 11 years old, complaints on weakness, headache, high temperature, pain
in the muscles; at night suddenly the temperature of body rose to 39,5 °C, decrease level
of urine. 4 days ago swimming in a lake, injured a leg. What preparations must be
appointed?
a. Prednizolonum, hot foot-baths
b. Febrifuge
c. Euphyllin, vitamin C
d. *Antibiotics
e. Sorbents
712.
Patient E., 24 years old, was hospitalized in the infectious department in the severe
condition. He complains of headache, pain in muscles, joints, vomiting. Objectively:
temperature of body 39 °C. BP is 90 and 60 mm Hg, appeared tachycardia, hyperesthesia
of skin. Meningeal signs are doubtful. It is known from anamnesis, that he lives in a
village, has a pet, rats. What preparations must be given?
a. Veroshpironum, euphilinum, dimedrol
b. *Manitol, lazix, prednizolonum, penicillin
c. Analgin, dimedrol, acetophen
d. Manitol, acetophen
e. Lazix, analgin, ampicillin
713.
Patient T., 22 years old, was hospitalized in the infectious department with
complaints of chill, fever, severe headache, pain in the gastrocnemius muscles. The state
is severe, hyperemia of face, icterus of skin and scleras, liver and spleen are enlarged,
olygouriA. What preparations must be given first?
a. Introduction of serum
b. *Ftorchinolons preparations
c. Sulphanilamid preparations
d. Dethintoxikation therapy
e. Sorbents
714.
Patient T., 22 years old, was hospitalized in the infectious department with
complaints of chill, fever, severe headache, pain in the gastrocnemius muscles. The state
is severe, hyperemia of face, icterus of skin and scleras, liver and spleen are enlarged,
olygouriA. BP is 60 and 20 mm Hg, pulse – 120 per a min. What preparations must be
given?
a. Hormones, dethintoxikation therapy, antibacterial preparations
b. Diuretic, dethintoxikation therapy, antibacterial preparations
c. Sorbents, diuretic, dethintoxikation therapy
d. *Hormones diuretic, dethintoxikation therapy, antibacterial preparations
e. Hormones, diuretic, antibacterial preparations
715.
Patient A., 57 years old, farmer, came to the doctor on the 3rd day of illness with
complaints of headache, pain in the gastrocnemius muscles, fever, icterus of skin and
scleras, dark urine and decrease level of urine, temperature of body – 38,2 °C, petehial
rash on overhead part of thorax, hepatosplenomegaliA. What preparations must be given?
a. Salts solutions
b. Transfusion of fresh-frozen placma
c. *Antibiotics
d. Hyperbaric oxygenation
e. Hemosorbcion, sympathomimetics
716.
Patient K., 43 years old, miner, on the 4th day of disease complained of weakness,
headache, pain in the gastrocnemius muscles, fever, icterus of skin and scleras, dark urine,
temperature of body – 38,2 °C, hepatosplenomegaliA. What preparations must be entered?
There was the nose-bleed. Diuresis – 200 ml. What preparations must be given?
a. *Specific immunoglobulin
b. Holynomimetics
c. Spacmolytics
d. Miorelaxants
e. Vitamins of group B
717.
At a patient, 39 years old: disease beginning suddenly, t° – 39,5 °C, pain in
muscles (especially in gastrocnemius), hepatosplenomegly, icterus, hemorrhagic rash,
oligouria, rigidity of the neck muscles. What is the most credible complications?
a. Renal insufficiency, reno-hepatic insufficiency, ІТS, ACVI, meningitis
b. Renal insufficiency, hepatic insufficiency, ІТS, ACVI, meningitis
c. Renal insufficiency, hepatic insufficiency, ІТS, ACVI, break of spleen
d. *Renal insufficiency, reno-hepatic insufficiency, ІТS, ACVI, uveit
e. Renal insufficiency, hepatic insufficiency, ІТS, ACVI, uveit
718.
Patient K, 32 years old, of no fixed residence went to see a doctor for 5-days illness
with complaints of fever, severe headache, insomniA. On body temperature 40 °C, pulse
110/min. The patient is excited and talkative. Hyperemic, scleritis. At whole over the body
there is pink petehial rashes. Positive Govorov-Godele‘s symptom. Enlarged liver and
spleen. What do you need to find out the epidemic status?
a. *The existence of head lice
b. Use of poor food
c. Availability of parenteral interventions
d. Contact with rodents
e. Drinking water from unhygienic sources
719.
A patient L., 32 years old, who complains of severe headache and fever, on the 6thday illness positive agglutination test with rickettsia reaction. What is the vector of the
disease?:
a. Flea
b. Fly
c. Mosquitoes
d. Bee
e. *Lice
720.
Patient A., 37 years old, entered to infectious hospital on the third day of the
disease in the severe condition. He complains of high fever with chills and sweat, general
weakness, pain in right under a rib. Objectively: temperature of body 41 °С, icterus of
skin, liver +2 cm, pain at palpation in abdomen, positive symptoms of Ortner and Mussy,
a spleen is normal, tachycardiA. What is the previous diagnosis?
a. Malaria
b. *Cholangitis
c. Viral hepatitis
d. Sepsis
e. Leptospirosis
721.
Patient W., 38 years old, entered to infectious hospital on the 5th days of disease in
the severe condition. He complaints of the high fever with chills and sweat, general
weakness, pain in sacrum. Objectively: temperature of body of 41 °С, tachycardia,
positive symptom of Pasternasky, liver and spleen is not normal. Preliminary diagnosis?
a. Malaria
b. Cholangitis
c. *Pyelonephritis
d. Sepsis
e. Leptospirosis
722.
Patient D., 39 years old, entered to infectious hospital on the second week of
disease in the severe condition. Local habitant, nowhere arrived. She is complaints for the
high fever with chills and sweat, general weakness. Attacks of fever without correct
periodicity. Objectively: the temperature of body – 41 °С, subicterus of sclera,
tachycardia, Pasrernacky‘s symptom positive, liver + 2 cm, spleen + 2 cm. What is the
preliminary diagnosis?
a. Malaria
b. Cholangitis
c. Pyelonephritis
d. *Sepsis
e. Leptospirosis
723.
Patient D., 33 years old, came to the infectious hospital on the second week of
disease in the severe condition. She is complains of high fever with chills and sweat,
general weakness, pharyngalgia at swallowing. Objectively: temperature of body 37,7 °С,
hyperemia of mucus of pharynx, tonsils are enlarged, loose, festering raid in lacunas,
enlarged submandibular, neck and axillar lymph nodes, icterus of sclera and skin,
bradycardia, liver + 2 cm, spleen + 1 cm. Urine is color of beer, an excrement is
discolored. What is the preliminary diagnosis?
a. Malaria
b. Infectious mononucleosis
c. Viral hepatitis
d. *Sepsis
e. Leptospirosis
724.
Patient K, 32 years old, of no fixed residence went to see a doctor for 5-days
illness with complaints of fever, severe headache, insomniA. Body temperature 40 °C, pulse
110/min. The patient is excited and talkative. Hyperemic, scleritis. The body is ocvered
pink petehial rashes. Positive Govorova-Godele‘s symptom. Enlarged liver and spleen.
What do you need to find out the epidemic status?
a. *The existence of head lice
b. Use of poor food
c. Availability of parenteral interventions
d. Contact with rodents
e. Drinking water from unhygienic sources
725.
A patient L., 32 years old, who complains of severe headache and fever, on the
6th-day of illness has positive agglutination test with rickettsia reaction. What is the vector
of the disease?:
a. Flea
b. Fly
c. Mosquitoes
d. Bee
E. *Lice
726.
A patient L, 72 years old, who complaints of severe headaches and fever, had 6day illness with positive agglutination test with rickettsia reaction. In the past he was sick
with typhus. Vector of the disease is:
a. Flea
b. Fly
c. Mosquitoes
d. Lice
e. *Carrier does not need a vector
727.
A patient 29 years old, few days ago, had a chill, the temperature for 2-3 days
increased to 39-40 °C. There was hyperemia and edema on the person, significant sclera
like "drunk" person and "rabbit" eyes. On the third day of illness – on mucous of soft
palate, parenthesis bright red enanthema is seen. In 3-4-days patient tried to protrude the
tongue, there was hyper movement, tremor, moving it to the side. Diagnosis: epidemic
typhus. Which disease is likely in the patient?
a. *Epidemic typhus
b. Typhoid fever
c. The disease Brill
d. Paratyphoid A
e. Paratyphoid B
728.
A patient on the 4th-day of fever has profuse rosy-petechial exanthema
predominantly located on the lateral surface of the torso and limbs. Hyperemic, vascular
conjunctivitis, petechial transition of anterior fold conjunctivA. What kind of illness do
you think it is?
a. Typhoid fever
b. *Epidemic typhus
c. Measles
d. Hemorrhagic fever with renal syndrome
e. Crimean hemorrhagic fever
729.
Patient B., complaints of headaches, delirium. When inspection: a body
temperature of 39 °C, initiated, a person hyperemic, positive symptom Govorova-Godele,
petechial rash on the trunk, limbs, tachycardia, hypotension, hepatosplenomegaly.
Drunker, drug user, living in the basement room type. What is your preliminary
diagnosis?
a. AIDS infection
b. Influenza
c. Leptospirosis
d. * Epidemic typhus
e. Alcohol psychosis
730.
A patient 28 years old, of no fixed abode, hospitalized with a preliminary diagnosis
«flu», a 5-day illness appeared rosy-petehia rashes on the body and interior surfaces of the
extremities. The temperature of 41 °C, euphoria, hyperemia person, redness sclera, tongue
tremor, tachycardia, enlarged spleen; excitement. What is the likely diagnosis?
a. Typhoid fever
b. Leptospirosis
c. Alcohol deliry
d. Measles
e. *Epidemic typhus
731.
A watchman 42 years old, desperately ill. Entered the 6-day illness with fever 39,7
°C, severe headaches, noise in the ears, insomniA. OBJECTIVE: instituted, euphoric,
talkative. Faceis red, his eyes shining, sclera and conjunctiva enhanced vascular pattern.
On the inside shoulder and the side surfaces of chest rosy-petechia rash. Found on head
lice and nits. Reaction Vidal 1:40. What is diagnosis?
a. Brill disease
b. Malaria
c. *Epidemic typhus
d. Typhoid fever
e. Q-fever
732.
A patient 28 years at the 4th-day of fever has profuse rosy-petechial exanthema
predominantly located on the lateral surface of the torso and limbs flexion surfaces. An
individual patient hyperemic expressed vessels conjunctival injection, petechial transition
to fold conjunctivA. Analysis of urine protein single hyaline and granular cylinders. What
kind of illness do you think it is?
a. Typhoid fever
b. Hemorrhagic fever with renal syndrome
c. Crimean hemorrhagic fever
d. Measles
e. *Epidemic typhus
733.
For a patients of 78 years acute illness began, which is characterized by fever with
chilling, rash dominated rosy petechia elements. Agent and the source of infection is not
detected, but it is known that 45 years ago patient had severe typhus. What is your
diagnosis?
a. Yersinia
b. *The Brill‘s disease
c. Typhoid fever
d. Drug allergy
e. Food Allergies
734.
The patient, a train conductor, 39 years old, hospitalized for a 4-day illness with
complaints of headache, weakness, dizziness, sweating, insomnia, fever. Hyperemic,
edema, conjunctivitis. At the transition fold conjunctiva – single petehies. At the skin
torso, chest, abdomen, limbs abundance rosy-petechia rash. TachycardiA. AD 100 and 60.
Tremor of the tongue. Palpable liver, spleen. What is the most likely diagnosis?
a. Leptospirosis
b. Typhoid fever
c. Influenza
d. Meningococcemia
e. *Epidemic typhus
735.
A patient 30 years on 9th day of illness that began gradually, the slow rise of fever
and intoxication, a painless rosy rashes on the skin of the abdomen. OBJECTIVE: pale,
temperature 40 ° C, pulse 80/min, AD 100 and 65 mm RT. Art. Language is coached,
abdomen bubbles, enlarged spleen and liver. For what disease you can suggest in the first
place?
a. *Typhoid fever
b. Epidemic typhus
c. Measles
d. Scarlet fever
e. Sepsis
736.
Patient 20 years old, complained about the high temperatures of up to 39 ° C,
headache in the frontal area, pain in the eyeball, photophobia, pain in muscles, dry cough.
Acutely ill the day before. Objective: condition serious, hyperaemic, eyes shining,
vascular injection sclerA. Pulse 96/min, rhythmical. Tone heart weakened. In the lungs
scattered dry bubbling rale. Faces hyperemic, granular. Meningeal symptoms are not
present. Blood tests: Leu. 3?109, eoz. 1 %, yang neu. 6 %, neu. 51 %, lymph. 35 %, mon.
7 %. What is the most likely diagnosis?
a. Measles
b. *Influenza
c. Meningococcal disease
d. Pneumonia
e. Typhus
737.
Often, in patient with epidemic typhus arise transition petechias in the conjunctivA.
What term is it called?
a. Symptom of Heller
b. Conjunctivitis
c. Symptom of Govorov-Godele
d. *Symptom of Zorohovich-Chiari-Avtsyna
e. Exanthema Rosenberg‘s
738.
Often, in patient with epidemic typhus arises petechia on mucosal soft palate. What
term did it call?
a. Symptom of Heller
b. Conjunctivitis
c. Symptom of Govorova-Godele
d. Symptom of Zorohovich-Chiari
e. *Exanthema Rosenberg‘s
739.
Often, in patient with epidemic typhus the tongue is in tremor when protruded it
sticks on the lower teeth. What term did it call?
a. Symptom of Heller
b. Conjunctivitis
c. *Symptom of Govorova-Godele
d. Symptom of Zorohovich-Chiari
e. Exanthema Rosenberg‘s
740.
A patient 29 years, a few days ago, a chill, the temperature for 2-3 days increased
to 39-40 ° C. There hyperemia and edema person, significant sclera like "drunk" person
and "rabbit" eyes. On the third day of illness – on mucous of soft palate, parenthesis bright
red exanthema is seen. At 3-4-day patient when trying to protrude the tongue, there was
hyper movement, tremor, rejecting it to the side. Diagnosis: epidemic typhus. What is the
mechanicm of infection?
a. Air-dropping
b. Fecal-oral
c. Contact Residential
d. Parenteral
e. *Vector borne
741.
Patient A., 37 years old, entered to infectious hospital on the third days of disease
in the severe condition. He complaints of the high fever with chills and sweat, general
weakness, pain in right under a rib. Objectively: temperature of body 41 °С, icterus of
skin, liver + 2 cm, pain at palpation in abdomen, positive symptoms of Ortner and Mussy,
a spleen is normal, tachycardiA. What methods is it possible to confirm a previous
diagnosis by?
a. General blood analysis
b. *Ultrasonic research
c. Biochemical blood test
d. Hem culture
e. Parasitoscopy of blood
742.
Patient W., 38 years old, entered to infectious hospital on the 5th days of disease in
the severe condition. He complaints of the high fever with chills and sweat, general
weakness, pain in sacrum. Objectively: temperature of body of 41 °С, tachycardia, and
positive symptom of Pasternascky sign, liver and spleen not is normal. What is the
previous diagnosis? What methods is it possible to confirm a previous diagnosis by?
a. Global analysis of blood
b. Ultrasonic research
c. Biochemical blood test
d. Hem culture
e. *Parasitoscopy of blood
743.
Student of 22 years, gradually ill: hyperemia and erythema in area of right forearm,
which was gradually increased to 10 cm in a diameter, appeared. Erythema has a center
and raised bright red edges. The temperature of body is sub febrile, general state
satisfactory. 5 days prior to beginning of illness gather mushrooms in the forest. What is
our preliminary diagnosis?
a. Erysipelas
b. *Lime disease
c. Ebola
d. Erysipelas
e. Herpetic infection
744.
Student F., 22 years old, gradually ill: hyperemia and erythema in area of right
forearm, which was gradually increased to 10 cm in a diameter, appeared. Erythema has a
center and raised bright red edges. The temperature of body sub febrile, general state is
satisfactory. 5 days prior to beginning of illness walked in the forest after mushrooms.
What laboratory methods will confirm the diagnosis?
a. Common analysis of blood
b. Hem culture
c. parasitoscopy of blood
d. RMA with leptospirosis
e. *RIGA with borrelliea
745.
Student of 22 years, gradually ill: hyperemia and erythema in area of right forearm,
which was gradually increased to 10 cm in a diameter, appeared. Erythema has a center
and raised bright red edges. The temperature of body is sub febrile, general state
satisfactory. 5 days prior to beginning of illness gather mushrooms in the forest. What is
etiotropic therapy?
a. *Tetracycline
b. Metronidazole
c. Normal immunoglobulin
d. Glucocorticoids
e. Enter sorbents
746.
Patient A., 28 years old, was treated in a neurological department during 2 months
concerning neuritis of facial nerve. In anamnesis - 2 years ago was treated due to
polyarthritis (right knee and talocrural joints). Lives in a mud flow, likes to gather
mushrooms. Presence of ticks does not deny. What is laboratory and instrumental methods
of research will allow specifying a diagnosis?
a. Biological test on botulicm
b. EEG
c. Computer tomography of a brain
d. RNGA with borrelliea
e. *Lumbar puncture
747.
Patient A., 28 years old, was treated in a neurological department during 2 months
concerning neuritis of facial nerve. In anamnesis - 2 years ago was treated due to
polyarthritis (right knee and talocrural joints). Lives in a mud flow, likes to gather
mushrooms. Presence of ticks does not deny. What is etiotropic therapy?
a. *Benzyl penicillin
b. Sulfanilamide’s
c. Normal immunoglobulin
d. Glucocorticoids
e. Antiviral drugs
748.
Patient B., 38 years old, was treated from myocarditis in a cardiologic department.
There are violations of cardiac conductivity on the type of atrioventricular blockade,
tachycardiA. The temperature of body is sub febril. Arthritis of right knee-joint. Works as
a forest ranger, likes to gather mushrooms and berries. What is treatment?
a. Benzyl penicillin
b. Nonsteroid ant inflammatory drugs
c. Glucocorticoids
d. Physical therapy
e. *All above enumerated
749.
Patient P., 35 years old, was treated from myocarditis in a cardiologic department.
There are violations of cardiac conductivity on the type of atrioventricular blockade,
tachycardiA. The temperature of body is. Arthritis of right knee-joint. Works as a forest
ranger, likes to gather mushrooms and berries. What laboratory and instrumental methods
of research will allow you to specify the diagnosis?
a. Serology
b. ECG
c. X-rays investigation
d. *RIGA with borrelliea
e. Biochemical blood test
750.
A man 28 years old was appealed to the policlinic. The day before he found a tick
in the surface of his skin. He was in-field 2 days ago. He feels satisfactory. What measures
of Lime disease prophylaxis?
a. Treatment of the place of bite
b. *Urgent antibiotic therapy
c. Using of specific immunoglobulin
d. Enter sorbents
e. Vitamins
751.
Citizen A. was a participant in a business geologic expedition. Area is endemic to
Lyme disease. How to prevent possible infection?
a. *Individual protecting from ticks
b. Urgent antibiotic prophylaxis
c. Vaccination
d. Using of specific immunoglobulin
e. All above enumerated
752.
Patient of 52 years fell ill sharply from a chill and head pain, fevers to 40°C. On a
3th day illnesses are nausea and vomiting, dark urine. On a 4th day a temperature went
down to 37 °, but an icterus appeared and the amount of urine diminished to 600 ml. What
disease is such development of symptoms characteristic for?
a. Hepatitis A
b. Hemorrhagic fever with a kidney syndrome
c. *Leptospirosis
d. Sepsis
e. Acute glomerulonephritis
753.
A patient L, 72 years old, who complaints of severe headache and fever, the 6-day
illness positive agglutination, test with rickettsia reaction. Past sick was typhus. Vector of
the disease is:
a. Flea
b. Fly
c. Mosquitoes
d. Lice
e. *Carrier does not need
754.
A patient 29 years, a few days ago, a chill, the temperature for 2-3 days increased
to 39-40 °C. There hyperemia and edema person, significant sclera like "drunk" person
and "rabbit" eyes. On the third day of illness – on mucous of soft palate, parenthesis bright
red enantema is seen. At 3-4-day patient when trying to protrude the tongue, there was
hyper movement, tremor, rejecting it to the side. Diagnosis: epidemic typhus. Which
disease is likely in a patient?
a. *Epidemic typhus
b. Typhoid fever
c. The disease Brill
d. Paratyphoid A
e. Paratyphoid B
755.
A patient at the 4-day fever has profuse rosy-petechial exanthema predominantly
located on the lateral surface of the torso and limbs flexion surfaces. Hyperemic, vascular
conjunctivitis, petechial transition of anterior fold conjunctivA. What kind of illness can
think?
a. Typhoid fever
b. *Typhus
c. Measles
d. Hemorrhagic fever with renal syndrome
e. Crimean hemorrhagic fever
756.
Patient B. complaints of headaches, delirium. When inspection: a body temperature
of 39 ° C, initiated, a person hyperemic, positive symptom Govorova-Godele, petechial
rash on the trunk, limbs, tachycardia, hypotension, hepatosplenomegaly. Drunker, drug
user, living in the basement room type. What is your preliminary diagnosis?
a. AIDS infection
b. Influenza
c. Leptospirosis
d. *Typhus
e. Alcohol psychosis
757.
A patient 28 years old, of no fixed abode, hospitalized with preliminary diagnosis
«flu», a 5-day illness appeared rosy-petechial rashes on the body and interior surfaces of
the extremities. The temperature of 41 °C, euphoria, hyperemia person, redness sclera,
tongue tremor, tachycardia, enlarged spleen; excitement. What is the likely diagnosis?
a. Typhoid fever
b. Leptospirosis
c. Alcohol delirium
d. Measles
e. *Epidemic typhus
758.
A patient 28 years at the 4-day fever has profuse rosy-petechial exanthema
predominantly located on the lateral surface of the torso and limbs flexion surfaces. An
individual patient hyperemic expressed vessels conjunctiva injection, petechias transition
to fold conjunctivA. Analysis of urine protein single hyaline and granular cylinders. What
kind of illness can think?
a. Typhoid fever
b. Hemorrhagic fever with renal syndrome
c. Crimean hemorrhagic fever
d. Measles
e. *Epidemic typhus
759.
For patients 78 years of acute illness began, which is characterized by fever with
chilling, rash dominated rosy petechial elements. The condition of the patient violated
moderately. Agent and the source of infection are not detected, but it is known that 45
years ago patient had severe typhus. What is your diagnosis?
a. Lyme disease
b. * Brill‘s disease
c. Typhoid fever
d. Drug allergy
e. Food Allergies
760.
The patient, a train conductor, 39 years old, hospitalized for a 4-day illness with
complaints of headache, weakness, dizziness, sweating, insomnia, fever. Hyperemic,
edema, conjunctivitis. At the transition fold conjunctiva – single petechias. At the skin
torso, chest, abdomen, limbs abundance rosy-petechial rash. TachycardiA. AD 100 and
60. Tremor of the tough. Palpable liver, spleen. Stool arrested. What is the most likely
diagnosis?
a. Leptospirosis
b. Typhoid fever
c. Influenza
d. Meningococcemia
e. *Epidemic typhus
761.
A patient 30 years on 9th day of illness that began gradually, the slow rise of fever
and intoxication, a painless rosy rashes on the skin of the abdomen. OBJECTIVE: pale,
temperature 40 ° C, pulse 80/min, AD 100 and 65 mm RT. Art. Language is coached,
abdomen bubbles, enlarged spleen and liver. For what disease you can suggest in the first
place?
a. *Typhoid fever
b. Epidemic typhus
c. Measles
d. Scarlet fever
e. Sepsis
762.
Patient 20 years, complained about the high temperatures of up to 39 ° C, headache
in the frontal area, pain in the eyeball, photophobia, pain in muscles, dry cough. Acutely
ill the day before. Objective: condition serious, hyperemic, eyes shining, vascular
injection sclerA. Pulse 96/min, rhythmical. Tone heart weakened. In the lungs scattered
dry bubbling rile. Faces hyperemic, granular. Meningeal symptoms are not present. Blood
tests: Leu. 3*109, eoz. 1 %, yang neu. 6 %, neu. 51 %, lymph. 35 %, mon. 7 %. What is
the most likely diagnosis?
a. Measles
b. *Influenza
c. Meningococcal disease
d. Pneumonia
e. Typhus
763.
Patient K., 23 years old., hospitalized on the 3rd day of illness, which was
accompanied by mild running nose, high fever to 40,2 °C, headache and hemorrhagic rash
on the skin. In the 2 hours after the introduction of penicillin blood pressure dropped to 40
and 10 mm RT. Art. Distal pulse and meningeal signs are not defined. What is the
diagnosis in a patient?
a. *Meningococcemia, infectious-toxic shock
b. Epidemic Typhus, severe course
c. Measles, severe course
d. Scarlet fever, severe course
e. Flu, anaphylactic shock
764.
A patient 25 years old, who returned from the Far East, suddenly increased body
temperature to 39 °C, a pain in backbone, reddening skin type «hood», single
hemorrhages on the skin. After 3 days, along with declining fever, weakness, thirst,
decreased diuresis to 300 ml, decreased blood pressure. Much pronounced Pasternatsky‘s
symptom. What is the most likely diagnosis?
a. Leptospirosis
b. Typhus
c. Hemorrhagic fever Crimean-Congo
d. Acute glomerulonephritis
e. *Hemorrhagic fever with renal syndrome
765.
Male 45 years old, fell ill after 2 weeks after returning from Afghanistan, where six
months ago underwent malariA. In return were found lice. Suddenly, there were severe
headache, weakness, body temperature 39 ?C with a temporary decrease in the 4th day of
illness, followed by general weakness, much intoxication, headache, appeared on the body
abundance polymorphic rash. At the 7-day state of heavy, the phenomenon of
encephalitis, excitation, hallucinations periodically. Objective – erythematos-petechial
exanthemA. The temperature of the body 40,1 °C, and pulse was 136 for 1 min, blood
pressure 120/70 mm RT art. Moderately enlarged liver and spleen. In the blood analys
neutrofil mild leukocytosis, eozinofilia, accelerated ESR. Which of the studies of blood
will be positive?
a. At typhoid fever
b. At the three-day malaria
c. Tropical malaria
d. *At epidemic typhus
e. At brucellosis
766.
Male 26 years old, who last month returned from Africa and was processing about
lice, sick 5 days. Home was a sudden: severe headache, weakness, body temperature 38,8
°C, which persisted all day at a constant level, but at the 4th day of the disease declined
for several hours. Then the patient condition has deteriorated significantly, a rash on the
body, delay urine. OBJECTIVE: pulse was 110 for 1 min, blood pressure 115/70 mm RT.
Art., temperature 39,7 °C. Face red, a significant injection of vascular sclera, and cmall
hemorrhages in the conjunctiva, mucous membrane soft palate, and abundant rosypetechial exanthema on the body. Signs interstitial pneumonia, encephalitis. Enlarged
liver and spleen. What kind of illness can think?
a. *Epidemic typhus
b. Typhoid fever
c. Tropical malaria
d. Leptospirosis
e. Yellow fever
767.
Patient P., 68 years old, fell ill suddenly 7 days ago from the increase of
temperature to 39,3 °C, appearances of headache and insomniA. Objectively: excited,
inadequate. Face is hyperemic. The tongue is very dry, trembles at pulling out. On a trunk
are polymorphic rash, tachycardia, and blood pressure low. Hepatosplenomegaly. Stool is
absent. In age 10 years had epidemic typhus. What is the previous diagnosis?
a. Yersiniosis
b. Typhoid fever
c. *Brill‘s disease
d. meningococcemia
e. Sepsis
768.
Patient P., 76 years old, complied during 7 days for the permanent increase of
temperature to 38,2-38,7 °C, headache, insomnia, horrific dreams, dry cough, myalgia’s,
arthalgias. Polymorphic rash appeared on a trunk on the 4th day of diseases. In childhood
was ill by epidemic fever, three-day malariA. Temperature of body – 38,4 °C, pulse – 98
per a min, bilateral pneumonia confirmed roentgenologic. In the analysis of blood
moderate neutrophil leucocytosis. What is the previous diagnosis?
a. *Brill‘s disease
b. Typhoid fever
c. Lime‘s disease
d. Malaria
e. Leptospirosis
769.
Patients I., 78 years old, became ill saddenly, the temperature of body rose to 39,2
°C, euphoric, vessels of scleras are injection, hyperemia of face, appears enantema of
Rozenberg. Reaction with Rickettsia prowazekii 1:160, IGG – 87 %. What is your
diagnosis?
a. *Brill‘s disease
b. Typhoid fever
c. Meningococcal infection
d. Flu
e. Leptospirosis
770.
Patients D., 30 years old, became ill saddenly, when the temperature of body rose
to 40,2 °C, appeared headache, weakness, euphoria, injection of vascular sclera,
hyperemia of face, appears the positive symptom of Govorov-Godele. Reaction with
Rickettsia prowazekii 1:160, IgG – 87 %. What is your diagnosis?
a. Brill‘s disease
b. Typhoid fever
c. Paratyphoid A
d. Paratyphoid B
e. *Epidemic typhus
771.
A patient with a pediculosis the temperature of body rose to 41,6 °C, appeared
headache, euphoria, in 4th days from the beginning of illness – rose-petechial rash.
Reaction with Rickettsia prowazekii 1:640, IgG – 89 %. What is your diagnosis?
a. Flu
b. Typhoid fever
c. Meningococcal infection
d. *Epidemic typhus
e. Leptospirosis
772.
25 years sick, which got back from Far East, suddenly temperature of body rose up
to 39,9 °C, pain appeared, rash as a “jacket”, single hemorrhages on a skin. In 3 days
weakness, thirst, grew together with the decline of fever, diminished to 300 ml diuresis
considerably expressed Pasternatsky symptom. What is most credible diagnosis?
a. *Hemorrhagic fever with a kidney syndrome
b. Congo-Crimea hemorrhagic fever
c. Epidemic typhus
d. Glomerulonephritis
e. Leptospirosis
773.
Person 26 years old, month ago got back from Africa and passed treatment on an
occasion a pediculosis, are ill 5 days. Beginning was sudden: great head pain, a weakness,
38,8 °C, was saved all days at permanent level, but on the 4th day of illness went down on
a few hours. Farther the state of patient became worse considerably, a rash, coughing,
appeared on a trunk. Objectively: temperature – 39,7 °C, pulse – 110, BP - 115/70 mm/hg.
The skin of person is red, considerable injection of sclera, shallow hemorrhages on a
conjunctiva, mucous shell of soft palate, abundant petechial-еrythematos Signs of
іnterstitial pneumonia, encephalitis. A liver and spleen is megascopic. What disease is
most credible?
a. Dengue fever
b. Typhoid
c. Tropical malaria
d. Leptospirosis
e. *Epidemic typhus
774.
Patient B., 45 years old, appealed to the hospital on the fourth day of illness with
complaints of a high temperature, head pain, in muscles Two weeks ago a patient arrived
from Far East. Objectively: temperature – 39,6 °C, face and neck hyperemic, vessels of
the sclera’s and conjunctivitis. On a skin under collar-bones, shoulder-blades, necks, face
is petechial rash in the type of chains. Pasternatsky‘s symptom is positive on both sides.
What changes is characteristic for this disease in hemo gramme?
a. *Leukocytosis
b. neutropenia
c. Monocytosis
d. ESR is low
e. Eosinophilia
775.
Sick patient, 27 age, 3th day of illness, became ill sharply. Grumbles about a chill,
head myalgia, pain in muscles and joints, nausea, vomit. Objectively: hyperemia and
puffiness of person, neck, thorax, petechial, “rabbit eyes”, photophobiA. Pulse – 110.
Hepatomegaly. A patient arrived from Zaire. Your diagnosis?
a. Viral hepatitis
b. Flu
c. Leptospirosis
d. *Yellow fever
e. Malaria
776.
A patient, 39 years old, became ill sharply 3 days ago after return from JamaicA.
Complaints on a high temperature, severe headache, nausea, vomit by “coffee- brown”,
pain in the muscles and joints. Objectively: temperature 39,9 °C, face is swollen, red,
conjunctivitis, photophobiA. Rashes. Sclerotic, and skin is yellow, petechial rash,
acrocyanosis. A liver is latge3 сm, painful on palpation. Tachycardia, hypotension. In
hemograme are leukopenia, thrombocytopeniA. Describe the most likely diagnosis.
a. *Yellow fever
b. Viral hepatitis B
c. Malaria
d. Leptospirosis
e. Hemorrhagic fever with a kidney syndrome
777.
At two nurses who care for the patients with fever from southern Sudan in a
hospital, appeared a fever, headache, pain in muscles, joints, abdomen, general weakness
and diarrheA. On the 4th day of the disease maculo-papular rash appeared on a body, on a
5th day was the bloody vomiting, melena, hypotension, and deafness of cardiac tones. The
condition was extremely heavy. Put a previous diagnose.
a. Leptospirosis
b. Spotted fever
c. Dengue fever
d. *Ebola-fever
e. Malaria
778.
A patient 26 years entered permanent establishment in 3 weeks upon termination of
reaping. The state is heavy, chronotaraxis and space, high fever to 40 °C, on a skin
petechial and еchimosis, on the back linear hemorrhages, sanguifluousness from a nose
and gums, there was vomit by “coffee-grounds”. Pasternatsky symptom is sharply
positive. At the laboratory inspection: neutrophilic leukocytosis, thrombocytes – 100
thousand in 1 ml, hematocrit – 0,55, day's amount of urine 70 ml, protienuria,
еrythrocyturiA. What disease does it follow to suspect?
a. *Hemorrhagic fever with a kidney syndrome
b. Leptospirosis, anicteric form
c. Salmonellosis, septic form
d. Lassa hemorrhagic fever
e. Crimean hemorrhagic fever
779.
A boy 12 years old is hospitalized in an infectious department. Became ill 7 days
ago from appearance of a headache, hyperthermia up to 40,3 °C, pain in muscles, general
weakness. In 2 days after normalization of temperature of body the appeared hemorrhagic
rash on the skin and mucus’s, nasal bleeding, icterus, hepatospleenomegaly. Three weeks
ago got back from rest in South AfricA. There have been repeated mosquito bites. What
infectious disease does it follow to suspect above all things?
a. Dengue fever
b. *Yellow fever
c. Leptospirosis
d. Malaria
e. Q-fever
780.
A boy 10 years of age, who lives in Transcarpathia, on the 4th day of fever
appeared painless hemorrhagic petechial rushes, sometimes in the form of red and purple
stripes with overwhelming localization on a neck, thorax, in the axillary, above collarbones. On hyperemic mucus of oro-pharing point hemorrhages, bleeding from a nose. In
lungs was hard breathing, tones of the heart deaf, bradycardia, swelling and abdomenache, enlarged a liver and spleen. Oliguria, proteinuria, hematuria, cylindruriA. In blood
neutrophilic leukocytosis, thrombocytopenia, accelerated ESR. What diagnose is previous
the most reliable.
a. Meningococcal infection
b. *Hemorrhagic fever
c. Rheumato cells
d. Leptospirosis
e. Thrombocytopenic purpura
781.
In ambulance delivered a girl 14 years, which became ill 6 days ago from
appearance of weakness, increase body temperature to 39,3 °C, enanthema appeared on
the skin and mucous and had hemorrhagic character, icterus, nasal bleeding, increases of
liver and spleen. 2 weeks ago got back from a festival in Brazil. What infectious
pathology should be eliminated first?
a. *Yellow fever
b. Q-fever
c. Dengue fever
d. Malaria
e. Viral hepatitis
782.
A 30-year-old resident of Peru brought to the hospital at the 4th day of illness,
there was vomiting with blood, nose bleeding, icteric skin, petechial. In urine revealed
erythrocytes protein. Volume of urine decreased. What is the preliminary diagnosis?
a. *Yellow fever
b. Dengue Fever
c. Malaria
d. Lassa fever
e. Ebola fever
783.
Family week ago got back from the trip on AfricA. In a few days at a child 10
years the temperature of body rose to 40 °C, on a next day vomit, diarrhea with the
admixtures of mucus, blood. The state gets worse gradually. On a 4th day on the skin of
extremities, trunk appeared single hemorrhage, hemorrhage on a soft palate. What
credible disease from will you settle tactic of conduct of patient coming?
a. *Hemorrhagic fevers
b. Grigoriev-Shiga dysentery
c. Leptospirosis
d. Meningococcal infection
e. Salmonellosis
784.
The soldier, who arrived on vacation from Sierra Leone, was taken to hospital in
connection with the febrile illness. Suspected Lassa fever. What kind of laboratory
indicators is characteristic for this disease?
a. The decline in serum indicators KFK and LDG
b. ESR greater after 60 mm/hour
c. Neutrophil in liquor
d. Elevated levels of alkaline phosphatase in the serum
e. *High proteinuria
785.
At patient, who acted to permanent establishment, on clinical epidemiological
indexes of Lassa fever is suspected. What from the resulted clinical indexes are not
characteristic for this disease?
a. *Hemolytic icterus
b. Generalized lymphadenopathy
c. Conjunctivitis
d. Ulcerous pharyngitis
e. Encephalopathy
786.
At patient, who acted to permanent establishment, the especially dangerous
hemorrhagic fever is suspected. For the selection of exciter can be used all materials,
except for?
a. Blood
b. *Puncture from lymph nodes
c. Urines
d. Pleura liquid
e. Swab from nasopharyngeal
787.
In a patient who is at the hospital about high fever disease, which arose up
suddenly, a diagnosis of Ebola fever. The severity of the state can be attributed to the
development of the following emergency conditions, except for:
a. *Acute respiratory insufficiency
b. Infectious-toxic shock
c. Hypovolemic shock
d. Hemorrhagic shock
e. Acute kidneys insufficiency
788.
In a serviceman, who acted to permanent establishment, set previous diagnosis of
especially dangerous hemorrhagic fever. What from the transferred terms of latent period
does follow at determination of duration of primary disease measures?
a. 6 days
b. 10 days
c. 40 days
d. *21 day
e. 72 hours
789.
At a patient with the hemorrhagic fever with a kidney syndrome a diuresis
decreased to 300 ml, the рН of blood is 7,0; increase the level of creatinine and urea on
the blood. Intensive therapy is conducted. Which of the funds should not be considered
appropriate?
a. *Hypotension drugs
b. Heparin
c. Introduction of entero sorbent solution
d. Introduction 4 % solution of sodium bicarbonate
e. Saluretics
790.
At a resident of Colombia on the 3th day of illness was a high temperature, head
pain, marked pains in back and extremities appear. At the inspection: face is hyperemic,
edema, scleritis. Tongue is assessed by the white coats, pulse frequent. Abdomen is soft,
sickly in a еpigastric areA. 7 days ago got back from forests. What is previous diagnosis?
a. Ebola fever
b. Dengue fever
c. Hemorrhagic fever with a kidney syndrome
d. Lassa fever
e. *Yellow fever
791.
You are flying in an airplane from the Republic of Zaire. Waitress asks to help a
child 12 years old. In inspection – at the torso, limbs, palms and soles are determined
multi bubbles. Parents indicate that over 4 days before the child's fever was 40 °C. Rash
developed gradually, on the first day of the face, on the second day on the body, on the
third day in the limbs. What is the best tactic?
a. *The immediate message to ground air traffic controllers on the case, the
suspect cmallpox
b. Reassure parents, said that the state is a manifestation of allergic reaction
c. Immediately enter dexamethasone
d. Immediately begin antibiotic therapy
e. Immediately enter vaccine
792.
To you, as to the graduating student of medical university, is possibly to work upon
termination of studies in CrimeA. What endemic ticks infections is in this territory?
a. *Crimean hemorrhagic fever, tick encephalitis, Q-fever
b. Crimean hemorrhagic fever, malaria, epidemic typhus
c. Tick encephalitis, Ebola fever, Lassa fever, Crimean hemorrhagic fever
d. Q-fever, spotted fever, leptospirosis
e. Q-fever, Crimean hemorrhagic fever, psittacosis, tick encephalitis
793.
Sick 37 years person on Crimea grumbles about a high temperature, nonpermanent vomit head pain. On the 4th day of illness: the state is severe, temperature of
body 38,9 °C, skin pale, mucous shells are hyperemic. On a soft palate is hemorrhagic
enanthema, hemorrhages in sclera reveal. On lateral area of trunk – petechial rash. Pulse is
weak, 120/min, BP – 90/50 mm/hg. Abdomen is soft, painless. What from therapeutic
facilities not are prescribed in this case?
a. Cyclopheron
b. *Biseptol
c. Glucocorcoids
d. Suprastine
e. Depiridamol
794.
Patient 25 years, which got back from Far East, suddenly temperature of body
raised to a 39 °C, pain appeared in back, erubescence as “jacket”, single hemorrhage on a
skin. In 3 days a weakness, thirst, grew together with the decline of fever, diminished to a
300 ml diuresis, the arteriotony went down. Considerably expressed Pasternasky
symptom. What diagnosis is most credible?
a. Typhoid fever
b. Crimea-Congo hemorrhagic fever
c. *Hemorrhagic fever with a kidneys syndrome
d. Acute glomerulonephritis
e. Leptospirosis
795.
Patient 39 years, grumbles about the high temperature, sharp pain in a head, back,
muscles of extremities, photophobiA. At a review is pallor of nasolabial triangle,
hyperemia of person, neck, overhead half of trunk, by the positive Pasternatsky symptom
on either side. Eyeing cracks lips, injection of sclera’s. Mucous of mouth bright red with
point hemorrhages. On a neck, lateral surfaces of thorax, in arm-pits fossulas, above the
collar-bones of petechial rushes in the form of asterisks and forms group as red or violet
strips. What diagnosis is most credible?
a. Marburg fever
b. Congo-Crimean hemorrhagic fever
c. *Hemorrhagic fever with a kidney syndrome
d. Ebola fever
e. Lassa fever
796.
Patient complaints of the increasing body temperature up to 40,2 °C, pain in the
head, joints and muscles, limbs and back, abdomen, frequent vomits. Patient is excited.
Neck, overhead part of breasts are hyperemic, scleratis and conjunctivatis of both eyes.
Mucous of oral cavity is hyperemic with punctulate еnanthema, soft palate is filling out.
On the 2-4th day of illness on the skin of lateral surface of trunk, and arm-pits areas, on
abdomen and extremities appeared petechial rushes oval form with the clear contours of
darkly-cherry blossom. Simultaneously mucosal bleeding appeared from gums, nose,
lights, digestive channel uterus. Liver is megascopic. Positive Pasternaskiy‘s symptom.
What diagnosis is most credible?
a. Yellow fever
b. Congo-Crimea hemorrhagic fever
c. *Hemorrhagic fever with a kidneys syndrome
d. Lassa fever
e. Ebola fever
797.
Sick person 34 years old, became ill suddenly – rose temperatures of body,
vomiting. Patient disturbs pain in a head, back, joints, and bones. Hyperemia and
puffiness of face and neck, eyes are poured by a blood. Patient is excited. The yellow
coloring of skin and sclera’s appeared on the 3th day of illness, point hemorrhages on a
skin, liver and spleen was multiplied. What diagnosis is most credible?
a. *Yellow fever
b. Congo-Crimea hemorrhagic fever
c. Hemorrhagic fever with a kidneys syndrome
d. Lassa fever
e. Ebola fever
798.
Sick C., 33, appealed on the 5th day of illness. In second day the temperature of
body was raised to 40 °C, delirium happened. An icterus grew. Vomits was dark-brown
color, stool – dark (melena). Petechial and ecchymosis exanthema appeared on the skin of
trunk and extremities. There was the nasal and uterine bleeding. Oliguria, blood and
cylinders in urine, azotemiA. In the blood test – leukopenia – (1,5-2,0?109), neutropenia,
increasing of ESR. Hyperbilirubinemia (due to both fractions of pigment), increasing of
activity of aminotransferases, in urine was bilirubin, urobilin. What diagnosis is the most
credible?
a. *Yellow fever
b. Viral hepatitis A
c. Viral hepatitis B
d. Viral hepatitis hepatic encephalopathy
e. Crimean hemorrhagic fever
799.
Patient, 30 years old, lives in Egypt. Became ill sharply. A temperature with a chill
rose to 39 °C and was contained next 3 days. Disturbed headache, severe muscle pain. The
skin of person is hyperemic, injection of sclera and conjunctiva vessels, hyperemia of
nasopharyngs mucous. Pulse – 80/min, lowered blood pressure. Increasing of lymphatic
nodes. Phenomena of bronchitis in lungs, cough insignificant, not productive. Petechial
rush on the skin of overhead and lower extremities. Bleeding of gums, nose-bleeds.
Enlarged liver. How do to confirm a credible diagnosis?
a. Bacteriological
b. *Virologic
c. Research of drop of blood in the dark field
d. Research of hanging drop of blood
e. Research of thick drop of blood
800.
The Patient, a hunter, lives in Russia (Ural), arrived to Ukraine 2 weeks ago.
Became ill sharply. A temperature rose to 39 °C. Disturbed severe headache, pains in the
back, muscles, extremities. There was vomit, photophobiA. Decline of sharpness of sight.
Consciousness is some darkened. Hyperemia of face, pallor of nasolabial triangle,
injection of sclera’s vessels. Petechial rush pouring out trunk as strips. Pulse 110 per 1
min. Tones of heart is hypostheniA. Tongue is dry, abdomen is blown away. Megascopic
liver. Sharply positive Pasternatskiy‘s symptom. OliguriA. In urine: protein, hailing, fibrin
cylinders, renal epithelium cells. About what disease is it possible to think?
a. hemorrhagic fever
b. Yellow fever
c. Leptospirosis
d. *Hemorrhagic fever with a kidneys syndrome
e. Crimea hemorrhagic fever
801.
Sick, 35 years, habitant of CrimeA. Became ill sharply. With a chill a temperature
rose to 39,5 °C. Disturbed pain of head, joints, in muscles, frequent vomits. Face, neck,
overhead part of thorax, is hyperemic. Sclerotic, conjunctivitis. Petechial rush over the
trunk. The state got worse. Uterine and intestinal nose-bleeds joined. Hyperemia of face
changed by a pallor and puffiness. Patient a dynamic, consciousness is darkened.
OliguriA. In spite of haematostatic therapy patient died. About what disease is it possible
to think?
a. Lassa fever
b. Yellow fever
c. Q-fever
d. *Crimea hemorrhagic fever
e. Omsk hemorrhagic fever
802.
A man, a resident of Nigeria, became ill sharply. Appeared a chill, repeated
vomiting, body temperature had risen to 39,5 °C. A patient is excited. Hyperemia and
puffiness was marked of face, expressed sclerotic, hyperemia of mycoses mouth cavity.
Pulse 130 per 1 min. The icterus of skin and sclera appeared on a 3th day, hemorrhages on
a skin. A liver and spleen is enlarged. After the brief improvement in the 5th day of illness
the state became worse sharply. A temperature again rose to 39,9 °C. Raves. An icterus
grew sharply. Vomit moderate. Vomiting and feces of black color. Nose bleeding.
Petechial enanthemA. Tachycardia changed on bradycardiA. Blood pressure 80/50
mm/hg. OlyguriA.About what disease is it possible to think?
a. Lassa fever
b. *Yellow fever
c. Q-fever
d. Crimea hemorrhagic fever
e. Omsk hemorrhagic fever
803.
A woman, a resident of Nigeria, became ill sharply. Appeared a chill, repeated
vomiting, body temperature had risen to 39,7 °C. A patient is excited. Hyperemia and
puffiness was marked of face, expressed sclerotic, hyperemia of mycoses mouth cavity.
Pulse 130 per 1 min. The icterus of skin and sclera appeared on a 3th day, hemorrhages on
a skin. A liver and spleen is enlarged. After the brief improvement in the 5th day of illness
the state became worse sharply. A temperature again rose to 39,9 °C. Raves. An icterus
grew sharply. Vomit moderate. Vomiting and feces of black color. Nose bleeding.
Petechial enanthemA. Tachycardia changed on bradycardiA. Blood pressure 80/50
mm/hg. OliguriA.
a. Viral hepatitis A
b. Leptospirosis
c. *Yellow fever
d. Crimean fever
e. Malaria
804.
At workers laboratories, which looked after marmosets which were brought from
Uganda, great pain appeared heads, myalgia’s, dryness in a mouth cavity, nausea, vomits,
frequent watery emptying, afterwards pains joined in the area of lower ribs which
increased at a cough. On the 4th day of disease a hemorrhagic syndrome appeared are
hemorrhages in a conjunctiva, sanguifluousness of gums, blood in vomiting. Maculopapular pouring out on a trunk. Credible diagnosis?
a. Lassa fever
b. *Marburg fever
c. Leptospirosis
d. Ebola fever
e. Crimean fever
805.
A patient, habitant of Nigeria, grumbles about the increase of temperature, general
weakness. Gradually a temperature rose to 38,9 °C, pain of head increased, pains appeared
in muscles, nausea, and conjunctivitis. On a 3th day necrotizing ulcer pharyngitis
developed. Ulcers appeared on soft palatine tonsils. The groups of lymphatic nodes of
neck were multiplied. The state got worse. Abdomen-aches joined, vomit, diarrhea, pain
in a thorax, cough. Relative bradycardiA. About what disease is it possible to think?
a. Flu
b. Typhoid fever
c. *Hemorrhagic fever
d. Quinsy
e. Spotted fever
806.
Patient 45 years old, approached into the hospital on a 5th day of illness, 8 days
ago he arrival from Laos, complaining of fever, headache, general weakness.
OBJECTIVE: body temperature is 40,2 °C, skin moist, sclera inject, subichteric,
acrocyanosis, cardinal tone dull, the increase of liver and spleen. What does the most often
complications develop in malignant forms of this disease?
a. Coma
b. Lung edema
c. *IVD syndrome
d. Acute liver failure
e. Acute kidney failure
807.
Patient G., complaints on the increased of body temperature to 39-40 °C, a sharp
pain in the head, in the back pain, in the muscles of limbs, photophobiA. Objectively:
nosolabial triangle was pallor; face, neck, half upper torso was hyperemic, a positive
Pasternatskiy‘s symptom on both sides. Gaps narrowed eyes, sclerotic. Mucous membrane
of faces bright was red with point hemorrhages. At the neck, thorax side surfaces, in
axillary, over collarbone was petechial rash in the form of stars and grouped in the form of
red or violet strips. What is the most likely diagnosis?
a. Yellow fever
b. Crimean-Congo hemorrhagic fever
c. *Hemorrhagic fever with renal syndrome
d. hemorrhagic fever
e. Crimean hemorrhagic fever
808.
Patient A., 25 years old, is being treated concerning tetanus. Choose the specific
treatment.
a. Antibiotics
b. *Immunoglobulin
c. Anticonvulsant medicine
d. Cardiac preparations
e. Respiratory analeptics
809.
Patient D., 47 years old, veterinary doctor, admitted to infections department on
2nd day of disease in quite severe condition with expressed intoxication, high temperature
(40 °C); general weakness, exhausting cough with runny bloody sputum. Objectively:
cyanosis of mouth, pulse 128 per minute, mucous membranes of mouth cavity lightly
hyperemic, due to significant edema of nasal and pharyngeal mucous present impediment
breathing, during lungs auscultation – dry and moist rales, in permutation – areas of
consolidations. Deaf heart tones, AP - 85/50 mmHg. From epidemiologic anamnesis
revealed, that patient during a week till the onset of disease work with ill animals. Part of
the animals died. What is the most possible diagnosis?
a. Community-acquired pneumonia
b. *Anthrax, lung form
c. Plague, lung form
d. Atypical pneumonia (SARS)
e. Q-fever
810.
In a villager, on back surface of right fist appeared itching papule, in the center of
which is present swelling with ichoric content. In next 2 days develops edema of fist and
forearm. On 4th day increase of body temperature, in axillaries region of right revealed
large painful lymphatic nodule. One day before the beginning of disease, patient had
contact with dead calf. What is the most possible diagnosis?
a. Plague, skin-bubonic form
b. *Anthrax ulcer, skin form
c. Common carbuncle
d. Tularemia, skin-bubonic form
e. Sepsis
811.
To villager an itching papula appeared on the back surface of right hand, in the
center of which a bubble with ichor content appeared. During the next two days developed
edema of hand and forearm . At 4th day appear body temperature, in right axillary region
observe large painful lymph node. Patient examined a dead bull a day ago to beginning of
illness. Most credible diagnosis?
a. Plague, skin-bubonic form
b. *Anthrax, skin form
c. Banal carbuncle
d. Tularemia, skin-bubonic form
e. Sepsis
812.
Young woman, a milkmaid, was admitted to the infectious department. Became ill
3 days ago. An itching node appeared on the neck. She continued to work, her state
became worse on the eve of hospitalization, body temperature – 38,1?C, edema at neck
and upper part of thorax, with serous-hemorrhagic excretions, not painful. No animal
disease was registered during the last year in diary form, where she works. What diagnosis
is it necessary to suspect?
a. Furuncle
b. Tularemia
c. Cu-fever
d. *Anthrax
e. Sap
813.
2 men admitted in the clinic of infectious diseases, attendant of cattle farm. Both
complaints on increase of body temperature, headache, and injury of skin of hands like
ulcers uncoated by crusty black covering. Became ill 3 days ago. At first at the skin of
hands appeared 2-3 red spots, then in the place of spots appear bubbles, bursting with
formation of the ulcers covered by a dense black scab. Around the scab appeared inflamed
red colored secondary vesicles, all is surrounded by the soft studnevidnim edema but not
painful. Patients care for the ill bull 4 days ago to illness. What is the most credible
diagnosis?
a. Plague
b. Erysipelas
c. Carbuncle
d. *Anthrax
e. Tularemia
814.
Men D., 44 years old, received a suspicious letter. A letter must be read, however a
recipient fears that his content can be potentially dangerous in regard to anthrax. In this
situation, it is necessary:
a. To through it, not unsealing
b. *Cover a letter by a moist gauze and iron it on heated till 250°C
c. Burn out a letter
d. Deliver it in sanepidemic unit
e. Send back
815.
A patient has the suddenly expressed edema in half face and neck. In the center of
edema is the black scab surrounded by cmall vesicles. Paracervicle and submandibular
lymph nodes are increased in size. The changes on face are not accompanied by pain.
During examination, patient has temperature 37,5 °C. What is the most credible
diagnosis?
a. *Anthrax
b. Plague
c. Tularemia
d. Erysipelas
e. Furuncle
816.
Patient A., 43 years old, was admitted in infectious hospital with complaints on a
high body temperature and severe headache. He is ill 2 days. By examination observe
carbuncle on forearm with the expressed edema around it and insignificant painful.
Regional lymphadenitis. It is known from anamnesis that a patient works on a stockraising farm. About what disease is it necessary to think about?
a. Erisipelias
b. Cancer of skin
c. Erysipelas
d. *Anthrax
e. Eczema
817.
A patient W., 40 years old, veterinary, is ill 3 day: insignificant weakness, body
temperature – 38 °C, at right forearm – ulcer of diameter 1,5 cm, covered by dark brown
scab, surrounded by hyperemia with cmall vesicles. Expressed edema of soft tissue of
forearm. Enlarged and sensible lymph nodes on the right elbow and arm pit. On the hands
fresh scratches. What is the most credible diagnosis?
a. *Anthrax
b. Erysipelas
c. Felines
d. Staphylococcal carbuncle
e. Tularemia, ulcerous-bubonic form
818.
Patient A., complaints of redness of skin and edema at the right cheek. During
examination: body temperature – 38,7 °C, painful and edematous submandibular lymph
nodes on the right side, can clearly differentiate between hyperemia and healthy skin,
bubbles with a dark liquid inwardly, painful palpation. Your preliminary diagnosis?
a. *Erysipelas, hemorrhagic form
b. Anthrax, skin form
c. Herpetic infection
d. Chicken-pox
e. Phlegmon of cheek
819.
Patient V., appealed to the surgeon with expressed edema of left half of neck.
During examination a doctor observe a carbuncle on the lateral surface of neck and a few
cmall bubbles near it, filled with a rather yellow liquid. Submandibular and anterior
cervical lymph nodes are enlarged and painful. What diseases is the most credible?
a. *Anthrax
b. Diphtheria
c. edema
d. Plague
e. Carbuncle of neck
820.
Patient P., 40 years old, is a farmer. He is ill during 3 days. He complaints of sub
febrile temperature, headache. There is a large edema on the left arm. There is a vesicle
(0,3x0,5 cm) in the center of it; which contains the serous and bloody liquid, painless,
with considerable peripheral erythemA. What is clinical diagnose?
a. Plague
b. Erysipelas
c. Carbuncle
d. Tularemia
e. *Anthrax
821.
Young woman, a milkmaid, was admitted to the infectious department. Became ill
3 days ago. An itching node appeared on the neck. She continued to work, her state
became worse on the eve of hospitalization, body temperature – 38,1 °C, edema at neck
and upper part of thorax, with serous-hemorrhagic excretions, not painful. No animal
disease was registered during the last year in diary form, where she works.
a. What method is possible to confirm a diagnosis?
b. Examination of blood in the dark field
c. RIFA with anthrax antigen
d. *Bacteriologic examination of content of ulcer
e. Bacteriologic examination of blood
822.
Patient C. has got significant edema of half of face and neck. In the middle of
edema there is a black scab, surrounded by cmall vesicular. There are enlarged
submandibular and frontal neck lymph nodes. The changes on face are not accompanied
with pain. Temperature of body at examination is 37,5 °C. What is the most credible
diagnosis:
a. *Anthrax
b. Plague
c. Rabbit-fever
d. Erysipelas
e. Furuncule
823.
A teenager was bitten by a neighbor’s dog. This dog is not instilled; it’s on a leash
and bit the boy after provocation in the area of hand. What should be done for the
prophylaxis of rabies?
a. Elimination of dog
b. Inoculations on vital testimonies
c. *Conditional course of inoculations
d. Surgical treatment of the wound
e. All answers are correct
824.
Woman of 33 years was attacked and bitten by a bat in the area of hand
aggressively and bit a woman on her hand. During the laboratory research the diagnosis of
rabies was confirmed. What should be done for the urgent prophylactic of the rabies in
this case?
a. Human rabies immunoglobulin 18 doses of anti rabies vaccine
b. 12 doses of anti rabies vaccine
c. *Human rabies immunoglobulin and 6 doses of vaccine
d. Human rabies immunoglobulin and 21 dose of vaccine
e. 2 doses of vaccine
825.
Wild dog bite man for fingers of the left hand. What kind of specific prophylaxis
should be conducted for this patient?
a. *Human rabies immunoglobulin 6 doses of vaccine
b. 12 doses of vaccine
c. Human rabies immunoglobulin and 16 doses of vaccine
d. Human rabies immunoglobulin and 21 dose of vaccine
e. 2 doses of vaccine
826.
Patient A., 25 years old, is being treated concerning tetanus. Choose the specific
treatment.
a. Antibiotics
b. *Immunoglobulin’s
c. Anticonvulsant medicine
d. Cardiac preparations
e. Respiratory analeptics
827.
During a walk in-field a fox attacked on a child, bit his feet and racemes of hands
and disappear. An animal disappeared. What is medical tactic.
a. To process a wound surgically
b. *To process a wound soapy solution, to enter an immunoprotein and vaccine
c. Preventive vaccination
d. To process wounds surgically, to enter an human rabies immunoglobulin and
vaccine
e. Preventive human rabies immunoglobulin
828.
A 23 years old person, became ill sharply: fever 38.2 °C, moderate diffuse
pharyngalgia at swallowing, pain and itching in the right eye. Objectively: tonsillitis,
pharyngitis, conjunctivitis. What is previous diagnosis?
a. Adenoviral infection
b. Enteroviral infection
c. Para influenza
d. Flu
e. Acute respiratory infection
829.
A 25 years old patient, fell ill rapidly, with chills and temperature rose to 39,9,
headache appeared in front temporal regions, pain in eyeballs, dull pain in all trunk, closed
nose. Dry cough after 2 days of illness, there was nose bleeding. Objectively: hyperemia
with sputum, isolated petechial rash and shallow grittiness of soft palate. Difficult
breathing in lungs. What is the most possible diagnosis?
a. Leptospirosis
b. Adenoviral infection
c. Typhoid fever
d. Flu
e. Epidemic typhus
830.
A child 10 years old with temperature 38,0 °C, conjunctivitis, moist cough,
hyperemia of the mucous membranes of cheeks and lips. Gums are pallor. What is your
diagnosis?
a. Measles
b. *Adenoviral infection
c. Acute respiratory viral infection
d. Enteroviral infection
e. Infectious mononucleosis
831.
A child 3 years old is found in the grave condition – naughty, forced breathing, dry
«barking» coughing, voice is hoarse, perioral cyanosis. The third day, temperature of body
is sub febrile, mild common cold. In lungs single dry wheezes can be heard. Moderate
tachycardiA. For which disease these symptoms are characteristic?
a. Localized diphtheria of oropharynx
b. Whooping-cough
c. *Para influenza, false croup
d. Bronchopneumonia
e. Adenoviral infection
832.
A child of age 2 years has temperature of body 37.3 °C, cold, hoarse voice
“barking cough” appeared suddenly the anxiety, shortness of breath, appeared with
participation of auxiliary muscles. Supposed diagnosis?
a. *Para influenza, false croup
b. Diphtheria croup
c. Allergic laryngitis, croup
d. Flu, laryngitis
e. Acute exudative pleurisies
833.
A patient 14 years old, hospitalized in the infectious department in severe condition
with considerable headache mainly in frontal and temporal area, pain in eyeballs, in
muscles and joints. Objectively: patient is excited, temperature of the body is 39 °C.
Bradycardia changed by tachycardiA. Muscles tonic and colonic cramps. Positive
meningeal signs. It is found in epidemic anamnesis, his brother is also sick. What is your
diagnosis?
a. *Flu with pneumonia and edema of brain
b. Flu, typical course
c. Para influenza, false croup
d. Respiratory-syncytial infection
e. Adenoviral infection, pneumonia
834.
A patient 17 years old, became suddenly ill: temperature rose to 40,3 °C. Severe
headache, motive excitation, frequent vomiting, tremor of fingers of extremities.
Hemorrhagic spots of round form and different sizes, more frequently as stars, mainly on
buttocks and trunk. Meningeal signs are positive. What is the most possible diagnosis?
a. Encephalitis
b. Flu with a hemorrhagic syndrome
c. *Meningococcal infection
d. Measles
e. Leptospirosis
835.
A patient 17 years, 11th class student, were a lot of cases of ARI (acute respiratory
infection) have happened, appealed to a doctor in clinic at 3rd day of disease with
complaints of chills, general weakness, a moderate sore throat, running nose, swelling of
face, watering from eyes. Objective examination: minor palatal hyperemia brackets and
tonsillitis, on a background of moderate edema of tissues. Conjunctivitis. During palpation
not painful enlarged inframaxillary lymph nodes, and enlarged neck lymph nodes were
found. Crepitation can’t be found. Liver and spleen moderately increased. What is the
most likely diagnosis?
a. Diphtheria
b. *Adenoviral infection
c. Meningococcal nasopharyngitis
d. Influenza
e. Infectious mononucleosis
836.
A patient 18 years old, with complaints of headache, pharyngalgia, weakness, high
temperature. Objectively: all groups of lymph nodes, 1-3 cm in a diameter, dense, elastic,
enlarged, hepatospleenomegaly. Blood analysis: leukocytosis, mononuclear – 15 %. What
is possible diagnosis?
a. *Infectious mononucleosis
b. Adenoviral infection
c. Angina
d. Diphtheria
e. Acute lymphocytosis
837.
A patient 20 years old fell ill rapidly with increasing of temperature to 39.9 °C.
Complaints of headache in front temporal region, pain in eyeballs, dull ache in whole
trunk, closed nose, scrapes in the throat, dry cough. There was nose-bleeding. What
diagnosis is most possible?
a. *Influenza
b. Adenoviral infection
c. Para influenza
d. RS-infection
e. Enteroviral infection
838.
A patient 20 years old, complaints of increasing of temperature up to 39 act,
headache in frontal area, pain in eyeballs, photophobia, pain in muscles, dry cough.
Became ill sharply. Objectively: severe state. Face is hyperemic, eyes brilliant, injections
of sclera’s. Pulse 96/min, rhythmic, tones of heart arehypostheniac. Meningeal symptoms
are not present. Blood analysis: leuk 9*109, е 1 %, bands 6 %, seg 51 %, lymp 35 %,
mono 7 %. What is the most possible diagnosis?
a. *Influenza
b. Adenovirus infection
c. Leptospirosis
d. Pneumonia
e. Epidemic typhus
839.
A patient 26 years old, became sick rapidly: temperature 39.5 оC, severe headache,
mainly in frontal and temporal areas, pain in muscles and joints. Examined on the 2nd day
of illness: state of middle weight, skin is clean. Moderate hyperemia with cyanosis, pulse
120 per min, rhythmic. Heart activity is rhythmic, tones are muffled, in lungs there is
vesicular breathing. What is the treatment of this patient?
a. Aspirin
b. *Remantadin
c. Ampicillin
d. Ascorbic acid
e. Ribonuclease
840.
A patient 27 years old, entered clinic on the 4th day of illness with a diagnosis
ARVI, allergodermiA. Fell ill with the rise of temperature to 38,0 °C, headache,
hyperemia of the throat, then barking cough appeared. On the 3rd day rash appeared on
the skin and neck. Was treated by aspirin. Objectively: temperature 38.8 °C. Face is puffy,
conjunctivitis. On the skin of neck and upper part of chest is an abundant red-papular rash
as rings which do not itch. Mucosa of epiglottis is brightly hyperemic. Submandibular and
neck lymphadenitis. Liver and spleen were not enlarged. What is your diagnosis?
a. Measles
b. *Allergic dermatitis
c. Infectious mononucleosis
d. German measles
e. Scarlet fever
841.
A patient 56 years old, the day before felt easy indisposition, insignificant
headache, and weakness. Afterwards the increasing of temperature appeared to 38,5 оC
with chills, headache increased considerably, mainly in forehead and temples. Skin and
conjunctiva is hyperemic, dry, barking cough. Pharynx is hyperemic. On soft palate
present grainy granules, placed point hemorrhages. Difficult breathing. What is the most
possible diagnosis?
a. Typhoid fever
b. Leptospirosis
c. Epidemic typhus
d. *Flu
e. Enteroviral infection
842.
A patient becomes sick very fast: chills, increasing of temperature to 40,1 оC,
headache in frontal and temporal regions, pain in eyeballs, close nose, dry cough and pain
in the chest. The nose bleeding, nausea, double vomits. Objectively: conjunctivitis,
hyperemia, edema, hemorrhages in mucous of otopharhynx, tachycardiA. Blood pressure
is low. Difficult breathing . What is the most possible diagnosis?
a. Meningococcemia
b. Epidemic typhus
c. Leptospirosis
d. *Flu
e. Typhoid fever
843.
A patient 56 years old, workwomen of pig farm, on a background chills appeared,
the temperature rose to 39,9 °C, headache, and nauseA. The next day marked pains in the
muscles of lower extremities appeared, the nose bleeding began. At the receipt, on the 3rd
day common state deteriorated. Hyperemic spots, subecteric appeared. Liver +3 cm. daily
– diuresis 700 ml. What is the previous diagnosis?
a. Hemorrhagic fever with a kidney syndrome
b. Hepatitis A
c. Escerichiosis
d. Flu
e. *Leptospirosis
844.
A patient A., 30 years old, on the 4th day of illness a district doctor marked such
subjective and objective data: insignificant indisposition, mild headache, hoarseness of
voice, itching in throat, breaking dry cough, temperature of the body 37,4 °C. Pulse
86/min., difficult nasal breathing, insignificant serous excretions from nose. Which acute
respiratory infection does the patient carry?
a. Influenza
b. *RS-viral infection
c. Para influenza
d. Adenoviral infection
e. Enteroviral infection
845.
A patient admitted in the infectious department with diagnosis of acute respiratory
viral infection. Became ill suddenly, the disease is accompanied by the increase of
temperature of body till 39 °C, by severe headache, mainly in area of frontal, temporal,
above eyes, dryness in nose, itching in throat, dry cough, and dull pain in all body. He had
bleeding from nose twice at home. Which acute respiratory disease has the patient?
a. Adenoviral infection
b. RS-infection
c. *Flu
d. Para influenza
e. Enteroviral infection
846.
A patient C., was hospitalized on the 2nd day of illness with complaints of
hoarseness of voice, rough barking cough, labored breathing. Objectively: the state is
severe, uneasy, pallor, temperature 37.1 °C, BR 30/min., breathing is noisy, can hear from
the distance, with participation of auxiliary musculature. Which viruses could cause
development of similar state?
a. Rhino virus
b. Influenza virus
c. Adenovirus
d. *Para influenza virus
e. Cytomegalovirus
847.
A patient caused a doctor home. Age – 75 years. Complaints of a sub febrile
temperature, general weakness, pharyngalgia, conjunctivitis. In family a child is ill the
acute adenoviral disease. A patient considers itself a patient the second day. At a review
are found out the signs of acute blepharoconjunctivitis, pharyngitis. There are megascopic
lymphatic knots: neck front and back, arm-pits and inguinal, to 1 cm in a diameter, soft,
not is soldered between itself and with surrounding cellulose. A pharynx is hyperemic,
tonsils are hypertrophied and hyperemic. In lights of wheezes it is not. Breathing clean.
Tones of heart are muffled. BP is 140/80 mm Hg. Ps – 80 per 1 minute. Abdomen soft. A
megascopic liver which comes forward on 3 cm below costal arc and spleen are palpated –
soft, painless. Choose the most credible diagnosis:
a. *Adenoviral infection
b. Flu
c. Megablastoma
d. Infectious mononucleosis
e. Hepatitis A
848.
A patient fell ill very rapidly: chills, increase of temperature to 40.1 °C, headache
in front temporal regions, pain in eyeballs, close nose, dry cough, and chest pain. Nose
bleeding, nausea, vomiting appeared after 4 hours. Objectively: conjunctivitis, hyperemia,
edema, point hemorrhages in mucus of epiglottis, tachycardiA. Blood pressure is low.
Weaken breathing in the lungs. What is the most possible diagnosis?
a. Leptospirosis
b. Epidemic typhus
c. *Flu
d. Meningococcemia
e. Enteroviral infection
849.
A patient H., 22 years old, with flu was hospitalized into infectious department
with the acute worsening of the common state. Consciousness is stored. The patient
strangles. Pallor of skin with cyanosis. Respiratory rate 50 per min, AP 80/55 mmHg,
pulse 110 per a min, temperature 39.8 оC. During percussion of lungs tympanic sound
with dullness in lower quadrant was found. Crackles in the lower-back parts of lungs.
What complication of influenza has developed in that patient?
a. Pneumonia
b. Edema of lungs
c. Edema of brain
d. *Infectious-toxic shock
e. Meningoencephalitis
850.
A patient on the background of ARVI the fever developed to 40,1 °C, frequency of
breathing is 40 for a minute. What measures are necessary?
a. *Decreasing of patients temperature
b. Artificial ventilation
c. Oxygen. inhalation
d. Infusion therapy
e. Antibiotic therapy
851.
A patient P., 14 years old, is hospitalized in the infectious dept. in grave condition.
Complaints on headache, mainly in frontal and temporal regions, superciliary arcs,
vomiting on severe pain, pain on movement of eyeballs, in muscles, joints. Objectively – a
patient is excited, temperature of the body 39 °C. BP 100/60 mmHg. Bradycardia was
replaced by tachycardiA. Tonic cramps appeared. Doubtful meningeal signs. From
anamnesis it is found that at home his brother has flu. What preparations must be injected?
a. Verospiron, euphyllin, dimedrol
b. *Mannitol, paracetamol, prednisolone, euphyllin
c. Analgin, dimedrol, aspirin, ampicillin
d. Mannitol, aspirin
e. Lasix, analgin, ampicillin
852.
A patient with flu complicated by pneumonia, during some days there are the
displays of infectious-toxic shock of ІІ degree. In BA the level of urea and keratinize
increases. What from these preparations is not recommended to enter in such a situation?
a. *Adrenalin
b. Prednisolone
c. Polyconic solutions
d. Dofaminum
e. Heparin
853.
A patient Т., 45 years old, was hospitalized at the 2nd day of disease. One week
ago got back from India (sailor of the distant swimming). Complaints of temperature 41.3
°C, great headache, shortness of breathing; cough with foamy pink color sputum.
Objectively: pale of face, cyanosis of mucous, breath rate 24/min, tachycardiA. Lungs:
breathing is hyposthenia, moist wheezes in both lungs, crepitation. What is possible
diagnosis?
a. Flu
b. Miliary tuberculosis
c. *Plaque, pulmonary form
d. Leptospirosis
e. Sepsis
854.
A patient, 20 years old, during few days complaints of pharyngalgias. After
supercoiling the state became worse: sudden chills, increase of temperature to 40.6 °C,
headache. On the skin of low extremities, trunk and buttocks there are a lot of different
sizes hemorrhagic spots, acrocyanosis. Consciousness is preserved. Meningeal signs are
absent. What is the previous diagnosis?
a. *Meningococcal infection
b. Flu
c. Epidemic typhus
d. Hemorrhagic fever
e. Leptospirosis
855.
A student, 18 years old, for 7 days complaints of weakness, hyperthermia to 37.8
°C, mucous excretions from a nose, pharyngalgia at swallowing, pain in eyeballs.
Objectively: increased lymph nodes of neck and mandible, lymphadenitis, edema and
injection of conjunctiva, hyperemia of mucous of epiglottis, hypertrophy of tonsils. What
is the most reliable diagnosis:
a. *Adenoviral infection
b. Influenza
c. Infectious mononuleosis
d. Rhinoviral infection
e. Parainfluenza
856.
A woman 27 years old, complaints of the general weakness, absence of appetite,
coughing, fever up to 37.5 °C for three weeks. Ulcerous illness of abdomen, myocarditis
is in anamnesis. What inspection is primarily need to do?
a. Electrocardiography
b. Fibrobronchoscopy
c. Fibrogastroscopy
d. *Fluorography
e. Common blood analysis
857.
In a patient of 16 years old, the disease began gradually, from the catarrhal
syndrome. For 2-3 days the temperature of body increase till 38,5 °C, cold, severe cough
with the negligible quantity of mucous sputum, «souring» eyes. Peripheral lymph nodes
are soft, painless and some enlarged. Mucous of pharynx is hyperemic, granules on
posterior part of pharynx. General state is satisfactory. What is the drug of choice?
a. Ascorbic acid
b. Aspirin
c. *Deoxyribonucleas
d. Remantadin
e. Aminocapronic acid
858.
In girl V., 1 year old, appeared the thump of nose, dry cough, body temperature
rose till 37,5 °C. Next day cough become attack like with the excretion of cmall amount of
viscid sputum. Noisy breathing. Sharply expressed expiratory dyspnea, breating rate 40
times/minute. During examination: acrocyanosis and emphysematous thorax, at lungs
dissipated dry and single moist rales. Tear of frenulum of tongue. What will be the
preliminary diagnosis?
a. *Parainfluenza
b. Pneumonia
c. Influenza
d. Respiratory-syncytial infection
e. Whooping-cough
859.
A sick person, 45 years old, was hospitalized after 2 days of disease. On Sunday he
came back from India (sailor). Complaints of increasing of temperature to 41 оC, severe
headache, shortness of breath, cough, with sputum. Objectively: pallor, cyanosis of
mucous, tachycardiA. Breathing is weaken, crackles in the lower-back parts of the lungs,
crepitation. What is the possible diagnosis?
a. *Flu complicated by pneumonia
b. Miliary tuberculosis
c. Plague, pulmonary form
d. Leptospirosis
e. Sepsis
860.
A sick woman, 42 years old, complaints of temperature 39.3 °C, headache in the
frontal area, pain in the eyeballs, photophobia, pain in muscles, dry cough. Became ill
suddenly one day before. Objectively: state is severe. Hyperemia of the face, eyes shinny,
injection of scleras. Pulse 96/min., rhythmic. Tones of heart are hypotonic. Both lungs are
dissipated. Dry wheezes. Mucosa of epiglottis is hyperemic, grainy, vessels are extended.
Meningeal symptoms are not present. Analysis of blood: leuk – 3?109/l, еos – 1 %, band –
6 %, seg – 51 %, lymp – 35 %, mono – 7 %. What is the most possible diagnosis?
a. *Flu
b. Measles
c. Meningococcal infection
d. Pneumonia
e. Epidemic typhus
861.
In a patient of 16 years old, the disease began gradually, from the catarrhal
syndrome. For 2-3 days the temperature of body increase till 38,5 °C, cold, severe cough
with the negligible quantity of mucous sputum, «souring» eyes. Peripheral lymph nodes
are soft, painless and some enlarged. Mucous of pharynx is hyperemic, granules on
posterior part of pharynx. General state is satisfactory. What is the drug of choice?
a. Ascorbic acid
b. Aspirin
c. *Desoxyribonucleas
d. Remantadin
e. Aminocapronic acid
862.
Patient 22 years old, has increase temperature of body till 37,8 °C. Treated under
the supervision of district doctor with a diagnosis of influenzA. On the 5th day of illness
temperature remained the same; it began difficultly in opening eyes. On examination –
edema on face, expressed conjunctivitis with film stratifications. Mucous pharynx is
heperemia, on the back wall of gullet considerable graininess. Lymph nodes are enlarged
in neck. The general state of patient is satisfactory. This disease is related to cold. What
disease you suspect?
a. Leptospirosis
b. Infectious mononucleosis
c. *Adenoviral infection
d. Allergic dermatitis
e. Meningococcal infection
863.
Patient A., 28 years old, hospitalized with a previous diagnosis of flu. On the 5th
day of illness, rash appeared on the trunk and internal surfaces of extremities.
Temperature 41.5 °C, hyperemia of sclera, tremor of tongue, tachycardia, spleenomegaly,
excitation. What is the most possible diagnosis?
a. Measles
b. Meningococcal infection
c. Leptospirosis
d. *Epidemic typhus
e. Typhoid fever
864.
Patient B., 20 years old, complaints of severe headache in temples and orbits, dull
ache in the trunk, dry cough. Temperature of the body 39.6 °C. Inflammatory changes of
mucous membrane of oropharynx. Normal breathing in the lungs. What is the most
credible diagnosis?
a. Pneumonia
b. Parainfluenza
c. Respiratory micoplacma
d. *Flu
e. Meningococcal infection
865.
Patient L., 18 years old is sick with fever till 38 °C which proceeds 5 days. he has
moderate dry cough, common cold, badly opens eyes. On examination –edema on face,
expressed conjunctivitis with film raids. Mucous of pharynx is hyperemic, posterior wall
of pharynx is grainy. Internal organs are without pathology. What form of disease does the
described picture correspond to?
a. Viral conjunctivitis
b. Allergic dermatitis
c. *Adenoviral infection
d. Influenza
e. Rhinoviral infection
866.
Patient M., 11 years old, complaints on general weakness, cough, at night suddenly
temperature rose till 39,5 °C, appeared restlessness, barking cough, noisy whistling
breathing with drowing in supra- and subclavicular cavities, intercostal spaces. He was in
contact with the patient acute respiratory viral infection. What should recommend him the
first line?
a. *Prednisolon, hot foot-baths
b. Seduxsen, euphylin
c. Euphylin, vitamin C
d. Antibiotics, dimedrol
e. Astmopen, diazolin
867.
Patient P., 14 years old, is hospitalized in the infectious department in the severe
condition. Complaints on expressed headache, mainly in frontal and temporal regions,
supercilliary arcs, origin of vomiting appear in condition of severe pain, pains by moving
the eyeballs, in muscles and joints. Objectively: patient is excited, body temperature-39
?C. BP-100/60 mmHg. Bradycardia was replaced by tachycardiA. Appeared tonic cramps.
Doubtful meningeal signs. From anamnesis it is clear that his brother has flu at home.
What will be your diagnosis?
a. Influenza, typical flow
b. *Influenza with the phenomena of edema of brain
c. Respiratory-syncytial infection
d. Parainfluenza
e. Adenoviral infection
868.
Patient R., 16 years old, hospitalized for 5-day illness with complaints of moderate
headache in fronto-temporal region, laid nose, sore throat, pain in the left eye, rise in
temperature to 38.1-38.5 °C. General condition is satisfactory. Shortness of nasal breath,
mucous discharging from the nose, hyperemia of face, enlargment of the neck and
submaxillary lymph glands, left foamy conjunctivitis. What is preliminary diagnosis?
a. Influenza
b. Infectious mononucleosis
c. Enteroviral infection
d. *Adenoviral infection
e. parainfluenza
869.
Sick M., 22 years old, complaints of increasing of body temperature to 39 оC,
headache in frontal area, pain in eyeballs, photophobia, pain in a muscles, dry cough.
Became ill suddenly. The state is heavy. Objectively face is hyperemic, injection of
scleras. Pulse 96 per min, rhythmic. Tones of heart are hypotonic. In the lungs – dissipated
dry wheezes. Mucous membrane of oropharynx is hyperemic, grainy, vessels are
extended. menengial symptoms are not present. Analysis of blood: leukocytes 3*109/L, е
1 %, band neut. 6 %, seg. neut 51 %, lymphocytes 35 %, мonocytes 7 %. What is most
probable diagnosis?
a. Measles
b. *Flu
c. Meningococcal disease
d. Epidemic typhus
e. Pneumonia
870.
The patient P., 14 years old, is suffering from flu. He is hospitalized in infectious
dept. due to worsening of his condition. He is conscious. A patient is suffocated. Pallor of
skin covers with cyanosis, breathing rate 50 times/minute. BP-80/55 mmHg, pulse 110
times /minute. Body temperature-39,5 °C. Excretion of rose foamy sputum. On percussion
of lungs there is tympanic sound with dullness in lower part of lung .On auscultation there
is moist rales in lower posterior part of lungs. What complication of flu appeared in
patient?
a. Bronchitis
b. Edema of brain
c. *Pneumonia
d. Edema of lungs
e. infectious-toxic shock
871.
To the district doctor a patient, complaints of abundant excretions from a nose,
moderate headache, hearing loss, perversion of taste. On examination – dry of skin, nose
excoriation, in a pharynx – mild hyperemiA. Temperature of body is subfebrile.
Pathological changes of internal organs are absent. Which acute respiratory viral infection
carries the patient?
a. Adenoviral infection
b. Parainfluenza
c. *Rhinoviral infection
d. RC-infection
e. Influenza
872.
At a child with the clinical displays of ARVI a generilized lymphadenopathy, onesided conjunctivitis increase of liver and spleen, is marked. Most reliable diagnosis?
a. Infectious mononucleosis
b. Leptospirosis
c. *Adenoviral infection
d. Flu
e. Pseudotuberculosis
873.
A patient caused a doctor home. Age – 75 years. Complaints of a subfebrile
temperature, general weakness, pharyngalgia, conjunctivitis. In family a child is ill the
acute adenoviral disease. A patient considers itself a patient on second day. At a review
are found out the signs of acute blepharoconjunctivitis, pharyngitis. There are megascopic
lymphatic knots: neck front and back, arm-pits and inguinal, to 1 cm in a diameter, soft,
not is soldered between itself and with a surrounding cellulose. A pharynx is hyperemic,
tonsills are hypertrophied and hyperemic. In lights of wheezes it is not. Breathing clean.
Tones of heart are muffled. BP is 140/80 mm Hg. Ps – 80 per 1 minute. Abdomen soft. A
megascopic liver which comes forward on 3 cm below costal arc and spleen are palpated –
soft, painless. Choose the most credible diagnosis:
a. *Acute adenoviral infection
b. Flu
c. Megacaryoblastoma
d. Infectious mononucleosis
e. Hepatitis A
874.
Sick M., 22 years old, complaints of increasing of body temperature to 39 оC,
headache in frontal area, pain in eyeballs, photophobia, pain in a muscles, dry cough.
Became ill suddenly. The state is heavy. Objectively face is hyperemic, injection of
scleras. Pulse 96 per min, rhythmic. Tones of heart are hypotonic. In the lungs – dissipated
dry wheezes. Mucous membrane of oropharynx is hyperemic, grainy, vessels are
extended. menengial symptoms are not present. Analysis of blood: leukocytes 3*109/L, е
1 %, band neut. 6 %, seg. neut 51 %, lymphocytes 35 %, мonocytes 7 %. What is most
probable diagnosis?
a. Measles
b. *Flu
c. Meningococcal disease
d. Epidemic typhus
e. Pneumonia
875.
A patient 14 years old, hospitalized in the infectious department in severe condition
with considerable headache mainly in frontal and temporal area, pain in eyeballs, in
muscles and joints. Objectively: patient is excited, temperature of the body is 39 оC.
Bradycardia changed by tachycardiA. Muscles tonic and clonic cramps. Positive
meningeal signs. It is found in epidemic anamnesis, his brother is also sick. What is your
diagnosis?
a. *Flu with pneumonia and edema of brain
b. Flu, typical course
c. Parainfluenza, false croupe
d. Respiratory-sencytial infection
e. Adenoviral infection, pneumonia
876.
A patient H., 22 years old, with flu was hospitalized to infectious department with
the acute worsening of the common state. Consiousness is clear. The patient strangles.
Pallor of skin with cynosis. Respiratory rate 50 per min, AP 80/55 mmHg, pulse 110 per a
min, temperature 39.8 оC. During percussion of lungs tympanic sound with dullness in
lower quadrant was found. Crackles in the lower-back parts of lungs. What complication
of influenza has developed in that patient?
a. Pneumonia
b. *Edema of lungs
c. Edema of brain
d. Infectious-toxic shock
e. Meningoencephalitis
877.
A sick, 54 years old, hospitalized in infectious department in the grave condition.
Complaints are headache, mainly in frontal and temporal areas superciliary arcs, origin of
vomiting on peak of pain. Objectively: patient is excited, temperature of body 39 оC, AP
100/60 mm Hg. Bradycardia changed to tachycardiA. Tonic cramps, meningeal signs
appeared. From anamnesis it is known that father is also sick. What treatment should be
prescribed?
a. *Mannitol, lasix, prednisolone, еuphyllin, suprastain
b. Mannitol, acetophene
c. Lasix, analgin, ampicillin
d. Verospiron, euphyllin, demidrol
e. Aspirin, analgin, demidrol
878.
A patient becomes sick very fast: chills, increasing of temperature to 40,1 оC,
headache in frontal and temporal regions, pain in eyeballs, close nose, dry cough and pain
in the chest. The nose bleeding, nausea, double vomits. Objectively: conjunctivitis,
hyperemia, edema, hemorrhages in mucous of otopharhynx, tachycardiA. Blood pressure
is low. Difficult breathing . What is the most possible diagnosis?
a. Meningococcemia
b. Epidemic typhus
c. Leptospirosis
d. *Flu
e. Typhoid fever
879.
A patient 56 years old, the day before he felt easy indisposition, insignificant
headache, and weakness. Afterwards the increasing of temperature appeared to 38,5 оC
with chills, headache increased considerably, mainly in forehead and temples. Skin and
conjunctiva is hyperemic, dry, barking cough. Pharynx is hyperemic. On soft palate
present grainy granules, placed point hemorrhages. Difficult breathing. What is the most
possible diagnosis?
a. Typhoid fever
b. Leptospirosis
c. Epidemic typhus
d. *Flu
e. Enteroviral infection
880.
A sick person, 45 years old, was hospitalized after 2 days of disease. On Sunday he
came back from India (sailor). Complaints of increasing of temperature to 41 оC, severe
headache, shortness of breath, cough, with sputum. Objectively: pallor, cyanosis of
mucous, tachycardiA. Breathing is weaken, crackles in the lower-back parts of the lungs,
crepitation. What is the possible diagnosis?
a. *Flu complicated by pneumonia
b. Miliary tuberculosis
c. Plague, pulmonary form
d. Leptospirosis
e. Sepsis
881.
A 25 years old patient, fell ill rapidly, with chills and temperature rose to 39,9 оC,
headache appeared in frontotemporal regions, pain in eyeballs, dull pain in all trunk,
closed nose. Dry cough after 2 days of illness, there was nose bleeding. Objectively:
hyperemia with sputum, isolated petechial rash and shallow grittiness of soft palate.
Difficult breathing in lungs. What is the most possible diagnosis?
a. Leptospirosis
b. Adenoviral infection
c. Typhoid fever
d. *Flu
e. Epidemic typhus
882.
A patient 26 years old, became sick rapidly: temperature 39.5 оC, severe headache,
mainly in frontal and temporal areas, pain in muscles and joints. By examining on the 2nd
day of illness: state of middle weight, skin is clean. Moderate hyperemia with cyanosis,
pulse 120 per min, rhythmic. Heart activity is rhythmic, tones are muffled, in lungs there
is vesicular breathing. What is the treatment of this patient?
a. Aspirin
b. *Remantadin
c. Ampicillin
d. Ascorbic acid
e. Ribonuclease
883.
A patient fell ill very rapidly: chills, increase of temperature to 40.1 °C, headache
in frontotemporal regions, pain in eyeballs, close nose, dry cough, and chest pain. Nose
bleeding, nausea, vomiting appeared after 4 hours. Objectively: conjunctivitis, hyperemia,
edema, point hemorrhages in mucus of epiglottis, tachycardiA. Blood pressure is low.
Weaken breathing in the lungs. What is the most possible diagnosis?
a. Leptospirosis
b. Epidemic typhus
c. *Flu
d. Мeningococcemia
e. Enteroviral infection
884.
A student, 18 years old, for 7 days complaints of weakness, hyperthermia to 37.8
°C, mucous excretions from a nose, pharyngalgia at swallowing, pain in eyeballs.
Objectively: increased lymph nodes of neck and mandible, lymphadenitis, edema and
injection of conjunctiva, hyperemia of mucous of epiglottis, hypertrophy of tonsils. What
is the most reliable diagnosis?
a. *Adenoviral infection
b. Influenza
c. Infectious mononuleosis
d. Rhinoviral infection
e. Parainfluenza
885.
A patient Т., 45 years old, was hospitalized on the 2nd day of disease. One week
ago he came back from India (sailor of the distant swimming). Complaints of temperature
41.3 °C, great headache, shortness of breathing; cough with foamy pink colour sputum.
Objectively: pale of face, cyanosis of mucous, breath rate 24/min, tachycardiA. Lungs:
breathing is hyposthenic, moist wheezes in both lungs, crepitation. What is possible
diagnosis?
a. Flu
b. Miliary tuberculosis
c. *Plaque, pulmonary form
d. Leptospirosis
e. Sepsis
886.
A patient 20 years old sick rapidly with increasing of temperature to 39.9 °C.
complaints of headache in frontotemporal region, pain in eyeballs, dull ache in whole
trunk, closed nose, scrapes in the throat, dry cough. There was nose-bleeding. What
diagnosis is most possible?
a. *Influenza
b. Adenoviral infection
c. Parainfluenza
d. RS-infection
e. Enteroviral infection
887.
4 years old child complaints of: cough, temperature of body 38.1 °C. Conjunctiva
is hyperemic. On mucous of cheeks there are points of hyperemia gum blushs. Weaken
breathing in the lungs. What is the most possible diagnosis?
a. Scarlet fever
b. Rubella
c. *Measles
d. Herpetic infection
e. Flu
888.
A patient, 20 years old, during few days complaints of pharyngalgias. After
supercooling the state became worse: sudden chills, increase of temperature to 40.6 °C,
headache. On the skin of low extremities, trunk and buttocks there are a lot of different
sizes hemorrhagic spots, acrocyanosis. Consiouness is preserved. Meningeal signs are
absent. What is the previous diagnosis?
a. *Meningococcal infection
b. Flu
c. Epidemic typhus
d. Hemorrhagic fever
e. Leptospirosis
889.
A patient 56 years old, workwomen of pig farm, on a background chills appeared,
the temperature rose to 39,9 °C, headache, nauseA. The next day marked pains in the
muscles of lower extremities appeared, the nose bleeding began. At the receipt, on the 3rd
day common state deteriorated. Hyperemic spots, subicteric appeared. Liver +3 cm. Daily
– diuresis 700 ml. What is the previous diagnosis?
a. Hemorrhagic fever with a kidney syndrome
b. Hepatitis A
c. Escerichiosis
d. Flu
e. *Leptospirosis
890.
A sick 19 years old, sick rapidly, when a temperature rose to 39,2 °C, coughing
appeared, closed nose. Pains in muscles and joints. On the 3rd day of disease, shallow
spots on the trunk appeared, extremities with hyperemia and edematous feet. Generalized
lymphadenopathy, hyperemia of cheeks, enlargement of the liver were found out. What is
previous diagnosis?
a. *Pseudotuberculosis
b. Flu
c. Infectious mononucleosis
d. Herpetic infection
e. Epidemic typhus
891.
A woman who came back from a tour trip, the next day called emergency help. It is
known from the anamnesis, that within a week the temperature of body was moderately
high. Complaints of bad sleep and bad appetite, pain in the abdomen. During the
assescment of the sick it is found out roseolas on the pale skin of breasts and abdomen.
Pulse is normal, temperature of body 38,2 °C, hepatospleenomegaly. What is your
previous diagnosis?
a. *Typhoid fever
b. Epidemic typhus
c. Flu
d. Enteroviral infection
e. Leptospirosis
892.
A patient C., 25 years old, fell suddenly ill. Every morning severe headache,
frequent vomiting, temperature of the body is 39.9 °C. Adopted fatigue, then state got
much worse. In the evening lost of consciousness. Expressed muscles pains of back and
head. Positive Кеrning’s symptom. Leukocytes – 18,0?109. What is the most reliable
diagnosis?
a. Flu
b. Epidemic typhus, typhus state
c. Viral menigoencephalitis
d. Sepsis, infectious-toxic shock
e. *Bacterial menigoencephalitis
893.
A patient 17 years old, became suddenly ill: temperature rose to 40,3 °C. Severe
headache, motive excitation, frequent vomiting, tremor of fingers of extremities.
Hemorrhagic spots of round form and different sizes, more frequently as stars, mainly on
buttocks and trunk. Meningeal signs are positive. What is the most possible diagnosis?
a. Encephalitis
b. Flu with a hemorrhagic syndrome
c. *Meningococcal infection
d. Measles
e. Leptospirosis
894.
Patient B., 20 years old, complaints of severe headache in temples and orbits, dull
ache in the trunk, dry cough. Temperature of the body 39.6 °C. Inflammatory changes of
mucous membrane of oropharynx. Normal breathing in the lungs. What is the most
credible diagnosis?
a. Pneumonia
b. Parainfluenza
c. Respiratory micoplacma
d. *Flu
e. Meningococcal infection
895.
A sick woman, 42 years old, complaints of temperature 39.3 °C, headache in the
frontal area, pain in the eyeballs, photophobia, pain in muscles, dry cough. Became ill
suddenly one day before. Objectively: state is severe. Hyperemia of the face, eyes shinny,
injection of scleras. Pulse 96/min., rhythmic. Tones of heart are hypotonic. Both lungs are
dissipated. Dry wheezes. Mucosa of epiglottis is hyperemic, grainy, vessels are extended.
Meningeal symptoms are not present. Analysis of blood: leuk – 3?109/l, еos – 1 %, band –
6 %, seg – 51 %, lymp – 35 %, mono – 7 %. What is the most possible diagnosis?
a. *Flu
b. Measles
c. Meningococcal infection
d. Pneumonia
e. Epidemic typhus
896.
Patient A., 28 years old, hospitalized with a previous diagnosis of flu. On the 5th
day of illness, rash appeared on the trunk and internal surfaces of extremities.
Temperature 41.5 °C, hyperemia of sclera, tremor of tongue, tachycardia, spleenomegaly,
excitation. What is the most possible diagnosis?
a. Measles
b. Meningococcal infection
c. Leptospirosis
d. *Epidemic typhus
e. Typhoid
897.
A sick explorer of train, 39 years old is hospitalized on the 4th day of illness with
complaints of headache, weakness, dizziness, chills, insomnia, fever. The person is
hyperemic, conjunctivitis. On the transitional fold of conjunctiva there is a single rash. On
the skin of trunk, thorax, abdomen, extremities there are abundant red coloured rashes.
TachycardiA. AP 100/60 mm of Hg. Tremor of tongue. Liver and spleen were enlarged.
Stool fistula is detained. What is the most reliable diagnosis?
a. *Epidemic typhus
b. Typhoid
c. Flu
d. Меnigococcemia
e. Leptospirosis
898.
Sick, 52 years old, with complaints of pain in lumbar region, headache edema of
chin. It is known from anamnesis that the sick suffers from obesity of ІІ degree. Recently
carried heavy neurological stress and had flu. He has chronic bronchitis for 5 years,
chronic gastritis for 8 years. Objectively: Temperature of the body 38.2 °C, AP – 140/90
mm Hg. It is proposed the diagnosis of acute glomerulonephritis. What transferred factors
could be the reason of disease?
a. Neuro psycologic stress
b. Chronic bronchitis
c. Chronic gastritis
d. Obesity
e. *Flu
899.
A patient C., was hospitalized on the 2nd day of illness with complaints of
hoarseness of voice, rough barking cough, labored breathing. Objectively: the state is
severe, uneasy, pallor, temperature 37.1 °C, BR 30/min., breathing is noisy, can hear from
the distance, with participation of auxiliary musculature. Which viruses could cause
development of similar state?
a. Rhino virus
b. Influenza virus
c. Adenovirus
d. *Parainfluenza virus
e. Cytomegalovirus
900.
A sick 70 years old, became ill sharply, the temperature of body rose to 39.2 °C,
excited, euphoric, hyperemia of face, Rozenberg’s exanthema appears. Ricketsia titer is
1:160, IgG – 87 %. What is diagnosis?
a. *Epidemic typhus
b. Meningococcal infection
c. Epidemic spotted fever
d. Flu
e. Parainfluenza
901.
At patient with pediculosis rapidly rise temperature of body up to 41.2 °C,
headache, euphoria appeared in 4 days from the beginning of illness. Red colour rash on
the lateral thorax and back. Titer of Rickettsia antibodies 1:640, Ig M – 89 %. What is
diagnosis?
a. Flu
b. Enteroviral infection
c. Brill-Zinsser disease
d. *Epidemic typhus
e. Parainfluenza
902.
A patient 27 years old, entered clinic on the 4th day of illness with a diagnosis
ARVI, allergodermiA. Fell ill with the rise of temperature to 38,0 °C, headache,
hyperemia of the throat, then barking cough appeared. On the 3rd day rash appeared on
the skin and neck. Was treated by aspirin. Objectively: temperature 38.8 °C. Face is puffy,
conjunctivitis. On the skin of neck and upper part of chest is abundant red-papular rashes
as rings which does not itch. Mucosa of epiglottis is brightly hyperemic. Submandibular
and neck lymphadenitis. Liver and spleen were not enlarged. What is your diagnosis?
a. *Measles
b. Allergic dermatitis
c. Infectious mononucleosis
d. German measles
e. Scarlet fever
903.
A patient K., 23 years old, with 3 days of moderate illness, with high temperature
of body to 40.0 °C, headache and petechial rash on skin, is hospitalized. After introduction
of penicillin at 2 o’clock, the BP fell down to 40/10 mm of hg. Peripheral pulse and
мeningeal signs does not concerne. What is the diagnosis of the patient?
a. *Меningococcemia, infectious-toxic shock
b. ARVI, anaphylactic shock
c. Measles, severe course
d. Epidemic typhus, severe course
e. Scarlet fever, severe course
904.
A patient, 75 years old, called a doctor at home. Rashes and subfebrile temperature,
general weakness, pharyngalgia, conjunctivitis. In family a child is ill with acute
adenoviral disease. A patient considers himself ill on the second day. At a review there are
signs of pharyngitis. There are enlarged lymphatic nodes: of neck, front and back, armpits
and inguinal up to 1 cm in diameter, soft. Pharynx is hyperemic, tonsils are hypertrophy
and hyperemic. Both lungs have wheezing sounds. Not clean breathing. Tones of heart are
muffled. AP 140/80 mm Hg. Heart rate 80 for 1 minute. Abdomen is soft. Enlarged liver 3
cm below costal arch and spleen is palpable. Palpation is soft, painless. Choose the most
possible diagnosis?
a. *Adenoviral infection
b. Flu
c. Hepatitis B
d. Infectious mononucleosis
e. Hepatitis A
905.
A patient 18 years old, with complaints of headache, pharyngalgia, weakness, high
temperature. Objectively: all groups of lymphonodes, 1-3 cm in a diameter, dense, elastic,
enlarged, hepatospleenomegaly. Blood analysis: leukocytosis, mononuclear – 15 %. What
is possible diagnosis?
a. *Infectious mononucleosis
b. Adenoviral infection
c. Angina
d. Diphtheria
e. Acute lympoleycosis
906.
A child of age 2 years has temperature of body 37.3 °C, cold, hoarse voice
“barking cough” appeared suddenly the anxiety, shortness of breath, appeared with
participation of auxiliary muscles. Supposed diagnosis?
a. *Parainfluenza, false croup
b. Diphtheria croup
c. Allergic laryngitis, croup
d. Flu, laryngitis
e. Acute exudative pleuritis
907.
A patient of 5 years old, which treated at home on an occasion of flu by aspirin,
calcium gluconatis, on the second day from the beginning of disease “coffee grounds”
vomiting appeared, melenA. What complication arises?
a. Neurotoxicosis
b. Pneumonia
c. *Hemorrhagic syndrome
d. Infectious-toxic
e. Bowel obstruction
908.
A patient with flu complicated by pneumonia, during some days there are the
displays of infectious-toxic shock of ІІ degree. In BA the level of urea and creatinine
increases. What from these preparations is not recommended to enter in such a situation?
a. *Adrenalin
b. Prednisolone
c. Polioniic solutions
d. Dofaminum
e. Heparin
909.
A sick 15 years old, 3rd day of illness. On the background the catarrhal pneumonia,
weakness in hands appeared, double vision, cross-eye. Voice is weak. Palatoplegia and
extended extremities. Pulse 90/min. AP 130/90 mm Hg .What is your previous diagnosis?
a. Diphtheria
b. *Poliomyelitis
c. Botulicm
d. Epidemic encephalitis
e. Enteroviral infection
910.
A 23 years old person, became ill sharply: fever 38.2 °C, moderate diffuse
pharyngalgia at swallowing, pain and itching in the right eye. Objectively: tonsillitis,
pharyngitis, conjunctivitis. What is previous diagnosis?
a. *Adenoviral infection
b. Enteroviral infection
c. Parainfluenza
d. Flu
e. Acute respiratory infection
911.
A patient 52 years old, hospitalized with the severe form of viral hepatitis B. The
signs of flu appeared in the department. The indexes of bilirubin rose up and transaminase
falls down. What complication can arise in that patient?
a. *Acute hepatic insufficiency
b. Infectious-toxic shock
c. Gastric bleeding
d. Neurotoxicosis
e. Cerebral comma
912.
A child 10 years old with temperature 38,0 °C, conjunctivitis, moist cough,
hyperemia of the mucous membranes of cheeks and lips. Gums are pallor. What is your
diagnosis?
a. *Measles
b. Adenoviral infection
c. Acute respiratory viral infection
d. Enteroviral infection
e. Infectious mononucleosis
913.
A patient with temperature of body 40.0 °C, nonproductive cough, photophobia,
puffiness of face, dots on gums, blushes on the mucus of cheeks your diagnosis?
a. Tuberculosis
b. Меningococcemia
c. *Measles
d. Enteroviral infection
e. Staphylococcal sepsis
914.
A patient on the background of ARVI the fever developed to 40,1 °C, frequency of
breathing is 40 for a minute. What measures are necessary?
a. *Decreasing of patients temperature
b. Artificial ventillation
c. Oxygen. inhalation
d. Infusion therapy
e. Antibioticotherapy
915.
A woman 27 years old, complaints of the general weakness, absence of appetite,
coughing, fever up to 37.5 °C for three weeks. Ulcerous illness of abdomen, myocarditis
is in anamnesis. What inspection is primarily need to do?
a. Electrocardiography
b. Fibrobronchoscopy
c. Fibrogastroscopy
d. *Fluorography
e. Common blood analysis
916.
A patient 17 years, 11th class student, were a lot of cases of ARI (acute respiratory
infection) have happened, appealed to a doctor in clinic at 3rd day of disease with
complaints of chills, general weakness, a moderate sore throat, running nose, swelling of
face, watering from eyes Objective examination: minor palatal hyperemia brackets and
tonsillitis, on a background of moderate edema of tissues. Conjunctivitis. During palpation
not painful enlarged inframaxillary lymph nodes, and enlarged neck lymph nodes were
found. Crepitation can’t be find. Liver and spleen moderately increased. What is the most
likely diagnosis?
a. Diphtheria
b. *Adenoviral infection
c. Meningococcal nasopharyngitis
d. Influenza
e. Infectious mononucleosis
917.
Patient R., 16 years old, hospitalized for 5-day illness with complaints of moderate
headache in fronto-temporal region, laid nose, sore throat, pain in the left eye, rise in
temperature to 38.1-38.5 °C. General condition is satisfactory. Shortness of nasal breath,
mucous discharging from the nose, hyperemia of face, enlargment of the neck and
submaxillary lymph glands, left foamy conjunctivitis. What is preliminary diagnosis?
a. Influenza
b. Infectious mononucleosis
c. Enteroviral infection
d. *Adenoviral infection
e. Influenza
918.
Sick M., 22 years old, complaints of increasing of body temperature to 39 оC,
headache in frontal area, pain in eyeballs, photophobia, pain in a muscles, dry cough.
Became ill suddenly. The state is heavy. Objectively face is hyperemic, injection of
scleras. Pulse 96 per min, rhythmic. Tones of heart are hypotonic. In the lungs – dissipated
dry wheezes. Mucous membrane of oropharynx is hyperemic, grainy, vessels are
extended. menengial symptoms are not present. Analysis of blood: leukocytes 3*109/L, е
1 %, band neut. 6 %, seg. neut 51 %, lymphocytes 35 %, мonocytes 7 %. What is most
probable diagnosis?
a. Measles
b. *Flu
c. Meningococcal disease
d. Epidemic typhus
e. Pneumonia
919.
A patient 14 years old, hospitalized in the infectious department in severe condition
with considerable headache mainly in frontal and temporal area, pain in eyeballs, in
muscles and joints. Objectively: patient is excited, temperature of the body is 39 оC.
Bradycardia changed by tachycardiA. Muscles tonic and clonic cramps. Positive
meningeal signs. It is found in epidemic anamnesis, his brother is also sick. What is your
diagnosis?
a. *Flu with pneumonia and edema of brain
b. Flu, typical course
c. Parainfluenza, false croupe
d. Respiratory-sencytial infection
e. Adenoviral infection, pneumonia
920.
A patient H., 22 years old, with flu was hospitalized into infectious department
with the acute worsening of the common state. Consiousness is stored. The patient
strangles. Pallor of skin with cynosis. Respiratory rate 50 per min, AP 80/55 mmHg, pulse
110 per a min, temperature 39.8 оC. During percussion of lungs tympanic sound with
dullness in lower quadrant was found. Crackles in the lower-back parts of lungs. What
complication of influenza has developed in that patient?
a. Pneumonia
b. *Edema of lungs
c. Edema of brain
d. Infectious-toxic shock
e. Meningoencephalitis
921.
A sick, 54 years old, hospitalized in infectious department in the grave condition.
Complaints of expressed headache, mainly in frontal and temporal areas superciliary arcs,
origin of vomiting on peak of pain. Objectively: patient is excited, temperature of body 39
оC, AP 100/60 mm Hg. Bradycardia changed to tachycardiA. Tonic cramps, meningeal
signs appeared. From anamnesis it is known that father is also sick. What treatment should
be prescribed?
a. *Mannitol, lasix, prednisolone, еuphyllin, suprastain
b. Mannitol, acetophene
c. Lasix, analgin, ampicillin
d. Verospiron, euphyllin, demidrol
e. Aspirin, analgin, demidrol
922.
A patient becomes sick very fast: chills, increasing of temperature to 40,1 оC,
headache in frontal and temporal regions, pain in eyeballs, close nose, dry cough and pain
in the chest. The nose bleeding, nausea, double vomits. Objectively: conjunctivitis,
hyperemia, edema, hemorrhages in mucous of otopharhynx, tachycardiA. Blood pressure
is low. Difficult breathing . What is the most possible diagnosis?
a. Meningococcemia
b. Epidemic typhus
c. Leptospirosis
d. *Flu
e. Typhoid fever
923.
A patient 56 years old, the day before felt easy indisposition, insignificant
headache, weakness. Afterwards the increasing of temperature appeared to 38,5 оC with
chills, headache increased considerably, mainly in forehead and temples. Skin and
conjunctiva is hyperemic, dry, barking cough. Pharynx is hyperemic. On soft palate
present grainy granules, placed point hemorrhages. Difficult breathing. What is the most
possible diagnosis?
a. Typhoid fever
b. Leptospirosis
c. Epidemic typhus
d. *Flu
e. Enteroviral infection
924.
A sick person, 45 years old, was hospitalized after 2 days of disease. On Sunday he
came back from India (sailor). Complaints of increasing of temperature to 41 оC, severe
headache, shortness of breath, cough, with sputum. Objectively: pallor, cyanosis of
mucous, tachycardiA. Breathing is weaken, crackles in the lower-back parts of the lungs,
crepitation. What is the possible diagnosis?
a. *Flu complicated by pneumonia
b. Miliary tuberculosis
c. Plague, pulmonary form
d. Leptospirosis
e. Sepsis
925.
A 25 years old patient, fell ill rapidly, with chills and temperature rose to 39,9 оC,
headache appeared in frontotemporal regions, pain in eyeballs, dull pain in all trunk,
closed nose. Dry cough after 2 days of illness, there was nose bleeding. Objectively:
hyperemia with sputum, isolated petechial rash and shallow grittiness of soft palate.
Difficult breathing in lungs. What is the most possible diagnosis?
a. Leptospirosis
b. Adenoviral infection
c. Typhoid fever
d. *Flu
e. Epidemic typhus
926.
A patient 20 years old, complaints of increasing of temperature up to 39 оC,
headache in frontal area, pain in eyeballs, photophobia, pain in muscles, dry cough.
Became ill sharply. Objectively: severe state. Face is hyperemic, injections of scleras.
Pulse 96/min, rhythmic, tones of heart are hyposthenic. Menengial symptoms are not
present. Blood analysis: leuk 9?109, е 1 %, bands 6 %, seg 51 %, lymp 35 %, mono 7 %.
What is the most possible diagnosis?
a. *Influenza
b. Adenovirus infection
c. Leptospirosis
d. Pneumonia
e. Epidemic typhus
927.
A patient 26 years old, became sick rapidly: temperature 39.5 оC, severe headache,
mainly in frontal and temporal areas, pain in muscles and joints. Examined on the 2nd day
of illness: state of middle weight, skin is clean. Moderate hyperemia with cyanosis, pulse
120 per min, rhythmic. Heart activity is rhythmic, tones are muffled, in lungs there is
vesicular breathing. What is the treatment of this patient?
a. Aspirin
b. *Remantadin
c. Ampicillin
d. Ascorbic acid
e. Ribonuclease
928.
A patient fell ill very rapidly: chills, increase of temperature to 40.1 °C, headache
in frontotemporal regions, pain in eyeballs, close nose, dry cough, and chest pain. Nose
bleeding, nausea, vomiting appeared after 4 hours. Objectively: conjunctivitis, hyperemia,
edema, point hemorrhages in mucus of epiglottis, tachycardiA. Blood pressure is low.
Weaken breathing in the lungs. What is the most possible diagnosis?
a. Leptospirosis
b. Epidemic typhus
c. *Flu
d. Мeningococcemia
e. Enteroviral infection
929.
A student, 18 years old, for 7 days complaints of weakness, hyperthermia to 37.8
°C, mucous excretions from a nose, pharyngalgia at swallowing, pain in eyeballs.
Objectively: increased lymph nodes of neck and mandible, lymphadenitis, edema and
injection of conjunctiva, hyperemia of mucous of epiglottis, hypertrophy of tonsils. What
is the most reliable diagnosis:
a. *Adenoviral infection
b. Influenza
c. Infectious mononuleosis
d. Rhinoviral infection
e. Parainfluenza
930.
A patient Т., 45 years old, was hospitalized at the 2nd day of disease. One week
ago got back from India (sailor of the distant swimming). Complaints of temperature 41.3
°C, great headache, shortness of breathing; cough with foamy pink colour sputum.
Objectively: pale of face, cyanosis of mucous, breath rate 24/min, tachycardiA. Lungs:
breathing is hyposthenic, moist wheezes in both lungs, crepitation. What is possible
diagnosis?
a. Flu
b. Miliary tuberculosis
c. *Plaque, pulmonary form
d. Leptospirosis
e. Sepsis
931.
A patient 20 years old become sick rapidly with increasing of temperature to 39.9
°C. complaints of headache in frontotemporal region, pain in eyeballs, dull ache in whole
trunk, closed nose, scrapes in the throat, dry cough. There was nose-bleeding. What
diagnosis is most possible?
a. *Influenza
b. Adenoviral infection
c. Parainfluenza
d. RS-infection
e. Enteroviral infection
932.
A 4 years old child complaints of: cough, temperature of body 38.1 °C.
Conjunctiva is hyperemic. On mucous of cheeks there are points of hyperemia gum
blushs. Weaken breathing in the lungs. What is the most possible diagnosis?
a. Scarlet fever
b. Rubella
c. *Measles
d. Herpetic infection
e. Flu
933.
A patient, 20 years old, during few days complaints of pharyngalgias. After
supercooling the state became worse: sudden chills, increase of temperature to 40.6 °C,
headache. On the skin of low extremities, trunk and buttocks there are a lot of different
sizes hemorrhagic spots, acrocyanosis. Consiouness is preserved. Meningeal signs are
absent. What is the previous diagnosis?
a. *Meningococcal infection
b. Flu
c. Epidemic typhus
d. Hemorrhagic fever
e. Leptospirosis
934.
A patient 56 years old, workwomen of pig farm, on a background chills appeared,
the temperature rose to 39,9 °C, headache, nauseA. The next day marked pains in the
muscles of lower extremities appeared, the nose bleeding began. At the receipt, on the 3rd
day common state deteriorated. Hyperemic spots, subecteric appeared. Liver +3 cm. Daily
– diuresis 700 ml. What is the previous diagnosis?
a. Hemorrhagic fever with a kidney syndrome
b. Hepatitis A
c. Escerichiosis
d. Flu
e. *Leptospirosis
935.
A sick 19 years old, fell ill rapidly, when a temperature rose to 39,2 °C, coughing
appeared, closed nose. Pains in muscles and joints. On the 3rd day of disease, shallow
spots on the trunk appeared, extremities with hyperemia and edematous feet. Generalized
lymphadenopathy, hyperemia of cheeks, enlargement of the liver were found out. What is
previous diagnosis?
a. *Pseudotuberculosis
b. Flu
c. Infectious mononucleosis
d. Herpetic infection
e. Epidemic typhus
936.
A woman which came back from a tour trip, the next day she called emergency
help. It is known from the anamnesis, that within a week the temperature of body was
moderately high. Complaints of bad sleep and bad appetite, pain in the abdomen. During
the assescment of the sick it is found out roseolas on the pale skin of breasts and abdomen.
Pulse is normal, temperature of body 38,2 °C, hepatospleenomegaly. What is your
previous diagnosis?
a. *Typhoid fever
b. Epidemic typhus
c. Flu
d. Enteroviral infection
e. Leptospirosis
937.
A patient C., 25 years old, fell suddenly ill. Every morning severe headache,
frequent vomiting, temperature of the body is 39.9 °C. Adopted fatigue, then state got
much worse. In the evening lost of consciousness. Expressed muscles pains of back and
head. Positive Кеrning’s symptom. Leukocytes – 18,0*109. What is the most reliable
diagnosis?
a. Flu
b. Epidemic typhus, typhus state
c. Viral menigoencephalitis
d. Sepsis, infectious-toxic shock
e. *Bacterial menigoencephalitis
938.
A patient 17 years old, became suddenly ill: temperature rose to 40,3 °C. Severe
headache, motive excitation, frequent vomiting, tremor of fingers of extremities.
Hemorrhagic spots of round form and different sizes, more frequently as stars, mainly on
buttocks and trunk. Meningeal signs are positive. What is the most possible diagnosis?
a. Encephalitis
b. Flu with a hemorrhagic syndrome
c. *Meningococcal infection
d. Measles
e. Leptospirosis
939.
Patient B., 20 years old, complaints of severe headache in temples and orbits, dull
ache in the trunk, dry cough. Temperature of the body 39.6 °C. Inflammatory changes of
mucous membrane of oropharynx. Normal breathing in the lungs. What is the most
credible diagnosis?
a. Pneumonia
b. Parainfluenza
c. Respiratory micoplacma
d. *Flu
e. Meningococcal infection
940.
A patient B., 17 years old, student of 11th class, where cases of acute respiratory
infection were noted came to the policlinic. On the 3rd day of illness, start with
complaints of chills, general weakness, moderate pharyngalgia, cold, edema of face.
Objectively: insignificant hyperemia of palatal tonsils. Moderate edema of the face,
conjunctivitis. Enlargement of lymphatic nodes. Liver and spleen were moderately
enlarged. What is the most possible diagnosis?
a. *Adenoviral infection
b. Diphtheria
c. Meningococcal pharyngitis
d. Flu
e. Infectious mononucleosis
941.
A sick woman, 42 years old, complaints of temperature 39.3 °C, headache in the
frontal area, pain in the eyeballs, photophobia, pain in muscles, dry cough. Became ill
suddenly one day before. Objectively: state is severe. Hyperemia of the face, eyes shinny,
injection of scleras. Pulse 96/min., rhythmic. Tones of heart are hypotonic. Both lungs are
dissipated. Dry wheezes. Mucosa of epiglottis is hyperemic, grainy, vessels are extended.
Meningeal symptoms are not present. Analysis of blood: leuk – 3?109/l, еos – 1 %, band –
6 %, seg – 51 %, lymp – 35 %, mono – 7 %. What is the most possible diagnosis?
a. *Flu
b. Measles
c. Meningococcal infection
d. Pneumonia
e. Epidemic typhus
942.
Patient A., 28 years old, hospitalized with a previous diagnosis of flu. On the 5th
day of illness, rash appeared on the trunk and internal surfaces of extremities.
Temperature 41.5 °C, hyperemia of sclera, tremor of tongue, tachycardia, spleenomegaly,
excitation. What is the most possible diagnosis?
a. Measles
b. Meningococcal infection
c. Leptospirosis
d. *Epidemic typhus
e. Typhoid
943.
A sick explorer of train, 39 years old is hospitalized on the 4th day of illness with
complaints of headache, weakness, dizziness, chills, insomnia, fever. The person is
hyperemic, conjunctivitis. On the transitional fold of conjunctiva there is a single rash. On
the skin of trunk, thorax, abdomen, extremities there are abundant red coloured rashes.
TachycardiA. AP 100/60 mm of Hg. Tremor of tongue. Liver and spleen were enlarged.
Stool fistula is detained. What is the most reliable diagnosis?
a. *Epidemic typhus
b. Typhoid
c. Flu
d. Меnigococcemia
e. Leptospirosis
944.
Sick, 52 years old, with complaints of pain in lumbar region, headache edema of
chin. It is known from anamnesis that the sick suffers from obesity of ІІ degree. Recently
carried heavy neurological stress and had flu. He has chronic bronchitis for 5 years,
chronic gastritis for 8 years. Objectively: Temperature of the body 38.2 °C, AP – 140/90
mm Hg. It is proposed the diagnosis of acute glomerulonephritis. What transferred factors
could be the reason of disease?
a. Neuro psycologic stress
b. Chronic bronchitis
c. Chronic gastritis
d. Obesity
e. *Flu
945.
A patient C., was hospitalized on the 2nd day of illness with complaints of
hoarseness of voice, rough barking cough, labored breathing. Objectively: the state is
severe, uneasy, pallor, temperature 37.1 °C, BR 30/min., breathing is noisy, can hear from
the distance, with participation of auxiliary musculature. Which viruses could cause
development of similar state?
a. Rhino virus
b. Influenza virus
c. Adenovirus
d. *Parainfluenza virus
e. Cytomegalovirus
946.
A sick 70 years old, became ill sharply, the temperature of body rose to 39.2 °C,
excited, euphoric, hyperemia of face, Rozenberg’s exanthema appears. Ricketsia titer is
1:160, IgG – 87 %. What is diagnosis?
a. *Epidemic typhus
b. Meningococcal infection
c. Epidemic spotted fever
d. Flu
e. Parainfluenza
947.
At patient with pediculosis rapidly rise temperature of body up to 41.2 °C,
headache, euphoria appeared in 4 days from the beginning of illness. Red colour rash on
the lateral thorax and back. Titer of Rickettsia antibodies 1:640, Ig M – 89 %. What is
diagnosis?
a. Flu
b. Enteroviral infection
c. Brill-Zinsser disease
d. *Epidemic typhus
e. Parainfluenza
948.
A patient 27 years old, entered clinic on the 4th day of illness with a diagnosis
ARVI, allergodermiA. Fell ill with the rise of temperature to 38,0 °C, headache,
hyperemia of the throat, then barking cough appeared. On the 3rd day rash appeared on
the skin and neck. Was treated by aspirin. Objectively: temperature 38.8 °C. Face is puffy,
conjunctivitis. On the skin of neck and upper part of chest is abundant red-papular rashes
as rings which does not itch. Mucosa of epiglottis is brightly hyperemic. Submandibular
and neck lymphadenitis. Liver and spleen were not enlarged. What is your diagnosis?
a. *Measles
b. Allergic dermatitis
c. Infectious mononucleosis
d. German measles
e. Scarlet fever
949.
A patient K., 23 years old, with 3 days of moderate illness, with high temperature
of body to 40.0 °C, headache and petechial rash on skin, is hospitalized. After introduction
of penicillin at 2 o’clock, the BP fell down to 40/10 mm Hg. Peripheral pulse and
мeningeal signs does not concerne. What is the diagnosis of the patient?
a. *Меningococcemia, infectious-toxic shock
b. ARVI, anaphylactic shock
c. Measles, severe course
d. Epidemic typhus, severe course
e. Scarlet fever, severe course
950.
A patient, 75 years old, called a doctor at home. Rashes and subfebrile temperature,
general weakness, pharyngalgia, conjunctivitis. In family a child is ill with acute
adenoviral disease. A patient considers himself ill on the second day. At a review there are
signs of pharyngitis. There are enlarged lymphatic nodes: of neck, front and back, armpits
and inguinal up to 1 cm in diameter, soft. Pharynx is hyperemic, tonsils are hypertrophy
and hyperemic. Both lungs have wheezing sounds. Not clean breathing. Tones of heart are
muffled. AP 140/80 mm Hg. Heart rate 80 for 1 minute. Abdomen is soft. Enlarged liver 3
cm below costal arch and spleen is palpable. Palpation is soft, painless. Choose the most
possible diagnosis:
a. *Adenoviral infection
b. Flu
c. Hepatitis B
d. Infectious mononucleosis
e. Hepatitis A
951.
A patient 18 years old, with complaints of headache, pharyngalgia, weakness, high
temperature. Objectively: all groups of lymphonodes, 1-3 cm in a diameter, dense, elastic,
enlarged, hepatospleenomegaly. Blood analysis: leukocytosis, mononuclear – 15 %. What
is possible diagnosis?
a. *Infectious mononucleosis
b. Adenoviral infection
c. Angina
d. Diphtheria
e. Acute lympoleycosis
952.
A child of age 2 years has temperature of body 37.3 °C, cold, hoarse voice
“barking cough” appeared suddenly the anxiety, shortness of breath, appeared with
participation of auxiliary muscles. Supposed diagnosis?
a. *Parainfluenza, false croup
b. Diphtheria croup
c. Allergic laryngitis, croup
d. Flu, laryngitis
e. Acute exudative pleuritis
953.
A patient of 5 years old, which treated at home on an occasion of flu by aspirin,
calcium gluconatis, on the second day from the beginning of disease “coffee grounds”
vomiting appeared, melenA. What complication arises?
a. Neurotoxicosis
b. Pneumonia
c. *Hemorrhagic syndrome
d. Infectious-toxic
e. Bowel obstruction
954.
A patient with flu complicated by pneumonia, during some days there are the
displays of infectious-toxic shock of ІІ degree. In BA the level of urea and creatinine
increases. What from these preparations is not recommended to enter in such a situation?
a. *Adrenalin
b. Prednisolone
c. Polioniic solutions
d. Dofaminum
e. Heparin
955.
A sick 15 years old, 3rd day of illness. On the background the catarrhal pneumonia,
weakness in hands appeared, double vision, cross-eye. Voice is weak. Palatoplegia and
extended extremities. Pulse 90/min. AP 130/90 mm Hg .What is your previous diagnosis?
a. Diphtheria
b. *Poliomyelitis
c. Botulicm
d. Epidemic encephalitis
e. Enteroviral infection
956.
A 23 years old person, became ill sharply: fever 38.2 °C, moderate diffuse
pharyngalgia at swallowing, pain and itching in the right eye. Objectively: tonsillitis,
pharyngitis, conjunctivitis. What is previous diagnosis?
a. *Adenoviral infection
b. Enteroviral infection
c. Parainfluenza
d. Flu
e. Acute respiratory infection
957.
A patient 52 years old, hospitalized with the severe form of viral hepatitis B. The
signs of flu appeared in the department. The indexes of bilirubin rose up and transaminase
falls down. What complication can arise in that patient?
a. *Acute hepatic insufficiency
b. Infectious-toxic shock
c. Gastric bleeding
d. Neurotoxicosis
e. Cerebral comma
958.
A child 10 years old with temperature 38,0 °C, conjunctivitis, moist cough,
hyperemia of the mucous membranes of cheeks and lips. Gums are pallor. What is your
diagnosis?
a. *Measles
b. Adenoviral infection
c. Acute respiratory viral infection
d. Enteroviral infection
e. Infectious mononucleosis
959.
A patient with temperature of body 40.0 °C, nonproductive cough, photophobia,
puffiness of face, dots on gums, blushes on the mucus of cheeks your diagnosis?
a. Tuberculosis
b. Меningococcemia
c. *Measles
d. Enteroviral infection
e. Staphylococcal sepsis
960.
A patient on the background of ARVI the fever developed to 40,1 °C, frequency of
breathing is 40 for a minute. What measures are necessary?
a. *Decreasing of patients temperature
b. Artificial ventillation
c. Oxygen. inhalation
d. Infusion therapy
e. Antibioticotherapy
961.
A woman 27 years old, complaints of the general weakness, absence of appetite,
coughing, fever up to 37.5 °C for three weeks. Ulcerous illness of abdomen, myocarditis
is in anamnesis. What inspection is primarily need to do?
a. Electrocardiography
b. Fibrobronchoscopy
c. Fibrogastroscopy
d. *Fluorography
e. Common blood analysis
962.
A man, 47 years old, became ill suddenly, appeared chills, fever 39,0 °C, head and
muscular pain, nausea, vomiting and diarrheA. Cough, shortness of breath, appeared on
the 3rd day of illness. Pulse is 68 per min, rhythmic. Breath rate 44/min. Physical
examination revealed. Right sided pneumonia in roentgenologic diagnosed. Liver +2,5
cm. It is known that the patient, week prior to illness lived in a hotel with conditioners.
What disease does it follow to think about?
a. Bacterial pneumonia
b. Psittacosis
c. Tuberculosis
d. Myocard infarction
e. *Legionellosis
963.
A 55 years old businescman died from the edema of lungs on the background of
severe bilateral pneumonia which is confirmed roentgenologically. A patient 5 days prior
to beginning of illness got back from Sweden, where he was in the business trip for 2
weeks. In the hotel, where he lived, he used a swimming pool and conditioner. What
etiology of pneumonia most possible in his death?
a. Tubercular
b. Micoplacma
c. Chlamydia
d. *Legionellosis
e. Pneumococcal
964.
Among the tourists who lived in a fashionable hotel, where was the outbreake of
respiratory disease in daylight saving time of the year. The illness of three patients have
pneumonic form. In the blood after inspection found out with antibodies to L.
pneumophila in a titer 1:256 (reaction of indirect immunofluorescence). Antibiotics of
what group is the most expedient to appoint in this case?
a. Penicillin
b. Aminoglycosides
c. *Macrolides
d. Теtracyclines
e. Cephalosporines
965.
A sick , 45 years old, is on treatment with a diagnosis of pneumoniA. Gets the
antibiotics of penicillin group. Condition does not get better. In blood analysis found
growth of titre of protomycoplacma antibodies in 4 times. What from the resulted
preparations does follow to appoint for etiotropic treatment of patient?
a. *Теtracyclines, еrythromycin
b. Ampicillin, levomycetin
c. Acyclovir, zovirax
d. Gentamycin, penicillin
e. Furazolidon, nifuroksazid
966.
A patient 23 years old with pregnancy of 12 weeks. Scars do not appear. She has
passed the inspection in a woman consultation. Protomycoplacma antibodies in a titre of
1:10 was found in her analysis. What tactics of gynecologist should be?
a. *To repeat analysis with mycoplacma diagnostics in 14 days
b. To appoint tetracyclin
c. To appoint еrythromycin
d. To cut pregnancy short
e. To repeat analysis with a mycoplacma diagnostic after childbirth
967.
In a newborn, the septic state was accompanied with the increase of temperature to
40 oC which developed on 5th day, with pneumonia expressed intoxication: pallor,
vomiting, shortness of breath, disturbance, cramps. Appeared rash (blisters with
hemorrhages) on skin, mucous membrane of mouth cavity, throat and conjuctivA. The
child suffers with congenital herpes. Lungs: difficult breathing and vesicular wheezing.
Mild enlargement of liver. What is the most reliable diagnosis?
a. *Herpetic infection
b. Cytomegaloviral infection
c. Chicken pox
d. Rubella
e. AIDS
968.
A new born child on 10th day of life became worse: Temperature 39.2 °C, no
frequent vomiting, generalized cramps, violations of consciousness, spastic paresis of left
extremities. Month prior to his birth herpes virus was present in the mother, which she did
not treated. What disease is most possible?
a. *Herpetic encephalitis
b. Meningococcal meningitis
c. Subarechnoid hemorrhage
d. Cerebral abscess
e. Violation of blood cerebral circulation
969.
A patient has herpetic meningitis. What preparation of specific therapy for viral
neuro infection should be given?
a. *Acyclovir
b. Cefataxime
c. Ceftriaxone
d. Gentamycin
e. Furazolidon
970.
A patient, 22 years old, became ill sharply. History showed fever up to 38.2 °C
with headache, repeated vomiting, olfactory and tastes hallucinations. Quickly got
complex of meningeal symptoms, pyramidal paresis. The general epileptic attack and
comatose state also developed. Neurolymph is with mixed lymphocytosis, cytochrome,
single red corpuscles. What is previous diagnosis?
a. Brain abscess
b. Subdural empyema
c. *Herpetic encephalitis
d. Tumor of brain
e. Encephalopathy
971.
A boy 11 years old, complaints of sickness at mastication, increasing of
temperature to 37.1 °C, enlargment of parotid salivary glands . At the age of 8 years
carried a paraflu infection. Objectively: in the region of right parotid salivary gland
tubular sickly at palpation, a skin above it is not changed. A pharynx is moderate
hyperemic, tonsils are not coated. What is your previous diagnosis?
a. *Cytomegaloviral syaloadenitis
b. Lymphadenitis
c. Parotitis
d. Infectious mononucleosis
e. Cholylithiasis
972.
At junior nurse, who works in child’s infectious department, herpes simplex was
found. What manager of department must do?
a. *Create a quarantine in the department
b. To appoint an immunoprotein to the children
c. Discharge all children from the department
d. To appoint immunomodulators with a prophylactic purpose
e. To inspect a junior nurse on a staphylococcus
973.
A woman 65 years old had the disease beginning sharply from increase of
temperature to 39.0 °C, weakness, and pain in the left part of thorax that increased with
breathing motions. On 3rd day of disease vesicular breathing appeared after motion of rib
on the left on a hyperemic background. Together with sick a grandchild lives 4 years.
What measures of prophylaxis of disease need to be adopted?
a. Vaccination
b. Final disinfection
c. Reception of specific immunoprotein
d. *Isolation of patient
e. Acyclovir administration
974.
A patient 60 years old for 2 days has disturbed severe pain in a right arm. On 3rd
day appeared blisters, pouring out as a chain on the skin of shoulder, forearm and brush.
Sensitiveness in the area of pouring out is mionectic. What disease can be diagnosed?
a. Dermatitis
b. *Herpetic ganglionitis
c. Neck-pectoral redicals
d. Psoriasis
e. Allergy
975.
The patient, 58 years old, was hospitalised in the infectious department with
complaints of pain in the left half of thorax, fever. At a review: temperature of body 37.5
°C, in XI-XII intercostal area the grouped shallow blisters on a hyperemic-filling
background are filled by transparent maintenance. Preparation of choice for treatment of
this patient is:
a. Suprastin
b. Prednisolone
c. Biseptolum-480
d. Semavin
e. *Laferon
976.
A pregnant woman, 27 years (pregnancy ІІ, 8-10 weeks.), temperature of body
increased. At the inspection on a TORCH-infection antibodies are found to the herpes
virus, ІІ types of class IGM. What we must recommend to pregnant women?
a. *To cut pregnancy
b. To prolong the supervision
c. Treatment with acyclovir
d. Symptomatic treatment
e. Appoint of alpha-fetoprotein
977.
A patient appealed to the doctor with complaints of difficulties in opening of the
mouth. Two weeks ago fell down and head was hurt, did not have medication. At a review
mouth opens on 1.5 cm, moderate expressed pain of muscles at the back of head. The
paralysis of muscles of person, eyeballs, are more prominent. In a temporal area dry
bloody crusts in the place of traumA. What are your diagnosis?
a. Neuritis of facial nerve
b. Throat abscess
c. *Facial paralytic stupor of Rоsе
d. Bulbar encephalitis
e. Herpetic ganglionitis of knot of trifacial
978.
A patient came with complaints of sickly erosions on his penis. From anamnesis
frequent appearance of similar rashes is found out during a year. Objectively: on a balanus
are the grouped blisters and erosions, soft on palpation. What is your diagnose?
a. *Recurrent herpes of ІІ type
b. Vulvar pemphigus
c. Primary syphyllis
d. Shankoform pyoderma
e. Recurrent herpes of ІІІ type
979.
Sick patient, 25 years old, was consulted by a doctor on the third day of illness
with complaints of pouring out on lips, wings of nose and ears, pain and swelling in the
places of these pouring out. Objectively: temperature of body – 37.7 °C, unchanged skin
of overhead lip, wings of nose, auricles, there is vesicular scars 1-2 mm with the group
location. What disease you may think about?
a. Erysipelas, bullous form
b. Anthrax
c. *Herpetic infection
d. Anthrax, skin form
e. Chicken pox
980.
Sick A., complaints of turning red rash and edema on a right cheek. During a
review: temperature of body – 38.7 °C, submandibular lymph nodes enlarged and painful,
border between turning red and healthy skin is clear, there are blisters with a dark liquid
inwardly, palpation is painful. What is your previous diagnosis?
a. *Erysipelas, hemorrhagic form
b. Anthrax, skin form
c. Herpetic infection
d. Chicken pox
e. Phlegmon of cheek
981.
A 32 years old patient appealed to a doctor on a background of fever and increase
of temperature to 38.0 °C, pain in the right ear area, left side “deviation” appear during the
next day . On examination: right side cmoothed out frontal and occipital skinning folds,
right eyelids fissure is wider than left, a mouth is overtighten to the left, a right eyebrow
does not rise upwards, and a cheek “hyperemia”, xerostomia and violation of taste
receptors on front 2/3 right halves of tongue. Herpetic blisters in right external auditory
canal and auricle. What is the most possible diagnosis?
a. *Hunt’s syndrome
b. Post herpetic neuralgia of the I- and II- branches of right trifacial nerve
c. Post herpetic neuralgia of all branches of right trifacial nerve
d. Sludder’s syndrome
e. Rosolimo-Меlkerson-Rozantalia syndrome
982.
A s 40 years old scientist, became ill sharply with chill, temperature of body – 39.8
°C, severe headache, vomiting, pain in muscles. Did not went to the doctor, the state had
become worse, hyperemia of sclera, appeared, on lips herpes with hemorrhagic
maintenance, the nose-bleeding, skin and sclera, became icteric, urine color is of strong
tea, diuresis 200 ml, an anacholia was not present. What is the most reliable source of
infection?
a. *Rats
b. Cats
c. Infected people
d. Bacillocarrier
e. Mosquito
983.
A patient came with complaints of erosions of his penis. From anamnesis frequent
appearance of similar rashes during a year is found out. Objectively: on a balanus are the
grouped blisters and erosions of polycyclic outlines, with clear margin, soft during
palpation. What is your diagnosis?
a. *Recurrent congenital herpes
b. Pemphigus vulgaris
c. Primary syphillis
d. Pyodermia
e. Scabies
984.
At a 36 years old sick person, 4 days ago a rash appeared on a skin that is
accompanied by itching. Swelling of abdomen. Disturbed dull pain in right subcostal area
had constipation. Day prior to it he ate the smoked meat. Atypical reactions appeared after
the use of tomatoes, strawberry, chocolate. Objectively: on the skin of body, trunk,
extremities a lot of rashes. Level of general IgE in normal. What is most possible
diagnosis?
a. *Pseudoallergy
b. Idiosyncrasy
c. Food allergy
d. Herpetic infection
e. Erysipelas
985.
Sick 65 years old patient, complaints of pain in a subscapular region. Objectively:
on a skin surface of subscapular region the placed arcwise rose-red filling out hearths
some infiltrative, with clear scopes was present. On-the-spot hearths grouped vesicles with
transparent maintenance. What is the diagnosis?
a. Impetigo
b. Herpes simplex
c. Erysipelas
d. Allergodermia
e. *Herpes zoster
986.
A sick person, 65 years old, complaints of rash, pain in a subscapular region.
Objectively: on a skin surface of the subscapular region present the arcwise rose-red
filling out hearths some infiltrative, with clear scopes. On-the-spot hearths grouped
vesicles with transparent maintenance. What preparation he should take?
a. Suprastyn
b. Prednisolone
c. Biseptolum-480
d. Loratidin
e. *Laferon
987.
A patient N., 45 years old, complaints of headache, general weakness increasing of
temperature, to 37.4 °C. In 2 days pain appeared in the pectoral region of spine with an
irradiation in a right between scapular regions. After some time skin in this region turned
red as a strip from a spine to the subarmpit line, and in 2 days red knots which through the
set time grew into blisters with transparent maintenance appeared in this place. What is
your diagnosis?
a. *Herpes zoster
b. Thoracal rediculopathy
c. Neurology of intercostal nerves
d. Neurology of superscapular nerve
e. Herpetiform dermatitis
988.
A 37 years old patient, 2 days ago a spot on a hand appeared, which for days grew
into pustule with a black bottom, painless at touch, with the crown of daughters vesicles
on periphery. There are painless edema on a hand and shoulder. Temperature rise to 39.0
°C, dizziness appeared. Pulse – 100 beats per min, AP – 95/60 mm Hg. BR – 30 per a
minute. What is the most possible diagnosis?
a. *Anthrax
b. Plague
c. Tularemia
d. Brucellosis
e. Herpes
989.
A 70 years old sick person, after supercooling severy pain in the left half of head in
the area of forehead and left eye appeared. 3 days ago the temperature of body increased
to 37.6 °C, the blister of pouring out at the head and left overhead eyelid appeared. What
disease can be diagnosed?
a. *Herpetic ganglionitis
b. Encephalitis
c. Allergy
d. Dermatitis
e. Trifacial neuritis
990.
A sick 3 years old child came to the doctor with symptoms of the fever, languor,
waiver of meal. A boy is capricious, temperature of body 37.9 °C. On the mucus shell of
soft palate, cheeks are single vesicle hypersalivation. What is the diagnosis?
a. *Herpetic stomatitis
b. Candidosis of oral cavity
c. Leucoplacia
d. Follicular tonsillitis
e. Lacunar tonsillitis
991.
Sick, 49 years old, came to the doctor with complaints of pain. On the 3rd day he
became ill, first marked heartburn and pain in thorax, yesterday is pouring out. At
examination: temperature of body 37.8 °C, after motion of the V-VI intercostals intervals
on a hyperemic skin group of the blisters filled by transparent maintenance. What is the
diagnosis?
a. *Herpes zoster
b. Chicken pox
c. Erysipelas, erythematous-bulous form
d. Allergic dermatitis
e. Myositis
Test to figures
1. A mechanism of transmission at this illness is (Fig. 49):
A. Air-drop
B. Fecal-oral
C. * Contact
D. Transmissiv
E. Vertical
2. A mechanism of transmission at this illness is (Fig. 40):
A. Air-drop
B. Fecal-oral
C. * Contact
D. Transmissiv
E. Vertical
3. A mechanism of transmission at this illness is (Fig. 46):
A. Air-drop
B. Fecal-oral
C. * Contact
D. Transmissiv
E. Vertical
4. Appoint adequate therapy to this patient (Fig. 61) in blood of which is RNA HCV.
A. * Intron A 3 millions 3 times for a week
B. Intron A 3 millions 5 times for a week
C. Intron A 5 millions 3 times for a week
D. Intron A 5 millions 5 times for a week
E. Intron A 10 millions 3 times for a week
5. At what infectious disease does conduct this symptom (Fig. 58)?
A. Poisoning mushrooms
B. Meningo-encefalit
C. Poliomyelitis
D. Rabies
E. * Tetanus
6. Curent treatment for HIV infection (Fig. 3) consists of highly active artiretroviral therapy. Choose the
correct combination of preparations:
A. 1 NRTIs + 2 IP
B. 1 NRTIs + 1 IP + 2 NNRTI
C. * 3 NRTIs
D. 2 NRTIs + 2 IP
E. 2 NNRTIs + 1 IP
7. Curent treatment for HIV infection (Fig. 3) consists of highly active artiretroviral therapy. Choose the
correct combination of preparations:
A. 1 NRTIs + 2 IP
B. 1 NRTIs + 1 IP + 2 NNRTI
C. 2 NRTIs
D. 2 NRTIs + 2 IP
E. * 2 NRTIs + 1 IP
8. Curent treatment for HIV infection (Fig. 3) consists of highly active artiretroviral therapy. Choose the
correct combination of preparations:
A. 1 NRTIs + 2 IP
B. 1 NRTIs + 1 IP+ 2 NNRTI
C. * 2 NRTIs + 1 NNRTI
D. 2 NRTIs + 2 IP
E. 2 NNRTIs + 1 IP
9. Curent treatment for HIV infection (Fig. 3) consists of highly active artiretroviral therapy. Choose the
correct combination of preparations:
A. 1 NRTIs + 2 IP
B. 1 NRTIs + 1 IP + 2 NNRTI
10.
11.
12.
13.
14.
15.
16.
17.
18.
C. * 3 NRTIs
D. 2 NRTIs + 2 IP
E. 1 NNRTIs + 1 IP + 1 NNRTI
?For what family of viruses does this exciter (Fig. 1) belong to?
A. Paramyxovirus
B. Reovirus
C. Rabdovirus
D. * Retrovirus
E. Herpesvirus
For what group does this infectious diseases caused by this exciter (Fig. 1) belong to by L. Gromashevskij
classification?
A. Intestinal infection
B. Infections of respiratory tract
C. Behave to all indicated groups
D. Blood infection
E. * Infection of external covers
For what infectious pathology is it characteristically (Fig. 51)?
A. Poisoning mushrooms
B. Meningo-encefalit
C. Poliomyelitis
D. Rabies
E. * Tetanus
For what infectious pathology is it characteristically (Fig. 52)?
A. Poisoning mushrooms
B. Encefalomeningit
C. Poliomyelitis
D. Rabies
E. * Tetanus
For what infectious pathology is it characteristically (Fig. 53)?
A. Poisoning mushrooms
B. Meningo-encefalit
C. Poliomyelitis
D. Rabies
E. * Tetanus
For what infectious pathology is it characteristically (Fig. 54)?
A. Poisoning mushrooms
B. Meningo-encefalit
C. Poliomyelitis
D. Rabies
E. * Tetanus
For what infectious pathology is it characteristically (Fig. 55)?
A. Poisoning mushrooms
B. Meningo-encefalit
C. Poliomyelitis
D. Rabies
E. * Tetanus
For what infectious pathology is it characteristically (Fig. 56)?
A. Poisoning mushrooms
B. Meningo-encefalit
C. Poliomyelitis
D. Rabies
E. * Tetanus
For what infectious pathology is it characteristically (Fig. 57)?
A. Poisoning mushrooms
B. Meningo-encefalit
C. Poliomyelitis
D. Rabies
E. * Tetanus
19. How is this symptom named (Fig. 51)?
A. * Opisthotonos
B. Emprostotonus
C. Plevrostotonus
D. Risus sardonicus
E. Lockjaw
20. How is this symptom named (Fig. 53)?
A. Opisthotonos
B. Emprostotonus
C. Plevrostotonus
D. Risus sardonicus
E. Lockjaw
21. How is this symptom named (Fig. 54)?
A. Opisthotonos
B. Emprostotonus
C. Plevrostotonus
D. Risus sardonicus
E. * Lockjaw
22. How is this symptom named (Fig. 55)?
A. Opisthotonos
B. Emprostotonus
C. Plevrostotonus
D. * Risus sardonicus
E. All right
23. How is this symptom named (Fig. 56)?
A. Opisthotonos
B. Emprostotonus
C. Plevrostotonus
D. All right
E. * Lockjaw
24. How is this symptom named (Fig. 57)?
A. Opisthotonos
B. Emprostotonus
C. Plevrostotonus
D. * Risus sardonicus
E. Lockjaw
25. How is this symptom named (Fig. 58)?
A. * Lorin-Epshteyn
B. Blyumberg
C. Murson
D. Rozenberg
E. Koplik
26. Man V., 26 years, has HIV infection 5 years. What is your diagnosis (Fig. 5)?
A. HIV infection. Pneumococal pneumonia
B. HIV infection. Megakaryoblastoma
C. * HIV infection. Pneumocystis pneumonia
D. HIV infection. Visceral leshmaniasis
E. HIV infection. Pneumococal pneumonia. Megakaryoblastoma
27. Man V., 26 years, has HIV infection 5 years. What is your diagnosis (Fig. 6)?
A. Dementsia of AIDS
B. Patient has еncephalopathy (AIDS-related complex)
C. Rabies
D. Anxiously depressed syndrome for HIV infection
E. * Toxoplasmosis
28. Put a preliminary diagnosis (Fig. 40)?
A. * Erysipelas
29.
30.
31.
32.
33.
34.
35.
36.
37.
B. Phlegmon
C. Sepsis
D. Herpetic infection
E. Erysipelotrix
Put a preliminary diagnosis (Fig. 41)?
A. * Erysipelas, erytematous form
B. Erysipelas, erytematous-bulous form
C. Erysipelas, erytematous-hemorrhagic form
D. Erysipelas, bulous-hemorrhagic form
E. Erysipelas, bulous form
Put a preliminary diagnosis (Fig. 42)?
A. * Erysipelas, erytematous form
B. Erysipelas, erytematous-bulous form
C. Erysipelas, erytematous-hemorrhagic form
D. Erysipelas, bulous-hemorrhagic form
E. Erysipelas, bulous form
Put a preliminary diagnosis (Fig. 44)?
A. Erysipelas, erytematous form
B. * Erysipelas, erytematous-bulous form
C. Erysipelas, erytematous-hemorrhagic form
D. Erysipelas, bulous-hemorrhagic form
E. Erysipelas, bulous form
Put a preliminary diagnosis (Fig. 45)?
A. * Erysipelas, erytematous form
B. Erysipelas, erytematous-bulous form
C. Erysipelas, erytematous-hemorrhagic form
D. Erysipelas, bulous-hemorrhagic form
E. Erysipelas, bulous form
Put a preliminary diagnosis (Fig. 46)?
A. Erysipelas, erytematous form
B. Erysipelas, erytematous-bulous form
C. Erysipelas, erytematous-hemorrhagic form
D. * Erysipelas, bulous-hemorrhagic form
E. Erysipelas, bulous form
Put a preliminary diagnosis (Fig. 47)?
A. Erysipelas, erytematous form
B. Erysipelas, erytematous-bulous form
C. Erysipelas, erytematous-hemorrhagic form
D. * Erysipelas, bulous-hemorrhagic form
E. Erysipelas, bulous form
Put a preliminary diagnosis (Fig. 49)?
A. * Erysipelas
B. Phlegmon
C. Sepsis
D. Herpetic infection
E. Erysipelotrix
Put a preliminary diagnosis (Fig. 50)?
A. * Erysipelas
B. Phlegmon
C. Sepsis
D. Herpetic infection
E. Erysipelotrix
Put a previous diagnosis (Fig. 61)?
A. Viral hepatitis
B. Leptospirosis
C. Malaria
D. Amebiaz, visceral form
E. All is right
38. Specify the measures of urgent prophylaxis of this desease (Fig. 15).
A. Anti-anthrax bacteriofag
B. * Penicillinum or tetracyclinum during 5 days
C. Vaccination
D. Medical supervision
E. Biseptolum 5 days
39. Specify the measures of urgent prophylaxis of this desease (Fig. 16).
A. Anti-anthrax bacteriofag
B. * Penicillinum or tetracyclinum during 5 days
C. Vaccination
D. Medical supervision
E. Biseptolum 5 days
40. Specify the measures of urgent prophylaxis of this desease (Fig. 18).
A. Anti-anthrax bacteriofag
B. * Penicillinum or tetracyclinum during 5 days
C. Vaccination
D. Medical supervision
E. Biseptolum 5 days
41. The duration of incubation period of the most possible disease (Fig. 20) is:
A. 3 to 8 days;
B. 2 to 12 days;
C. 2 to 10 days;
D. 1 to 8 days.
E. * 2 to 6 days;
42. The duration of incubation period of the most possible disease (Fig. 21) is:
A. 3 to 8 days;
B. 2 to 12 days;
C. 2 to 10 days;
D. 1 to 8 days.
E. * 2 to 6 days;
43. The duration of incubation period of the most possible disease (Fig. 22) is:
A. 3 to 8 days;
B. 2 to 12 days;
C. 2 to 10 days;
D. 1 to 8 days.
E. * 2 to 6 days;
44. The experts of WHO consider suspicious in relation to AIDS (Fig. 12):
A. Increase 2 and more lymph nodes in two anatomical groups (except for inguinal), by sizes more
than 2 cm in diameter, which lasts more than 2 months
B. Increase 3 and rmore lymph nodes in two anatomical topographic groups (except for inguinal) by
sizes more than 2 cm in diameter, which lasts more than 3 months
C. Increase 2 and more lymph nodes in two аnatomical groups (except for inguinal), by sizes more
than 1 cm in diameter, which lasts more than 2 months
D. * Increase 2 and more lymph nodes in two аnatomical groups (except for inguinal), by sizes more
than 1 cm in diameter, which lasts more than 3 months
E. Increase 3 and more lymph nodes in two anatomical groups (except for inguinal), by sizes more
than 2 cm in diameter, which lasts more than 1 months
45. The experts of WHO consider suspicious in relation to AIDS (Fig. 7):
A. Loss of weight (9 % and more)
B. Loss of weight (5 % and more)
C. Loss of weight (6 % and more)
D. * Loss of weight (10 % and more)
E. Loss of weight (3 % and more)
46. The experts of WHO consider suspicious in relation to AIDS (Fig. 8):
A. Diarhea which lasts more than 6 months
B. Diarhea which lasts more than 4 months
47.
48.
49.
50.
51.
52.
53.
54.
55.
C. Diarhea which lasts more than 2 months
D. Diarhea which lasts more than 3 months
E. * Diarhea which lasts more than 1 months
What cells are infected by this virus (Fig. 1)?
A. CD 1
B. CD 2
C. CD 3
D. * CD 4
E. CD 5
What clinical features of sarcoma Kaposhi in such (Fig. 2) patients?
A. A sarcoma metastasis in internal and marked high lethality
B. Pouring out with necrosis and ulceration
C. Primary elements appear on a head and trunk
D. Will strike the persons of young and middle age
E. All adopted features
What clinical features of sarcoma Kaposhi in such (Fig. 2) patients?
A. * A sarcoma metastasis in internal and marked high lethality
B. Pouring out without necrosis and ulceration
C. Primary elements appear on a leg
D. Will strike the persons of old age
E. All adopted features
What clinical features of sarcoma Kaposhi in such (Fig. 2) patients?
A. A sarcoma do not metastasis in internal and marked high lethality
B. * Pouring out with necrosis and ulceration
C. Primary elements appear on a leg
D. Will strike the persons of old age
E. All adopted features
What clinical features of sarcoma Kaposhi in such (Fig. 2) patients?
A. A sarcoma do not metastasis in internal and marked high lethality
B. Pouring out without necrosis and ulceration
C. * Primary elements appear on a head and trunk
D. Will strike the persons of old age
E. All adopted features
What clinical features of sarcoma Kaposhi in such (pic. 2) patients?
A. A sarcoma do not metastasis in internal and marked high lethality
B. Pouring out without necrosis and ulceration
C. Primary elements appear on a leg
D. * Will strike the persons of yung and middle age
E. All adopted features
What complication may appear only in patient with HIV infection (Fig. 5)?
A. Pneumococal pneumonia
B. Megakaryoblastoma
C. * Pneumocystis pneumonia
D. Visceral leshmaniasis
E. Tularemia
What drug did use for the treatment of the most possible disease (Fig. 20)?
A. Amoxicillin
B. * Streptomycin
C. Penicillin
D. Biseptol
E. 5-NOK
What drug did use for the treatment of the most possible disease (Fig. 21)?
A. Amoxicillin
B. * Streptomycin
C. Penicillin
D. Biseptol
E. 5-NOK
56. What drug did use for the treatment of the most possible disease (Fig. 22)?
A. Amoxicillin
B. * Streptomycin
C. Penicillin
D. Biseptol
E. 5-NOK
57. What drug did use for the treatment of the most possible disease (Fig. 9)?
A. Merapenem
B. Flukonazol
C. * Interferon
D. Vitamin K
E. Levomicetyn
58. What drug did use for the treatment of the most possible disease (Fig. 10)?
A. Merapenem
B. Flukonazol
C. * Interferon
D. Vitamin K
E. Levomicetyn
59. What is optimum preparation for the prophylaxis of relapse of this disease (Fig. 42)?
A. Bicillinum 1
B. Glyukokortikoides
C. Bicillinum 3 and glyukokortikoides
D. * Bicillinum 5
E. Bicillinum 5 and glyukokortikoides
60. What is optimum preparation for the prophylaxis of relapse of this disease (Fig. 50)?
A. Bicillinum 1
B. Glyukokortikoides
C. Bicillinum 3 and glyukokortikoides
D. * Bicillinum 5
E. Bicillinum 5 and glyukokortikoides
61. What is optimum preparation for treatment of this disease (Fig. 40)?
A. * Bicillinum
B. Glyukokortikoides
C. Levomicetin
D. Furagin
E. Tetracyclinum
62. What is optimum preparation for treatment of this disease (Fig. 41)?
A. * Bicillinum
B. Glyukokortikoides
C. Levomicetin
D. Furagin
E. Tetracyclinum
63. What is optimum preparation for treatment of this disease (Fig. 42)?
A. * Bicillinum
B. Glyukokortikoides
C. Levomicetin
D. Furagin
E. Tetracyclinum
64. What is optimum preparation for treatment of this disease (Fig. 44)?
A. * Bicillinum
B. Glyukokortikoides
C. Levomicetin
D. Furagin
E. Tetracyclinum
65. What is optimum preparation for treatment of this disease (Fig. 45)?
A. * Bicillinum
B. Glyukokortikoides
66.
67.
68.
69.
70.
71.
72.
73.
74.
C. Levomicetin
D. Furagin
E. Tetracyclinum
What is optimum preparation for treatment of this disease (Fig. 46)?
A. * Bicillinum
B. Glyukokortikoides
C. Levomicetin
D. Furagin
E. Tetracyclinum
What is optimum preparation for treatment of this disease (Fig. 47)?
A. * Bicillinum
B. Glyukokortikoides
C. Levomicetin
D. Furagin
E. Tetracyclinum
What is the most possible diagnosis (Fig. 10)?
A. Plague, septic form
B. Anthrax, septic form
C. * Hemorrhagic fever
D. Tularemia, septic form
E. Sepsis
What is the most possible diagnosis (Fig. 15)?
A. Plague, skin-bubonic form
B. * Anthrax, skin form
C. Common carbuncle
D. Tularemia, skin-bubonic form
E. Sepsis
What is the most possible diagnosis (Fig. 16)?
A. Plague, skin-bubonic form
B. * Anthrax, skin form
C. Common carbuncle
D. Tularemia, skin-bubonic form
E. Sepsis
What is the most possible diagnosis (Fig. 18)?
A. Plague, skin-bubonic form
B. * Anthrax, skin form
C. Common carbuncle
D. Tularemia, skin-bubonic form
E. Sepsis
What is the most possible diagnosis (Fig. 20)?
A. * Plague, skin-bubonic form
B. Anthrax, skin form
C. Common carbuncle
D. Tularemia, skin-bubonic form
E. Sepsis
What is the most possible diagnosis (Fig. 21)?
A. * Plague, skin-bubonic form
B. Anthrax, skin form
C. Common carbuncle
D. Tularemia, skin-bubonic form
E. Sepsis
What is the most possible diagnosis (Fig. 22)?
A. * Plague, skin-bubonic form
B. Anthrax, skin form
C. Common carbuncle
D. Tularemia, skin-bubonic form
E. Sepsis
75. What is the most possible diagnosis (Fig. 9)?
A. Plague, septic form
B. Anthrax, septic form
C. * Hemorrhagic fever
D. Tularemia, septic form
E. Sepsis
76. What is your diagnosis (Fig. 13)?
A. * Sarcoma Kaposhi
B. Candidos
C. CMV-infection
D. Toxoplasmosis
E. Diphtheria
77. What is your diagnosis (Fig. 2)?
A. * Sarcoma Kaposhi
B. Candidos
C. CMV-infection
D. Toxoplasmosis
E. Diphtheria
78. What is your diagnosis (Fig. 4)?
A. * Sarcoma Kaposhi
B. Toxoplasmosis
C. Chicken pox
D. CMV-infection
E. Small pox
79. What is your diagnosis (Fig. 4)?
A. Leycoplakia
B. * Sarcoma Kaposhi
C. Furunculosis
D. Rubella
E. CMV-infection
80. What kind of specific prophylaxis should be conducted for this patient which was bitten (Fig. 60)?
A. Gamma-globulin and 18 doses of Kab
B. 12 doses of Kab
C. * 6 doses of Kab
D. Gamma-globulin and 21 dose of Kab
E. 2 doses of Kab
81. What kind of specific prophylaxis should be conducted for this patient which was bitten (Fig. 60)?
A. Gamma-globulin
B. * Vaccine
C. Vaccine and gamma-globulin
D. Gamma-globulin and serum
E. Serum
82. What kind of specific prophylaxis should be conducted for this patient which was bitten (Fig. 60)?
A. Gamma-globulin + tetanus antitoxin
B. * Antyrabies vaccine + tetanus antitoxin
C. Antyrabies vaccine and gamma-globulin
D. Gamma-globulin and serum
E. Serum + tetanus antitoxin
83. What mechanism of transmission of this virus (Fig. 1)?
A. Droplet
B. Fecal-oral
C. Transmissiv
D. * Contact
E. Vertical
84. What specific test is used for diagnostic of this disease (Fig. 15)?
A. Compliment fixation test
B. Indirect hemaglutination test
C. * Coetaneous test with antraxin
D. Hemaglutination test
E. RIFA with anthrax antigen
85. What specific test is used for diagnostic of this disease (Fig. 16)?
A. Compliment fixation test
B. Indirect hemaglutination test
C. * Coetaneous test with antraxin
D. Hemaglutination test
E. RIFA with anthrax antigen
86. What specific test is used for diagnostic of this disease (Fig. 18)?
A. Compliment fixation test
B. Indirect hemaglutination test
C. * Coetaneous test with antraxin
D. Hemaglutination test
E. RIFA with anthrax antigen